You are on page 1of 536

P

ICAP
Financial accounting and reporting I

* The student must refer original handbook of IFRS.

© Emile Woolf International i The Institute of Chartered Accountants of Pakistan


Ninth edition published by
Emile Woolf International
Bracknell Enterprise & Innovation Hub
Ocean House, 12th Floor, The Ring
Bracknell, Berkshire, RG12 1AX
United Kingdom
Email: info@ewiglobal.com
www.emilewoolf.com

© Emile Woolf International, September 2021

All rights reserved. No part of this publication may be reproduced, stored in a retrieval system, or transmitted,
in any form or by any means, electronic, mechanical, photocopying, recording, scanning or otherwise, without
the prior permission in writing of Emile Woolf International, or as expressly permitted by law, or under the
terms agreed with the appropriate reprographics rights organisation.

You must not circulate this book in any other binding or cover and you must impose the same condition on
any acquirer.

Notice
Emile Woolf International has made every effort to ensure that at the time of writing the contents of this study
text are accurate, but neither Emile Woolf International nor its directors or employees shall be under any
liability whatsoever for any inaccurate or misleading information this work could contain.

© Emile Woolf International ii The Institute of Chartered Accountants of Pakistan


Certificate in Accounting and Finance

C
Financial accounting and reporting I

Contents
Page

Chapter

1 IAS 16: Property, plant and equipment 1

2 Correction of errors 73

3 Non-Current Assets: Sundry standards 113

4 IAS 36: Impairment of assets 155

5 Statement of changes in equity 185

6 IAS 8: Accounting Policies, Changes in Accounting Estimates and 211


Errors

7 IAS 33: Earnings per Share 243

8 IAS 7: Statement of cash flows 273

9 Conceptual framework for financial reporting 373

10 Interpretation of financial statements 397

11 Accounting for Not-for-Profit Organisations 447

© Emile Woolf International iii The Institute of Chartered Accountants of Pakistan


Financial accounting and reporting I

© Emile Woolf International iv The Institute of Chartered Accountants of Pakistan


1
Certificate in Accounting and Finance

CHAPTER
Financial accounting and reporting I

IAS 16: Property, plant and equipment

Contents
1 Recognition and measurement
2 Depreciation
3 Derecognition
4 Revaluation
5 Disclosure
6 Objective based questions and answers

* The student must refer original handbook of IFRS.

© Emile Woolf International 1 The Institute of Chartered Accountants of Pakistan


Financial accounting and reporting I

1 RECOGNITION AND MEASUREMENT


Section overview

 Introduction
 Recognition
 Measurement at recognition
 Exchange transactions
 Subsequent expenditure
 Measurement after recognition

1.1 Introduction
Property, plant and equipment (PPE) are a company’s long term, tangible fixed assets that are vital
to business operations and cannot be easily liquidated.

Definitions
Property, plant and equipment are tangible items that:
 are held for use in the production or supply of goods or services, for rental to others, or for
administrative purposes; and
 are expected to be used during more than one period.

1.2 Recognition
The cost of an item of PPE shall be recognised as an asset if, and only if:
(a) it is probable that future economic benefits associated with the item will flow to the entity; and
(b) the cost of the item can be measured reliably.
Spare parts and servicing equipment are usually carried as inventory and recognised in profit or
loss as consumed. However, major spare parts and stand-by equipment qualify as property, plant
and equipment when an entity expects to use them during more than one period. Similarly, if the
spare parts and servicing equipment can be used only in connection with an item of property, plant
and equipment, they are accounted for as property, plant and equipment.
As a practical expedient, immaterial items are not recognised as PPE even if they meet the
definition criteria, for example, staplers and calculators etc.

1.3 Measurement at recognition


Property, plant and equipment are initially recorded in the accounts of a business at their cost.

Definition
Cost is the amount of cash or cash equivalents paid and the fair value of the other consideration
given to acquire an asset at the time of its acquisition or construction.

The cost of an item of property, plant and machinery consists of:


 its purchase price after any discounts and rebates have been deducted plus any import taxes
or non-refundable sales tax; plus
 the directly attributable costs of ‘bringing the asset to the location and condition necessary
for it to be capable of operating in the manner intended by management’. These directly
attributable costs may include:
 employee costs arising directly from the installation or construction of the asset
 the cost of site preparation

© Emile Woolf International 2 The Institute of Chartered Accountants of Pakistan


Chapter 1: IAS 16: Property, plant and equipment

 initial delivery and handling costs (‘carriage inwards is the shipping and handling
costs incurred by a company that is receiving goods from suppliers. ’)
 installation and assembly costs
 testing costs
 professional fees such as architect and surveyor fee.
 When the entity has an obligation to dismantle and remove the asset at the end of its life, its
initial cost should also include an estimate of the costs of dismantling and removing the asset
and restoring the site where it is located.
The cost of a non-current asset cannot include any administration costs or other general overhead
costs.
Illustration 01: Cost
A company has purchased a large item of plant. The following costs were incurred.
Rupees
List price of the machine 1,000,000
Trade discount given 50,000
Delivery cost 100,000
Installation cost 125,000
Cost of site preparation 200,000
Architect’s fees 15,000
Administration expense 150,000
Local government officials have granted the company a license to operate the asset on
condition that the company will remove the asset and return the site to its former condition at
the end of the asset’s life.
The company has recognised a liability of Rs. 250,000 in respect of the expected clearance
cost.
The cost of the asset is as follows:
Purchase price of the machine (1,000,000 – 50,000) 950,000
Delivery cost 100,000
Installation cost 125,000
Cost of site preparation 200,000
Architect’s fees 15,000
Decommissioning cost 250,000
1,640,000

When the land is acquired, certain costs are necessary and should be part of the cost of land.
These costs include the cost of the land, title and legal fees, site preparation costs like grading and
draining and survey costs etc. All of these costs may be considered necessary to get the land ready
for its intended use. Some costs are land improvements. This asset category includes the cost
of parking lots, sidewalks, landscaping, irrigation systems etc. Care must be taken to distinguish
land and land improvement costs. Land is considered to have an indefinite life and is not
depreciated. Whereas land improvements do wear out and must be depreciated.
The costs of day-to-day servicing of an asset are not included in the carrying amount of the asset
but expensed when incurred.
Costs are no longer recognised when the item is ready for use. This is when it is in the location
and condition necessary for it to be capable of operating in the manner intended by management.
Examples of costs that are not costs of an item of property, plant and equipment are:
(a) costs of opening a new facility;
(b) costs of introducing a new product or service (including costs of advertising and promotional
activities);

© Emile Woolf International 3 The Institute of Chartered Accountants of Pakistan


Financial accounting and reporting I

(c) costs of conducting business in a new location or with a new class of customer (including costs
of staff training); and
(d) administration and other general overhead costs.
(e) Interest paid to finance the purchase of property, plant and equipment is expensed. An
exception is interest incurred on funds borrowed to finance construction of plant and
equipment. Such interest related to the period of time during which active construction is
ongoing is capitalized.
(f) The acquisition of new machinery is accompanied by employee training. The normal rule is
that training costs are expensed. The logic is that the training attaches to the employee not the
machine and the employee is not owned by the company. However, on rare occasion,
justification for capitalization of very specialized training costs (where the training is company
specific and benefits many periods) is made, but this is the exception rather than the rule.
Recognition of costs in the carrying amount of an item of property, plant and equipment ceases
when the item is in the location and condition necessary for it to be capable of operating in the
manner intended by management. Therefore, costs incurred in using or redeploying an item is not
included in the carrying amount of that item.
The following costs are not included in the carrying amount of an item of property, plant and
equipment:
(a) costs incurred while an item capable of operating in the manner intended by management has
yet to be brought into use or is operated at less than full capacity;
(b) initial operating losses; and
(c) relocating cost.
Exception
If a company main business is selling machines then that machine does not classify as property,
plant and equipment rather the machinery used to produce the machine for sales is PP&E. The
machine manufactured for sale is classified as inventory. The same goes for real estate companies.
Their offices are PP&E but the houses they sell are inventory.

1.4 Exchange transactions


An asset may be acquired in exchange for another asset. The cost of such an asset is measured
at its fair value unless:
 the exchange transaction lacks commercial substance; or
 the fair value of neither the asset received nor the asset given up is reliably measurable.
If the new asset is measured at fair value, the fair value of the asset given up is used to measure
the cost of the asset received unless the fair value of the asset received is more clearly evident.
If the new asset is not measured at fair value, its cost is measured at the carrying amount of the
asset given in exchange for it. This would be the case when the exchange lacked commercial
substance or when the fair value of either asset cannot be measured.

Example 01: Exchange of assets


Question: Following information pertains to three exchange transactions relating to fixed assets:
(i) (ii) (iii)
--------- Rs. in million ---------
Cash received/(paid) 1.1 (2.1) -

Assets given-up:
Original cost 10.3 12.4 14.5
Book value 6.4 7.3 3.4
Estimated fair value 8.5 6.6 4.6
Assets received:
Estimated fair value 7.1 9.0 4.1

© Emile Woolf International 4 The Institute of Chartered Accountants of Pakistan


Chapter 1: IAS 16: Property, plant and equipment

Additional information:
 In case of transaction (i), fair values of both assets are reliably measurable.
 In case of transaction (ii), fair value of the asset received is clearly more evident.
 In case of transaction (iii), fair value of neither asset is reliably measurable.
Required:
Calculate the cost of asset received at recognition for each of above transactions.

Answer:
Transaction (i)
Fair value of asset given up Rs. 8.5m – cash received Rs. 1.1m = Rs. 7.4 million
Transaction (ii)
Fair value of asset received = Rs. 9.0 million
Transaction (iii)
Carrying amount = Rs. 3.4 million

1.5 Subsequent expenditure


Subsequent expenditure relating to non-current assets, after their initial acquisition, should be
capitalised if it meets the criteria for recognising an asset.
In practice, this means that expenditure is capitalised if it:
 improves the asset (for example, by enhancing its performance or extending its useful life);
or
 is for a replacement part (provided that the part that it replaces is treated as an item that has
been disposed of).

1.6 Measurement after recognition


Property, plant and equipment is recognised at cost when it is first acquired.
IAS 16 allows a business to choose one of two measurement models as its accounting policy for
property, plant and equipment after initial recognition. However, the same measurement model
should be applied to all assets in the same class (e.g. if an entity has three different factories at
different geographical locations then it cannot discretionally revalue two factories and applies cost
model on one of its factory.
The two measurement models for property, plant and equipment after acquisition are:
 cost model (i.e. cost less accumulated depreciation); and
 revaluation model (i.e. revalued amount less accumulated depreciation since the most
recent revaluation).
For example, a company’s policy might be to value all its motor vehicles at cost, but to apply the
revaluation model to all its land and buildings.
Understanding the concept and calculation of depreciation is necessary to apply both
measurement models. However, revaluation model has few more issues which will be discussed
later in this chapter.

© Emile Woolf International 5 The Institute of Chartered Accountants of Pakistan


Financial accounting and reporting I

2 DEPRECIATION
Section overview

 Concept and definitions


 Method of calculating depreciation
 Depreciation charge and period
 Change in estimates
 Accounting for depreciation

2.1 Concept and definitions


In simple words, depreciation is an expense that matches the cost of a non-current asset to the
benefit earned from its ownership. It is calculated so that a business recognises the full cost
associated with a non-current asset over the entire period that the asset is used. In effect, the cost
of the asset is transferred to the statement of comprehensive income over the life of the asset. This
may be several years.

Definitions
Depreciation: The systematic allocation of the depreciable amount of an asset over its useful life.
Depreciable amount: The cost of an asset (or its revalued amount, in cases where a non-current
asset is revalued during its life) less its residual value.
The residual value of an asset is the estimated amount that an entity would currently obtain from
disposal of the asset, after deducting the estimated costs of disposal, if the asset were already of
the age and in the condition expected at the end of its useful life.
Useful life is:
(a) the period over which an asset is expected to be available for use by an entity; or
(b) the number of production or similar units expected to be obtained from the asset by an
entity.
Accumulated Depreciation is the depreciation charged to date (cumulative) on a non-current asset.
This is contra asset account.
Carrying amount (also called net book value or written down value) is the amount at which an asset
is presented in statement of financial position. Carrying amount = Cost – accumulated depreciation

Illustration 02: Depreciation


ABC Enterprise bought a machine for Rs. 700,000 in early 2021.
Although the machine can be used for seven years but ABC Enterprises expects to use it for 5 years
only.
The cost of machine is Rs. 700,000 and its useful life is 5 years.
ABC Enterprise estimated that machine can be disposed of for Rs. 620,000 (at current prices) and
for Rs. 680,000 (at prices expected at end of 2025). Further, a 5 year old similar machine can be
disposed of for Rs. 150,000 (at current prices) and for Rs. 330,000 (at prices expected at end of
2025).
The residual value is Rs. 150,000 that can be currently obtained if the machine was already in 5
years old condition.
Depreciation is a way of allocating the depreciable amount of Rs. 550,000 (= Rs. 700,000 - Rs.
150,000) over the five years of the asset’s expected life.

© Emile Woolf International 6 The Institute of Chartered Accountants of Pakistan


Chapter 1: IAS 16: Property, plant and equipment

Example 02: Definitions


Question: An asset costs Rs. 100,000 and can be easily used for ten years. The management of
the business entity intends to use the asset for six years at which point expected residual value will
be Rs. 40,000 (at current prices).
Required:
What is depreciable amount and useful life of above asset?
Answer:
Depreciable amount = Rs. 100,000 – 40,000 = Rs. 60,000
Useful life is six years as intended by the management of the entity.

2.2 Method of calculating depreciation


The depreciation method used should reflect as fairly as possible the pattern in which the asset’s
economic benefits are consumed by the entity.
Straight-line method

Definition: Straight line depreciation


Where the depreciable amount is charged in equal amounts to each reporting period over the
expected useful life of the asset.

Formula: Straight-line depreciation

Depreciation charge for Cost of asset less expected residual value


=
the year Expected useful life (years)
Alternatively,

Depreciation charge = Cost x Rate

Where,

Depreciation charge for = Depreciable amount


the year Cost x Useful life

With the straight-line method, the annual depreciation charge is the same for each full financial
year over the life of the asset.
This is the most common method in practice and the easiest to calculate.

Illustration 03: Straight line depreciation


A machine cost Rs. 250,000. It has an expected economic life of five years and an expected
residual value of Rs. 50,000 at the end of that time.
Annual depreciation is:

250,000  50,000 Rs. 40,000 per


Depreciation charge = =
5 years annum

© Emile Woolf International 7 The Institute of Chartered Accountants of Pakistan


Financial accounting and reporting I

Illustration 04: Straight line depreciation – mid-year acquisition


A machine cost Rs. 250,000. It has an expected economic life of five years.
It is expected that the machine will have a zero scrap value at the end of its useful life.
The machine was bought on the 1st September and the company has a 31st December year end.
The depreciation charge in the first year of ownership is:
250,000 4
Depreciation charge = × = Rs. 16,667
5 years 12

Reducing balance method

Definition: Reducing balance method


Where the annual depreciation charge is a fixed percentage of the carrying amount of the asset at
the start of the period.

Formula: Reducing balance depreciation

Depreciation charge for Carrying amount at the


=  Fixed %
the year start of the year

The annual depreciation charge is highest in the first year and lowest in the final year of the asset’s
economic life.

Illustration 05: Reducing balance method


A machine cost Rs. 100,000. It has an expected life of five years, and it is to be depreciated by the
reducing balance method at the rate of 30% each year.
Annual depreciation and carrying amount over the life of the asset will be as follows.

Annual depreciation
Carrying amount at charge (at (30% of the Carrying amount at
Year start of year reducing balance) end of year

Rs. Rs. Rs.

1 100,000 30,000 70,000

2 70,000 21,000 49,000

3 49,000 14,700 34,300

4 34,300 10,290 24,010

5 24,010 7,203 16,807

Calculating the reducing balance


The reducing balance reduces the cost of an asset down to its expected residual value over its
expected useful life.
The reducing balance percentage can be calculated using the following formula.

© Emile Woolf International 8 The Institute of Chartered Accountants of Pakistan


Chapter 1: IAS 16: Property, plant and equipment

Formula: Calculation of reducing balance percentage

𝑛 Residual value
𝑥 = 1− √
Cost

Where:

x = The reducing balance percentage

n = Expected useful life.

Illustration 06: Reducing balance


An asset cost Rs. 10,000 and has an expected residual value of Rs. 2,000 at the end of its expected
useful life which is 5 years.
The reducing balance percentage is calculated as follows.

n Residual value 5 2,000


𝑥 =1− √ = 1− √ = 0.275 or 27.5%
Cost 10,000

This percentage reduces Rs.10,000 to Rs. 2,000 over 5 years.

Annual depreciation
Carrying amount Carrying amount at
Year charge (at (27.5%
at start of year end of year
reducing balance)

Rs. Rs. Rs.

1 10,000 2,750 7,250

2 7,250 1,994 5,256

3 5,256 1,445 3,811

4 3,811 1,048 2,763

5 2,763 763 2,000

Note that the depreciation charge in year 5 contains a rounding difference of 3.

Sum-of-the-digits method

Definition
Where depreciation is calculated by multiplying the depreciable amount by a fraction where
numerator is the remaining life of the asset at the start of the period and the denominator is the
sum of all the years’ useful life at the start of ownership.

This is another method of depreciation that charges the highest amount in the first year and the
lowest amount in the final year.

© Emile Woolf International 9 The Institute of Chartered Accountants of Pakistan


Financial accounting and reporting I

Illustration 07: Sum of the digits


A machine cost Rs. 500,000 and was expected to have a useful life of 5 years.
Annual depreciation over the life of the asset will be as follows.

Step 1: Calculate the sum of the digits (the denominator in the fraction)
Year
1
2
3
4
5
Sum of the digits 115

Illustration 08: Sum of the digits (continued)

Step 2: Calculate the annual depreciation charge in each year

Year Remaining useful life at Annual depreciation charge


the start of the year

5
1 5 ×500,000=166,667
15

4
2 4 ×500,000=133,333
15

3
3 3 ×500,000=100,000
15

2
4 2 ×500,000=66,667
15

1
5 1 ×500,000=33,333
15

There following formula can be used to calculate the sum of the digits:

Formula: Sum of the digits

n(n+1)
Sum of the digits=
2

Where:

n = the useful life at the start of ownership

© Emile Woolf International 10 The Institute of Chartered Accountants of Pakistan


Chapter 1: IAS 16: Property, plant and equipment

Illustration 09: Sum of the digits by formula

5 year useful life


n(n+1) 5(5+1) 30
Sum of the digits= = = =15
2 2 2
25 year useful life
n(n+1) 25(25+1) 650
Sum of the digits= = = =325
2 2 2

Depreciation by the number of units produced

Definition
Where depreciation is calculated by expressing the useful life of an asset in terms of its expected
total output and allocating the annual charge to depreciation based on actual output.

Formula: Depreciation by number of units produced

Cost of asset less expected


Depreciation charge residual value Output this
= 
for the year Expected total output of asset year
over its life

Illustration 10: Number of units produced


A machine cost Rs. 500,000.
It is expected to produce 5,000,000 units over its useful life.
47,850 units were made in the first year of production.
The depreciation charge in the first year of ownership is:

Rs.500,000
= ×47,850=Rs. 4,785
5,000,000

2.3 Depreciation charge and period


Depreciation should be charged as an expense in the statement of comprehensive income each
year over the life of the asset even if the asset is idle and not being used. However, under usage
methods of depreciation (sum of units’ method), the depreciation charge can be zero while there
is no production.
Land is not depreciated because it has indefinite useful life.

Commencement of depreciation
Depreciation must be charged from the date the asset is available for use. This may be earlier
than the date it is actually brought into use.

End of depreciation
The depreciation is no more charged when the asset is derecognized or disposed of. In simple
words, depreciation is an expense that matches the cost of a non-current asset to the benefit
earned from its ownership. It is calculated so that a business recognises the full cost associated
with a non-current asset over the entire period that the asset is used. In effect, the cost of the asset
is transferred to the statement of comprehensive income over the life of the asset. This may be
several years.

© Emile Woolf International 11 The Institute of Chartered Accountants of Pakistan


Financial accounting and reporting I

2.4 Change in estimates

Change in depreciation method


A change from one method of providing depreciation to another method is permissible only on
the grounds that the new method will give a fairer presentation of the results and of the financial
position.
The carrying amount should be written off over the remaining useful life, commencing with the
period in which the change is made.

Illustration 11: Marden Fabrics


Marden Fabrics owns a machine which originally cost Rs. 30,000 on 1 January 2014. It has no
residual value.
It was being depreciated over its useful life of 10 years on a straight-line basis. At the end of 2017,
when preparing the financial statements for 2017, Marden Fabrics decided to change the method
of depreciation, from straight-line to the reducing balance method, using a rate of 25%.
The depreciation charge for 2017 shall be calculated as follows:
The change in accounting estimate is made at the end of 2017, but is applied to the financial
statements from 1 January 2017. The reducing balance method of depreciation is applied to the
2017 statements.
Rs.
Cost on 1 January 2014 30,000
Depreciation for 2014 to 2016 (30,000 × 3/10) (9,000)
Carrying amount at end of 2016 21,000
Depreciation for 2017 will therefore be Rs. 21,000 × 25% = Rs. 5,250.

Review of useful life and residual values


Useful life and residual value of tangible non-current assets should be reviewed at end of each
reporting period and revised if expectations are significantly different from previous estimates.
The carrying amount of the asset at the date of revision less any residual value should be
depreciated over the revised remaining useful life.

Illustration 12: Chiniot Engineering


Chiniot Engineering owns a machine which originally cost Rs. 60,000 on 1 January 2014. The
machine was being depreciated over its useful life of 10 years on a straight-line basis and has no
residual value. On 31 December 2017 Chiniot Engineering revised the total useful life for the
machine to eight years (down from the previous 10).
The depreciation charge for 2017 and subsequent years shall be calculated as follows:
The change in accounting estimate is made at the end of 2017 but may be applied to the financial
statements from 2017 onwards.
Rs.
Cost on 1 January 2014 60,000
Depreciation for 2014 to 2016 (60,000 × 3/10) (18,000)
Carrying amount at end of 2016 42,000
Remaining useful life at the end of 2016 = 8 – 3 years = 5 years.
Depreciation for 2017 and subsequent years = Rs. 42,000 ÷ 5 years = Rs. 8,400.

© Emile Woolf International 12 The Institute of Chartered Accountants of Pakistan


Chapter 1: IAS 16: Property, plant and equipment

Illustration 13: Review of useful life


A machine was purchased three years ago on 1 January Year 2. It cost Rs.150,000 and its expected
life was 10 years with an expected residual value of Rs.30,000.
Due to technological changes, the estimated life of the asset was re-assessed during Year 5. The
total useful life of the asset is now expected to be 7 years and the machine is now considered to
have no residual value.
The financial year of the entity ends on 31 December.
The depreciation charge for the year ending 31 December Year 5 shall be as follows:
Original depreciation=(150,000 – 30,000) /10 = Rs.12,000 per annum
Carrying amount at start of year 5 = 150,000 – (12,000 3) = Rs.114,000
If the total useful life is anticipated to be 7 years then there are four years remaining.
Depreciation charge for year 5 =Rs.114,000/4 = Rs.28,500

2.5 Accounting for depreciation


The double entry for depreciation is as follows:

Illustration 14: Depreciation double entry

Debit Credit

Depreciation expense X

Accumulated depreciation X

The balance on the depreciation expense account is taken to the statement of comprehensive
income as an expense for the period.
The accumulated depreciation account contains all of the depreciation recognised to date. When
the final statement of financial position is prepared it is deducted from the cost of the assets. The
non-current asset figure in the statement of financial position is made up of two figures, the cost
less accumulated depreciation.
The balance on the accumulated depreciation account is carried forward as a (credit) balance at
the end of the period and appears in the statement of financial position as a deduction from the cost
of the non-current assets. The figure that appears in the statement of financial position is known
as the carrying amount (or net book value).

Illustration 15: Carrying amount of a non-current asset

Rs.
Non-current asset at cost X
Less accumulated depreciation (X)
This figure appears on the face of the
Carrying amount (net book value) X statement of financial position

Accounts in the ledger for non-current assets and accumulated depreciation


There are separate accounts in the general ledger for each category of non-current assets (for
example, an account for land and buildings, an account for plant and machinery, an account for
office equipment, an account for motor vehicles, and so on) and the accumulated depreciation for
each of these categories of non-current assets.

© Emile Woolf International 13 The Institute of Chartered Accountants of Pakistan


Financial accounting and reporting I

This means that each category of non-current assets can be shown separately in the financial
statements.

Illustration 16: Accounting for depreciation


A company purchases a non-current asset in Year 1 for Rs. 90,000.
In Year 1, the depreciation charge is Rs. 15,000.
These transactions should be recorded as follows:
Asset account
Year 1 Rs. Rs.
Cash/creditors 90,000 Balance c/f 90,000
90,000 90,000
Year 2
Balance b/f 90,000

Illustration 17: Accounting for depreciation (continued)


Accumulated depreciation account
Year 1 Rs. Rs.
Balance c/f 15,000 Depreciation account 15,000
15,000 15,000
Year 2
Balance b/f 15,000

Depreciation account
Year 1 Rs. Rs.
Accumulated depreciation 15,000 Statement of comprehensive
income 15,000
15,000 15,000
At the end of Year 1, the carrying amount of the asset in the statement of financial position is:
Rs.
Non-current asset at cost (or valuation) 90,000
Less: Accumulated depreciation (15,000)
Carrying amount 75,000

Illustration 18:
The depreciation charge In Year 2 is also Rs. 15,000.
The ledger accounts in Year 2 will be as follows:

Asset account
Year 2 Rs. Rs.
Balance b/f 90,000 Balance c/f 90,000
90,000 90,000
Year 3
Balance b/f 90,000

© Emile Woolf International 14 The Institute of Chartered Accountants of Pakistan


Chapter 1: IAS 16: Property, plant and equipment

Accumulated depreciation account


Year 2 Rs. Rs.
Balance b/f 15,000
Balance c/f 30,000 Depreciation account 15,000
30,000 30,000
Year 3
Balance b/f 30,000

Depreciation account
Year 2 Rs. Rs.
Accumulated depreciation 15,000 Statement of comprehensive
income 15,000

At the end of Year 2, the carrying amount of the asset in the statement of financial position is:
Rs.
Non-current asset at cost (or valuation) 90,000
Less: Accumulated depreciation (30,000)
Carrying amount 60,000

© Emile Woolf International 15 The Institute of Chartered Accountants of Pakistan


Financial accounting and reporting I

3 DERECOGNITION
Section overview

 Gain or loss on disposal of a non-current asset


 Accounting for the disposal

3.1 Gain or loss on disposal of a non-current asset


Property, plant and equipment are eventually disposed of:
 by sale, or
 if they have no sale value, through disposal as scrap.
Disposal can occur at any time, and need not be at the end of the asset’s expected useful life.
The effect of a disposal on the statement of financial position (or accounting equation) is that:
 the asset (at cost or valuation) is no longer in the statement of financial position, and
 the accumulated depreciation on the asset is also no longer in the statement of financial
position.
The carrying amount of the asset is therefore removed from the accounting equation.
Gains on disposal should not be classified in the profit and loss account as revenue. Rather, it
should be classified in other income.
There is a gain or loss on disposal of the asset calculated as:
= Net sale proceeds – Carrying amount
Net sale proceeds are consideration received or receivable net of disposal costs, if any.

Illustration 19: Gain or loss on disposal


A non-current asset originally cost Rs.75,000. Accumulated depreciation is Rs.51,000. The asset
is now sold for Rs.18,000. Disposal costs are Rs.500.
The gain or loss on disposal shall be calculated as follows:
Net sale proceeds = Rs. 18,000 – 500 = Rs. 17,500
Carrying amount = Rs. 75,000 – 51,000 = Rs. 24,000
Gain (loss) on disposal = Rs. 17,500 – 24,000 = Rs. (6,500) Loss

In case of exchange of assets, net sale proceeds are:


= Cost of asset received + Cash received (deduct, if paid) – disposal costs
= Trade in value – disposal costs

Example 03: Exchange of assets and disposal


Question: Following information pertains to three exchange transactions relating to fixed assets:
(i) (ii) (iii)
--------- Rs. in million ---------
Cash received/(paid) 1.1 (2.1) -

Assets given-up:
Original cost 10.3 12.4 14.5
Book value 6.4 7.3 3.4
Estimated fair value 8.5 6.6 4.6
Assets received:
Estimated fair value 7.1 9.0 4.1

© Emile Woolf International 16 The Institute of Chartered Accountants of Pakistan


Chapter 1: IAS 16: Property, plant and equipment

Additional information:
 In case of transaction (i), fair values of both assets are reliably measurable.
 In case of transaction (ii), fair value of the asset received is clearly more evident.
 In case of transaction (iii), fair value of neither asset is reliably measurable.
Required:
Calculate the gain or loss on disposal for each of above transactions.

Answer:
Transaction (i)
Fair value of asset given up Rs. 8.5m – cash received Rs. 1.1m = Rs. 7.4 million
Gain (loss) on disposal = [Rs. 7.4m + 1.1m] – 6.4m = Rs. 2.1m gain
Transaction (ii)
Fair value of asset received = Rs. 9.0 million
Gain (loss) on disposal = [Rs. 9.0m – 2.1m] – 7.3m = Rs. (0.4)m loss
Transaction (iii)
Carrying amount = Rs. 3.4 million
Gain (loss) on disposal = [Rs. 3.4m – 0] – 3.4m = Rs. 0 (neither gain nor loss)

3.2 Accounting for the disposal


In the general ledger the gain or loss on disposal of a non-current asset is recorded in a disposal
of asset account. The double entry transactions required are as follows for an asset recorded at
cost rather than at a revalued amount.
Step 1: Transfer the cost of the non-current asset from the asset account to the disposal account:

Illustration 20:

Debit Credit

Disposal account X

Non-current asset account (cost of the asset) X


Step 2: Transfer the accumulated depreciation on the asset from the accumulated depreciation
account to the disposal account:

Illustration 21:

Debit Credit

Accumulated (or Allowance for) depreciation account X

Disposal account X

The carrying amount of the asset is now in the disposal account.

© Emile Woolf International 17 The Institute of Chartered Accountants of Pakistan


Financial accounting and reporting I

Step 3: Record the sale proceeds in the disposal account:

Illustration 22:

Debit Credit

Bank or Receivables account X

Disposal account (sale proceeds) X

Step 4: Record the asset received in part exchange (if any).

Illustration 23:

Debit Credit

Non-current asset account (cost of the new asset) X

Disposal account X

Step 5: Record the disposal costs in the disposal account.

Illustration 24:

Debit Credit

Disposal account (disposal expenses) X

Bank or Payables account X

Step 6: Record the cash paid in part-exchange (if any)

Illustration 25:

Debit Credit

Disposal account X

Bank or Payables account X

Step 7: The balance on the disposal account is the gain or loss on disposal. This is transferred to
the statement of comprehensive income.

Illustration 26:

Debit Credit

Disposal account X

Gain on disposal X

OR

Loss on disposal X

Disposal account X

© Emile Woolf International 18 The Institute of Chartered Accountants of Pakistan


Chapter 1: IAS 16: Property, plant and equipment

Illustration 27:
A non-current asset cost Rs.82,000 when purchased. It was sold for Rs.53,000 when the
accumulated depreciation was Rs.42,000. Disposal costs were Rs.2,000.
The book-keeping entries to record the disposal shall be as follows:
Disposal of asset account
Rs. Rs.
Non-current asset account 82,000 Accumulated depreciation 42,000
account
Disposal expenses (Bank) 2,000 Sales value (Receivables) 53,000
Gain on disposal (statement of 11,000
comprehensive income)
95,000 95,000

Non-current asset account


Rs. Rs.
Opening balance 82,000 Disposal account 82,000

Accumulated depreciation account


Rs. Rs.
Disposal account 42,000 Opening balance 42,000

Receivables account
Rs. Rs.
Disposal account
(sale value of disposal) 53,000

Bank account
Rs. Rs.
Disposal account
(disposal expenses) 2,000

Statement of comprehensive income


Rs. Rs.
Disposal account
(gain on disposal) 11,000

Non-current asset accounts in the general ledger are usually maintained for a category of assets
rather than for individual assets. This means that when a non-current asset is disposed of, there
will be a closing balance to carry forward on the asset account and the accumulated depreciation
account.

© Emile Woolf International 19 The Institute of Chartered Accountants of Pakistan


Financial accounting and reporting I

Illustration 28 (continued):
In the previous example, suppose that the balance on the non-current asset account before the
disposal was Rs.500,000 and the balance of the accumulated depreciation account was
Rs.180,000.
The accounting entries would be as follows:
Property, plant and equipment account
Rs. Rs.
Opening balance b/f 500,000 Disposal account 82,000
Closing balance c/f 418,000
500,000 500,000
Opening balance b/f 418,000

Accumulated depreciation account


Rs. Rs.
Disposal account 42,000 Opening balance b/f 180,000
Closing balance c/f 138,000
180,000 180,000
Opening balance b/f 138,000

Example 04: Alpha Enterprises


Question: Following information pertains to plant and machinery of Alpha Enterprises (AE):
(i) As at 1 January 2018, balances of cost and accumulated depreciation amounted to Rs.
12,700,000 and Rs. 6,240,000 respectively.
(ii) On 1 April 2018, an old machine having fair value of Rs. 340,000 was exchanged for a
new machine. The balance of the purchase price was paid through a cheque of Rs.
680,000. The list price of the new machine was Rs. 1,130,000. The old machine had
been acquired for Rs. 870,000 on 1 September 2015.
(iii) On 1 February 2018, a plant having a list price of Rs. 10,000,000 was acquired. A trade
discount of 5% was allowed on the list price. The plant was ready for use on 1 August 2018
after incurring the following costs:
Rs. in '000
Freight charges 660
Consultant fees 540
Installation and testing 600
Administration and other general overheads 160
Staff training 120
Opening ceremony 100
2,180
(iv) On 31 October 2018, another machine was sold for Rs. 334,000. It was acquired on 1
January 2015 and had a net book value of Rs. 512,000 on 1 January 2018. A cost of Rs.
25,000 was incurred on its disposal.
(v) AE depreciates plant and machinery at 20% per annum using the reducing balance
method.
Required:
Prepare following ledger accounts pertaining to the plant and machinery for the year ended 31
December 2018:
(a) Cost
(b) Accumulated depreciation
(c) Assets disposal

© Emile Woolf International 20 The Institute of Chartered Accountants of Pakistan


Chapter 1: IAS 16: Property, plant and equipment

Answer:
Alpha Enterprises
Plant and machinery - Cost
Date Description Rs. 000 Date Description Rs. 000
1-Jan-18 Balance 12,700 1-Apr-18 Assets disposal 870
1-Apr-18 Assets disposal (340+680) 1,020 31-Oct-18 Assets disposal (W2) 1,000
1-Aug-18 Capital WIP (W4) 11,300 31-Dec-18 Balance 23,150
25,020 25,020

Accumulated depreciation - Plant and machinery


1-Apr-18 Assets disposal (W-1) 376 1-Jan-18 Balance 6,240
31-Oct-18 Assets disposal (W-2) 573 31-Dec-18 Depreciation (W3) 2,292
31-Dec-18 Balance 7,583
8,532 8,532

Assets disposal - Plant and machinery


1-Apr-18 P&M 870 1-Apr-18 Acc. depreciation (W-1) 376
1-Apr-18 Cash paid 680 1-Apr-18 P&M (New) 1,020
31-Oct-18 Cost 1,000 31-Oct-18 Acc. depreciation (W-2) 573
31-Oct-18 Bank (disposal cost) 25 31-Oct-18 Bank (Sales proceeds) 334
31-Dec-18 Loss on disposal (P&L) 272
2,575 2,575

W1: Accumulated depreciation – Machine exchange Rs. 000


Depreciation for 2015 870×20%×4/12 58
Depreciation for 2016 (870–58)×20% 162
Depreciation for 2017 (870–58–162)×20% 130
Accumulated depreciation up to 31-12-2017 350
Depreciation for 2018 W3 26
376

W2: Accumulated depreciation – Machine sold Rs. 000


Cost [512 / (0.8)3] 1,000

Accumulated depreciation up to 01-01-2018 1,000 – 512 488


Depreciation for 2018 W3 85
Accumulated depreciation at the date of disposal 573

Acc. Dep. WDV


W3: Depreciation Cost
1 Jan 2018 Rate Months Rs. 000
(Year 2018)
Rs. 000
Disposed 1 870 350 520 20% 3/12 26
Disposed 2 1,000 488 512 20% 10/12 85
Other remaining 10,830 5,402 5,428 20% 12/12 1,086
Total opening 12,700 6,240 6,460

Addition 1 1,020 20% 9/12 153


Addition 2 11,300 20% 5/12 942
2,292

W4: Cost of the plant: Rs. in '000


Purchase price of the plant 10,000×95% 9,500
Other relevant cost 660+540+600 1,800
11,300
.

All disposal entries may be combined into one compound entry and in that case disposal account
is not required to be created.

© Emile Woolf International 21 The Institute of Chartered Accountants of Pakistan


Financial accounting and reporting I

Illustration 29:

Debit Credit

Accumulated depreciation X

Bank / Other receivable (Sale proceeds) X

PPE (New acquired in part exchange, if any) X

Loss on disposal (balancing figure if debit side is short) X

Bank / Payable (Cash paid and disposal expenses) X

PPE (cost of asset disposed) X

Gain on disposal (balancing figure if credit side is short) X

© Emile Woolf International 22 The Institute of Chartered Accountants of Pakistan


Chapter 1: IAS 16: Property, plant and equipment

4 REVALUATION
Section overview

 Revaluation model – issues


 Accounting for revaluation
 Changing the carrying amount of a revalued asset
 Depreciation of a revalued asset
 Realisation of the revaluation surplus
 The frequency of revaluations

4.1 Revaluation model - issues


The following accounting issues have to be addressed when using the revaluation model:

Issue

1 What happens to the other side of the entry when the carrying amount of an asset is
changed as a result of a revaluation adjustment?
An asset value may increase or decrease.
What happens in each case?

2 How the carrying amount of the asset being revalued is changed? The carrying amount is
located in two accounts (cost and accumulated depreciation) and it is the net amount that
must be changed so how is this done?

3 How often should the revaluation take place?

4.2 Accounting for revaluation


When a non-current asset is revalued, its ‘carrying amount’ in the statement of financial position is
adjusted from carrying amount to its fair value (normally current market value) at the date of the
revaluation.
Asset carried at cost revalued upwards
An increase in value is credited to other comprehensive income and accumulated in equity under
the heading of revaluation surplus.

Illustration 30: Upward revaluation


Land was purchased for Rs.100 million on the first day of the 2019 accounting period.
The business applies the IAS 16 revaluation model to the measurement of land after initial
recognition.
The land was revalued to Rs.130 million at the end of the first year of ownership.

Other comprehensive Statement of profit or


Land
income loss
------------------ Rs. in million ------------------

At start 100   The surplus is


taken to other
Adjustment 30 30 Cr comprehensive
income
31/12/19 130 30 Cr

© Emile Woolf International 23 The Institute of Chartered Accountants of Pakistan


Financial accounting and reporting I

Double entry:

Debit Credit
Land 30 m
Revaluation surplus 30 m

Extract from the statement of financial position as at 31/12/19

ASSETS
Non-current assets
Property, plant and equipment 130 m
EQUITY AND LIABILITIES
Revaluation surplus 30 m

Asset carried at cost revalued downwards


A decrease in value is debited as an expense to the statement of comprehensive income.

Illustration 31: Downward revaluation


Land was purchased for Rs.100 million on the first day of the 2019 accounting period.
The business applies the IAS 16 revaluation model to the measurement of land after initial
recognition.
The land was revalued to Rs.90 million at the end of the first year of ownership.

Other comprehensive Statement of profit or


Land
income loss
------------------- Rs. in million ------------------

At start 100  
Adjustment (10) 10Dr
31/12/19 90

Double entry:

Debit Credit

Statement of profit or loss 10 m

Land 10 m

Asset carried at a revaluation deficit is revalued upwards


An asset might be carried at an amount lower than its original cost as a result of being revalued
downwards. If the asset is later revalued upwards, the revaluation increase is recognised in the
statement of comprehensive income to the extent of the previously recognised expense. The
amount that will be transferred to statement of comprehensive income is equal to the net expense
(expense already charged in SOCI – decrease in depreciation due to revaluation loss). That part
of any increase above the previously recognised expense is recognised in the usual way, directly
in other comprehensive income.

© Emile Woolf International 24 The Institute of Chartered Accountants of Pakistan


Chapter 1: IAS 16: Property, plant and equipment

Asset carried at a revaluation surplus revalued downwards


An asset might be carried at an amount higher than its original cost as a result of being revalued
upwards. If the asset is later revalued downwards, the revaluation decrease is recognised in other
comprehensive income to the extent of the previously recognised surplus. That part of any
decrease above the previously recognised surplus is recognised in the statement of
comprehensive income.

Illustration 32: Downward revaluation


A business purchased a plot of land on the first day of the 2015 accounting period.
The business applies the IAS 16 revaluation model to the measurement of land after initial
recognition. The business has a policy of revaluing land annually.
The initial amount recognised and the year-end values are shown below:

Rs.
Measurement on initial recognition 100
Valuation as at:
31 December 2015 130
31 December 2016 110
31 December 2017 95
31 December 2018 116

The double entries are as follows

As at 31 December 2015 Debit Credit

Land (130 – 100) 30

Other comprehensive income 30

As at 31 December 2016 Debit Credit

Other comprehensive income 20

Land (110 – 130) 20

The fall in value reverses a previously recognised surplus. It is recognised in OCI to the extent
that it is covered by the surplus.

As at 31 December 2017 Debit Credit


Other comprehensive income 10
Statement of profit or loss 5
Land (95 – 110) 15
The fall in value in part reverses a previously recognised surplus. It is recognised in OCI to the
extent that it is covered by the surplus. This reduces the revaluation surplus to zero.
Any amount not covered by the surplus is recognised as an expense in the statement of
comprehensive income.

© Emile Woolf International 25 The Institute of Chartered Accountants of Pakistan


Financial accounting and reporting I

As at 31 December 2018 Debit Credit


Land (116 – 95) 21
Statement of profit or loss 5
Other comprehensive income 16
A rise in value that reverses a previously recognised expense is recognised in the statement
of profit or loss to the extent that it reverses the expense. Any amount above this is recognised
in other comprehensive income.
Overview
Other
Statement of profit or
Land comprehensive
loss
income

At start 100  
Double entry 30 30 Cr
31/12/15 130

b/f 130

Adjustment (20) 20Dr 


31/12/16 110

b/f 110
Adjustment (15) 10Dr 5Dr

31/12/17 95

b/f 95
Adjustment 21 16Cr 5Cr
31/12/18 116

4.3 Changing the carrying amount of a revalued asset


In the previous illustration land was revalued. Land is not depreciated so the carrying amount of
land is represented in a single account. This made it easy to change:
The carrying amount of depreciable assets is the net of balances on two separate accounts. The
double entry to revalue the asset must take this into account.
A simple approach (and one that accords with IAS 16) is as follows:
 Step 1: Transfer the accumulated depreciation to the asset account. The result of this is that
the balance on the asset account is now the carrying amount of the asset and the
accumulated depreciation account in respect of this asset is zero.
 Step 2: Change the balance on the asset account to the revalued amount.

© Emile Woolf International 26 The Institute of Chartered Accountants of Pakistan


Chapter 1: IAS 16: Property, plant and equipment

Illustration 33:
A building owned by a company is carried at Rs. 8,900,000 (Cost of Rs.9m less accumulated
depreciation of Rs. 100,000. The company’s policy is to apply the revaluation model to all its land
and buildings.
A current valuation of this building is now Rs.9.6 million.

Step 1 Rs. (000) Rs. (000)


Accumulated depreciation 100
Asset 100
Step 2
Asset (Rs.9.6m – Rs.8.9m) 700
Other comprehensive income 700
Alternatively this could be done with a single journal
Asset (Rs.9.6m – Rs.9m) 600
Accumulated depreciation 100
Other comprehensive income 700

Illustration 34:
An office building was purchased four years ago for Rs.3 million.
The building has been depreciated by Rs. 100,000.
It is now re-valued to Rs.4 million.
Based on above Information the analysis and calculation are as under

Building account
Rs. Rs.
Opening balance b/f 3,000,000 Accumulated depreciation 100,000
Revaluation account 1,100,000 Closing balance c/f 4,000,000

4,100,000 4,100,000

Opening balance b/f 4,000,000


.

Accumulated depreciation of building account

Rs. Rs.

Building account 100,000 Opening balance b/f 100,000

Revaluation surplus

Rs. Rs.

Revaluation account 1,100,000

© Emile Woolf International 27 The Institute of Chartered Accountants of Pakistan


Financial accounting and reporting I

4.4 Depreciation of a revalued asset


After a non-current asset has been revalued, depreciation charges are based on the new valuation
over the new useful life (if changed).

Illustration 35:
An asset was purchased three years ago, at the beginning of Year 1, for Rs. 100,000.
Its expected useful life was six years and its expected residual value was Rs. 10,000.
It has now been revalued to Rs. 120,000. Its remaining useful life is now estimated to be three
years and its estimated residual value is now Rs. 15,000.
The straight-line method of depreciation is used.
Based on above Information the analysis and calculation are as under

(a) Annual depreciation originally (for Years 1 – 3)


= Rs.(100,000 – 10,000)/6 years = Rs.15,000.

Rs.
Cost 100,000
Less: Accumulated depreciation at the time of revaluation (= 3 years x
Rs.15,000) (45,000)
Carrying amount at the time of the revaluation 55,000
Re-valued amount of the asset 120,000

Transfer to the revaluation surplus 65,000

(b) Revised annual depreciation = Rs. (120,000 – 15,000)/3 years = Rs. 35,000.
(c) The annual depreciation charge in Year 4 will therefore be Rs. 35,000.

Rs.
Re-valued amount 120,000
Less: depreciation charge in Year 4 (35,000)
Carrying amount at the end of Year 4 85,000

4.5 Realisation of the revaluation surplus


All assets eventually disappear from the statement of financial position either by becoming fully
depreciated or because the company disposes them.
If nothing were done this would mean that there was a revaluation surplus on the face of the
statement of financial position that related to an asset that was no longer owned. IAS 16 allows
(but does not require) the transfer of a revaluation surplus to retained earnings when the asset to
which it relates is derecognised (realised).
This might happen over several years as the asset is depreciated or at a point in time when the
asset is disposed of.
Revalued assets being depreciated
Revaluation of an asset causes an increase in the annual depreciation charge. The difference is
known as excess depreciation.

© Emile Woolf International 28 The Institute of Chartered Accountants of Pakistan


Chapter 1: IAS 16: Property, plant and equipment

Excess depreciation is the difference between:


 the depreciation charge on the re-valued amount of the asset, and
 depreciation that would have been charged on historical cost.
Each year a business might make a transfer from the revaluation surplus to the retained profits
equal to the amount of the excess depreciation.

Illustration 36:

Debit Credit

Revaluation surplus X

Retained earnings X

Revalued assets being sold


When a revalued asset is sold the business might transfer the balance on the revaluation surplus
in respect of the asset into retained earnings. The journal entry would be the same as above.

Illustration 37:
An asset was purchased two years ago at the beginning of Year 1 for Rs. 600,000. It had an
expected life of 10 years and nil residual value.
Annual depreciation is Rs. 60,000 (= Rs. 600,000/10 years) in the first two years.
At the end of Year 2 the carrying value of the asset -Rs. 480,000.
After two years it is revalued to Rs.640,000.
Double entry: Revaluation
Debit Credit
Asset (Rs.640,000 – Rs.600,000) 40,000
Accumulated depreciation 120,000
Other comprehensive income 160,000
Each year the business is allowed to make a transfer between the revaluation surplus and retained
profits:
Double entry: Transfer
Debit Credit
Revaluation surplus (160,000/8) 20,000
Retained profits 20,000

4.6 The frequency of revaluations


When the revaluation model is applied to the measurement of property, plant and equipment, the
frequency of revaluations should depend on the volatility in the value of the assets concerned.
When the value of assets is subject to significant changes (high volatility), annual revaluations may
be necessary.

Illustration 38:
Depreciation calculation for revalued asset
ABC Ltd has a Factory that is revalued to Rs. 250,000 in the fourth year of the acquisition of the
Factory. Original cost of the building as Rs. 150,000 with estimated useful life of 10 years. The
company depreciates the factory on straight-line basis.

© Emile Woolf International 29 The Institute of Chartered Accountants of Pakistan


Financial accounting and reporting I

1. The Revaluation surplus amount is calculated as under:


Revaluation surplus amount
To calculate this, we need to know the carrying amount of asset at the time of revaluation
which is cost less accumulated depreciation of five years. Depreciation for four years is:
= 150,000 / 10 = 15,000 x 4 = Rs. 60,000
Carrying amount is therefore Rs. 90,000 (150,000 – 60,000)
As the factory was revalued to Rs. 250,000, therefore, the revaluation surplus amount is
Rs. 160,000 (250,000 – 90,000)
2. The Depreciation charge for the period is calculated as under:
Depreciation charge for the period
Divide the revalued amount over the remaining useful life to get depreciation charge for
the year:
= 250,000 / 6 = Rs. 41,667
3. The Excess depreciation to be transferred to retained earnings is calculated as under:
Excess depreciation
Depreciation on revalued amount = Rs.41,667
Depreciation on original cost = Rs.15,000
The difference is Rs. 26,667 (41,667 – 15,000) and this amount will be transferred from
revaluation surplus to retained earnings account if entity chose to do so. The journal entry
will be:
(Dr) Revaluation surplus a/c = Rs.26,667

(Cr) Retained earnings a/c = Rs.26,667

Illustration 39:
An asset was purchased four years ago at the beginning of Year 1 for Rs. 1,000,000. It had an
expected life of 10 years and nil residual value.
Annual depreciation is Rs. 100,000 (Rs. 1,000,000/10 years) in the first four years.
At the end of Year 4 the carrying value of the asset - Rs. 600,000.
At that time, it is re-valued to Rs. 1,200,000.
The amount to be posted in the revaluation surplus is as under:
Double entry: Revaluation
Debit Credit
Asset (Rs. 1,200,000 – Rs. 1,000,000) 200
Accumulated depreciation 400
Revaluation surplus 600
Each year the business is allowed to make a transfer between the revaluation surplus and retained
profits:
Double entry: Transfer
Debit Credit
Revaluation surplus (600/6) 100
Retained profits 100

© Emile Woolf International 30 The Institute of Chartered Accountants of Pakistan


Chapter 1: IAS 16: Property, plant and equipment

5 DISCLOSURE
Section overview
 General disclosure requirements
 Disclosure requirements under certain circumstances
 Disclosure for revalued assets
 Additional disclosure encouraged by IAS 16

5.1 General disclosure requirements


IAS 16 Property, plant and equipment requires the following disclosures in the notes to the
financial statements, for each major class of property, plant and equipment.
 The measurement bases used (cost or revaluation model) for determining the gross
carrying amount
 The depreciation methods used
 The useful lives or depreciation rates used
 Gross carrying amounts and the accumulated depreciation at the beginning and at the end
of the period
 A reconciliation between the opening and closing values for gross carrying amounts and
accumulated depreciation, showing:
 additions during the year
 disposals during the year
 depreciation charge for the year
 increase or decrease in asset resulting from revaluation and impairment losses
The following illustrations of how a simple table for tangible non-current assets may be presented
in a note to the financial statements are relevant:
Illustration 40: (Method 1)
Plant and
Property Total
equipment
Rs. m Rs. m Rs. m
Gross carrying amount at beginning of period 7,200 2,100 9,300
Less: Accumulated depreciation (800) (1,100) (1,900)
Net carrying amount at beginning of period 6,400 1,000 7,400
Additions 920 340 1,260
Disposals(carrying value) (205) (40) (245)
Depreciation expense (120) (250) (370)
Net carrying amount at end of period 6,995 1,050 8,045

Gross carrying amount at end of period 7,860 2,270 10,130


Less: Accumulated depreciation (865) (1,220) (2,085)
Net carrying amount at end of period 6,995 1,050 8,045

© Emile Woolf International 31 The Institute of Chartered Accountants of Pakistan


Financial accounting and reporting I

Illustration 41: (Method 2)

Plant and
Property Total
equipment

Cost Rs. m Rs. m Rs. m

At the start of the year 7,200 2,100 9,300

Additions 920 340 1,260

Disposals (260) (170) (430)

At the end of the year 7,860 2,270 10,130

Accumulated depreciation
At the start of the year 800 1,100 1,900
Depreciation expense 120 250 370
Accumulated depreciation on disposals (55) (130) (185)
At the end of the year 865 1,220 2,085
Carrying amount
At the start of the year 6,400 1,000 7,400
At the end of the year 6,995 1,050 8,045

5.2 Disclosure requirements under certain circumstances


An entity must also disclose:
 the existence and amounts of restrictions on title, and property, plant and equipment pledged
as security for liabilities;
 the amount of expenditures recognised in the carrying amount of an item of property, plant
and equipment in the course of its construction;
 the amount of contractual commitments for the acquisition of property, plant and equipment;
and
 if it is not disclosed separately in the statement of comprehensive income, the amount of
compensation from third parties for items of property, plant and equipment that were
impaired, lost or given up that is included in profit or loss.
5.3 Disclosure for revalued assets
When items of property, plant and equipment are stated at revalued amounts the following must
be disclosed:
 the effective date of the revaluation;
 whether an independent valuer was involved;
 for each revalued class of property, plant and equipment, the carrying amount that would
have been recognised had the assets been carried under the cost model; and
 the revaluation surplus, indicating the change for the period and any restrictions on the
distribution of the balance to shareholders.

© Emile Woolf International 32 The Institute of Chartered Accountants of Pakistan


Chapter 1: IAS 16: Property, plant and equipment

5.4 Additional disclosures encouraged by IAS 16


IAS 16 encourages disclosure of the following information as users of financial statements might
find it to be useful.
 the carrying amount of temporarily idle property, plant and equipment;
 the gross carrying amount of any fully depreciated property, plant and equipment that is
still in use;
 the carrying amount of property, plant and equipment retired from active use and held for
disposal; and
 when the cost model is used, the fair value of property, plant and equipment when this is
materially different from the carrying amount.

Example 05: ALI LIMITED


Question: Ali Limited (AL) uses the revaluation model for subsequent measurement of its property,
plant and equipment and has a policy of revaluing its assets on an annual basis using the net
replacement value method.
The following information pertains to AL’s building:
i. The building was purchased on 01 January 2010 for Rs. 2 hundred million with expected useful
life of ten years.
ii. AL depreciates buildings on the straight line basis over their useful life.
iii. The results of revaluations carried out during the last three years by Standard Valuation Service,
an independent firm of values, are as follows:
Revaluation date Fair value Rs. in million
1 January 2011 280
1 January 2012 170
1 January 2013 180

Required:
The journal entries relating to the above transactions including revaluations for the year ended
December 31, 2010, 2011, 2012 and 2013.
Answer:

Debit Credit
Date - Property, plant and equipment
---- Rs. in million ----
January Building 200
01, 2010 Account Payable 200
December Depreciation (200/10 years) 20
31, 2010 Accumulated depreciation 20
January Accumulated depreciation 20
01, 2011 Building 20
January Building (280 – (200 – 20)) 100
01, 2011 Revaluation surplus (OCI) 100
December Depreciation (200/9 years) 31
31, 2011 Accumulated depreciation 31
December Revaluation surplus (OCI) (31 – 20) 11
31, 2011 Retained earnings 11
January Accumulated depreciation 31
01, 2012 Building 31
January Revaluation surplus (OCI) ((280 – 31) – 170)) 79
01, 2012 Building 79

© Emile Woolf International 33 The Institute of Chartered Accountants of Pakistan


Financial accounting and reporting I

Debit Credit
Date - Property, plant and equipment
---- Rs. in million ----
December Depreciation (170/8 years) 21.25
31, 2012 Accumulated depreciation 21.25
December Revaluation surplus (21.25 – 20) 1.25
31, 2012 Retained earnings 1.25
January Accumulated depreciation 21.25
01, 2013 Building 21.25
January Building 31.25
01, 2013 Revaluation surplus (180 – (170 – 21.25)) 31.25
December Depreciation (180/7 years) 25.7
31, 2013 Accumulated depreciation 25.7
December Revaluation surplus (OCI) (25.7 – 20) 5.7
31, 2013 Retained earnings 5.7

Example 06: SUNDRY QUESTION 1


Question: The following are two independent scenarios:
1. A business purchased some land and buildings on 1 January 2011 for Rs.800million (land
Rs.250 million and buildings Rs.550 million). The buildings are to be depreciated over a
period of 50 years.
On 1 January 2015 the land and buildings were revalued to Rs. 1,500 million (land Rs.400
million and buildings Rs. 1,100 million). At this date the buildings were believed to have a
remaining useful life of 40 years.
Required:
What is the original depreciation charge for the buildings and the revised charge from 1
January 2015?
Answer:
The original depreciation charge for the buildings and the revised charge from 1 January
2015 are calculated as under:
Original depreciation = Rs.550 million /50 years = Rs.11 million
Revised depreciation (revalued amount) = Rs.1,100 million /40 years = Rs.27 million
Revised depreciation (cost model)
= Rs. 550m – (11 x 4) = Rs. 506m/ 40 years = Rs. 12.65 million
2. A business purchased land for Rs.250 million and buildings for Rs.400 million on 1 January
2011. The buildings were to be depreciated over a period of 50 years. On 1 January 2015
the land was revalued to Rs.520 million and the buildings were revalued at Rs.750 million.
Required:
What amount is to be taken to the revaluation reserve on 1 January 2015?
Answer:
Rs. m Rs. m
Land (520 - 250) 270
Buildings – cost 400
depreciation (400/50 x 4 years) (32)
Net book value 368
Revaluation 750
Transfer to revaluation reserve 382
Total revaluation surplus 652

© Emile Woolf International 34 The Institute of Chartered Accountants of Pakistan


Chapter 1: IAS 16: Property, plant and equipment

Example 07: ROONEY


Question: Rooney has recently finished building a new item of plant for its own use. The item is a
press for use in the manufacture of industrial diamonds. Rooney commenced construction of the
st st
asset on 1 April 2013 and completed it on 1 April 2015. The cost of manufacturing the asset were
Rs. 30,800,000. The cost of the hydraulic system is 30% of the total cost of manufacture.
The press comprises two significant parts, the hydraulic system and the ‘frame’. The hydraulic system
has a three-year life and the ‘frame’ has an eight-year life. Rooney depreciates plant on a straight
line basis.
Rooney uses the IAS 16 revaluation model in accounting for diamond presses and revalue these
assets on an annual basis.
Revaluation surpluses or deficits are apportioned between the hydraulic system and the ‘frame’ on
the basis of their year-end book values before the revaluation.
Required:
Explain the IAS 16 rules on accounting for significant parts of property, plant and equipment and
show the accounting treatment of the diamond press in the financial statements for the financial
years ending:
(i) 31st March 2016 (assume that the press has a fair value of Rs. 21 million)
(ii) 31st March 2017 (assume that the press has a fair value of Rs. 19.6 million).

Answer:
The IAS 16 rule on accounting for significant parts of property, plant and equipment is as under:
Accounting Rule
IAS 16 requires that each part of an item (that has a cost that is significant in relation to the total
cost) is depreciated separately. Therefore, the cost recognised at initial recognition must be allocated
to each part accordingly.
(i) 31st March 2016

Carrying
Cost
Depreciation value
1.4.2015 31.3.2016
Rs.000 Rs.000 Rs.000
Hydraulic system 9,240 (3,080) 6,160
“Frame” 21,560 (2,695) 18,865
30,800 (5,775) 25,025
Revaluation loss (to profit and loss) (4,025)
Fair value. 21,000

The carrying value of the assets should be written down by a factor of 21,000/25025. This
gives a carrying value for the hydraulic system (in Rs.000) of 5,169 and for the ‘frame’
15,831.
The hydraulic plant should be depreciated over two more years and the ‘frame’ over 7 more
years.
Based on above analysis if the press has a fair value of rupees 19.6 million at 31 st March
2017 then the relevant calculation is as under:

© Emile Woolf International 35 The Institute of Chartered Accountants of Pakistan


Financial accounting and reporting I

(ii) 31st March 2017

Carrying Depreciation Carrying


value Charge Value
1.4.2016 31.3.2017
Rs.000 Rs.000 Rs.000
Hydraulic system 5,169 (2,585) 2,584
“Frame” 15,831 (2,262) 13,569
21,000 (4,847) 16,153
Revalued amount 19,600
Total gain 3,447

To statement of profit or loss 3,097


Other comprehensive income 350
Fair value 19,600

The total revaluation gain is 3,447. Of this total amount, 3096 reverses the loss in the
previous year net of the benefit obtained through reduced depreciation and is therefore
reported in profit and loss for the year. The remaining 350 is reported as other
comprehensive income.
Working

Hydraulic Frame Total


Carrying amount 5,169 15,831 21,000
Depreciation (2,585) (2,262) (4,847)
Carrying amount 2,585 13,569 16,153
FV 3,136 16,464 19,600
Value increased 551 2,895 3,447

Loss recognised last year 991 3,034 4,025


Depreciation reduced by (495) (433) (929)
Loss reversal possible thru P/L 495 2,601 3,097

Gain in OCI 56 294 350

(Tutorial note: Deferred tax has been ignored in this question.)

Example 08: EHTISHAM


Question: The following information relates to the financial statements of Ehtisham for the year to
31 March 2015.
The head office of Ehtisham was acquired on 1 April 2012 for Rs. 1 million. Ehtisham intend to occupy
the building for 25 years. On 31 March 2014 it was revalued to Rs. 1.15 million. On 31 March 2015,
a surplus of vacant commercial property in the area had led to a fall in property prices and the fair
value was now only Rs. 0.8 million.
Required:
Explain the correct accounting treatment for the above (with calculations).

© Emile Woolf International 36 The Institute of Chartered Accountants of Pakistan


Chapter 1: IAS 16: Property, plant and equipment

Answer:
IAS 16 permits assets to be carried at cost or revaluation. Where the latter is chosen, the asset must
be stated at its fair value.
The original depreciation was Rs. 40,000 (Rs. 1,000,000/25 years) per annum.
On 31st March 2014 the asset is two years old. Its carrying value before revaluation was therefore
Rs.1million less accumulated depreciation of Rs.80,000 ( 2/25 × Rs. 1 million).

Rs.
Cost/valuation 1,000,000
Accumulated depreciation (80,000)
Net book value 920,000
In order to effect the revaluation, the cost is uplifted to fair value of Rs.1.15m, the accumulated
depreciation is eliminated, and the uplift to the net book value is credited to a revaluation surplus
account.
Debit Credit
Building 150,000
Accumulated depreciation 80,000
Revaluation surplus 230,000

The impact of the journal is as follows:

Before Adjustment After


Cost/valuation 1,000,000 150,000 1,150,000
Accumulated depreciation (80,000) 80,000 nil
Net book value 920,000 1,150,000
The asset is depreciated over its remaining useful economic life of 23 years giving a charge of Rs.
50,000 (Rs. 1,150,000/23 years) per annum in the year to 31st March 2015.
Debit Credit
Depreciation - Statement of profit or loss 50,000
Accumulated depreciation 50,000
This results in a carrying value as at 31st March 2015 of:
Rs.
Cost/valuation 1,150,000
Accumulated depreciation (50,000)
Net book value 1,100,000

Transfer from revaluation surplus to retained earnings


As a result of the revaluation, the annual depreciation has increased from Rs.40,000 to Rs.50,000.
This extra depreciation of Rs.10,000 is transferred from the revaluation reserve to accumulated
profits each year.

Debit Credit
Revaluation surplus 10,000
Accumulated profits 10,000

By the 31st March 2015, the balance remaining on the revaluation reserve will be Rs.220,000.

© Emile Woolf International 37 The Institute of Chartered Accountants of Pakistan


Financial accounting and reporting I

Rs.
Surplus recognised at 31 March 2014 230,000
Transfer to accumulated profits (10,000)
Net book value 220,000

The fall in property values at the year-end. The asset must be revalued downwards to Rs.0.8million,
a write-down of Rs.300,000.
Rs.220,000 of this is charged against the revaluation reserve relating to this asset, and the
remaining Rs.80,000 must be charged against profits.
The reduction of the carrying amount of the asset is achieved by removing the accumulated
depreciation and adjusting the asset account by the balance.

Debit Credit
Revaluation surplus 220,000
Revaluation Loss - Statement of profit or loss (Working - 1) 80,000
Asset at valuation 350,000
Accumulated depreciation 50,000

The impact of the journal is as follows:

Before Adjustment After


Cost/valuation 1,150,000 350,000 800,000
Accumulated depreciation (50,000) 50,000 nil
Net book value 1,100,000 800,000

This balance is depreciated over the remaining useful life of the asset (22 years).
Working – 1

Asset Account 2012-13 2013-14 2014-15


Opening Balance 1,000,000 960,000 1,150,000
Depreciation (40,000) (40,000) (50,000)
960,000 920,000 1,100,000
Revaluation - 230,000 (300,000)
Carrying amount 960,000 1,150,000 800,000
Revaluation Surplus
Opening balance - 230,000
Transfer - (10,000)
Balance - 220,000
Revaluation 230,000 (220,000)
230,000 -
Loss in P/L (80,000)

© Emile Woolf International 38 The Institute of Chartered Accountants of Pakistan


Chapter 1: IAS 16: Property, plant and equipment

Example 09: CARLY


Question: The following is an extract from the financial statements of Carly on 31 December 2014.

Property, plant and Land and Plant and Computer


Total
equipment buildings machinery equipment
Rs. Rs. Rs. Rs.
Cost
On 31 December 2014 1,500,000 340,500 617,800 2,458,300
Accumulated depreciation
On 31 December 2014 600,000 125,900 505,800 1,231,700
Carrying amount
On 31 December 2014 900,000 214,600 112,000 1,226,600

Depreciation is provided at the following rates.


On land and buildings Over 50 years on straight line basis on buildings only
On plant and equipment 25% reducing balance
On computers 33.33% per annum straight line
During 2015 the following transactions took place.
(1) On 31 December the land and buildings were revalued to Rs. 1,750,000. Of this amount, Rs.
650,000 related to the land (which had originally cost Rs. 500,000). The remaining useful life
of the buildings was assessed as 40 years.
(2) A machine which had cost Rs. 80,000 and had accumulated depreciation of Rs. 57,000 at
the start of the year was sold for Rs. 25,000 in the first week of the year.
(3) A new machine was purchased on 31 March 2015. The following costs were incurred:
Rs.
Purchase price, before discount, inclusive of reclaimable sales tax of Rs.3,000 20,000
Trade Discount 1,000
Delivery costs 500
Installation costs 750
Interest on loan taken out to finance the purchase 300
(4) On 1 January it was decided to change the method of providing depreciation on computer
equipment from the existing method to 40% reducing balance.
Required:
Produce the analysis of property, plant and equipment as it would appear in the financial statements
of Carly for the year ended 31 December 2015.
Answer:
Property, plant and Land and Plant and Computer
Total
equipment buildings machinery equipment
Rs. Rs. Rs. Rs.
Cost/valuation
At 1 January 2015 1,500,000 340,500 617,800 2,458,300
Cancelation (620,000) (620,000)
Revaluation 870,000 - - 870,000
Additions (W2) - 17,250 - 17,150
Disposals - (80,000) - (80,000)
------------------------ ------------------------ ------------------------ ------------------------
At 31 December 2015 1,750,000 277,750 617,800 2,645,550
------------------------ ------------------------ ------------------------ ------------------------

© Emile Woolf International 39 The Institute of Chartered Accountants of Pakistan


Financial accounting and reporting I

Accumulated depreciation
At 1 January 2015 600,000 125,900 505,800 1,231,700
Charge for the year (W1) 20,000 51,134 44,800 115,991
Cancelation (620,000) - - (620,000)
Disposals - (57,000) - (57,000)
------------------------ ------------------------ ------------------------ ------------------------
At 31 December 2015 Nil 120,034 550,600 670,691
------------------------ ------------------------ ------------------------ ------------------------
Carrying amount
At 31 December 2014 900,000 214,600 112,000 1,226,600
------------------------ ------------------------ ------------------------ ------------------------
At 31 December 2015 1,750,000 157,716 67,200 1,974,859
------------------------ ------------------------ ------------------------ ------------------------

Workings
(1) Depreciation charges
Buildings = (1,500,000 – 500,000) / 50 years = 20,000.

Plant and machinery: Rs.


New machine (17,250  25% 9/12) 3,234
Existing plant (((340,500 – 80,000) – (125,900 – 57,000))  25%) 47,900
51,134
Computer equipment = 112,000  40% = Rs.44,800
(2) Cost of new machine

Rs.
Purchase price (20,000 – 3,000 – 1,000) 16,000
Delivery costs 500
Installation costs 750
17,250

Example 10: ADJUSTMENTS LIMITED


Question: Adjustments Limited has carried out a review of its non-current assets.
(a) A grinder was purchased on 1 January 2012 for Rs. 100,000. The plant had an estimated useful
life of ten years and a residual value of nil. Depreciation is charged on the straight line basis.
On 1 January 2015, when the asset’s net book value is Rs. 70,000, the directors decide that
it would be more appropriate to depreciate this asset using the sum of digits’ approach. The
remaining useful life is unchanged.
(b) The company purchased a fifty year lease some years ago for Rs. 1,000,000. This was being
depreciated over its life on a straight line basis. On 1 January 2015, when the net book value
is Rs. 480,000 and twenty-four years of the lease are remaining, the asset is revalued to Rs.
1,500,000. This revised value is being incorporated into the accounts.
Required:
Explain the effects of these changes on the depreciation for the year to 31 December 2015.

© Emile Woolf International 40 The Institute of Chartered Accountants of Pakistan


Chapter 1: IAS 16: Property, plant and equipment

Answer:
(a) The grinder was purchased in 2012 and was originally being depreciated on a straight line basis.
It has now been decided to depreciate this on the sum of digits basis.
IAS 16 requires that depreciation methods be reviewed periodically and if there is a significant
change in the expected pattern of economic benefits, the method should be changed. Depreciation
adjustments should be made in current and future periods. This change might be appropriate if, for
instance, usage of the machine is greater in the early years of an asset’s life when it is still new and
consequently it is appropriate to have a higher depreciation charge.
If the change is implemented, the unamortised cost (the net book value) of the asset should be
depreciation over the remaining useful life commencing with the period in which the change is
made.
The depreciation charge for the remaining life of the asset will therefore be as follows.
Year Digits Depreciation
Rs.
2015 7 7/28  Rs.70,000 17,500
2016 6 6/28  Rs.70,000 15,000
2017 5 12,500
2018 4 10,000
2019 3 7,500
2020 2 5,000
2021 1 2,500
—— —————
1/2  7 (7 + 1) 28 Rs. 70,000
—— —————
Disclosure will need to be made in the accounts of the details of the change, including the effect on
the charge in the year.
The reassessment of the depreciation method is not a change in accounting policy and neither
rectification of a fundamental error so the effects of the change will not affect the previously reported
financial statements (opening retained earnings)
(b) Leasehold land
IAS 16 requires that the subsequent charge for depreciation should be based on the revalued
amount. The annual depreciation will therefore be Rs. 62,500, i.e. Rs.1,500,000 divided by the 24
years of remaining life.
There will then be a difference between the revalued depreciation charge and the historical
depreciation charge.
The resulting excess depreciation may be dealt with by a movement in reserves, i.e. by transferring
from the revaluation reserve to retained earnings a figure equal to the depreciation charged on the
revaluation surplus each year.

Example 11: FAM


Question: FAM had the following tangible fixed assets at 31 December 2014.
Cost Depreciation NBV
Rs. 000 Rs. 000 Rs. 000
Land 500 – 500
Buildings 400 80 320
Plant and machinery 1,613 458 1,155
Fixtures and fittings 390 140 250
Assets under construction 91 – 91
——— —— ———
2,994 678 2,316
════ ════ ════

© Emile Woolf International 41 The Institute of Chartered Accountants of Pakistan


Financial accounting and reporting I

In the year ended 31 December 2015 the following transactions occur.


(1) Further costs of Rs. 53,000 are incurred on buildings being constructed by the company. A
building costing Rs. 100,000 is completed during the year.
(2) A deposit of Rs. 20,000 is paid for a new computer system which is undelivered at the year
end.
(3) Additions to plant are Rs. 154,000.
(4) Additions to fixtures, excluding the deposit on the new computer system, are Rs. 40,000.
(5) The following assets are sold.
Depreciation brought
Cost Proceeds
forward
Rs. 000 Rs. 000 Rs. 000
Plant 277 195 86
Fixtures 41 31 2
(6) Land and buildings were revalued at 1 January 2015 to Rs. 1,500,000, of which land is worth
Rs. 900,000. The revaluation was performed by Messrs Jackson & Co, Chartered Surveyors,
on the basis of existing use value on the open market.
(7) The useful economic life of the buildings is unchanged. The buildings were purchased ten
years before the revaluation.
(8) Depreciation is provided on all assets in use at the yearend at the following rates. Buildings
2% per annum straight line
Buildings 2% per annum straight line
Plant 20% per annum straight line
Fixtures 25% per annum reducing balance
Required:
Show the disclosure under IAS 16 in relation to fixed assets in the notes to the published accounts
for the year ended 31 December 2015.

Answer:
Property, plant and equipment is stated at historical cost less depreciation, or at valuation.
Depreciation is provided on all assets, except land, and is calculated to write down the cost or
valuation over the estimated useful life of the asset.
The rates are as follows.
Buildings 2% pa straight line
Plant and machinery 20% pa straight line
Fixtures and fittings 25% pa reducing balance
Fixtures, fittings,
Land and Plant and
Fixed asset movements tools and Total
buildings machinery
equipment
Cost/valuation Rs.000 Rs.000 Rs.000 Rs.000
Cost at 1 January 2015 900 1,613 390 2,903
Cancelation (80)
Revaluation adjustment 680 - - 600
Additions 100 154 40 294
Disposals (277) (41) (318)
Cost at 31 December 2015 1600 1,490 389 1,979
2015 valuation 1,500

© Emile Woolf International 42 The Institute of Chartered Accountants of Pakistan


Chapter 1: IAS 16: Property, plant and equipment

Fixtures, fittings,
Land and Plant and
Fixed asset movements tools and Total
buildings machinery
equipment
Depreciation
At 1 January 2015 80 458 140 678
Revaluation adjustment (80) – – (80)
Provisions for year (W2) 17 298 70 385
Disposals – (195) (31) (226)
At 31 December 2015 17 561 179 757
Net book value
At 31 December 2015 1,583 929 210 2,722
At 31 December 2014 820 1,155 250 2,225
Land and buildings have been revalued during the year by Messrs Jackson & Co on the basis of an
existing use value on the open market.
The corresponding historical cost information is as follows.
Land and buildings
Rs.000
Cost
Brought forward 900
Reclassification 100
———
Carried forward 1,000
———
Depreciation
Brought forward 80
Provided in year 10
———
Carried forward 90
———
Net book value 910
———
Payments on account and assets in the course of construction
Cost at 1 January 2015 91
Additions (W1) 73
Reclassifications (100)
As at 31 December 2015 64
At 31 December 2014 91

WORKINGS Rs. 000


(1) Additions to assets under construction 53
Deposit on computer 20
——
73
——
600
(2) Depreciation on buildings 40 + (100  2%) 17
2% straight line depreciation is equivalent to a 50 year life.
The buildings are ten years old at valuation and therefore
have 40 years remaining.
Depreciation on plant (1,613 + 154 – 277)  20% 298
Depreciation on fixtures (390 + 40 – 41 – 140 + 31)  25% 70

© Emile Woolf International 43 The Institute of Chartered Accountants of Pakistan


Financial accounting and reporting I

Example 12: ORCHID LIMITED


Question: The following information pertains to property, plant and equipment of Orchid Limited
(OL), a listed company:

Cost Subsequent
Date of Original Depreciation
Description Rs. In measurement
purchase useful life method
million model
Buildings 1-Jan-15 600 30 years Straight line Revaluation
Plant 1-Jan-15 475 25 years Straight line Cost
Buildings
The revalued amount of buildings as determined by Shabbir Associates, an independent valuer, on
31 December 2015 and 2017 was Rs. 700 million and Rs. 463 million respectively.
On 30 June 2017 a building having original cost of Rs. 66 million was sold to Baqir Limited for Rs.
85 million. It was last revalued at Rs. 87 million. OL incurred a cost of Rs. 2 million on disposal.
OL transfers the maximum possible amount from revaluation surplus to retained earnings on an
annual basis.
Plant
On 31 December 2016 the recoverable amount of the plant was assessed at Rs. 360 million with
no change in useful life.
During 2017, OL has decided to change the depreciation method for plant from straight line to
reducing balance. The new depreciation rate would be 10%.
Required:
Prepare following notes (along with comparative figures) to be presented in the financial statements
of OL for the year ended 31 December 2017 in accordance with the requirements of relevant IFRSs
and Companies Act, 2017:
(a) Property, plant and equipment
(b) Change in depreciation method

Answer:
Notes to the financial statement
For the year ended 31 December 2017
Property, plant and equipment:

2017 2016
Building Plant Building Plant
-------------------------------- Rs. in million ----------------------------
Gross carrying amount - 700.00 475.00 700.00 475.00
opening
Accumulated dep. & (24.14) (115.00) - (19.00)
impairment (475÷25)
Opening carrying 675.86 360.00 700.00 456.00
amount
Depreciation (22.64) (36) (24.14) (19.00)

© Emile Woolf International 44 The Institute of Chartered Accountants of Pakistan


Chapter 1: IAS 16: Property, plant and equipment

2017 2016
Building Plant Building Plant
-------------------------------- Rs. in million ----------------------------
[21.14 (700-87) ÷ 29] (360 × 10%) (700÷29) (475÷25)
+
[1.5 (87÷ 29 × 6 ÷12)]
Disposal (82.50) - - -
[87-{(87÷ 29)+
(87÷29×6 ÷ 12)}]
Impairment (P&L) (77) (456–
19–360)
Revaluation
- surplus [W-1] (90.12) - - -
- P&L [W-1] (17.60) - - -
Closing carrying amount 463.00 324.00 675.86 360.00
Gross carrying amount - 463.00 475.00 700.00 475.00
closing
Accumulated dep. & - (151) (24.14) (115.00)
impairment
Closing carrying amount 463.00 324.00 675.86 360.00

Building Plant
Measurement base Revaluation model Cost model
Useful life (years)/depreciation rate % 30 10%
Depreciation method Straight line Reducing balance
The last revaluation was performed on 31 December 2017 by Shabbir Associates, an independent
firm of valuers.

2017 2016

Carrying value of building had the cost 480.6 560


model been used instead (600–66)÷30×27 (600÷30×28)

Following disposal has been made during the year.

Cost /
Book Sale
Name of revalued Gain/(loss) Mode of
value price
purchaser amount disposal
------------------------- Rs. in million -------------------------

Building Baqir 87.00 82.50 85.00 0.5 (85–2) – 82.5) Tender


Limited
Change in estimate
In lieu of significant change in the expected pattern or consumption of the future economic benefits
embodied in the plant, company decided to change the depreciation method of plant from straight
line to reducing balance method. The new depreciation rate would be 10%.
Had the depreciation method been not changed, profit of 2017 would have been higher by Rs. 20.35
million. (360×10%-360÷23)

© Emile Woolf International 45 The Institute of Chartered Accountants of Pakistan


Financial accounting and reporting I

W-1: Revaluation of building as on 31 December 2017 Rs. in


million

Carrying value of building (675.86-22.64-82.5) 570.72

Revalued amount 463.00

107.72

Available surplus [120 (700-580) - 23.2 (W-2)] × (27÷29)


OR [120-4.14 (120÷29) - 3.74 {4.14 - (23.2÷29×0.5)}- 22 (23.2÷29×27.5)] (90.12)

Expense (P&L) 17.60

W-2:

Revalued amount of building sold 87.0

Carrying value (66÷30×29) 63.8

23.2

Example 13: ABID LIMITED


Question: Abid Limited (AL) uses the revaluation model for subsequent measurement of its
property, plant and equipment and has a policy of revaluing its assets on an annual basis using the
net replacement value method.
The following information pertains to AL’s buildings:
i. Four buildings were acquired in same vicinity on 1 January 2012 at a cost of Rs. 300 million.
The useful life of the buildings on the date of acquisition was 20 years.
ii. AL depreciates buildings on the straight line basis over their useful life.
iii. The results of revaluations carried out during the last three years by Premier Valuation Service,
an independent firm of valuers, are as follows:

Revaluation date Fair value Rs. in million

1 January 2013 323

1 January 2014 252

1 January 2015 272

On 30 June 2015, one of the buildings was sold for Rs. 80 million.
Required:
Prepare a note on “Property, plant and equipment” (including comparative figures) for inclusion in
AL’s financial statements for the year ended 31 December 2015 in accordance with International
Financial Reporting Standards. (Ignore taxation)

© Emile Woolf International 46 The Institute of Chartered Accountants of Pakistan


Chapter 1: IAS 16: Property, plant and equipment

Answer:

2015 2014
Property, plant and equipment
--------- Rs. in million ---------
Gross carrying amount 252 323
Accumulated depreciation and (14) (17)
impairment losses (323÷19)
Net carrying amount 238 306
Additions - -
Revaluation (expense)/Income (P/L) 17[272- {300-(300÷20x3)}] (18)(306-252-36)
Revaluation surplus increase/(decrease) 17 (272-238-17) (36)[{323-(300-
(OCI) 15)}-(38÷19)]
Depreciation (14) (14)
[(204÷17)+(68÷17×6÷12)] (252÷18)
Disposal (66) [68 - (68÷17×6÷12)] -
192 238

Gross carrying amount 204 252


Accumulated depreciation and (12) (14)
impairment losses
Net carrying amount 192 238

Useful life 20 years 20 years

The last revaluation was performed on 1 January 2015 by M/s Premier Valuation Services, an
independent firm of valuers. Revaluations are performed annually.

2015 2014

--------- Rs. in million ---------

Carrying value had the cost model been 180 255


used instead [225–(225÷20×4)] [300–(300÷20×3)]

4.1- Details of property, plant and equipment disposed of during the year

Cost /
Accumulated Carrying Sale
Revalued
depreciation amount proceeds Mode of Particulars of
amount
disposal buyers
---------------------- Rs. in million ----------------------

Building 68 2 66 80 Not mentioned Not mentioned

© Emile Woolf International 47 The Institute of Chartered Accountants of Pakistan


Financial accounting and reporting I

Example 14: SHAHWEZ LIMITED


Question: Shahwez Limited (SL) revalued its property on 1 April 20X1 to Rs.20 million (Rs.8 million
for the land). The property originally cost Rs.10 million (Rs.2 million for the land) 10 years ago. The
original useful life of 40 years is unchanged. SL’s policy is to make a transfer to realised profits in
respect of excess depreciation.
Required:
How will the property be accounted for in the year ended 31 March 20X2?

Answer:
Statement of comprehensive income extract for the year ended 31 March 20X2
Rs. In 000
Depreciation expense 400
Other comprehensive income:
Revaluation gain 12,000
Statement of financial position extract as at 31 March 20X2
Rs. In 000
Non-current assets Property 19,600
(20,000 – 400)
Equity Revaluation reserve 11,800
(12,000 – 200)

Statement of changes in equity extracts


Revaluation reserve Retained earnings
Revaluation gain 12,000
Reserves transfer (200) 200
Workings:
Gain on revaluation: Rs. In 000
Carrying value at revaluation date
(10,000 – ((10,000 – 2,000)*40 years/10 years)) 8,000
Valuation 20,000
Gain on revaluation 12,000
Double entry:
Dr Property 10,000
(20,000 – 10,000)
Dr Accumulated depreciation 2,000
((10,000 – 2,000)/40 years x 10 years)
Cr Revaluation reserve 12,000
Depreciation charge for year to 31 March 20X2:
Dr depreciation expense 400
((20,000 – 8,000)/30 years)
Cr Accumulated depreciation 400

© Emile Woolf International 48 The Institute of Chartered Accountants of Pakistan


Chapter 1: IAS 16: Property, plant and equipment

Reserves transfer:
Historical cost depreciation charge 200
((10,000 – 2,000)/40 years)
Revaluation depreciation charge 400
Excess depreciation to be transferred 200
Dr Revaluation reserve 200
Cr Retained earnings 200

Example 15: HAMZA LIMITED


Question: Hamza Limited (HL) acquired a building on 1 April 20X1 for Rs. 100,000 at which point
it was considered to have a useful life of 40 years. At the year-end 31 March 20X6, HL decided to
revalue the building to its current value of Rs. 98,000.
Required:
How will the building be accounted for in the year ended 31 March 20X6?
Answer:
Rupees
Statement of comprehensive income extract 31 March 20X6
Depreciation charge 2,500
Other comprehensive income:
Revaluation gain 10,500
Statement of financial position extract 31 March 20X6
Building at valuation 98,000

Statement of changes in equity extract


Revaluation gain
Revaluation reserve 10,500
Notes:
Revaluation takes place at year end, therefore a full year of depreciation must first be charged.
(W1) Depreciation year ended 31 March 20X6
100,0000 / 40 years = Rs.2,500 per annum

(W2) Revaluation as at 31 March 20X6


Carrying value of non-current asset at revaluation date
(100,000 – (100,000/40 years x 5 years)) 87,500
Valuation of non-current asset 98,000
Gain on revaluation 10,500
Double entry: Debit Credit
Accumulated depreciation 12,500
Building 2,500
Revaluation reserve 10,500

© Emile Woolf International 49 The Institute of Chartered Accountants of Pakistan


Financial accounting and reporting I

Example 16: Abbas Limited


Question: Abbas Limited (AL) is engaged in the business of manufacturing near the Karachi-
Hyderabad Motorway. Its Property, Plant and Equipment comprises of land and buildings, plant and
machinery, and equipment and fittings.

Details for the period up to 30 June 2018 are as follows:


1. The balances of the Property, Plant and Equipment as at 30 June 2018 are given
below:
Assets Gross Carrying Amount Accumulated Depreciation
(Rs. Million) (Rs. Million)
Land 12 N/A
Buildings 125 38
Plant and Machinery 500 300
Equipment 100 36
2. The relevant information pertaining these assets is given below:
Assets Depreciation Method Subsequent Measurement
Model
Land N/A Fair Value
Buildings Straight-line Cost
Plant and Machinery Units of Production Cost
Equipment Written down value Cost
3. Abbas Limited uses proportionate policy to depreciate its Property, Plant and
Equipment.
4. All of the plant and machinery pertains to factory use whereas all the equipment
pertains to office use. However, floor areas occupied by factory and office are in the
ratio 60:40 respectively.
5. The equipment was purchased on 1 July 2016. No disposals and acquisitions took
place in the period up to 30 June 2018.
6. Until 30 June 2018, 12,000 units had been produced by Abbas Limited in its factory.
The plant and machinery does not have any residual value. No additions or disposals
of plant and machinery took place till this date.
7. The buildings were acquired on 1 July 2014 with a residual value of Rs. 11 million. No
additions and disposals took place till 30 June 2018.
8. The land had actually cost Rs. 15 million on the date of its acquisition.
9. It is assumed that value of land and buildings is spread evenly across the area
occupied.
The following information pertains to the year ended 30 June 2019:
1. On 1 July 2018, land was revalued to Rs. 20 million on 1 July 2018. The value was
determined by an independent firm M/s Ashfaq& Co. Chartered Accountants.
2. This year, 5,000 units were product in the factory of AL.
3. On January 1, 2019, AL disposed 25% of its area comprising of land and buildings at a
price of Rs. 90 million. The portion of land was sold at its fair value as determined on
1 July 2018. The legal costs of drafting transfer agreements were Rs. 0.1 million. It is
assumed that this disposal will not affect the proportion of areas occupied by factory
and office.
4. Further equipment costing Rs. 60 million was acquired on 1 November 2018.

© Emile Woolf International 50 The Institute of Chartered Accountants of Pakistan


Chapter 1: IAS 16: Property, plant and equipment

5. In the meeting of its board of directors, it was decided to open a new factory premises
near Lahore-Islamabad motorway. An expenditure of Rs. 20 million was spent of the
construction of the factory on 1 December 2018, financed by a loan obtained from the
bank at the rate of 12% per annum. The construction had not been completed at the
end of the year.
6. Moreover, the directors also made a contract with M/s UniPower& Co. to purchase
plant and machinery worth Rs. 35 million once the construction of factory building is
completed.
Required:
(a) Prepare journal entries to record the revaluation of land and disposal of land and
buildings.
(b) Prepare the disclosure under IAS 16 in relation to Property, Plant and Equipment in the
notes to the published accounts for the year ended 30 June 2019.
Answer:
a) The journal entries to record the revaluation of land and disposal of land and buildings
are as under:
Date Particulars Dr. Cr.
Rs. 000 Rs.000
2018
July 1, Land
Revaluation Surplus (OCI) 8,000
Reversal of Revaluation Loss (PL) 5,000
3,000
2019
Jan 1, Cash (90m - 0.1m) 89,900
Accumulated Depreciation – Buildings (W1) 10,687.5
Buildings 31,250
Land 5,000
Gain on Disposal (Other Income) 64,337.5
Jan 1, Revaluation Surplus 1,250
Retained Earnings 1,250

(W1) Rs. 000


At the start of year (38 × 25%) 9,500
During the year [(125 – 11) × 25% ÷ 12 ÷ 2] 1,187.5
Accumulated Depreciation of Buildings Disposed 10,687.5

b) The disclosure under IAS 16 in relation to Property, Plant and Equipment in the notes
to the published accounts for the year ended 30 June 2019 is as under:
Abbas Limited uses the following subsequent measurement bases to value its
Property, Plant and Equipment, and methods to calculate its depreciation.

Depreciation Useful Life/Residual


Assets Subsequent Measurement
Method Value/Rate
Land N/A N/A Fair Value
Useful life of 12 years
Cost less Accumulated
Buildings Straight-line with a residual value of
Depreciation
8.8% of cost. (W1)
Plant and Units of Rs. 25,000 per unit Cost less Accumulated
Machinery production (W2) Depreciation
Written Cost less Accumulated
Equipment Rate of 20% (W3)
down value Depreciation

© Emile Woolf International 51 The Institute of Chartered Accountants of Pakistan


Financial accounting and reporting I

Furthermore, Abbas Limited uses proportionate policy to depreciate its assets.


Schedule of Movement in Property, Plant and Equipment
For the year ended 30 June 2019 (in Rs. 000)

Plant and
Land Buildings Equipment Total
Machinery

Gross Carrying Amount:


At 1 July 2018
Acquisitions 12,000 125,000 500,000 100,000 737,000
Revaluation 60,000 60,000
Disposals 8,000 - 8,000
(5,000) (31,250) - (36,250)

At 30 June 2019 15,000 93,750 500,000 160,000 768,750

Accumulated Depreciation
At 1 July 2018
Depreciation - 38,000 300,000 36,000
374,000
Revaluation - 8,312.5(W4) 125,000 20,800(W5)
154,112.5
Disposals - - -
(10,687.5)

(10,687.5)

At 30 June 2019 - 35,625 425,000 56,800 517,425

Carrying Amount at 1 July


12,000 87,000 200,000 64,000 363,000
2018

Carrying Amount at 30 June


15,000 58,125 75,000 103,200 251,325
2019

An amount of expenditure of Rs. 20 million was incurred on the construction of a factory near Lahore-
Islamabad Motorway on 1 December 2018. This amount was capitalized as capital work-in-progress.
A further borrowing costs of Rs. 1.4 million (W6) were capitalized in respect of interest on loan
obtained from the bank to finance this project.
A contract was made with M/s UniPower & Co. to purchase plant and machinery worth Rs. 35 million
once the construction of factory building is completed.
The following depreciations are either made part of inventory or expensed out in statement of profit
or loss:

Assets Inventory Expense Total


Rs.000 Rs.000 Rs.000

Buildings 4,987.5 3,325 8,312.5

Plant and Machinery 125,000 - 125,000

Equipment - 20,800 20,800

Total 129,987.5 24,125 154,112.5

© Emile Woolf International 52 The Institute of Chartered Accountants of Pakistan


Chapter 1: IAS 16: Property, plant and equipment

Revaluation Disclosures:
i. The revaluation of land took place on 1 July 2018. The value was determined by an
independent firm M/s Ashfaq& Co. Chartered Accountants.
ii. The carrying amount of land had the revaluation not taken place:

Rs. million

At 1 July 2018 15

Disposals during the year (3.75)

At 30 June 2019 11.25

iii. Revaluation Surplus

Rs. million

At 1 July 2018 -

Revaluation of land 5

Transfer to retained earnings (1.25)

At 30 June 2019 3.75

A further reversal of revaluation loss of Rs. 3 million was reversed during the year.
125−11
(W1) × 4 = 12 𝑦𝑒𝑎𝑟𝑠
38
300,000,000
(W2) = 𝑅𝑠. 25,000 𝑝𝑒𝑟 𝑢𝑛𝑖𝑡
12,000

2 100−36
(W3) 𝑟 =1− √
100

𝑟 = 20%

(W4) Depreciation during the year Rs. 000

Disposals 1,187.5
During the year [(125 – 11) × 75% ÷ 12] 7,125

Depreciation Expense 8,312.5

(W5) Depreciation during the year Rs. 000

Acquisitions (60 × 0.2 × 8/12) 8,000


Remaining Assets (64 × 0.2) 12,800

Depreciation Expense 20,800


7
(W6) 20 𝑚𝑖𝑙𝑙𝑖𝑜𝑛 × 12% × = 1.4 𝑚𝑖𝑙𝑙𝑖𝑜𝑛
12

© Emile Woolf International 53 The Institute of Chartered Accountants of Pakistan


Financial accounting and reporting I

Example 15: GAMES LIMITED


Question: Games Limited (GL) commenced a business of preparing and burning video game CDs on
1 July 2015.
The following information pertains to the year ended 31 March 2016:
1) GL purchased 30 computers on the date of commencement of business at a cost of Rs. 20,000
each, purely for the task of burning CDs. The management of GL estimates that since the
computers are subject to obsolescence, more of its benefit can derived in its early life. The total
useful life at the date of acquisition was estimated to be 4 years and residual value was
estimated to be Rs. 4,802 for each computer.GL decided to adopt historical cost model for
subsequently measurement of computers.
2) GL purchased an office building at the date of start of business worth Rs. 3 million. GL decided
to adopt fair value model due to fluctuations in property prices. 80% of the building is occupied
by computer labs, whereas 20% is used by administrative and selling departments. The useful
life is estimated to be 10 years at the date of acquisition with no residual value, and the
economic benefits are expected to be derived evenly over its useful life. At the end of the year,
the fair value of office buildings was assessed to be Rs. 3,237,500.
3) GL also purchased fittings for its administrative and selling departments, costing Rs. 120,000
on 1 July 2015. It is to be depreciated over 10 years using the straight-line method, with no
residual value.
4) GL made a contractual commitment with Al-Karim Computers to purchase 6 computers of Rs.
20,000 each to be delivered at GL’s premises on 1 May 2017.
The following information pertains to the year ended 31 March 2017:
1) The computers were delivered at the GL’s premises by Al-Karim Computers at the said date. It
was decided to use the same method and same rate to depreciate these computers. However,
no further space was utilised by the computer labs.
2) At the end of the year, the fair value of office building was assessed to be Rs. 2 million. At the
year-end GL mortgaged entire building with JS Bank to obtain a loan worth Rs. 1.75 million for
prospective investments in other divisions.
3) Fittings with a cost of Rs. 30,000 were disposed of for Rs. 22,000 on 1 January 2017. The
Suzuki Driver was paid Rs. 1,000 to transfer the fittings to customer’s premises.
The fair values of the office building were determined by an independent firm M/s Hafeez Yasir
Chartered Accountants& Co. Moreover, GL uses proportionate policy to depreciate its assets.
Required:
(a) Prepare the disposal account to record the sale of fittings on 1 January 2017.
(b) Prepare the disclosure under IAS 16 in relation to Property, Plant and Equipment in the
notes to the published accounts for the year ended 31 March 2017 (comparatives are
required).
Answer:
(a) The disposal account to record the sale of fittings on 1 January 2017 is as under:

Disposal Account - Fittings


2017 Rs. 2017 Rs.
Jan 1, Fittings – Cost Jan 1, Accumulated
30,000
Jan 1, Cash (Disposal Depreciation – 4,500
1,000 Fittings(W1)
Costs)
Jan 1, Cash 22,000
Mar Statement of Profit or
4,500
31, Loss (Loss on Disposal)
31,000 31,000
9
(W1) 30,000 × × 2 × 10% = 4,500
12

© Emile Woolf International 54 The Institute of Chartered Accountants of Pakistan


Chapter 1: IAS 16: Property, plant and equipment

(b) The disclosures under IAS 16 in relation to Property, Plant and Equipment in the notes to
the published accounts for the year ended 31 March 2017 (comparatives are required)
are as under:
Property, plant and equipment:
Games Limited (GL) uses the following subsequent measurement bases to value its
Property, Plant and Equipment, and methods to calculate its depreciation.

Depreciation
Assets Rate Subsequent Measurement
Method
Buildings Straight-line 10% Fair Value
Written-down Cost less Accumulated
Computers 30% (W1)
value Depreciation
Cost less Accumulated
Fittings Straight-line 10%
Depreciation
Furthermore, Games Limited uses proportionate policy to depreciate its assets.
Games Limited
Schedule of Movement in Property, Plant and Equipment
For the year ended 30 March 2017 (in Rs.)

2017 2016
Buildings Computers Fittings Buildings Computers Fittings
Gross Carrying Amount:
At start of year 3,237,500 600,000 120,000 - - -
Acquisitions - 120,000 - 3,000,000 600,000 120,000
Revaluations (1,237,500) - - 237,500 - -
Disposals - - (30,000) - - -
At end of year 2,000,000 720,000 90,000 3,237,500 600,000 120,000

2017 2016
Buildings Computers Fittings Buildings Computers Fittings
Accumulated Depreciation:
At start of year - 135,000 9,000 - - -
Depreciation
charge for the
year(W2) 350,000 172,500 11,250 225,000 135,000 9,000
Revaluation (350,000) - - (225,000) - -
Disposals - - (4,500) - - -
At end of year - 307,500 15,750 - 135,000 9,000
Carrying Amount
at year start 3,237,500 465,000 111,000 - - -
Carrying Amount
2,000,000 412,500 74,250 3,237,500 465,000 111,000
at year end
The entire office building was mortgaged with JS Bank on 31 March 2017, to obtain a
loan worth Rs. 1.75 million for prospective investments in other divisions.
No contractual commitments were made during the year ended 31 March 2017 to
purchase Property, Plant and Equipment.

© Emile Woolf International 55 The Institute of Chartered Accountants of Pakistan


Financial accounting and reporting I

A contract was made with Al-Karim Computers during the year ended 31 March 2016 to
purchase 6 computers of Rs. 20,000 each to be delivered on 1 May 2017.
The following depreciations are either made part of inventory or expensed out in
statement of profit or loss:

Assets 2017 2016


Inventory Expense Total Inventory Expense Total
Rs. Rs. Rs. Rs. Rs. Rs.
Buildings 280,000 70,000 350,000 180,000 45,000 225,000
Computers 172,500 - 172,500 135,000 - 135,000
Fittings - 11,250 11,250 - 9,000 9,000
Total 452,500 81,250 533,750 315,000 54,000 369,000

Revaluation Disclosures:
The revaluations of office buildings took place on 31 March 2017 and 31 March 2016
respectively. The fair values of the office building were determined by an independent
firm M/s Hafeez Yasir Chartered Accountants & Co.
The carrying amount of buildings had the revaluation not taken place:

2017 2016
Rs. Rs.

Cost:
At start of year 3,000,000 -
Acquisitions - 3,000,000
Disposals - -

At end of year 3,000,000 3,000,000

Accumulated Depreciation:
At start of year 225,000 -
Depreciation charge for the year 300,000 225,000
Disposals - -

At end of year 525,000 225,000

Carrying Amount at year start 2,775,000 -

Carrying Amount at year end 2,475,000 2,775,000

Revaluation Surplus

At start of year 462,500 -


Revaluation of Buildings (412,500) 462,500
Transfer to Retained Earnings (350,000 - 300,000) (50,000) -

At end of year - 462,500

Furthermore, a revaluation loss of Rs. 475,000 was recognized at 31 March 2017.

© Emile Woolf International 56 The Institute of Chartered Accountants of Pakistan


Chapter 1: IAS 16: Property, plant and equipment

4 4,802
(W1) 𝑟 = 1 − √
20,000

𝑟 = 30%
(W2) 2017 2016
3,000,000 ÷ 10
Buildings 3,237,500 ÷ 9.25 = 350,000 9
× = 225,000
12
Rs.
Acquisitions (120,000 × 30% ×
33,000 600,000 × 30%
11/12)
Computers 9
Remaining [(600,000 - 135,000) × = 135,000
139,500 12
× 30%]
Total 172,500
Rs.
Disposals (30,000×10% × 9/12) 2,250 9
Fittings 120,000 ÷ 10 ×
Remaining (90,000 × 10%) 9,000 12
= 9,000
Total 11,250

Example 18: INDIGO LIMITED


Question: Indigo Limited (IL) purchased a building on 1 April 2015 for Rs.10 million at which point
it was considered to have a useful life of 40 years. At the year ended 31 March 2020, IL decided to
revalue the building to its market value of Rs.9.8 million.
Required:
How the above building shall be accounted for in the books of IL for the year ended 31 March 2020?

Answer:
Statement of comprehensive income extract for the year ended 31 March 2020
Profit or loss account:
Depreciation charge = Rs.0.25 million (W1)
Other comprehensive income:
Revaluation gain = Rs.1.05 million (W2)
Statement of financial position extract as at 31 March 2020
Building at valuation = Rs.9.8 million
Statement of changes in equity extract for the year ended 31 March 2020
Revaluation reserve
Revaluation gain = Rs.1.05 million (W2)
Workings:
Revaluation takes place at year end, therefore a full year of depreciation must first be charged.
(W1) Depreciation for the year ended 31 March 2020
Rs.10 million / 40 years = Rs.0.25 million
(W2) Revaluation
Carrying value of building at revaluation date (10 m – (10 m/40 years x 5 years)) = Rs.8.75 m
Revaluation of building = Rs.9.8 m
Gain on revaluation = Rs.1.05 m

© Emile Woolf International 57 The Institute of Chartered Accountants of Pakistan


Financial accounting and reporting I

Example 19: Glacier Limited


Question: At 1 April 2019, Glacier Limited (GL) carried its office at its original cost of Rs.20 million
and depreciation of Rs.4 million (based on straight line method to its original life of 50 years). GL
decided to revalue the office on 1 October 2019 to its current value of Rs.22 million. The useful life
remaining was reassessed at the time of valuation and is considered to be 40 years at this date. It
is the company’s policy to charge depreciation proportionally.
Required:
How the office will be accounted for in the financial statements of GL for the year ended 31 March
2020?

Answer:
Statement of comprehensive income extract for the year ended 31 March 2020
Depreciation charge (200,000 (W1) + 275,000 (W3) = Rs. 475,000
Other comprehensive income:
Revaluation gain (W2) = Rs.6.2 million
Statement of financial position extract as at 31 March 2020
Revaluation (office) = Rs. 22 million
Subsequent depreciation (W3) = (Rs. 275,000)
Carrying value = Rs. 21,725,000
Statement of changes in equity extract for the year 31 March 2020
Revaluation reserve (gain) = Rs. 6,200,000 (W2)
Workings:
Revaluation takes place part way through the year and therefore depreciation must first be charged
for the period 1 April 2019 – 30 September 2019, then the revaluation occurs so the depreciation
needs to be charged for the period 1 October 2019 – 31 March 2020 on current market value.
(W1) Depreciation
1 April – 30 September 2019 = Rs.20 million x 6/12 / 50 years = Rs. 200,000
(W2) Revaluation
The carrying value of the asset at 1 October 2019 can now be found and revalued.
Carrying value of office at revaluation date (20,000,000 – (4,000,000 + 200,000)) = Rs. 15,800,000
Revaluation of office = Rs.22 million
Gain on revaluation = Rs. 6,200,000
(W3) Depreciation
1 October 2019 – 31 March 2020 => Rs. 22,000,000 x 6/12 / 40 years = Rs. 275,000

© Emile Woolf International 58 The Institute of Chartered Accountants of Pakistan


Chapter 1: IAS 16: Property, plant and equipment

6 OBJECTIVE BASED QUESTIONS


01. An entity purchased a property 15 years ago at a cost of Rs. 100,000 and have been depreciating
it at a rate of 2% per annum, on the straight-line basis. The entity has had the property
professionally revalued at Rs. 500,000.
What is the revaluation surplus that will be recorded in the financial statements in respect of this
property?

(a) Rs. 400,000

(b) Rs. 500,000

(c) Rs. 530,000

(d) Rs. 430,000

02. An entity owns two buildings, A and B, which are currently recorded in the books at carrying
amounts of Rs. 170,000 and Rs. 330,000 respectively. Both buildings have recently been valued
as follows:
Building A Rs. 400,000
Building B Rs. 250,000
The entity currently has a balance on the revaluation surplus of Rs. 50,000 which arose when
building A was revalued several years ago. Building B has not previously been revalued.
What double entry will need to be made to record the revaluations of buildings A and B?

(a) Dr Non-current assets Rs. 150,000


Dr Statement of profit or loss Rs. 80,000
Cr Other comprehensive income (revaluation surplus) Rs. 230,000
(b) Dr Non-current assets Rs. 150,000
Dr Statement of profit or loss Rs. 30,000
Cr Other comprehensive income (revaluation surplus) Rs. 180,000
(c) Dr Non-current assets Rs. 150,000
Cr Other comprehensive income (revaluation surplus) Rs. 150,000
(d) Dr Non-current assets Rs. 150,000
Dr Statement of profit or loss Rs. 50,000
Cr Other comprehensive income (revaluation surplus) Rs. 200,000

03. An entity purchased property for Rs. 6 million on 1 July 2013. The land element of the purchase
was Rs. 1 million. The expected life of the building was 50 years and its residual value nil. On 30
June 2015 the property was revalued to Rs. 7 million, of which the land element was Rs. 1.24
million and the buildings Rs. 5.76 million. On 30 June 2017, the property was sold for Rs. 6.8
million.
What is the gain on disposal of the property that would be reported in the statement of profit or
loss for the year to 30 June 2017?

(a) Gain Rs. 40,000

(b) Loss Rs. 200,000

(c) Gain Rs. 1,000,000

(d) Gain Rs. 1,240,000

© Emile Woolf International 59 The Institute of Chartered Accountants of Pakistan


Financial accounting and reporting I

04. Which of the following statements are correct?


1. If the revaluation model is used for property, plant and equipment, revaluations must
subsequently be made with sufficient regularity to ensure that the carrying amount does not
differ materially from the fair value at each reporting date.
2. When an item of property, plant and equipment is revalued, there is no requirement that the
entire class of assets to which the item belongs must be revalued.

(a) Only statement 1 is correct

(b) Only statement 2 is correct

(c) Both statements are correct

(d) None of the statement is correct

05. The following trial balance extract relates to a property which is owned by Maira Limited as at 1
April 2014.
Dr Cr
Rs. 000 Rs. 000
Property at cost (20 year original life) 12,000
Accumulated depreciation as at 1 April 2014 3,600
On 1 October 2014, following a sustained increase in property prices, Maira Limited revalued its
property to Rs. 10.8 million.
What will be the depreciation charge in Maira Limited’s statement of comprehensive income for
the year ended 31 March 2015?

(a) Rs. 540,000

(b) Rs. 570,000

(c) Rs. 700,000

(d) Rs. 800,000

06. A company purchased a building on 1 April 2007 for Rs. 10,000,000. The asset had a useful
economic life at that date of 40 years. On 1 April 2009 the company revalued the building to its
current fair value of Rs. 12,000,000.
What is the double entry to record the revaluation?

(a) Dr. Building 1,500,000


Dr. Accumulated depreciation 500,000
Cr. Other comprehensive income 2,000,000
(b) Dr. Building 2,000,000
Dr. Accumulated depreciation 500,000
Cr. Profit or loss 2,500,000
(c) Dr. Building 2,000,000
Dr. Accumulated depreciation 500,000
Cr. Other comprehensive income 2,500,000
(d) Dr. Building 1,500,000
Dr. Accumulated depreciation 500,000
Cr. Profit or loss 2,000,000

© Emile Woolf International 60 The Institute of Chartered Accountants of Pakistan


Chapter 1: IAS 16: Property, plant and equipment

07. The carrying value of property at the end of the year amounted to Rs. 108 million. On this date the
property was revalued and was deemed to have a fair value of Rs. 95 million. The balance on the
revaluation reserve relating to the original gain of the property was Rs. 10 million.
What is the double entry to record the revaluation?

(a) Dr. Profit or loss 3 million


Dr. Other comprehensive income 10 million
Cr. Property 13 million
(b) Dr. Profit or loss 10 million
Dr. Other comprehensive income 3 million
Cr. Property 13 million
(c) Dr. Profit or loss 13 million
Dr. Other comprehensive income 3 million
Cr. Property 16 million
(d) Dr. Profit or loss 13 million
Cr. Property 13 million

08. A company revalued its property on 1 April 2009 to Rs. 20m (Rs. 8m for the land). The property
originally cost Rs. 10m (Rs. 2m for the land) 10 years ago. The original useful economic life of 40
years is unchanged. The company’s policy is to make a transfer to realized profits in respect of
excess depreciation.
At which amount the property be presented at as at 31 March 2010?

(a) Rs. 20 million

(b) Rs. 19.6 million

(c) Rs. 12 million

(d) Rs. 11.6 million

09. A company revalued its property on 1 April 2009 to Rs. 20m (Rs. 8m for the land). The property
originally cost Rs. 10m (Rs. 2m for the land) 10 years ago. The original useful economic life of 40
years is unchanged. The company’s policy is to make a transfer to realized profits in respect of
excess depreciation.
What is amount of balance in revaluation surplus account as at 31 March 2010?

(a) Rs. 12 million

(b) Rs. 10 million

(c) Rs. 9.8 million

(d) Rs. 11.8 million

10. Which of the following is an optional disclosure requirement of IAS 16?

(a) Measurement bases for determining gross carrying amount

(b) Depreciation method

(c) Useful lives or depreciation rates

(d) The carrying amount of temporarily idle PPE

© Emile Woolf International 61 The Institute of Chartered Accountants of Pakistan


Financial accounting and reporting I

11. Following information is available for equipment account of a business on 1st January 2018:
Opening balance of equipment, a/c (Revalued amount) Rs. 7,500,000
Surplus on revaluation of equipment a/c Rs. 2,000,000
At start of year company sold equipment for Rs. 90,000,000.
Company has a policy of charging 20% depreciation on straight line basis.
What will be treatment of revaluation surplus at disposal of asset?

(a) Dr Surplus on revaluation Rs. 2,000,000


Cr Retained earnings Rs. 2,000,000
(b) Dr Retained earnings Rs. 2,000,000,
Cr Surplus on revaluation Rs. 2,000,000
(c) Dr Surplus on revaluation Rs. 3,500,000
Cr Retained earnings Rs. 3,500,000
(d) Dr Surplus on revaluation Rs. 2,0000,000
Cr Equipment account Rs. 2,000,000

12. A non–current asset costing Rs. 216,000 and carrying value Rs. 145,000 is revalued to Rs.
291,000.
How should revaluation be recorded?

(a) Dr Asset a/c Rs. 75,000,


Cr Surplus on revaluation Rs. 75,000
(b) Dr Asset a/c Rs. 75,000,
Dr Accumulated Depreciation Rs. 71,000,
Cr Surplus on revaluation Rs. 146,000
(c) Dr Surplus on revaluation Rs. 146,000,
Cr Asset a/c Rs. 75,000,
Cr Accumulated Depreciation Rs. 71,000
(d) Dr Accumulated depreciation Rs. 146,000,
Cr Surplus on revaluation Rs. 146,000

13. When items of property, plant and equipment are stated at revalued amounts the following must
be disclosed:
(i) the effective date of the revaluation
(ii) whether an independent valuer was involved
(iii) for each revalued class of property, plant and equipment, the carrying amount that would
have been recognised had the assets been carried under the cost model;
(iv) the revaluation surplus, indicating the change for the period and any restrictions on the
distribution of the balance to shareholders.

(a) (i), (ii) and (iv) only

(b) (i), (ii), and (iii) only

(c) (ii), (iii) and (iv) only

(d) (i) to (iv) all

© Emile Woolf International 62 The Institute of Chartered Accountants of Pakistan


Chapter 1: IAS 16: Property, plant and equipment

14. IAS 16 encourages disclosure of the following information as users of financial statements might
find it to be useful.
(i) the carrying amount of temporarily idle property, plant and equipment
(ii) the gross carrying amount of any fully depreciated property, plant and equipment that is still
in use
(iii) the carrying amount of property, plant and equipment retired from active use and held for
disposal
(iv) when the cost model is used, the fair value of property, plant and equipment when this is
materially different from the carrying amount

(a) (i), (ii) and (iii) only

(b) (i), (ii) and (iv) only

(c) (i), (iii) and (iv) only

(d) (i) to (iv) all

15. Which of the following statements is correct?

(a) An entity may present PPE at gross carrying amount or net carrying amount under IAS 16

(b) Either useful lives or depreciation rates are to be disclosed, both are not required.

(c) Under revaluation model, PPE are revalued at end of each year

(d) If an entity chooses revaluation model, it must apply revaluation model to all of its PPE.

16. Waqas Limited purchased a machine for Rs. 30,000 on 1 January 2015 and assigned it a useful
life of 12 years. On 31 March 2017 it was revalued to Rs. 32,000 with no change in useful life.
What will be depreciation charge in relation to this machine in the financial statements for the year
ending 31 December 2017?

Rs. ___________

17. A business purchased building costing Rs. 7,500,000 on 1 January 2018.


The policy of business is to charge straight line depreciation over its useful life of 20 years.
On 31 December 2020, building was revalued to Rs. 7,650,000.
What is the amount of incremental depreciation to be transferred to retained earnings at year
ending 31 December 2021?

Rs. ___________

18. A business purchased an asset on 1 January 2016 costing Rs. 5,000,000 having a useful life of
10 years with nil residual value. On 1 January 2018 balance of accumulated depreciation was Rs.
1,000,000. Asset is revalued to Rs. 4,500,000 on 1 January 2018 (start of the year).
Business has a policy to charge straight line depreciation.
What is the depreciation charge for the year ended 31 December 2018?

Rs. ___________

© Emile Woolf International 63 The Institute of Chartered Accountants of Pakistan


Financial accounting and reporting I

19. A business purchased an asset on 1 January 2016 costing Rs. 5,000,000 having a useful life of
10 years with nil residual value. On 1 January 2018 balance of accumulated depreciation was Rs.
1,000,000. Asset is revalued to Rs. 4,500,000 on 1 January 2018 (start of the year).
Business has a policy to charge straight line depreciation.
What is the amount of revaluation surplus at the date of revaluation?

Rs. ___________

20. A business purchased an asset on 1 January 2016 costing Rs. 5,000,000 having a useful life of
10 years with nil residual value. On 1 January 2018 balance of accumulated depreciation was Rs.
1,000,000. Asset is revalued to Rs. 4,500,000 on 1 January 2018 (start of the year).
Business has a policy to charge straight line depreciation.
What is the amount of incremental depreciation for the year ended 31 December 2018?

Rs. ___________

21. A revaluation gain is credited into?

(a) Revaluation reserve

(b) Capital reserve

(c) Profit and loss

(d) Any of the above

22. After initial recognition, an entity has a choice to choose cost and?

(a) Realizable model

(b) Replacement model

(c) Revaluation model

(d) Carrying value model

23. When an item of property, plant and equipment is revalued, what should be revalued?

(a) A selection of assets decided by management

(b) The whole class of assets to which it belongs

(c) The individual asset

(d) A selection of assets picked at random

24. If an asset increases in value, the increase is noted as?

(a) An increase in net profit in the SOCI

(b) An increase in retained earnings in SOFP

(c) An increase in revaluation surplus in the SOFP and other comprehensive income in the SOCI

(d) An increase in “other profit” in SOCI

© Emile Woolf International 64 The Institute of Chartered Accountants of Pakistan


Chapter 1: IAS 16: Property, plant and equipment

25. Which of the following is not a valid reason for reporting non-current assets at revaluation amount
rather than cost?

(a) To prevent long life assets from being reported at out of date historical costs

(b) To keep owners of the business better informed of their equity in the business.

(c) To report performance correctly by matching earnings with the proper costs of assets used.

(d) To avoid having to pay higher taxes

26. An entity has a policy of revaluing its PPE. An asset cost Rs.5m on 1 January 2020 and has a
useful life of five years and is depreciated on a straight-line basis to a zero residual value. The
value of the asset at 31 December 2020 was Rs.3.8m. The fall in value will be accounted for as
follows?

(a) Depreciation Rs.1m and fall in value of Rs.200,000 both to the reserves

(b) Depreciation Rs.1m to the income statement and fall in value of Rs.200,000 ignored until
there is a revaluation surplus

(c) Depreciation Rs.1m to income statement and fall in value of Rs.200,000 to the reserves

(d) Depreciation Rs.1m and fall in value of Rs.200,000 both to the income statement

27. During the financial year, Akmal Ltd had the following increases in reserves:
i. Rs. 5 million from a revaluation of freehold premises
ii. Rs.10 million in share premium
iii. Rs.25 million from trading profit retained
Which of these are increases in capital reserves?

(a) i only

(b) ii only

(c) i. and ii. Only

(d) iii. only

28. The following gains may legally be withdrawn from the company by shareholders:
i. gains that arise from the upward revaluation of non-current assets
ii. gains that arise from the sale of non-current assets
What is the validity of each statement?

(a) Both i. and ii are true

(b) i. is true and ii. is false

(c) Both i. and ii are false

(d) ii. is true and i. is false

© Emile Woolf International 65 The Institute of Chartered Accountants of Pakistan


Financial accounting and reporting I

29. The financial statements of Saadi Limited for the most recent year indicated the following:
i. a bonus issue of shares
ii. a transfer of profit retained to retained earnings
iii. an increase in the revaluation reserve due non-current assets
iv. a rights issue of shares
Which of the above involved a movement of cash?

(a) i. and ii

(b) ii. and iii.

(c) iii only

(d) iv only

30. An apartment is revalued upwards by Rs. 1 million. It was acquired 5 years ago for Rs. 5 million.
Its useful life remains same as 10 years.
What is the revised depreciation charge for the year after revaluation?

(a) Rs. 500,000

(b) Rs. 600,000

(c) Rs. 700,000

(d) Rs. 800,000

31. A building is revalued upwards by Rs. 2 million. It was acquired five years ago for Rs.10 million.
Its useful life remains same as 20 years. What is the incremental depreciation charge for the year?

(a) Rs.100,000

(b) Rs.133,333

(c) Rs.166,667

(d) Rs.200,000

32. An IT equipment being carried at revaluation model has revaluation reserve balance of Rs. 50,000.
During the year, it reduces its value due to technological obsolescence. It has Rs. 70,000 decrease
in value. What would be the impact of this revaluation decrease?

(a) The decrease of Rs.50,000 is debited to revaluation reserve and Rs.20,000 to profit or loss
for the year

(b) The decrease of Rs.50,000 is debited to profit and loss account and Rs.20,000 to revaluation
reserve for the year

(c) The whole decrease is debited to revaluation reserve

(d) The whole decrease is debited to profit or loss for the year

© Emile Woolf International 66 The Institute of Chartered Accountants of Pakistan


Chapter 1: IAS 16: Property, plant and equipment

33. The correct accounting treatment of initial operating losses incurred during the commercial
production due to under-utilization of the plant would be to:

(a) capitalise as a directly attributable cost

(b) defer and charge to profit or loss account when profit is earned from the plant

(c) charge directly to retained earnings since these are not considered to be normal operating
losses

(d) charge to profit or loss account

34. Which of the following is NOT considered as an item of property, plant and equipment?

(a) A standby generator expected to be used for seven years

(b) A plot of land held for resale

(c) A bus for pick and drop of staff members

(d) A generator for rental to others

35. An entity acquires a plant in exchange of old machinery which has carrying amount of Rs. 760,000
and fair value of Rs. 750,000 at the date of exchange. The list price of plant acquired is Rs.
850,000. The entity is also required to pay cash of Rs. 55,000 in this exchange transaction.
At which amount the acquired plant should be initially recognised?

(a) Rs. 850,000


(b) Rs. 760,000
(c) Rs. 815,000
(d) Rs. 805,000

36. An item of plant was purchased on 1 April 2008 for Rs. 2,000,000 and is being depreciated at
25% on a reducing balance basis. What would be its residual value after its useful life of 5 years?
(a) Rs. 632,809
(b) Rs. NIL
(c) Rs. 474,609
(d) Rs. 400,000

37. A non-current asset cost Rs.96,000 and was purchased on 1 June Year 1. Its expected useful life
was five years and its expected residual value was Rs.16,000. The asset is depreciated by the
straight-line method.
The asset was sold on 1 September Year 3 for Rs.68,000. There were no disposal costs. It is the
company policy to charge depreciation on a monthly basis. The financial year runs from 1 January
to 31 December.
What was the gain or loss on disposal?

Rs. ___________

© Emile Woolf International 67 The Institute of Chartered Accountants of Pakistan


Financial accounting and reporting I

38. A non-current asset was purchased on 1 June Year 1 for Rs.216,000. Its expected life was 8
years and its expected residual value was Rs.24,000. The asset is depreciated by the straight-
line method. The financial year is from 1 January to 31 December.
The asset was sold on 1 September Year 4 for Rs.163,000. Disposal costs were Rs.1,000.
It is the company policy to charge a proportionate amount of depreciation in the year of acquisition
and in the year of disposal, in accordance with the number of months for which the asset was
held.
What was the gain or loss on disposal?

Rs. ___________

39. A change in depreciation method is a?


(a) Change in accounting policy
(b) Change in accounting estimate
(c) Change in accounting method
(d) Change in accounting standard

40. When an asset is sold or disposed of, where is the gain or loss recognised?
(a) Asset disposal account
(b) Profit and loss
(c) Revaluation reserve
(d) Depreciation

41. How often should the useful life of an asset be reviewed?


(a) Every six months
(b) As and when the market value will significantly change
(c) At least at each financial year end
(d) Never

42. An entity acquired laptops in exchange of desktops which have carrying amount of Rs. 450,000
and fair value of Rs. 300,000 at the date of exchange. The list price of the laptops acquired is Rs.
600,000. The entity is also required to pay cash of Rs. 275,000 in this exchange transaction.
The laptops should be initially recognized at:

(a) Rs. 300,000


(b) Rs. 575,000
(c) Rs. 450,000
(d) Rs. 600,000

43. During the year 2019, an entity purchased a machine for Rs. 20 million to be used for 6 years.
Which of the following would represent residual value of this machine in 2019?
(a) Rs. 15 million can be currently obtained from disposal of the machine in present condition
(b) Rs. 4 million can be currently obtained from disposal of a 6 year old similar machine
(c) Rs. 18 million can be obtained in 2025 from disposal of the machine in present condition
(d) Rs. 7 million can be obtained in 2025 from disposal of a 6 year old similar machine

© Emile Woolf International 68 The Institute of Chartered Accountants of Pakistan


Chapter 1: IAS 16: Property, plant and equipment

6 OBJECTIVE BASED ANSWERS


01. (d) Rs.
Current value 500,000
Carrying amount
(100,000 – (100,000 × 2% × 15 years)) (70,000)
Revaluation gain 430,000
02. (a)
Building A Building B
Current value 400,000 250,000
Carrying amount (170,000) (330,000)
Revaluation gain/(loss) 230,000 (80,000)
The gain on Building A will be credited to other comprehensive income and
the revaluation surplus.
The loss on Building B will be debited to the statement of profit or loss
expenses because we do not have a balance on the revaluation surplus in
respect of building B to offset the loss.
We make an overall debit to non-current assets of Rs. 230,000 – Rs.
80,000 = Rs. 150,000

03. (a) Land Buildings Total


Rs. Rs. m Rs. m
Cost 1 July 2013 1.00 5.00 6.00
Building depreciation
Rs. 5 million/50 years x 2 years (0.2) (0.2)
Carrying amount 30 June 2015 1.00 4.80 5.80
Revaluation gain 0.24 1.96 1.20
Revalued amount 1.24 5.76 7.00
Building depreciation
Rs. 5.76m/48 years x 2 years (0.24) (0.24)
Carrying amount 30 June 2017 1.24 5.52 6.76
Disposal proceeds 6.80
Gain on disposal 0.04
The gain on disposal is Rs. 40,000. The Rs. 1.2 million balance on the
revaluation reserve is transferred from the revaluation reserve to another
reserve account (probably retained earnings) but is not reported through
the statement of profit or loss for the year.

04. (a) IAS 16 (para 31) states that when the revaluation model is used,
revaluations should be made with sufficient regularity to ensure that the
carrying value of the assets remains close to fair value. IAS 16 also states
(para 36) that, if one item in a class of assets is revalued, all the assets in
that class must be revalued.

05. (c) Six months’ depreciation to the date of the revaluation will be Rs. 300,000
(12,000/20 years × 6/12). Six months’ depreciation from the date of
revaluation to 31 March 2015 would be Rs. 400,000 (10,800/13.5 years
remaining life × 6/12). Total depreciation is Rs. 700,000.

© Emile Woolf International 69 The Institute of Chartered Accountants of Pakistan


Financial accounting and reporting I

06. (c) Building a/c


Particulars Rs. Particulars Rs.
b/d 10,000,000 Acc. dep 500,000
Surplus 2,500,000 c/d 12,000,000
12,500,000 12,500,000

Building net debited by Rs. 2,000,000 (2,500,000 – 500,000)

07. (a) Total loss Rs. 13 million, Rs. 10 will be charged to revaluation surplus and
remaining to profit or loss.

08. (b) Depreciation (20 – 8) / 30 years = Rs. 0.4 million


Carrying amount Rs. 20 million less 0.4 million = Rs. 19.6 million

09. (d) Depreciation Now (20 – 8) / 30 years = Rs. 0.4 million


Depreciation Cost (10 – 2) / 40 years = Rs. 0.2 million
Revaluation surplus
= Land Rs. 6 million + Building Rs. 6 million - incremental depreciation 0.2
million = Rs. 11.8 million

10. (d)

11. (a) On disposal of a revalued asset, the full balance of surplus on revaluation
is transferred to retained earnings.

12. (b) Accumulated depreciation = Rs. 216,000-Rs. 145,000=Rs. 71,000


Surplus = Rs. 291,000-Rs. 145,000=Rs. 146,000
Net amount debited to asset = Rs. 146,000-Rs. 71,000=Rs. 75,000

13. (d)

14. (d)

15. (b)

16. Rs. 3,087 The machine has been owned for 2 years 3 months, so the remaining
useful life at 31 March 2017 was 9 years 9 months.
Prior to revaluation it was being depreciated at Rs. 2,500 pa (30,000/12),
so the charge for the first three months of 2017 was Rs. 625.
The machine will now be depreciated over the remaining 9 years 9 months
= 117 months. So the charge for the remaining 9 months of 2017 is Rs.
2,462 ((32,000 / 117) × 9).
So total depreciation for the year ended 31.12.17 is (625 + 2,462) = Rs.
3,087

17. Rs. 75,000 Incremental depreciation = depreciation on revalued amount – depreciation


at cost
Dep. before revaluation = Rs. 7,500,000 / 20 years = Rs. 375,000
Dep. after revaluation = Rs. 7,650,000 / 17 years = Rs. 450,000
Incremental depreciation = Rs. 75,000

18. Rs. 562,500 Depreciation = Rs. 4,500,000/8= Rs. 562,500

19. Rs. 500,000 Revaluation surplus = Rs. 4,000,000 – 4,500,000= Rs. 500,000

© Emile Woolf International 70 The Institute of Chartered Accountants of Pakistan


Chapter 1: IAS 16: Property, plant and equipment

20. Rs. 62,500 Incremental depreciation = Dep on revalued amount – Dep on cost
= (4,500,000/8)– (5,000,000/10)
=Rs. 562,500 – 500,000 = 62,500
Alternatively, Rs. 500,000 surplus / 8 years = Rs. 62,500

21. (a)

22. (c)

23. (b)

24. (c)

25. (d)

26. (d)

27. (c)

28. (d)

29. (d)

30. (c)

31. (b)

32. (a)

33. (d) Charge to profit or loss account

34. (b) A plot of land held for resale

35. (d) Fair value of asset given up + cash paid


= Rs. 750,000 + 55,000 = Rs. 805,000
36. (c) Rs. 2,000,000 x (0.75)5 = 474,609
37. Rs. 8,000 gain Net sale proceeds = Rs. 68,000
Accumulated depreciation = Rs. 80,000 / 5 years x 2.25 years = Rs. 36,000
Carrying amount = Rs. 96,000 – 36,000 = Rs. 60,000
Gain on disposal = Rs. 68,000 – 60,000 = Rs. 8,000
38. Rs. 24,000 Net sale proceeds = Rs. 163,000 – 1,000 = Rs. 162,000
gain
Accumulated depreciation Rs.
Depreciation year 1
(Rs. 216,000 – 24,000) / 8 years = 24,000 x 7/12 14,000
Depreciation year 2  Rs. 24,000 x 12/12 24,000
Depreciation year 3  Rs. 24,000 x 12/12 24,000
Depreciation year 4  Rs. 24,000 x 8/12 16,000
78,000
Carrying amount = Rs. 216,000 – 78,000 = Rs. 138,000
Gain on disposal = Rs. 162,000 – 138,000 = Rs. 24,000

39. (b) Change in depreciation method is change in accounting estimate.

40. (b) Gain or loss is recognised in profit or loss

© Emile Woolf International 71 The Institute of Chartered Accountants of Pakistan


Financial accounting and reporting I

41. (c) Useful life is reviewed annually at each financial year end, at least.

42. (b) Rs. 575,000

43. (b) Rs. 4 million can be currently obtained from disposal of 6 year old similar
machine

© Emile Woolf International 72 The Institute of Chartered Accountants of Pakistan


Certificate in Accounting and Finance

CHAPTER
Financial accounting and reporting I

Correction of errors

Contents
1 Errors
2 Correcting errors
3 Impact of correcting errors on financial statements
4 Objective based questions and answers

* The student must refer original handbook of IFRS.

© Emile Woolf International 73 The Institute of Chartered Accountants of Pakistan


Financial accounting and reporting I

1 ERRORS
Section overview

 Introduction
 Errors where trial balance does not balance
 Errors where trial balance still balances

1.1 Introduction
Accounting errors can occur in double entry bookkeeping for a number of reasons including, but
not limited to, human factor. These errors can occur at any stage of accounting process, for
example, recording, posting, totalling, balancing, etc. Transposition errors are quite common where
the wrong sequence of individual characters within a number is entered, for example, Rs. 142
entered instead of Rs. 124.
An accounting error can cause the trial balance not to balance, which is easier to spot, or the error
can be such that the trial balance will still balance, usually making it more difficult to identify the
error.

1.2 Errors where trial balance does not balance


One way of finding some errors in the accounting records is to extract a trial balance from the
general ledger. If the total of the debit balances does not equal the total of the credit balances on
the general ledger accounts then an error or several errors have been made. The following six
types of errors will cause a trial balance not to tally i.e. total debits shall not agree with total credits.
Casting error
Incorrect addition in any individual account, i.e. under-casting or over-casting.

Example 01:
In the sales day book, the column for total sales has been added up incorrectly. The total should be
Rs. 26,420, but the total was undercast by Rs. 1,000. (The total was added up as Rs. 25,420). The
correct total amount receivable was entered in the receivables account in the general ledger.
This type of error is called casting error.

Balancing error
Opening balance not brought down or brought down on wrong side or with wrong amount.

Example 02:
Prepaid insurance amounting to Rs. 740,000 pertaining to the last year was not brought forward
from the previous year.
This type of error is called balancing error.

Extraction error
The ledger balance omitted or placed in trial balance at wrong side or with incorrect amount.

Example 03:
Fixture and fittings account balance (debit) of Rs. 460,000 has been included in the trial balance at
credit side.
This type of error is called extraction error because it is caused at the time trial balance is being
extracted from ledger accounts.

Posting error
Part of the transaction not posted or transaction posted with incorrect amount or posting to wrong
side of an account.

© Emile Woolf International 74 The Institute of Chartered Accountants of Pakistan


Chapter 2: Correction of errors

Example 04:
Salaries paid of Rs. 909,000 were correctly recorded in cash book but posted to credit side of
Salaries account in general ledger.
This type of error is called posting error.

Error of part omission


A debit entry has been made but no corresponding credit entry or vice versa i.e. single sided entry.

Example 05:
Cash sales of Rs. 6,000 was correctly recorded in cash book but not recorded anywhere else in the
general ledger.
This type of error is called error of part omission.

Different amounts
Debit and credit entries have been made but at different amounts.

Example 06:
Sales return of Rs. 4,600 was recorded correctly in Sales Return Account in general ledger but
included on credit side of relevant receivable account at Rs. 6,400.
This type of errors is called error of different amounts.

1.3 Errors where trial balance still balances


A trial balance is only useful in helping to identify errors where the debit and credit entries in the
general ledger accounts do not match. It does not help with the identification of errors where there
has not been a mismatch between debit and credit entries.
The following six types of errors result in situation where errors are not highlighted by extraction of
trial balance.
Error of omission
Entry missed from the accounting records completely.

Example 07:
Journal entry that should have been recorded
Date Particulars LF Debit Rs. Credit Rs.
17 Aug Purchases 450,000
ABC & Co (Payables) 450,000

Actually the journal entry was omitted (causing the error)


Date Particulars LF Debit Rs. Credit Rs.

This type of error is called error of omission.

Error of commission
Correct amount and type of account but wrong individual account.

Example 08:
Journal entry that should have been recorded
Date Particulars LF Debit Rs. Credit Rs.
17 Aug Purchases 450,000
ABC & Co (Payables) 450,000

© Emile Woolf International 75 The Institute of Chartered Accountants of Pakistan


Financial accounting and reporting I

Journal entry that was recorded actually (causing the error)


Date Particulars LF Debit Rs. Credit Rs.
17 Aug Purchases 450,000
ATC & Co (Payables) 450,000
This type of error is called error of commission.

Error of principle
Correct amount but wrong type of account.

Example 09:
Journal entry that should have been recorded
Date Particulars LF Debit Rs. Credit Rs.
17 Aug Purchases 450,000
ABC & Co (Payables) 450,000

Journal entry that was recorded actually (causing the error)


Date Particulars LF Debit Rs. Credit Rs.
17 Aug Plant & machinery 450,000
ABC & Co (Payables) 450,000
This type of error is called error of principle.

Compensating errors
Two or more errors balance each other out.

Example 10:
Journal entries that should have been recorded
Date Particulars LF Debit Rs. Credit Rs.
17 Aug Purchases 450,000
ABC & Co (Payables) 450,000
31 Aug Salaries expense 999,000
Bank 999,000

Journal entries that were recorded actually (causing the error)


Date Particulars LF Debit Rs. Credit Rs.
17 Aug Purchases 540,000
ABC & Co (Payables) 450,000
31 Aug Salaries expense 909,000
Bank 999,000
This type of errors are called compensating errors.

Error of original entry


Correct accounts but wrong amounts on both side of an entry.

Example 11:
Journal entry that should have been recorded
Date Particulars LF Debit Rs. Credit Rs.
17 Aug Purchases 450,000
ABC & Co (Payables) 450,000

© Emile Woolf International 76 The Institute of Chartered Accountants of Pakistan


Chapter 2: Correction of errors

Journal entry that was recorded actually (causing the error)


Date Particulars LF Debit Rs. Credit Rs.
17 Aug Purchases 540,000
ABC & Co (Payables) 540,000
This type of error is called error of original entry.

Complete reversal of entries


Correct accounts and amounts but sides (debit & credit) reversed.

Example 12:
Journal entry that should have been recorded
Date Particulars LF Debit Rs. Credit Rs.
17 Aug Purchases 450,000
ABC & Co (Payables) 450,000

Journal entry that was recorded actually (causing the error)


Date Particulars LF Debit Rs. Credit Rs.
17 Aug ABC & Co (Payables) 450,000
Purchases 450,000
This type of error is called complete reversal of entries.

© Emile Woolf International 77 The Institute of Chartered Accountants of Pakistan


Financial accounting and reporting I

2 CORRECTING ERRORS
Section overview

 Suspense account
 Approach to correction of errors

2.1 Suspense account


A suspense account is temporary account that is required until the sufficient information is received
and errors are identified and corrected. Suspense accounts are temporarily classified as a
statement of financial position account, usually under the heading of current assets or current
liabilities depending on the balance (debit or credit).
There are two main reasons for creation of suspense account:
Trial balance does not agree
Irrespective of the reasons why a trial balance may not balance, as a temporary measure the
difference in the trial balance is allocated to a suspense account and a suspense account
reconciliation is carried out at a later stage.

Example 13:
For example, a trial balance showed total debits of Rs. 97,000 but total credits of Rs. 94,000 leaving
a difference of Rs. 3,000.
Trial balance as at 30 June 2021 (extracts only)
Account Debit Credit
Particulars
code Rs. Rs.

Total 97,000 94,000


TMP-05 Suspense account 3,000
Total 97,000 97,000
This way a suspense account has been created to tally the trial balance temporarily until the error
is identified and corrected.

Unknown entry
In some instances, a suspense account will be opened deliberately by the book-keeper, if the
bookkeeper is uncertain of where to post one side of the double entry. This is later transferred to
proper account when relevant information is received.

Example 14:
A bookkeeper has noticed the credit transfer by a customer of Rs. 25,000 in bank statement but
which of the customers exactly made that payment. He may temporarily record it in suspense
account.
Date Particulars LF Debit Rs. Credit Rs.
17 Aug Bank 450,000
Suspense 450,000

Later, it was discovered that the credit transfer was from O&J Traders, the suspense account can be
eliminated now by transferring the amount to correct account.
Date Particulars LF Debit Rs. Credit Rs.
19 Oct Suspense 450,000
O&J Traders 450,000
.

© Emile Woolf International 78 The Institute of Chartered Accountants of Pakistan


Chapter 2: Correction of errors

Approach to correction of errors


The correction of errors would require the consideration of following three steps:
 What double entry should have been recorded?
 What double entry was recorded, in actual, including suspense account where the error
caused it?
 What correction is required?
Note: Assume that if one side of double entry is not mentioned, it has been correctly recorded.

The correction process would require:


 to reverse what had been recorded incorrectly; and
 to record what had been omitted.
Example 15:
Question: Sales returns of Rs. 800 were recorded correctly in the receivables account in the general
ledger, but they have been recorded incorrectly as a credit entry in the purchases returns account.

Required:
What shall be the correcting entry?

Answer:
Double entry that should have been recorded
Date Particulars LF Debit Rs. Credit Rs.
Sales return 800
Receivables 800

Double entry as recorded (including Suspense effect, if any)


Date Particulars LF Debit Rs. Credit Rs.
Suspense (balancing) 1,600
Purchase return 800
Receivable 800

The correcting entry shall be:


Date Particulars LF Debit Rs. Credit Rs.
Sales return 800
Purchase return 800
Suspense 1,600
Note: only correcting entry is answer (other entries have been mentioned to understand the
correction process).

Example 16:
Question: In the sales day book, the column for total sales has been added up incorrectly. The total
should be Rs. 26,420, but the total was undercast by Rs. 1,000. (The total was added up as Rs.
25,420). The correct total amount receivable was entered in the receivables account in the general
ledger.
Required:
What shall be the correcting entry?

© Emile Woolf International 79 The Institute of Chartered Accountants of Pakistan


Financial accounting and reporting I

Answer:
Double entry that should have been recorded
Date Particulars LF Debit Rs. Credit Rs.
Receivables 26,420
Sales 26,420

Double entry as recorded (including Suspense effect, if any)


Date Particulars LF Debit Rs. Credit Rs.
Receivables 26,420
Sales 25,420
Suspense (balancing) 1,000

The correcting entry shall be:


Date Particulars LF Debit Rs. Credit Rs.
Suspense 1,000
Sales 1,000
Note: only correcting entry is answer (other entries have been mentioned to understand the
correction process).

Example 17:
Question: A cheque of Rs. 10,800 was paid to a creditor who allowed 10% cash discount. The
payment was correctly entered in the bank book but was posted to purchase account as Rs. 1,080
only therefrom. No other entry was made.
Required:
What shall be the correcting entry?

Answer:
Double entry that should have been recorded
Date Particulars LF Debit Rs. Credit Rs.
Trade payables [10,800 x 100/90] 12,000
Purchases/Discount received 1,200
Bank 10,800

Double entry as recorded (including Suspense effect, if any)


Date Particulars LF Debit Rs. Credit Rs.
Purchases 1,080
Suspense (balancing) 9,720
Bank 10,800

The correcting entry shall be:


Date Particulars LF Debit Rs. Credit Rs.
Trade payables 12,000
Purchases/Discount received 1,200
Purchases 1,080
Suspense 9,720
Note: only correcting entry is answer (other entries have been mentioned to understand the
correction process).

© Emile Woolf International 80 The Institute of Chartered Accountants of Pakistan


Chapter 2: Correction of errors

Example 18:
Question: Sales commission of Rs. 3,500 was paid but was credited twice, once in the bank account
and again in the commission account.
Required:
What shall be the correcting entry?

Answer:
Double entry that should have been recorded
Date Particulars LF Debit Rs. Credit Rs.
Selling costs (commission) 3,500
Bank 3,500

Double entry as recorded (including Suspense effect, if any)


Date Particulars LF Debit Rs. Credit Rs.
Suspense (balancing) 7,000
Bank 3,500
Selling costs (commission) 3,500

The correcting entry shall be:


Date Particulars LF Debit Rs. Credit Rs.
Selling costs (commission) 7,000
Suspense 7,000
Note: only correcting entry is answer (other entries have been mentioned to understand the
correction process).

Example 19:
Question: Goods costing Rs. 10,000 were taken by the owner for personal use and goods worth of
Rs. 2,000 were used for office repairs. However, recording these was omitted by the bookkeeper.
Required:
What shall be the correcting entry?

Answer:
Double entry that should have been recorded
Date Particulars LF Debit Rs. Credit Rs.
Drawings 10,000
Office repairs 2,000
Purchases 12,000

No entry was recorded (Error of complete omission)

The correcting entry shall be:


Date Particulars LF Debit Rs. Credit Rs.
Drawings 10,000
Office repairs 2,000
Purchases 12,000
Note: only correcting entry is answer (other entries have been mentioned to understand the
correction process).

© Emile Woolf International 81 The Institute of Chartered Accountants of Pakistan


Financial accounting and reporting I

Example 20: Sky Electronics


Question: While reviewing the draft financial statements of Sky Electronics (SE) for the year ended
31 December 2017, following errors have been identified:
(i) Computers costing Rs. 240,000 purchased on 1 September 2017 for office use were debited
to purchases account. SE depreciates computers at 20% per annum using straight line
method.
(ii) Furniture costing Rs. 1,200,000 and having a book value of Rs. 670,000 as on 31 December
2017 had already been sold on 1 November 2017. The proceeds of Rs. 700,000 were
credited to sales. SE depreciates furniture at 10% per annum using straight line method.
(iii) On 1 April 2017, SE rented-out one of its premises at an annual rent of Rs. 900,000 payable
in advance. The rent received was credited to income.
(iv) Trade receivables include a balance of Rs. 180,000 which is irrecoverable but has not been
written-off. Further, a recovery of Rs. 96,000 against receivables written off in prior years was
credited to trade receivables. As per SE's policy, provision for doubtful receivables has already
been made at 5% on year-end balance.
(v) A cheque of Rs. 192,000 was received after a settlement discount of 4% from a customer.
However, in the cash book, the amount received was entered in the discount allowed column
and the amount of discount was entered in the bank column. SE does not maintain separate
settlement discount accounts in its ledgers.

Required: Prepare rectification entries to correct the above errors. (Narrations are not required)

Answer: Accounting entries for correction of the errors

Debit Credit
Date Description
Rupees
(i) Fixed assets (Computers) 240,000
Purchases 240,000
Depreciation expense [240,000×0.2÷12×4] 16,000
Accumulated depreciation 16,000
(ii) Accumulated depreciation (1,200,000×0.1×2÷12) 20,000
Depreciation expense 20,000
Sales 700,000
Accumulated depreciation [(1,200,000-670,000)-20,000] 510,000
Fixed assets (Furniture) (1,200,000-700,000) 1,200,000
Gain on disposal (Balancing figure) 10,000
(iii) Rent income (900,000÷12×3) 225,000
Advance rent 225,000
(iv) Bad debt expense 180,000
Trade receivables 180,000
Trade receivable 96,000
Bad debt expense/Bad debt recovered 96,000
Provision for doubtful debts (180,000-96,000)×5% 4,200
Bad debt expense 4,200
(v) Bank 192,000-(192,000÷0.96×0.04) 184,000
Revenue/Sales 184,000

© Emile Woolf International 82 The Institute of Chartered Accountants of Pakistan


Chapter 2: Correction of errors

Example 21: Grant Company


Question: The accountant of Grant Company has prepared a trial balance, but has found that the
total of debit balances is Rs.864,600 and the total of credit balances is Rs.862,150.
On investigation, he discovers the following errors in the book-keeping:
(1) Total purchases in the period were recorded at Rs.100 below their correct value, although
the total value of trade payables was correctly recorded.
(2) Total telephone expenses were recorded at Rs.1,000 above their correct amount, although
the total value of the amounts payable was correctly recorded.
(3) Purchase returns of Rs.550 were recorded as a debit entry in the sales returns account, but
the correct entry had been made in the trade payables control account.
(4) Equipment costing Rs.2,000 had been recorded as a debit entry in the repairs and
maintenance account.
(5) Rental expenses of Rs.5,490 were entered incorrectly as Rs.5,940 in the expense account
but were entered correctly in bank account in the ledger.
(6) Bank charges of Rs.200 have been omitted entirely from the ledger.
(Note: Individual debtors and creditors accounts are maintained in the general ledger)
Required
(a) Prepare journal entries for the correction of the errors.
(b) Open a suspense account. Record the appropriate corrections in the suspense account, so
that the balance on this account is eliminated.
Answer:
a) The journal entries for the correction of the errors shall be prepared as follows:

Transaction Debit Credit


Rs. Rs.
1 Purchases 100
Suspense account 100
Correction of error: purchases under-stated by Rs.100.

2 Suspense account 1,000


Telephone expenses 1,000
Correction of error: telephone expenses over-stated by Rs.1,000.

3 Suspense account 1,100


Purchase returns 550
Sales returns 550
Correction of error. Purchase returns of Rs.550 incorrectly recorded
as a debit entry in sales returns.

4 Equipment 2,000
Repairs and maintenance 2,000
Correction of error. Equipment purchase costs incorrectly recorded as repairs and
maintenance expenses

© Emile Woolf International 83 The Institute of Chartered Accountants of Pakistan


Financial accounting and reporting I

Debit Credit
Rs. Rs.
5 Suspense account 450
Rent expenses 450
Correction of error: rent expenses over-stated by Rs.450.

6 Interest expense 200


Bank account 200

b) The appropriate corrections in the suspense account shall be as follows:

Transaction omitted from the ledger.


Suspense account
Rs. Rs.
Telephone expenses 1,000 Opening balance 2,450
Purchase returns 550 (864,600 – 862,150)
Sales returns 550 Purchases 100
Rent expenses 450
2,550 2,550
Tutorial notes
(1) In the trial balance, total debits exceed total credits by Rs.2,450. A suspense account
is therefore opened with a credit balance of Rs.2,450, to make the total debits and
credits equal.
(2) It helps to think about the ledger account that needs correcting first, and then make
the suspense account entry as the matching debit or credit. For example, since
purchases were under-stated in the purchases account, the correction must be to
increase the debit side of the purchases account by Rs.100. The matching double
entry is a credit in the suspense account.
(3) You also need to recognise that one of the errors and the omission can be corrected
without using a suspense account, because the error/omission has not made the
total debits and credits unequal.

© Emile Woolf International 84 The Institute of Chartered Accountants of Pakistan


Chapter 2: Correction of errors

3 IMPACT OF CORRECTING ERRORS ON FINANCIAL STATEMENTS


Section overview

 Impact of errors on financial statements


 Effect of correcting entries on profit
 Effect of correcting entries on other line items of financial statements

3.1 Impact of errors on financial statements


The impact of errors on financial statements may be material or immaterial, therefore, it is important
to quantify this impact. This can be calculated as difference of ‘the amount if error is not corrected’
and ‘the corrected amount’. The same impact may also be quantified by looking at debit and credit
items of correcting journal entry.
Often the most common analysis in this respect is the effect of correcting entries on profit and non-
current assets.

3.2 Effect of correcting entries on profit


We must also understand the effect of an error on reported profit and effect of correcting an error
on reported profit as well. Sometimes, exam questions also provide the figure of reported profits
before the correction of errors and require recalculating the profit after the corrections have been
made.
The following points are relevant in this regard:
 All accounts of income and expenses are called nominal accounts.
 The net profit shall increase due to all nominal accounts credited in correcting entries.
 The net profit shall decrease due to all nominal accounts debited in correcting entries.
 The gross profit shall increase due to nominal accounts, related to revenue and cost of sales
only, credited in correcting entries.
 The gross profit shall decrease due to nominal accounts, related to revenue and cost of
sales only, debited in correcting entries.

Example 22:
Question: The following correcting entry has been passed to correct the error given in example 15.
Date Particulars LF Debit Rs. Credit Rs.
Sales return 800
Purchase return 800
Suspense 1,600

Required:
What shall be impact of correction on gross profit and net profit?

Answer:
Impact on gross profit: Rs. 800 debit + 800 debit = Rs. 1,600 Decrease
Impact on net profit: Same as gross profit = Rs. 1,600 Decrease

© Emile Woolf International 85 The Institute of Chartered Accountants of Pakistan


Financial accounting and reporting I

Example 23:
Question: The following correcting entry has been passed to correct the error given in example 16.
Date Particulars LF Debit Rs. Credit Rs.
Suspense 1,000
Sales 1,000

Required:
What shall be impact of correction on gross profit and net profit?

Answer:
Impact on gross profit: Rs. 1,000 credit = Rs. 1,000 Increase
Impact on net profit: Same as gross profit = Rs. 1,000 Increase

Example 24:
Question: The following correcting entry has been passed to correct the error given in example 17.
Date Particulars LF Debit Rs. Credit Rs.
Trade payables 12,000
Purchases/Discount received 1,200
Purchases 1,080
Suspense 9,720

Required:
What shall be impact of correction on gross profit and net profit?

Answer:
Impact on gross profit: Rs. 1,200 credit + 1,080 credit = Rs. 2,280 Increase
Impact on net profit: Same as gross profit = Rs. 2,280 Increase

Example 25:
Question: The following correcting entry has been passed to correct the error given in example 18.
Date Particulars LF Debit Rs. Credit Rs.
Selling costs (commission) 7,000
Suspense 7,000

Required:
What shall be impact of correction on gross profit and net profit?

Answer:
Impact on gross profit: No effect.
Impact on net profit: Rs. 7,000 debit = Rs. 7,000 Decrease

© Emile Woolf International 86 The Institute of Chartered Accountants of Pakistan


Chapter 2: Correction of errors

Example 26:
Question: The following correcting entry has been passed to correct the error given in example 19.
Date Particulars LF Debit Rs. Credit Rs.
Drawings 10,000
Office repairs 2,000
Purchases 12,000

Required:
What shall be impact of correction on gross profit and net profit?

Answer:
Impact on gross profit: Rs. 12,000 credit = Rs. 12,000 Increase
Impact on net profit: Rs. 12,000 credit – 2,000 debit = Rs. 10,000 Increase

The exam questions may also require to calculated revised profit after the corrections are made.

Example 27:
Question: A bookkeeper in error recorded the purchase cost of a new item of equipment as Rs.
36,000 when it should have been Rs. 360,000.
A draft profit of Rs. 2,560,000 for the period was calculated before the discovery of the error. This
included a depreciation charge of 10% (Rs. 3,600) for the equipment.
Required: What is the correct figure for profit?
Answer:
Rs.
Draft profit 2,560,000
Add back: Depreciation incorrectly charged 3,600
2,563,600
Deduct: Correct depreciation charge (10%  Rs. 360,000) (36,000)
Adjusted figure for profit 2,527,600

Example 28:
Question: A bookkeeper in error recorded the Rs. 60,000 purchase cost of a new machine as repairs
and maintenance costs.
A draft profit of Rs. 300,000 for the period was calculated before the discovery of the error.
Depreciation on machinery is charged at 20% on cost, with a full year’s charge in the year of
acquisition.
Required: What is the correct figure for profit?
Answer:
Rs.
Draft profit 300,000
Add back: Repair costs incorrectly charged 60,000
360,000
Deduct: Depreciation charge (20%  Rs. 60,000) (12,000)
Adjusted figure for profit 348,000

© Emile Woolf International 87 The Institute of Chartered Accountants of Pakistan


Financial accounting and reporting I

Example 29: BA Enterprises


The accountant of BA Enterprises prepared a statement of comprehensive income for the year
ended December 31, 2013 which showed gross profit of Rs. 1,050,000 and net profit of Rs.
650,000. The company sells goods at cost plus mark-up of 20%.
The following errors/omissions were found on a detailed review of the financial statements.
(a) Items not included in the statement of comprehensive income:
(i) Free samples costing Rs. 25,000 were sent to potential and regular customers.
(ii) Goods costing Rs. 10,000 were taken by the owner for personal use and goods having
sales value of Rs. 2,500 were used for office repairs.
(iii) Unpaid salaries and transportation (inward) expenses payable, amounting to Rs.
20,000 and Rs. 10,000 respectively.
(b) Old furniture items were sold for Rs. 3,000 and entered in the sales day book. The book
value of these items was Rs. 2,000.
(c) Rs. 24,500 were paid to a creditor as full and final settlement of an amount of Rs. 25,000
and debited to purchases.
(d) The sales day book was overcast by Rs. 30,000.
(e) An amount of Rs. 67,000 was carried forward in the purchase day book as Rs. 6,700.
Required:
Ascertain the correct amount of gross and net profit for the year.
Answer:
Statement for ascertaining correct gross and net profit
Gross Profit Net Profit
Profit as given 1,050,000 650,000
Add / (Less)
(a) (i) Cost of Goods distributed as free samples 25,000 -
(ii) Goods taken by owner for personal use 10,000 10,000
Goods used for office repair (2,500*100/120) 2,083 -
(iii) Outstanding expenses
Salaries - (20,000)
Transportation-in (10,000) (10,000)
(b) Sale of old furniture items posted as sale (3,000) (2,000)
(c) Payment to creditor wrongly debited to purchases 24,500 24,500
Purchase discount (settlement) 500 500
(d) Over-casting of sales day book (30,000) (30,000)
(e) Error in Carrying forward of purchases (67,000 
6,700) (60,300) (60,300)
Corrected figures 1,008,783 562,700

© Emile Woolf International 88 The Institute of Chartered Accountants of Pakistan


Chapter 2: Correction of errors

Example 30: Eastern Products


Question: The trial balance of Eastern Products showed a short credit of Rs. 6,264 as at June 30,
2013. A suspense account was opened for the difference and the profit for the year was then
calculated at Rs. 956,180.
The following errors and adjustments were discovered subsequently:
(i) An invoice of Rs. 3,700 was debited to purchases but the goods were received after year-end
and were not included in the closing inventory.
(ii) Store equipment costing Rs. 8,100 and having a book value of Rs. 3,600 was sold for Rs.
2,500. Cash was debited and store equipment was credited. No other entries were made.
(iii) A cheque of Rs. 1,850 received from a customer was dishonoured on June 25, 2013 but no
entry was made in the books. Cash there against was received after year-end.
(iv) Purchase of office equipment costing Rs. 15,200 was entered in the purchases account.
Depreciation on office equipment is provided at the rate of 10%.
(v) A purchase invoice of Rs. 197 was debited to the supplier account as Rs. 917.
(vi) Purchase returns book was under-casted by Rs. 650.
(vii) The opening balance of furniture account was brought forward as Rs. 18,300 instead of
Rs.13,800. Depreciation on furniture is provided at the rate of 10%.
(viii) A balance of Rs.730 in the sales ledger is to be offset against a balance of Rs. 880 in the
purchase ledger.

(Note: Individual debtors and creditors accounts are maintained in the general ledger)

Required:
(a) Prepare journal entries to adjust the above items.
(b) Recalculate the net profit for the year.

Answer:

(a) Debit Credit


1 Supplier/Creditor account /goods in transit 3,700
Purchases 3,700

2 Accumulated Depreciation 4,500


Loss on sale of store equipment 1,100
Store equipment a/c 5,600

3 Customer / debtors a/c 1,850


Bank 1,850

4 Office Equipment 15,200


Purchases 15,200
Depreciation expense 1,520
Accumulated depreciation – office equipment 1,520

5 Suspense a/c 1,114


Supplier a/c (197 + 917) 1,114

6 Suspense a/c 650


Purchase Returns 650

© Emile Woolf International 89 The Institute of Chartered Accountants of Pakistan


Financial accounting and reporting I

7 Suspense a/c 4,500


Furniture a/c 4,500
Accumulated Depreciation 450
Depreciation expenses 450

8 Payables a/c 730


Debtors a/c 730

(b) Recalculation of net profit for the year


Net profit as per question 956,180
Add: Reversal of purchases / goods in transit / increase in closing
inventory 3,700
Equipment wrongly debited to purchases 15,200
Purchase returns 650
Reversal of depreciation on furniture 450
976,180
Less: loss on sale of store equipment (1,100)
Depreciation on office equipment (1,520)
Adjusted net-profit 973,560

3.3 Effect of correcting entries on other line items of financial statements


Similarly, impact on other line items in the financial statements may be calculated, for example,
assets would increase when related account is debited in correcting entry and decrease when
related account is credited in correcting entry.

Example 31: Zeta Traders


Question: Financial statements of Zeta Traders (ZT) for the year ended 30 June 2019 is under
preparation. Following information has been gathered in this respect:
(i) Trade receivables as at 30 June 2019:
Rupees
Trade receivables 2,500,000
Provision for doubtful debts (400,000)
Net trade receivables 2,100,000
It was noted that:
 an old outstanding balance of Rs. 250,000 which was written off previously was settled during the
year at 20% discount. The amount received was credited to trade receivables.
 purchase return amounting to Rs. 500,000 was mistakenly debited to trade receivables.
 Rana and Sons having a balance of Rs. 80,000 due for more than one year was declared bankrupt
and its balance needs to be written off.
ZT maintains provision for doubtful debts:
 at 25% for balances outstanding for more than six months. As at 30 June 2019, such balances
are aggregated to Rs. 600,000 (excluding balance of Rana and Sons); and
 at 5% for the remaining balances

© Emile Woolf International 90 The Institute of Chartered Accountants of Pakistan


Chapter 2: Correction of errors

(ii) A cheque dated 25 June 2019 for Rs. 150,000 was received from an insurance company and
deposited by the owner in his personal bank account. The cheque was received in settlement
of an inventory loss claim. Actual inventory loss was determined at Rs. 180,000. No entries
have been made for loss of inventory and insurance claim.
(iii) The opening balance of accumulated depreciation was brought forward as Rs. 280,000
instead of Rs. 820,000. The error was tried to be corrected with the difference by crediting
accumulated depreciation and debiting depreciation expense.
(iv) Goods amounting to Rs. 350,000 received from a supplier on 30 June 2019 were included
in the year-end physical inventory count but recorded in purchases day book on 1 July 2019.
(v) Third party stock of Rs. 500,000 lying on ZT premises has been included in ZT’s year-end
inventory.
(vi) ZT uses periodic inventory method.
Required:
(a) Prepare adjusting / correcting entries for the year ended 30 June 2019. (Narrations are not
required)
(b) Compute the net effect of the above on ZT’s profit for the year ended 30 June 2019.
Answer:
Adjusting/correcting entries
General journal
Debit Credit
Description
------ Rupees ------
(i) Trade receivables 200,000
Bad debt expense/Bad debts recovered 200,000

Trade payables 500,000


Trade receivables 500,000

Bad debts expense 80,000


Trade receivables 80,000

Provision for doubtful debts (W-1) 174,000


Bad debt expense 174,000

(ii) Abnormal loss (P&L) 180,000


Purchases 180,000

Drawings 150,000
Abnormal loss (P&L) 150,000

(iii) Suspense (820,000–280,000) 540,000


Depreciation expense 540,000

(iv) Purchases 350,000


Trade payables 350,000

(v) Trading account/COGS 500,000


Closing inventory 500,000

W-1: Provision for bad debts: Rupees


Provision for bad debts as at 1 July 2018 400,000
Provision required as at 30 June 2019:
- 25% on balances outstanding for more than 6 months (600,000×25%) 150,000
- 5% on other balances [2,500+(250×0.8)–500–80–600]×5% 76,000
226,000

© Emile Woolf International 91 The Institute of Chartered Accountants of Pakistan


Financial accounting and reporting I

Excess provision to be reversed (174,000)


(b) Increase/ (decrease) in net profit Rupees
(i) Recovery of old outstanding balance previously written-off 200,000
Bad debts (write off) (80,000)
Reversal of provision for doubtful debts 174,000
(ii) Insurance claim 150,000
(iii) Correction in depreciation expense 540,000
(iv) Purchases of 30 June 2019 (350,000)
(v) Third party inventory (500,000)
Net increase in net profit 134,000
.

Example 32: Naltar Establishment


Question: Following draft statement of financial position as on 31 December 2019 of Naltar
Establishment (NE) is under review:
Assets Rs. in ‘000 Equity & liabilities Rs. in ‘000
Fixed assets – net 22,590 Opening capital 32,240
Current assets: Net profit for the year 9,360
Stock 15,320 41,600
Trade receivables 19,730 Current liabilities:
Drawings 1,400 Trade payables 17,332
Cash & bank 3,850 Other payables 2,680
40,300 Suspense account 798
Discount received 480
21,290
62,890 62,890
The following matters are identified:
(i) Goods costing Rs. 5,800,000 received on 31 December 2019 were included in the year-end
physical count. However, these goods were recorded in purchases day book on 5 January
2020 on receipt of the invoice.
(ii) Year-end physical count sheets include a third party stock of Rs. 1,320,000.
(iii) Goods sold on credit at a trade discount of 10% were recorded at gross amount of Rs.
6,400,000.
(iv) An unidentified credit of Rs. 294,000 appearing in the bank statement was accounted for
in the suspense account. It was discovered that the credit was a settlement of an old
outstanding balance previously written off. The amount was net of 2% bank charges.
(v) A cheque of Rs. 500,000 issued by a customer as an advance was dishonoured and
returned by the bank on 31 December 2019. No entry was made on return of cheque.
(vi) Operating expenses include insurance premium of Rs. 900,000 paid during the year, out of
which Rs. 200,000 pertain to owner's residential premises. The policy is valid up to 30 June
2020.
(vii) An upgradation of a plant to improve quality and efficiency was completed on 1 July
2019 at a cost of Rs. 2,500,000. The cost was charged to repair and maintenance expense.
(viii) Total sales of 26 December 2019 as per sales day book was Rs. 167,000. This was posted
in trade receivable control account as Rs. 671,000. Trial balance was balanced by taking
the difference to the suspense account.
Other information: NE uses periodic stock method. The plant is depreciated at 20% using
diminishing balance method.
Required: Prepare corrected statement of financial position as on 31 December 2019.

© Emile Woolf International 92 The Institute of Chartered Accountants of Pakistan


Chapter 2: Correction of errors

Answer:
Naltar Establishment
Statement of financial position as at 31 December 2019
Rs. in '000
Assets
Fixed assets – net 22,590+2,500–250 24,840
Current assets:
Closing stock 15,320–1,320 14,000
Trade receivables 19,730–640–504 18,586
Prepayment 350
Cash & bank 3,850–500 3,350
36,286
61,126
Equity & liabilities
Opening capital 32,240
Net profit for the year (W-1) 5,174
Drawings 1,400+200 (1,600)
Net equity 35,814
Current liabilities:
Trade payables 17,332+5,800 23,132
Other payables 2,680–500 2,180
Suspense account 798–294–(671–167) -
Discount received 480–480 -
25,312
61,126

W-1: Corrected net profit for the year Rs. in '000


Balances as per draft financial statements 9,360
Corrections:
(i) Purchases of Dec. 2019 recorded in Jan. 2020 (5,800)
(ii) Third party stock included in NE closing stock (1,320)
(iii) Goods sold recorded at gross of 10% trade discount 6,400×10% (640)
(iv) Clearance of unidentified credit taken to suspense account:
- Bank commission at 2% 294÷0.98×0.02 (6)
- Recovery of previously written off balance 294÷0.98 300
(vi) Insurance premium paid pertains to:
- Owner 200
- Prepayments (900–200)÷2 350
(vii) Upgradation cost of a plant charged to repair and maintenance:
- Repair and maintenance expense 2,500
- Depreciation expense (2,500×20%×6÷12) (250)
Discount received shown as a liability 480
Revised net profit 5,174
.

Example 33: Smetena’s Newsagents


Question: The bookkeeper has produced the following statement of financial position at 31
December for Smetena’s Newsagents.
Rs. Rs.
Non-current assets 72,208
Current assets
Inventory 18,826
Trade receivables 26,216
Drawings 8,260
Suspense account 3,830

© Emile Woolf International 93 The Institute of Chartered Accountants of Pakistan


Financial accounting and reporting I

Cash 700
57,832
130,040
Capital account 50,224
Loan – L Franks 12% 20,000
Trade payables 26,782
Bank overdraft 14,634
Profit for year 18,400
130,040

Jan Smetena, the proprietor, is unhappy with the statement of financial position and asks you to
revise it. You discover the following.
(1) The suspense account balance represents the difference on the trial balance.
(2) The purchases day book total for October of Rs.4,130 was posted to the purchases account
as Rs.4,310 although the correct entry was made to the payables ledger control account.
(3) Inventory sheets were overcast by Rs.2,000.
(4) Cash should be Rs.110.
(5) Fixtures and fittings account balance of Rs.4,600 has been omitted from the trial balance.
(6) Interest for a half year on the loan account has not been paid and no provision has been
made for it.

Required:
(a) Show the journal entries to correct the above errors.
(b) Write up the suspense account.
(c) Draw up a revised statement of financial position at 31 December. Clearly show the
adjustments to profit.
Answer:
Part (a) Journal Entries:
Date /# Particulars Dr. Rs. Cr. Rs.
(2) Suspense 180
Purchases 180
(3) CGS 2,000
Inventory 2,000
(4) Suspense 590
Cash 590
(5) Fixture & fittings 4,600
Suspense 4,600
(6) Interest expense 1,200
Interest payable 1,200

Part (b) Suspense Account


b/d (balancing fig.) 3,830 Fixtures and fittings (5) 4,600
Purchases (2) 180
Cash (4) 590
4,600 4,600

Part (c)
SMETENA NEWSAGENTS
Statement of financial position (revised)
As on 31st December
Rs. Rs.
Non- current assets (72,208+4,600) (5) 76,808
Current assets
Inventory (18,826-2,000) (3) 16,826
Trade Receivables 26,216
Cash (700-590) (4) 110 43,152
Total assets 119,960

Non – current liabilities


Loan – L Franks 20,000

© Emile Woolf International 94 The Institute of Chartered Accountants of Pakistan


Chapter 2: Correction of errors

Current liabilities
Bank overdraft 14,634
Trade payables 26,782
Accrued expenses 1,200 42,616
Total liabilities 62,616
Opening capital 50,224
Profit W 15,380
Drawings (8,260) 57,344
Total liabilities and capital 119,960

Working (W) Rs.


Profit (unadjusted) 18,400
Inventory overstated (2,000)
Interest on loan (1,200)
Purchases overstated 180
Net profit (adjusted) 15,380

Example 34: Sibi Brothers


Question: The trial balance of Sibi Brothers (SB), dealer of equipment and machines, did not agree
as at 31 December 2018 and the difference was carried to suspense account. The financial
statements prepared from the trial balance showed a gross profit of Rs. 854,000.

During review, following errors were detected:


(i) A sales invoice of Rs. 24,000 was debited to the debtor’s account as Rs. 42,000.
(ii) A purchase of Rs. 23,000 was entered in purchases day book as Rs. 32,000 and posted to
the creditor’s account as Rs. 3,200.
(iii) An item was included in closing inventory at its cost of Rs. 94,000. Due to lower demand,
it had a net realizable value of Rs. 81,000.
(iv) A sub-total of Rs. 49,000 was carried forward in the sales day book as Rs. 94,000.
(v) Return inward and return outward appearing in the trial balance were Rs. 82,000 and Rs.
99,000 respectively. While preparing the financial statements, the amount of return inward
was shown as return outward and vice versa.
(vi) Discount received of Rs. 4,100 was posted to the debit of discount allowed.
(vii) SB started using an inventory item as office machine effective from 1 October 2018. No
adjustment has been recorded and this item is included in closing inventory. The cost and
selling price of this item are Rs. 145,000 and Rs. 182,000 respectively.
(viii) Another office machine costing Rs. 270,000 with a carrying value of Rs. 127,200 as on 1
January 2018 was disposed of on 1 September 2018 for Rs. 80,000. The sale proceeds
were credited to accumulated depreciation account and whole year’s depreciation was
provided on the machine.

The balance as per bank statement as on 31 December 2018 was reconciled with cash book. During
review, following matters were noted in bank reconciliation statement:
(i) List of unpresented cheques included:
 a cheque issued to a creditor on 30 April 2018 amounting to Rs. 28,000.
 a cheque dated 30 December 2018 amounting to Rs. 16,000 which was handed over
to the creditor on 6 January 2019.
(ii) List of deposits in transit included a cheque dated 15 January 2019 from a debtor
amounting to Rs. 35,000.
(iii) Bank charges of Rs. 3,100 correctly debited by bank had been added back.

Other information:
SB uses periodic inventory method to record the inventory. Office machines are depreciated at 10%
from the month of addition to the month prior to disposal using reducing balance method. Control
accounts are not maintained for Debtors and Creditors.

Required:

© Emile Woolf International 95 The Institute of Chartered Accountants of Pakistan


Financial accounting and reporting I

Prepare journal entries to correct the above errors. (Narrations are not required), also compute the
corrected gross profit.
Answer:
Part (a) Sibi Brothers – General Journal
Date Particulars Dr. Rs. Cr. Rs.
(i) Suspense 18,000
Receivables [42,000 – 24,000] 18,000

(ii) Suspense 28,800


Purchases [32,000 – 23,000] 9,000
Payables [23,000 – 3,200] 19,800

(iii) Cost of sales 13,000


Inventory (closing) 13,000

(iv) Sales [94,000 – 49,000] 45,000


Suspense 45,000

(v) No entry. Trial balance is correct. However, correction in


financial statements is still required.
(vi) Suspense 8,200
Revenue/Discount allowed 4,100
Purchases/Discount received 4,100

(vii) PPE (Machine) 145,000


Purchases 145,000

Cost of sales 145,000


Inventory (closing) 145,000

Depreciation exp [145,000 x 10% x 3/12] 3,625


Accumulated depreciation 3,625

(viii) Acc. Dep. [incorrect exp (127,200 – 80,000) x 10%] 4,720


Depreciation exp 4,720

Depreciation exp [correct 127,200 x 10% x 8/12] 8,480


Accumulated depreciation 8,480

Acc. Dep. [incorrect credit] 80,000


Acc. Dep. [disposal 142,800 + 8,480] 151,280
Loss on disposal [balancing figure] 38,720
PPE (machine) 270,000

(i) Bank [outdated and next period cheque] 44,000


Payables [28,000 + 16,000] 44,000

(ii) Receivables 35,000


Bank [cheque related to next period] 35,000

(iii) Bank charges 3,100


Bank 3,100

Part (b) Corrected gross profit


Ref. Rs.
Given 854,000
Purchases (ii) 9,000
Inventory (closing) – Cost of sales impact (iii) (13,000)

© Emile Woolf International 96 The Institute of Chartered Accountants of Pakistan


Chapter 2: Correction of errors

Sales (iv) (45,000)


Return inwards overstated [99,000 – 82,000] (v) 17,000
Return outward understated [99,000 – 82,000] (v) 17,000
Revenue/Discount allowed (vi) 4,100
Purchases/Discount received (vi) 4,100
.
847,200

© Emile Woolf International 97 The Institute of Chartered Accountants of Pakistan


Financial accounting and reporting I

4 OBJECTIVE BASED QUESTIONS


01. Amna wrongly credited return inwards of Rs. 3,500 to returns outward although Receivable
Account was correctly credited.
What will be the difference in trial balance prepared at the year end?

(a) Total of trial balance on the debit side will be Rs. 3,500 more than total of credit side
(b) Total of trial balance on credit side will be Rs. 3,500 more than the total of debit side
(c) Total of trial balance on the debit side will be Rs. 7,000 more than total of credit side
(d) Total of trial balance on credit side will be Rs. 7,000 more than the total of debit side

02. For the year ended 31 December 2018 Ahmad showed a profit of Rs. 15,500.
It was further discovered; during the year he purchased a piece of equipment for Rs. 5,000.
Transaction was recorded as Debit Repairs account and Credit Cash account. It is policy to
depreciate equipment at 10% and charging full year’s deprecation in the year of purchase.
What is the impact of correcting the error on statement of profit or loss for the year?

(a) Profit will be increased by Rs. 5,000


(b) Profit will be reduced by Rs. 5,000
(c) Profit will be increased by Rs. 4,500
(d) Profit will be decreased by Rs. 4,500

03. Which of the following errors will require creating a suspense account?

(a) Repairs expense was considered a purchase of asset


(b) Purchase of inventory was considered purchase of non–current asset
(c) An invoice of Rs. 2,500 was totally omitted from the books
(d) Petty cash expenses of Rs. 500 were only credited to bank account

04. Zahid granted an early settlement discount of Rs. 1,500 to one of its customers. The discount
amount was correctly entered in the account receivable account but it was wrongly credited to
revenue account. The sales to this customer was originally recorded at gross amount and Zahid
does not maintain separate discount allowed account.
In order to balance the trial balance at year end, what should be the balance of suspense account
in trial balance?

(a) Rs. 3,000 Cr


(b) Rs. 3,000 Dr
(c) Rs. 1,500 Dr
(d) Rs. 1,500 Cr

05. The suspense account shows a debit balance of Rs. 500. Which of the following errors could be
the cause of suspense?

(a) Overstatement of salaries expense by Rs. 500


(b) Overcasting sales account by Rs. 500
(c) Under-casting of sales by Rs. 500
(d) Payment to supplier Rs. 500 was wrongly omitted (completely) from records

© Emile Woolf International 98 The Institute of Chartered Accountants of Pakistan


Chapter 2: Correction of errors

06. Interest expense of Rs. 100 has been wrongly debited to stationery expense. What entry is
required to correct the error?

(a) Dr Interest expense Rs. 100; Cr Suspense Account Rs. 100


(b) Dr Interest expense Rs. 100; Cr Stationery expense Rs. 100
(c) Dr Suspense Account Rs. 100; Dr Interest expense Rs. 100 Cr Stationery expense Rs.
200
(d) Dr Suspense Account Rs. 100, Cr Stationery expense Rs. 100

07. Which of the following errors will not affect a trial balance?

(a) Rs. 5,000 utility expenses were entirely omitted from recording
(b) Rent paid Rs. 2,500 has been recorded as Rs. 1,500 in rent account.
(c) Sales revenue account was under-casted by Rs. 10,000
(d) Cash paid Rs. 3,000 for repair of equipment was credited to repairs account

08. Which error will cause an entry in the suspense account?

(a) a transposition error when transferring a ledger account balance to the trial balance
(b) an error of commission where the wrong account is used for a transaction but it is the
correct type of account
(c) an error of omission
(d) an error of principle

09. The correction of which of the following error would require an entry in the suspense account?

(a) A cheque, Rs. 2,000, paid to Asif had been debited to Arif’s account.
(b) A purchase of stationery, Rs. 80, had been debited to the purchases account.
(c) Commission income, Rs. 120, had been debited to a loan interest account.
(d) Salaries account had been undercast by Rs. 300 and the entertainment account had
been overcast by Rs. 300.

10. A trial balance was extracted from the books of Nizam. It was found that debit side exceeded
credit side. Following errors were identified:
(i) Purchases account was over-casted by Rs. 120,000.
(ii) An amount paid to Sajjad was debited to his account as Rs. 98,000 instead of Rs. 89,000.
(iii) Sales account was under-casted by Rs. 11,000.
What was the balance of suspense account before correction of errors?

(a) Rs. 140,000 Credit


(b) Rs. 140,000 Debit
(c) 0
(d) Rs. 271,000 Debit

© Emile Woolf International 99 The Institute of Chartered Accountants of Pakistan


Financial accounting and reporting I

11. The credit side of a business trial balance is Rs. 2,000 more than the debit side. Which one of
the following could be the reason for that?

(a) Credit purchase of Rs. 6,000 was recorded in Purchase day book as Rs. 4,000
(b) Cash paid to supplier Rs. 2,000 was omitted from books
(c) Overpayment of Rs. 2,000 was received from a customer
(d) Credit sales of Rs. 7,000 was posted to account receivable control account as Rs. 5,000
while it was correctly entered in sales account

12. A suspense account was opened when a trial balance failed to agree. The following errors were
discovered afterwards:
(i) An electricity bill of Rs. 620 had been recorded in the electricity charges account as Rs.
250
(ii) A discount allowed of Rs. 200 was wrongly credited to revenue account
(iii) Interest given by bank Rs. 450 was debited to bank account only.
At what amount the suspense account should be shown in trial balance in order to make trail
balance agree?

(a) Rs. 320 Dr


(b) Rs. 320 Cr
(c) Rs. 120 Dr
(d) Rs. 120 Cr

13. Trial balance of a business did not agree and a suspense account was created. On investigation
it was revealed that while posting payments from bank book, insurance expense was posted as
Rs. 254,000 instead of Rs. 245,000.
What entry is required to correct the error?
(a) Dr Suspense Rs. 9,000 Cr Insurance expense Rs. 9,000
(b) Cr Suspense Rs. 9,000 Dr Insurance expense Rs. 9,000
(c) Dr Insurance expense Rs. 245,000 Cr Cash Rs. 245,000
(d) Cr Insurance expense Rs. 245,000 Dr Suspense Rs. 245,000

14. A sales return of Rs. 400 has been wrongly posted to the credit of the purchases return account,
but has been correctly entered in the customer’s account.
Which of the following will be the effects of the error?

(a) Profit for the year will be overstated by Rs. 400


(b) Profit for the year will be overstated by Rs. 800
(c) Credit side of the trial balance will be Rs. 400 more than debit side
(d) Debit side of the trial balance will be Rs. 800 more than credit side

15. A company’s trial balance failed to agree, the total being:

Debit Rs. 950,300


Credit Rs. 955,300

Which one of the following is the reason of this difference?

© Emile Woolf International 100 The Institute of Chartered Accountants of Pakistan


Chapter 2: Correction of errors

(a) Utilities expense of Rs. 5,000 entirely omitted from recording


(b) Transportation in of Rs. 5,000 is recorded as transportation out
(c) Purchase return Rs. 2,500 debited in error to the Sales return account, while transaction
was correctly posted to party’s account.
(d) Sales return Rs. 2,500 was wrongly recorded as Purchase return, while transaction was
correctly posted to party’s account.

16. Following errors occurred in books of Majid & Co.


(1) Carriage inwards of Rs. 100 was debited to carriage outwards
(2) Purchase of inventory Rs. 5,000 was debited to repairs account
(3) Rent expense was Rs. 200 and it was credited to rent account.
What will be the balance of Suspense account before correction of above errors?

(a) Rs. 100


(b) Rs. 200
(c) Rs. 400
(d) Rs. 5,400

17. A return inward of Rs. 180 has been wrongly recorded as carriage inwards and a repair expense
of Rs. 250 was wrongly debited to salaries account. What is the impact on net profit of the
correction of these errors?

(a) Rs. Nil


(b) Rs. 180 Decrease
(c) Rs. 250 Decrease
(d) Rs. 70 Decrease

18. A suspense account was opened when a trial balance failed to agree. The following errors were
discovered afterwards:
(i) A payment of Rs. 5,000 to a supplier was credited to his account
(ii) A return outwards of Rs. 400 was wrongly debited to return inwards account
(iii) Payment for establishment of petty cash fund by Rs. 1,000 was only credited to bank
account
At what amount the suspense account would have been created before correction of these
errors?

(a) Rs. 5,400 Credit


(b) Rs. 10,200 Debit
(c) Rs. 5,400 Debit
(d) Rs. 10,200 Credit

19. After extracting trial balance a business has identified following errors:
(i) Owner’s home rent paid Rs. 1,500 has been debited to business rent account
(ii) Purchase of stationery Rs. 500 has been debited to machinery account. Depreciation
rate is 10%

© Emile Woolf International 101 The Institute of Chartered Accountants of Pakistan


Financial accounting and reporting I

(iii) Freight paid Rs. 150 for inventory has been debited to stationery account
Profit for the year before correction of these errors was Rs. 10,500.
What is the amount of corrected profit?

(a) Rs. 11,500


(b) Rs. 11,550
(c) Rs. 11,600
(d) Rs. 11,750

20. A cash refund of Rs. 20,000 due to customer A was correctly treated in the cash book and then
credited to the accounts receivable ledger of customer B.
At what amount the suspense account should be shown in trial balance in order to make trial
balance agree?

(a) Rs. 20,000 debit


(b) Rs. 20,000 credit
(c) Rs. 40,000 debit
(d) Rs. 40,000 credit

21. “Treating a revenue expense as a capital expenditure" is an example of?

(a) Compensating errors


(b) Error of omission
(c) Error of commission
(d) Error of principle

22. If an effect of an error is cancelled by the effect of some other error, it is commonly known as
(a) Errors of principle
(b) Errors of omission
(c) Compensating errors
(d) Errors of commission

23. Goods of Rs.100,000 purchased from Abbas Traders were recorded in sales book, the
rectification of this error will
(a) Increase the gross profit
(b) Reduce the gross profit
(c) Have no effect on gross profit
(d) None of the given options

24. What would be the total of the trial balance if a purchase return of Rs.84,000 has been wrongly
posted to the debit of the sales return account, but had been correctly entered in the suppliers
account?

(a) The credit side to be Rs.84,000 more than debit side


(b) The debit side to be Rs.84,000 more than credit side
(c) The credit side to be Rs.168,000 more than debit side

© Emile Woolf International 102 The Institute of Chartered Accountants of Pakistan


Chapter 2: Correction of errors

(d) The debit side to be Rs.168,000 more than credit side

25. When opening stock is overstated, net profit for the accounting period will be

(a) Overstated
(b) Understated
(c) No effect
(d) None of the above

26. Difference of totals of both debit and credit side of the trial balance is transferred to

(a) Difference account


(b) Trading account
(c) Miscellaneous account
(d) Suspense account

27. Goods purchased from supplier worth Rs.200,000, no entry made in purchases book is an
example of

(a) Error of posting


(b) Error of omission
(c) Error of principle
(d) Compensating errors

28. Purchase of fuel for the car is capitalised to motor vehicles. It is a type of

(a) Error of posting


(b) Error of omission
(c) Error of principle
(d) Compensating errors

29. A company incorrectly recorded a credit sales invoice of Rs. 45,000 as Rs. 54,000. What is the
appropriate entry, the company should follow regarding the error of Rs. 9,000?

(a) Add Rs. 9,000 to its Cash account


(b) Sales (Dr) and Receivable (Cr) with Rs. 9,000
(c) Receivable (Dr) and Sales (Cr) with Rs. 9,000
(d) None of the above

30. Which of the following account(s) will be affected, while rectifying the error of Carriage paid Rs.
50,000 for the newly purchased machinery mistakenly debited to carriage account?

(a) Only machinery account


(b) Only carriage account
(c) Both carriage and machinery account
(d) Only cash account

© Emile Woolf International 103 The Institute of Chartered Accountants of Pakistan


Financial accounting and reporting I

31. If a cash sale is made for Rs. 400,000 and posted as follows:

Account Debit Credit


Sales 400,000
Cash 400,000
What would be the correcting entry?
(a) Sales (debit) = Rs. 800,000 and Cash (credit) = Rs. 800,000
(b) Cash (debit) = Rs. 400,000 and Sales (credit) = Rs. 400,000
(c) Cash (debit) = Rs. 600,000 and Sales (credit) = Rs. 600,000
(d) Cash (debit) = Rs. 800,000 and Sales (credit) = Rs. 800,000

32. Received cheque from debtor – Faraz worth Rs. 100,000 was treated as received from debtor-
Sarfaraz. What would be correcting entry?

(a) Sales (debit) = Rs. 100,000 and debtor - Faraz (credit) = Rs. 100,000
(b) Debtor - Faraz (debit) = Rs. 100,000 and debtor – Sarfaraz (credit) = Rs. 100,000
(c) Debtor - Sarfaraz (debit) = Rs. 100,000 and debtor –Faraz (credit) = Rs. 100,000
(d) Sales (debit) = Rs. 100,000 and debtor – Sarfaraz (credit) = Rs. 100,000

33. Which of the following errors will create balance in a suspense account?
(a) Repairs expense was considered as purchase of asset
(b) Purchase of inventory was considered as purchase of non-current asset
(c) An invoice of Rs. 2,500 was totally omitted from the books
(d) Petty cash expenses of Rs. 500 were only credited to bank account

34. Office supplies purchased (and held in stock) were mistakenly debited to Purchases account.
This type of error is called:
(a) error of omission
(b) compensating error
(c) error of principle
(d) error of transposition

35. While reviewing the draft financial statements of Sky Electronics (SE) for the year ended 31
December 2017, following error has been identified:
Computers costing Rs. 240,000 purchased on 1 September 2017 for office use were debited to
purchases account. SE depreciates computers at 20% per annum using straight line method.
What would be the impact of correcting the above error on property, plant and equipment as at
31 December 2017?

(a) Increase by Rs. 240,000


(b) Increase by Rs. 224,000
(c) Increase by Rs. 200,000
(d) Increase by Rs. 192,000

© Emile Woolf International 104 The Institute of Chartered Accountants of Pakistan


Chapter 2: Correction of errors

36. While reviewing the draft financial statements of Sky Electronics (SE) for the year ended 31
December 2017, following error has been identified:
Trade receivables include a balance of Rs. 180,000 which is irrecoverable but has not been
written-off. Further, a recovery of Rs. 96,000 against receivables written off in prior years was
credited to trade receivables. As per SE's policy, provision for doubtful receivables has already
been made at 5% on year-end balance.
If the profit before correcting the above was Rs. 800,000, what would be amount of corrected
profit?
(a) Rs. 716,000
(b) Rs. 720,200
(c) Rs. 711,800
(d) None of above

37. While reviewing the draft financial statements of Sky Electronics (SE) for the year ended 31
December 2017, following error has been identified:
A cheque of Rs. 192,000 was received after a discount of 4% from a customer. The related
revenue was recorded at gross amount. However, in the cash book, the amount received was
entered in the discount allowed column and the amount of discount was entered in the bank
column. SE does not maintain separate account for discount allowed and any revenue reversal
is directly posted to Sales account.
What would be the correcting entry for above?
(a) Debit Bank Rs. 192,000 and Credit Sales Rs. 192,000
(b) Debit Bank Rs. 184,000 and Credit Sales Rs. 184,000
(c) Debit Sales Rs. 192,000 and Credit Bank Rs. 192,000
(d) Debit Sales Rs. 184,000 and Credit Bank Rs. 184,000

38. Draft income statement of Timothy Enterprises (TE) for the year ended 31 December 2017 shows
gross profit of Rs. 850,000 and net profit of Rs. 460,000.
It was subsequently discovered that an invoice of Rs. 3,700 was debited to purchases but the
goods were received after year-end and were not included in the closing inventory.
Compute the corrected gross profit and net profit for the year.

(a) Gross Profit Rs. 846,300 & Net Profit Rs. 456,300
(b) Gross Profit Rs. 853,700 & Net Profit Rs. 463,700
(c) Gross Profit Rs. 850,000 & Net Profit Rs. 463,700
(d) Gross Profit Rs. 853,700 & Net Profit Rs. 460,000

39. Draft income statement of Timothy Enterprises (TE) for the year ended 31 December 2017 shows
gross profit of Rs. 850,000 and net profit of Rs. 460,000.
It was subsequently discovered that transportation inward amounting to Rs. 2,000 was included
in transportation outward.
Compute the corrected gross profit and net profit for the year.
(a) Gross Profit Rs. 848,000 & Net Profit Rs. 460,000
(b) Gross Profit Rs. 852,000 & Net Profit Rs. 462,000
(c) Gross Profit Rs. 848,000 & Net Profit Rs. 462,000
(d) Gross Profit Rs. 852,000 & Net Profit Rs. 458,000

© Emile Woolf International 105 The Institute of Chartered Accountants of Pakistan


Financial accounting and reporting I

40. Draft income statement of Timothy Enterprises (TE) for the year ended 31 December 2017 shows
gross profit of Rs. 850,000 and net profit of Rs. 460,000.
It was subsequently discovered that the sub-total of a closing stock sheet had been carried
forward as Rs. 21,830 instead of Rs. 21,380.
Compute the corrected gross profit and net profit for the year.
(a) Gross Profit Rs. 850,450 & Net Profit Rs. 460,450
(b) Gross Profit Rs. 849,550 & Net Profit Rs. 459,550
(c) Gross Profit Rs. 850,450 & Net Profit Rs. 459,550
(d) Gross Profit Rs. 849,550 & Net Profit Rs. 460,450

41. Draft income statement of Timothy Enterprises (TE) for the year ended 31 December 2017 shows
gross profit of Rs. 850,000 and net profit of Rs. 460,000.
It was subsequently discovered that a receipt of Rs. 24,000 was credited to sales. The amount
was received from a credit customer who availed a cash discount of Rs. 1,000 on this payment.
It was already expected that this customer will avail the cash discount at the time of sale.
Compute the corrected gross profit and net profit for the year.

(a) Gross Profit Rs. 826,000 & Net Profit Rs. 436,000
(b) Gross Profit Rs. 874,000 & Net Profit Rs. 484,000
(c) Gross Profit Rs. 825,000 & Net Profit Rs. 435,000
(d) Gross Profit Rs. 875,000 & Net Profit Rs. 485,000

42. Draft income statement of Timothy Enterprises (TE) for the year ended 31 December 2017 shows
gross profit of Rs. 850,000 and net profit of Rs. 460,000.
It was subsequently discovered that purchase of office computer on 1 April 2017 amounting to
Rs. 42,000 was entered in the purchase account. Depreciation on office computer is provided at
the rate of 25%.
Compute the corrected gross profit and net profit for the year.

(a) Gross Profit Rs. 892,000 & Net Profit Rs. 491,500
(b) Gross Profit Rs. 892,000 & Net Profit Rs. 494,125
(c) Gross Profit Rs. 808,000 & Net Profit Rs. 491,525
(d) Gross Profit Rs. 808,000 & Net Profit Rs. 428,500

43. Draft income statement of Timothy Enterprises (TE) for the year ended 31 December 2017 shows
gross profit of Rs. 850,000 and net profit of Rs. 460,000.
It was subsequently discovered that goods having sales value of Rs. 4,500 were used for office
repairs. No entry has been made in the books. TE uses periodic inventory method and goods
are sold at cost plus mark up of 25%.
Compute the corrected gross profit and net profit for the year.

(a) Gross Profit Rs. 853,600 & Net Profit Rs. 460,000
(b) Gross Profit Rs. 846,400 & Net Profit Rs. 464,500
(c) Gross Profit Rs. 854,500 & Net Profit Rs. 455,500
(d) Gross Profit Rs. 845,500 & Net Profit Rs. 459,100

© Emile Woolf International 106 The Institute of Chartered Accountants of Pakistan


Chapter 2: Correction of errors

44. Draft income statement of Timothy Enterprises (TE) for the year ended 31 December 2017 shows
gross profit of Rs. 850,000 and net profit of Rs. 460,000.
It was subsequently discovered that purchase return amounting to Rs. 6,700 has been recorded
as sales return.
Compute the corrected gross profit and net profit for the year.

(a) Gross Profit Rs. 856,700 & Net Profit Rs. 467,700
(b) Gross Profit Rs. 863,400 & Net Profit Rs. 473,400
(c) Gross Profit Rs. 843,300 & Net Profit Rs. 453,300
(d) Gross Profit Rs. 836,600 & Net Profit Rs. 446,600

45. Draft income statement of Timothy Enterprises (TE) for the year ended 31 December 2017 shows
gross profit of Rs. 850,000 and net profit of Rs. 460,000.
It was subsequently discovered that the owner had withdrawn goods costing Rs. 4,680 for
personal use. No entry has been made in the books. TE uses periodic inventory method and
goods are sold at cost plus mark up of 25%.
Compute the corrected gross profit and net profit for the year.

(a) Gross Profit Rs. 854,680 & Net Profit Rs. 464,680
(b) Gross Profit Rs. 855,850 & Net Profit Rs. 465,850
(c) Gross Profit Rs. 845,320 & Net Profit Rs. 455,320
(d) Gross Profit Rs. 844,150 & Net Profit Rs. 454,150

46. A cheque dated 25 June 2019 for Rs. 150,000 was received from an insurance company and
deposited by the owner in his personal bank account. The cheque was received in settlement of
an inventory loss claim. Actual inventory loss was determined at Rs. 180,000. No entries have
been made for loss of inventory and insurance claim. The business uses periodic inventory
method.
Which of the following effect would be part of correcting entry?

(a) Debit Abnormal loss Rs. 30,000


(b) Debit Drawings Rs. 150,000
(c) Credit Purchases Rs. 180,000
(d) All of above

47. Goods amounting to Rs. 350,000 received from a supplier after proper inspection on 30 June
2019 were included in the year-end physical inventory count but recorded in purchases day book
on 1 July 2019. The business uses periodic inventory method.
What would be appropriate correcting entry if the year-end is June 30, 2019?

(a) Trade payables Debit Rs. 350,000 and Purchases Rs. 350,000
(b) Drawings Debit Rs. 350,000 and Purchases Rs. 350,000
(c) Purchases Debit Rs. 350,000 and Trade payables Rs. 350,000
(d) None of above

48. Zeta Traders (ZT) has prepared its draft financial statements for the year ended on 30 June 2019.
On review it was discovered that third party stock of Rs. 500,000 lying on ZT premises has been
included in ZT’s year-end inventory. ZT uses periodic inventory method.

© Emile Woolf International 107 The Institute of Chartered Accountants of Pakistan


Financial accounting and reporting I

What correcting entry is required?

(a) No entry is required.


(b) Inventory (debit) Rs. 500,000 and Cost of Sales (credit) Rs. 500,000
(c) Third party (debit) Rs. 500,000 and Purchase return (credit) Rs. 500,000
(d) Cost of Sales (debit) Rs. 500,000 and Inventory (credit) Rs. 500,000

49. On 1 July 2021, a repair expense of Rs. 100,000 was debited to office machinery account.
Depreciation of 15% has been recorded at year end of 31 December 2021. What will be impact
of correcting above error on gross profit?

(a) Rs. Nil


(b) Rs. 100,000 Decrease
(c) Rs. 85,000 Decrease
(d) Rs. 92,500 Decrease

50. ‘Correct accounts, correct amounts but incorrect sides’, which of the following type of error is
indicated by preceding phrase?

(a) Error of commission


(b) Error of principle
(c) Error of original entry
(d) Complete reversal of entries

© Emile Woolf International 108 The Institute of Chartered Accountants of Pakistan


Chapter 2: Correction of errors

4 OBJECTIVE BASED ANSWERS


01. (d) Instead of debiting returns inwards; returns outwards have been credited. Hence the
effect of error is that credit side exceeds by double amount = Rs. 3,500x2 = Rs.
7,000
02. (c) Profit will be increased by Rs. 5,000 as repairs expense will be reversed.
Depreciation expense of Rs. 5,000x10%= Rs. 500 will be charged.
So the total net impact is Rs. 4,500 increase in profit.
03. (d) Petty cash transaction has not been debited anywhere. Suspense account is created
when debit and credit side of trial balance are not equal. This does not arise when
an entry is posted to incorrect account or totally omitted.
04. (b) Since the amount that should have been debited to revenue account has been
credited to revenue account; hence total of credit side exceeds debit side by double
amount i.e. Rs. 3,000.
05. (b) Sales are credit and over casting it is excess credit side (debit side short by Rs. 500),
so suspense account of Rs. 500 balance would be created on debit side.
06. (b) Interest expense was incorrectly not debited, it should be debited now.
Stationery expense was incorrectly debited; it should be credited now to reverse the
effect.
07. (a) There is no impact of complete omission of double entry on trial balance since
neither debit nor credit entry has been recorded.
08. (a) Any error while transferring balance of ledger account to trial balance would affect
one balance (and one side) only, and suspense account would be created.
09. (c) Commission income is credited but it was incorrectly debited to interest account,
creating a difference by double amount.
10. (a) Debit120,000+ Debit (98,000 - 89,000)+ Debit 11,000=Rs. 140,000
In all three corrections, the suspense account would be debited and it means, before
correction it had balance on credit side.
11. (d) Debit and credit sides of the entry are not equal hence suspense arises.

12. (a) (i) Dr side short (Rs. 620-250) =Rs. 370


(ii) Dr side short 200 x 2 =Rs. 400
(iii) Credit side short = Rs. 450
Total = Rs. 370 + Rs. 400 - Rs. 450= Debit side short by Rs. 320 (net).
13. (a) Insurance expense was overstated, and now to correct this, it is to be credited with
the difference of Rs. 9,000, and suspense account will be debited.

14. (b) Not recording sales return, increased the profit by Rs. 400 and Incorrect recording
of purchase returns, further increased the profit by Rs. 400 hence profit increased
by Rs. 800 in total due to error.
15. (d) Difference = Rs. 955,300 Credit – 950,300 Debit = Rs. 5,000 Debit side is shorter
The option (a) & (b) would not result in disagreement of trial balance. The option (c)
would create shorter credit side of trial balance.
Impact of recording sale return Rs. 2,500 as purchase return of Rs. 2,500 is that the
debit side is short by Rs. 5,000

16. (c) Error (1) and (2) do not affect trial balance

© Emile Woolf International 109 The Institute of Chartered Accountants of Pakistan


Financial accounting and reporting I

Error (3) is single side error and affects trial balance with double amount as rent
expense should have been debited and it was rather credited. Rs. 200 x 2 = Rs. 400
17. (a) The recording of return inwards (sales return) would have reduced the profit by Rs.
180, the same was effect of recoding carriage inwards (an expense), there is no
effect of correction of this error on profit. Same reason for the other transaction.
18. (b) (i) Double impact Rs. 10,000 debit side short
(ii) Double impact Rs. 800 credit side short
(iii) Rs. 1,000 debit side short
Net impact is = Rs. 10,000+1,000-800 = Rs. 10,200 debit

19. (b) Owner’s home rent is drawings not business expense, so expense shall be reversed.
Impact of asset related error would be net of depreciation. Debiting one expense
instead of other does not affect profit.
Corrected profit = Rs. 10,500 + 1,500 - (500 x 90%) = Rs. 11,550
20. (c) Effect on trial balance is double of the amount on debit side since amount has been
posted on wrong side (credit side).
21. (d) Error of principle: Correct amount but wrong type of account.
22. (c) Compensating errors: Two or more errors balance each other out.

23. (b) Correction would reduce revenue and increase purchases (and cost of sales), both
would reduce gross profit.

24. (d) Not recording purchase return (credit side short by Rs. 84,000)
Recording incorrect sales return (credit side short by further Rs. 84,000)
25. (b) Overstatement of opening stock would lead to overstatement of cost of sales
resulting in understatement of gross profit and net profit.
26. (d) Suspense account is created temporarily for the difference in trial balance.

27. (b) Error of omission: Entry missed from the accounting records completely.
28. (c) Error of principle: Correct amount but wrong type of account.

29. (b) Receivables and sales both were overstated by Rs. 9,000.
30. (c) Machinery account shall be debited and carriage account shall be credited to correct
the error.
31. (d) The error was complete reversal of entry. It has to be rectified by recording the
double amount now.
32. (c) Receipt of cheque from customer does not affect sales. The relevant customer
account should be credited for cheque received.
33. (d) Petty cash expenses of Rs. 500 were only credited to bank account, no account was
debited that would cause trial balance to disagree resulting in suspense account.
34. (c) Error of principle: Correct amount but wrong type of account.

35. (b) Recording of computer would increase PPE by Rs. 240,000


Then depreciation of Rs. 16,000 (Rs. 240,000 x 20% x 4/12) would decrease PPE
by year-end.
The net increase would be by Rs. 224,000
36. (b) Bad debts expense Rs. 180,000 would decrease profit.
Recovery of bad debts Rs. 6,000 would increase profit.

© Emile Woolf International 110 The Institute of Chartered Accountants of Pakistan


Chapter 2: Correction of errors

Receivables would decrease by Rs. 84,000 due to above two corrections (i.e.
credited by 180,000 and debited by Rs. 96,000)
Decrease in allowance for doubtful debts will increase profit by Rs. 4,200 (i.e. Rs.
84,000 x 5%).
Corrected profit = Rs. 800,000 – 180,000 + 96,000 + 4,200 = Rs. 720,200
37. (b) The revenue reversal should have been Rs. 8,000 (i.e. Rs. 192,000 x 4/96). Instead
it was recorded at Rs. 192,000 (overstating the debit to revenue by Rs. 184,000).
Similarly, the bank should have been debited by Rs. 192,000 but it was debited by
only Rs. 8,000, causing understatement of Rs. 184,000.
38. (b) Reversal of purchase would increase gross profit by Rs. 3,700 (same effect will carry
to net profit).
39. (a) Recording of transportation inwards would increase cost of sales, hence, decreasing
gross profit by Rs. 2,000. However, there would be nil impact on net profit (increase
in transportation in expense and same decrease in transportation out expense).
40. (b) Decreasing the closing stock would increase cost of sales, resulting in decrease of
gross profit by Rs. 450. The effect of gross profit shall carry to net profit as well.
41. (a) The receivable should have been credited by Rs. 24,000 and not sales. Therefore,
now sales shall be debited resulting in decrease in gross profit and net profit. No
impact of discount allowed is required as sales would have already been recorded
at net amount.
42. (b) Purchases shall be reduced by Rs. 42,000 (impact on GP and NP both).
Depreciation of Rs. 7,875 (i.e. Rs. 42,000 x 25% x 9/12) would affect net profit only.

43. (a) Correcting entry would be Repairs debit and Purchases credit by Rs. 3,600 (i.e. Rs.
4,500 / 125%). Gross profit would increase but there would be net impact on net
profit of Rs. Nil.
44. (b) Recording of purchase return would increase GP and NP by Rs. 6,700. Also,
reversal of sales return would increase GP and NP by further Rs. 6,700.
45. (a) The correcting entry would be Drawings debit and Purchases credit by Rs. 4,680.
The reduction in purchase would increase gross profit and net profit. The drawings
will decrease owner’s capital but would not affect profit.
46. (d) The option (a) to (c) reflect complete correcting entry.
47. (c) The control of goods was taken on June 30, 2019, therefore, purchase should be
recorded in year 2019 and not in next year. Purchases Debit Rs. 350,000 and Trade
payables Rs. 350,000
48. (d) Third party stock is not stock of the entity. Simply record the reversal of inventory
adjustment. Cost of Sales (debit) Rs. 500,000 and Inventory (credit) Rs. 500,000
49. (a) Repair expense and depreciation of office machinery will affect net profit but it has
no effect on gross profit.
50. (d) Complete reversal of entries: Correct accounts and amounts but sides (debit &
credit) reversed.

© Emile Woolf International 111 The Institute of Chartered Accountants of Pakistan


Financial accounting and reporting I

© Emile Woolf International 112 The Institute of Chartered Accountants of Pakistan


Certificate in accounting and Finance

3
Financial accounting and reporting I

CHAPTER
Non-current assets: sundry standards

Contents
1 IAS 20: Accounting for government grants and disclosure of
government assistance
2 IAS 23: Borrowing costs
3 IAS 40: Investment property
4 Objective based questions and answers

* The student must refer original handbook of IFRS.

© Emile Woolf International 113 The Institute of Chartered Accountants of Pakistan


Financial accounting and reporting I

1 IAS 20: ACCOUNTING FOR GOVERNMENT GRANTS AND DISCLOSURE OF


GOVERNMENT ASSISTANCE

Section overview

 Introduction and definitions


 Accounting treatment of government grants
 Repayment of government grants
 Government assistance
 Disclosure requirements

1.1 Introduction and definitions


In many countries the government provides financial assistance to industry. The most common
form of such assistance is a grant of cash from local or national government.

Definitions

Government assistance is action by government designed to provide an economic benefit specific


to an entity or range of entities qualifying under certain criteria. Government assistance does not
include benefits provided only indirectly through action affecting general trading conditions, such
as the provision of infrastructure in development areas or the imposition of trading constraints on
competitors.

Government grants are assistance by government in the form of transfers of resources to an entity
in return for past or future compliance with certain conditions relating to the operating activities of
the entity. They exclude those forms of government assistance which cannot reasonably have a
value placed upon them and transactions with government which cannot be distinguished from the
normal trading transactions of the entity.

Forgivable Loan is treated as a government grant when there is reasonable assurance that the
entity will meet the terms for forgiveness of the loan.

Low interest loans are loans which the government provides at lower interest rate as compared to
market interest rate.

IAS 20 Accounting for Government Grants and Disclosure of Government Assistance identifies two
types of government grants:
 grants related to assets, or
 Grants related to income.

Definitions

Grants related to assets are government grants whose primary condition is that an entity qualifying
for them should purchase, construct or otherwise acquire long-term assets. Subsidiary conditions
may also be attached restricting the type or location of the assets or the periods during which they
are to be acquired or held.

Grants related to income are government grants other than those related to assets.

Government grants are sometimes called by other names such as subsidies, subventions, or
premiums.

© Emile Woolf International 114 The Institute of Chartered Accountants of Pakistan


Chapter 3: Non-current assets: sundry standards

1.2 Accounting treatment of government grants


IAS 20 states that grants should not be recognized until there is reasonable assurance that:
 the entity will comply with any conditions attaching to the grant, and
 The grant will be received.
Once these recognition criteria are met, the grants should be recognized in profit or loss over the
periods necessary to match them with their related costs.
Neither type of grant should be credited directly to shareholders’ interests in the statement of
financial position. They must be reported on a systematic basis through the statement of profit or
loss (profit or loss).
Grants related to income
For grants related to income, IAS 20 states that an ‘income approach’ should be used, and the
grant should be taken to income over the periods necessary to match the grant with the costs that
the grant is intended to compensate.
IAS 20 allows two methods of doing this:
 Method 1. Include the grant for the period as ‘other income’ for inclusion in profit or loss
for the period
 Method 2. Deduct the grant for the period from the related expense.

Illustration 01: Grant related to income


A company receives a cash grant of Rs. 30,000 on 31 December Year 0.
The grant is towards the cost of training young apprentices, and the training programme is expected
to last for 18 months from 1 January Year 1.
Actual costs of the training were Rs. 50,000 in Year 1 and Rs. 25,000 in Year 2.
The grant would be accounted for as follows:
At 31 December Year 0 the grant would be recognized as a liability and presented in the statement
of financial position split between current and non-current amounts. Rs. 20,000 (12 months/18
months Rs. 30,000) is current and would be recognized in profit for Year 1. The balance is non-
current.
At the end of year 1 there would be a current balance of Rs. 10,000 (being the non-current balance
at the end of Year 0 reclassified as current) in the statement of financial position. This would be
recognized in profit in Year 2.
Extracts from the financial statements are as follows:
Statement of financial position (extracts)
31 31 31
December December December
Year 0 Year 1 Year 2
Current liabilities
Deferred income 20,000 10,000 
Non-current liabilities
Deferred income 10,000  

Statement of profit or loss (extracts)


Method 1
Training costs (50,000) (25,000)
Government grant received 20,000 10,000

© Emile Woolf International 115 The Institute of Chartered Accountants of Pakistan


Financial accounting and reporting I

31 31 31
December December December
Year 0 Year 1 Year 2
Method 2
Training costs (50,000 – 20,000) 30,000
Training costs (25,000 – 10,000) 15,000

Grants related to assets


For grants related to assets, IAS 20 allows two methods of doing this:
 Method 1. Deduct the grant from the cost of the related asset. The asset is included in the
statement of financial position at cost minus the grant. Depreciate the net amount over the
useful life of the asset.
 Method 2. Treat the grant as deferred income and recognize it as income on a systematic
basis over the useful life of the asset.
Both methods achieve the same effective result.

Illustration 02: Grant related to an asset


A company receives a government grant of Rs. 400,000 towards the cost of an asset with a cost of
Rs. 1,000,000.
The asset has an estimated useful life of 10 years and no residual value.
The amounts could be reflected in the financial statements prepared at the end of Year 1 in
accordance with IAS 20 in the following ways:
Method 1:
Statement of financial position (extract)
Property, plant and equipment Rs.
Cost (1,000,000 – 400,000) 600,000
Accumulated depreciation (60,000)
Carrying amount 540,000
Included in statement of profit or loss (extract)
Depreciation charge (Rs. 600,000/10 years) 60,000
Method 2:
Statement of financial position (extract)
Property, plant and equipment Rs.
Cost 1,000,000
Accumulated depreciation (100,000)
Carrying amount 900,000
Current liabilities
Deferred income 40,000
Non-current liabilities
Deferred income 320,000
At the end of year 1 there would be Rs. 360,000 of the grant left to recognise in profit in the
future at Rs. 40,000 per annum. Rs. 40,000 would be recognised in the next year and is
therefore current. The balance is non-current

© Emile Woolf International 116 The Institute of Chartered Accountants of Pakistan


Chapter 3: Non-current assets: sundry standards

Included in statement of profit or loss (extract) Rs.


Expense: Depreciation charge (Rs. 1,000,000/10 years) (100,000)
Income: Government grant (Rs. 400,000/10 years) 40,000

Illustration 03:
On January Year 1 Entity O purchased a non-current asset with a cost of Rs. 500,000 and received a
grant of Rs. 100,000 in relation to that asset.
The asset is being depreciated on a straight-line basis over five years.
The grant would be reflected in the financial statements at the end of the first year under both
methods of accounting for the grant allowed by IAS 20 are as under:
The amounts could be reflected in the financial statements prepared at the end of Year 1 in
accordance with IAS 20 in the following ways:

Method 1:
Statement of financial position
Property, plant and equipment Rs.
Cost (500,000 – 100,000) 400,000
Accumulated depreciation (80,000)
Carrying amount 320,000
Included in statement of profit or loss
Depreciation charge (Rs.400,000/5 years) 80,000
Method 2:
Statement of financial position
Property, plant and equipment Rs.
Cost 500,000
Accumulated depreciation (100,000)
Carrying amount 400,000
Current liabilities
Deferred income 20,000
Non-current liabilities
Deferred income 60,000
At the end of year 1 there would be Rs. 80,000 of the grant left to recognise in profit in
the future at Rs. 20,000 per annum. Rs. 20,000 would be recognised in the next year
and is therefore current. The balance is non-current.
Included in statement of profit or loss Rs.
Expense: Depreciation charge (Rs. 500,000/5 years) (100,000)

Income: Government grant (Rs. 100,000/5 years) 20,000

© Emile Woolf International 117 The Institute of Chartered Accountants of Pakistan


Financial accounting and reporting I

Grants as compensation for expenses or losses


A government grant may be awarded for the purpose of giving immediate financial support to an
entity rather than as an incentive to undertake specific expenditures.
The circumstances under which the grant is made may warrant immediate recognition of the grant
in profit or loss in the period in which the entity qualifies to receive it with disclosure to ensure that
its effect is clearly understood.
A government grant may become receivable by an entity as compensation for expenses or losses
incurred in a previous period. Such a grant is recognized in profit or loss of the period in which it
becomes receivable, with disclosure to ensure that its effect is clearly understood.

1.3 Repayment of government grants


A government grant might become repayable by the entity (e.g. when the entity fails to meet the
underlying conditions for the grant).
When a government grant becomes repayable it is accounted for as a change in accounting
estimate (IAS 8: Accounting policies, changes in accounting estimates and errors).
Repayment of a grant related to income
Repayment of a grant related to income is applied in the first instance against any unamortized
deferred credit recognized in respect of the grant. If the repayment exceeds any such deferred
credit any excess is recognized immediately in profit or loss.

Illustration 04: Repayment of grant related to income


On 1 January Year 1 X Limited received a cash grant of Rs. 500,000 towards the cost of employing
an environmental impact analyst on a new project for a 5 year period.
The grant is repayable in full if the project is not completed.
The analyst was employed and the project commenced from the 1 January Year 1.
On 1 January Year 3 the project was abandoned and the grant became repayable in full.
The grant and its subsequent repayment is accounted for as follows:

Year 1 Debit Credit


Cash 500,000
Deferred income 500,000
Statement of profit or loss
(reduction of employment cost) 100,000
Deferred income 100,000
Year 2
Statement of profit or loss
(reduction of employment cost) 100,000
Deferred income 100,000
The double entry to reflect the repayment is as follows
Deferred income 300,000
Statement of profit or loss 200,000
Cash 500,000

Repayment of a grant related to an asset


Accounting for a repayment of a grant related to an asset depends on how the grant was accounted
for originally.

© Emile Woolf International 118 The Institute of Chartered Accountants of Pakistan


Chapter 3: Non-current assets: sundry standards

If the grant was accounted for as reduction of the carrying amount of the related asset, its
repayment is recognized by increasing the carrying amount of the asset.
If the grant was accounted for as deferred income, its repayment is recognized by reducing the
deferred income balance by the amount repayable.
The cumulative additional depreciation that would have been recognized in profit or loss to date in
the absence of the grant must be recognized immediately in profit or loss.
Also note that the circumstances giving rise to repayment of the grant might indicate the possible
impairment of the new carrying amount of the asset.

1.4 Government assistance


The definition of government grants (see earlier) excludes:
 certain forms of government assistance which cannot reasonably have a value placed
upon them (e.g. free advice and the provision of guarantees); and
 Transactions with government which cannot be distinguished from the normal trading
transactions of the entity (e.g. a government procurement policy that is responsible for a
portion of the entity’s sales).
There is no specified accounting treatment for either of these. However, the significance of the
benefit may require disclosure of the nature, extent and duration of the assistance in order to
prevent the financial statements being misleading.
Forgivable Loans
A forgivable loan is treated as a government grant when there is reasonable assurance that the
entity will meet the terms for forgiveness of the loan. Until then, such a loan is treated as a liability
and recognised at fair value under IFRS 9 (Financial Instruments).

Illustration 05:
ABC Pharmaceutical Company received cash from government for a research and development
project of a children vaccine.
Scenario 1
As per the terms of the loan, the cash received from the government shall be waived off if the entity
is able to develop the vaccine within 3 years and sell it free of cost for 5 years.
If the entity takes more time than three years in the development or sells the vaccine for a price
before 5 years, it will be liable to repay the loan and the loan will not be considered a forgivable
loan.
Scenario 2
As per the terms of the loan, the cash received from the government is repayable in cash only if
the entity decides to commercialize the results of the research phase of the project. If the entity
decides not to commercialize the results of the research phase, the cash received is not repayable
in cash, but instead the entity must transfer to the government the rights to the research.
In this scenario, cash received from the government does not meet the definition of a forgivable
loan in IAS 20. This is because, in this arrangement, the government does not undertake to waive
repayment of the loan, but rather to require settlement in cash or by transfer of the rights to the
research. the cash receipt described in the submission gives rise to a financial liability to be dealt
with under IFRS 9.

Loans with Below-Market Rate of Interest


 The benefit of a government loan at a below market rate of interest is treated as a
government grant. The loan shall be recognized and measured as per IFRS 9.

© Emile Woolf International 119 The Institute of Chartered Accountants of Pakistan


Financial accounting and reporting I

 The benefit of below market rate of interest shall be measured as the difference
between the cash receipt under a government loan and the fair value of the liability the
benefit will be accounted for IAS 20.
 The entity shall consider the conditions and obligations to be met when identifying the
costs which the benefit of the loan is intended to compensate,
Non-monetary government grants
 A government grant may take the form of a transfer of a non-monetary asset, such as land
or other resources, for the use of the entity. In such case, the fair value of the non-
monetary asset should be assessed and to account for both grant and asset at that fair
value.
 Alternatively, both asset and grant may be recorded at a nominal amount.

1.5 Disclosure requirements


IAS 20 requires the following disclosures in the notes to the financial statements:
 the accounting policy adopted for government grants, including the method of presentation
in the financial statements
 The nature and extent of government grants recognized in the financial statements and an
indication of other forms of government assistance from which the entity has directly
benefitted.
 Unfulfilled conditions and other contingencies attaching to government assistance (if this
assistance has been recognized in the financial statements).
Government assistance may be significant so that disclosure of the nature, extent and duration of
the assistance is necessary in order that the financial statements may not be misleading.

Example 01: Accounting treatment


Question: Discuss how the following should be dealt with in the financial statements of relevant
entities according to IAS 20 Accounting for Government Grants and Disclosure of Government
Assistance:
(a) The government makes a grant to an entity which is planning to develop teaching software for
children with learning difficulties. The purpose of the grant is to help the entity to meet its
general financing requirement in the initial phase. There are no further conditions attached to
the grant.
(b) A manufacturing entity sets up a plant in an area of high unemployment. A government grant
of Rs. 4 million is received with a condition that the grant is repayable in full if the number of its
employees fell below 100 at any time during the next four years. It is highly probable that the
entity will comply with the condition attached to the grant.
(c) Free technical advice has been provided by the government’s export promotion department to
help an exporter to market his new technology in North America.
Answer:
Part (a)
The grant has been provided for the purpose of giving immediate financial support to the entity with
no further conditions, so this grant should be immediately recognised in profit or loss in full in the
period in which the entity qualifies to receive it (when it is receivable) with disclosure to ensure that
its effect is clearly understood.
Part (b)
Since there is reasonable assurance that conditions attaching to the grant will be met, the grant is
recognised in statement of profit or loss over the four year period in which the entity incurs the costs
of employing 100 people. Amount taken to statement of profit or loss may be either be presented

© Emile Woolf International 120 The Institute of Chartered Accountants of Pakistan


Chapter 3: Non-current assets: sundry standards

as other income or shown as deduction from the related expense. The remaining amount of grant
will be presented as deferred income under liabilities in the balance sheet.
Part (c)
Free technical advice is government assistance that cannot reasonably have a value placed upon it
and therefore should not be recognised. However, an indication of such assistance should be
disclosed in financial statements.

Example 02: KATIE


Question: During the year ended 30 June Year 2, Katie received three grants, the details of which
are set out below.
(1) On 1 September, a grant of Rs. 40,000 from local government. This grant was in respect of
training costs of Rs. 70,000 which Katie had incurred.
(2) On 1 November Katie bought a machine for Rs. 350,000. A grant of Rs. 100,000 was received
from central government in respect of this purchase. The machine, which has a residual value
of Rs. 50,000, is depreciated on a straight-line basis over its useful life of five years.
(3) On 1 June a grant of Rs. 100,000 from local government. This grant was in respect of
relocation costs that Katie had incurred moving part of its business from outside the local
area. The grant is repayable in full unless Katie recruits ten employees locally by the end of
Year 2. Katie is finding it difficult to recruit as the local skill base does not match the needs
of this part of the business.
Required:
Show how the above transactions should be reflected in the financial statements of Katie for the
year ended 30 June Year 2. Where any accounting standards allow a choice you should show all
possible options.
Answer:

The above transactions should be reflected in the financial statements of Katie for the year ended
30 June Year 2 based on IAS 20 as under:
Option 1 – Net grants off related expenditure
Statement of financial position as at 30 June Year 2 (extracts)

Rs.
Non-current assets
Property, plant and equipment 223,333
Current liabilities
Other current liabilities 100,000

Notes to the financial statements for the year ended 30 June Year 2 (extracts)

Property, plant and equipment Rs.


Cost (350,000 – 100,000) 250,000
Accumulated depreciation ((250,000 – 50,000) ÷ 5  8/12) (26,667)
Carrying amount 223,333

Included in statement of profit or loss for the year ended 30 June Year 2

Rs.
Depreciation charge 26,667
Training costs (70,000 – 40,000) 30,000

© Emile Woolf International 121 The Institute of Chartered Accountants of Pakistan


Financial accounting and reporting I

Option 2 – Show grants separately from related expenditure


Statement of financial position as at 30 June Year 2 (extracts)

Rs.
Non-current assets
Property, plant and equipment 310,000

Current liabilities
Other current liabilities 186,667

Notes to the financial statements for the year ended 30 June Year 2 (extracts)

Rs.
Property, plant and equipment
Cost 350,000
Accumulated depreciation ((350,000 – 50,000) ÷ 5  8/12) (40,000)
Carrying amount 310,000
Other current liabilities
Deferred income relating to government grants 86,667
(100,000 - (100,000 ÷ 5  8/12))
Government grant repayable 100,000
186,667

Included in statement of profit or loss for the year ended 30 June Year 2

Rs.

Depreciation charge 40,000

Training costs 70,000

Government grant received (40,000)

Release of deferred government grant (13,333)

Tutorial note
The Rs. 100,000 grant in (3) has conditions attached to it. In such a situation, IAS 20 states that
grants should not be recognised until there is reasonable assurance that the entity will comply with
any conditions attaching to the grant. Since Katie is struggling to recruit, and there is only one month
left for recruitment to meet these conditions, then it does not seem that there is ‘reasonable
assurance’. Hence the grant should not be recognised as such, but should be held in current
liabilities, pending repayment.

© Emile Woolf International 122 The Institute of Chartered Accountants of Pakistan


Chapter 3: Non-current assets: sundry standards

2 IAS 23: BORROWING COSTS

Section overview

 Introduction
 Borrowing costs eligible for capitalisation
 Period of capitalisation
 Disclosures

2.1 Introduction
A company might incur significant interest costs if it has to raise a loan to finance the purchase or
construction of an asset. IAS 23: Borrowing costs defines borrowing costs and sets guidance on
the circumstances under which they are to be capitalised as part of the cost of qualifying assets.

Definition: Borrowing costs


Borrowing costs are interest and other costs that an entity incurs in connection with the borrowing
of funds.

Definition: Qualifying asset


A qualifying asset is an asset that necessarily takes a substantial period of time to get ready for
its intended use or sale.

Any of the following may be qualifying assets depending on circumstances:


 inventories;
 items of property, plant and equipment;
 Intangible assets.
The following are not qualifying assets:
 inventories that are manufactured, or otherwise produced, over a short period of time, are
not qualifying assets
 Assets that is ready for their intended use or sale when acquired.
 Qualifying assets are usually self-constructed non-current assets.

2.2 Borrowing costs eligible for capitalisation


Borrowing costs that are directly attributable to the acquisition, construction or production of a
qualifying asset must be capitalised as part of the cost of that asset. All other borrowing costs are
recognized as an expense in the period in which they are incurred.
Borrowing costs that are directly attributable to the acquisition, construction or production of a
qualifying asset are those that would have been avoided if the expenditure on the qualifying asset
had not been made.
This includes the costs associated with specific loans taken to fund the production or purchase of
an asset and general borrowings. General borrowings are included because if an asset was not
being constructed it stands to reason that there would have been a lower need for cash.

© Emile Woolf International 123 The Institute of Chartered Accountants of Pakistan


Financial accounting and reporting I

Funds specifically borrowed to obtain a qualifying asset


When a specific loan is taken in order to obtain a qualifying asset the borrowing costs eligible for
capitalization are the actual borrowing costs incurred on that borrowing during the period less any
investment income on the temporary investment of those borrowings.

Illustration 06: Specific borrowings


On 1 January 2016 Okara Engineering issued a bond to raise Rs. 25,000,000 to fund a capital
project which will take three years to complete. Amounts not yet needed for the project are invested
on a temporary basis. During the year to 31 December 2016, Okara Engineering spent Rs.
9,000,000 on the project.
The cost of servicing the bond was Rs. 1,250,000 during this period and the company was able to
earn Rs. 780,000 through the temporary reinvestment of the amount borrowed.
The amounts recognized as capital work in progress in the period was:

Rs.
Costs incurred (labour, material, overhead etc.) 9,000,000
Interest capitalised:
Actual interest cost 1,250,000
Less: return on temporary investment (780,000)
470,000
Additions to capital work in progress 9,470,000

General funds used for the purpose of obtaining a qualifying asset.


When general borrowings are used the amount of borrowing costs eligible for capitalisation is
obtained by applying a capitalisation rate to the expenditures on that asset.
The capitalisation rate is the weighted average of the borrowing costs applicable to the borrowings
that are outstanding during the period except for borrowings made specifically for the purpose of
obtaining a qualifying asset.
The amount of borrowing costs capitalised cannot exceed the amount of borrowing costs it incurred
during a period.

Illustration 07: General borrowings: Capitalisation rate


Sahiwal Construction has three sources of borrowing:

Average loan in the year Interest expense incurred in


(Rs.) the year (Rs.)
7 year loan 8,000,000 800,000
10 year loan 10,000,000 900,000
Bank overdraft 5,000,000 900,000
The 7 year loan has been specifically raised to fund the building of a qualifying asset.
A suitable capitalisation rate for other projects is found as follows:
General borrowings Average loan in the year Interest expense incurred in
(Rs.) the year (Rs.)
10 year loan 10,000,000 900,000
Bank overdraft 5,000,000 900,000
15,000,000 1,800,000
Capitalisation rate = 1,800,000/15,000,000 100 = 12%

© Emile Woolf International 124 The Institute of Chartered Accountants of Pakistan


Chapter 3: Non-current assets: sundry standards

Alternatively:
Rate on 10 year loan = 900,000/10,000,000 100 = 9%
Rate on bank overdraft = 900,000/5,000,000 100 = 18%
Weighted average: 9% 10,000,000/15,000,000 + 18% 5,000,000/15,000,000
6% + 6% = 12%

The capitalisation rate is applied from the time expenditure on the asset is incurred.

Illustration 08: General borrowings: Capitalisation rate


Continuing the illustration above, Sahiwal Construction has incurred the following expenditure on
a project funded from general borrowings for year ended 31 December 2016.

Date incurred: Amount (Rs.)


31st March 1,000,000
31st July 1,200,000
30th October 800,000

The amount capitalised in respect of capital work in progress during 2016 is as follows:

Rs.
31st March  Expenditure 1,000,000
Interest (1,000,000  12% 9/12) 90,000

31st July  Expenditure 1,200,000


Interest (1,200,000  12% 5/12) 60,000

30th October  Expenditure 800,000


Interest (800,000  12% 2/12) 16,000
3,166,000

2.3 Period of capitalisation


Commencement of capitalisation
Capitalisation of borrowing costs should start only when:
 expenditures for the asset are being incurred; and
 borrowing costs are being incurred, and
 Activities necessary to prepare the asset have started.
Suspension of capitalisation
Capitalisation of borrowing costs should be suspended if development of the asset is suspended
for an extended period of time.
Cessation of capitalisation
Capitalisation of borrowing costs should cease when the asset is substantially complete. The costs
that have already been capitalised remain as a part of the asset’s cost, but no additional borrowing
costs may be capitalised.

© Emile Woolf International 125 The Institute of Chartered Accountants of Pakistan


Financial accounting and reporting I

Illustration 09: Commencement and suspension of capitalisation


Company A borrowed Rs. 9,000 @ 15% per annum to fund a project on 1st Jan 2016.
The following expenditures were made on the project during the year ending 31 December 2016
Date: 1st March 2016: Rs. 2,500
Date: 1st Oct 2016: Rs. 4,200
Date: 1st Dec 2016: Rs. 2,300
Surplus funds were invested @10% whenever available. The project commenced on 1st March
2016. Work on the project was suspended during the whole month of August and resumed on 1st
September. Construction was completed on 31 December 2016.
The relevant calculation of borrowing costs to be capitalised is as under:
Borrowing costs to be capitalised Rs.
Borrowing costs incurred 9,000 x 15% x 9/12 1,012.5
Less: Temporary investment income 6,500 x 10% x 6/12 (325)
2,300 x 10% x 2/12 (38.3)
649.2

The project commenced on 1st March resulting in a period of 10 months up to the year end.
However, interest cannot be capitalised during the period of suspension. Therefore, interest is
capitalised only for 9 months.

Borrowing costs to be charged as expense Rs.


Borrowing costs incurred 9,000 x 15% x 3/12 337.5
Less: Temp. Inv. income 9,000 x 10% x 2/12 (before commencement) (150)
6,500 x 10% x 1/12 (suspension) (54.17)
133.33
.

2.4 Disclosures
IAS 23 requires disclosure of the following:
 the amount of borrowing costs capitalised during the period; and
 The capitalisation rate used to determine the amount of borrowing costs eligible for
capitalisation.

Example 03: SHAYAN LIMITED


Question: Shayan Limited (SL) started the construction of its new factory on 1 January 2018 with
a loan of Rs. 50,000,000 borrowed at an interest rate of 8% per annum.
The loan was used on the factory as follows:

Date of Payment Rs. in million


Jan 1, 2018 25
May 1, 2018 15
Oct 1, 2018 10

The construction of the asset was completed on 31 December 2018. However, during the
accounting period SL invested the surplus funds at an interest rate of 3%.
Required:
How much the amount of borrowing cost eligible for capitalization at 31.12.2018?

© Emile Woolf International 126 The Institute of Chartered Accountants of Pakistan


Chapter 3: Non-current assets: sundry standards

Answer:
Borrowing costs to be capitalised Rs.
Borrowing costs incurred Rs. 50m x 8% 4,000,000
Less: Temporary investment income Rs. 25m x 3% x 4/12 (250,000)
Rs. 10m x 3% x 5/12 (125,000)
3,625,000

Example 04: SARA LIMITED


Question: On January 1, 2018 Sara Limited (SL) started the construction of an asset. To meet the
financing requirements, borrowing was made from three different banks at the start of the year
as follows:
Banks Amount (Rs.) Interest Rate per annum
A 70,000 10%
B 60,000 8%
C 50,000 12%

The funds were used on the assets as follows:


Date of Payment Rs.
Jan 1, 2018 30,000
May 1, 2018 20,000
Oct 1, 2018 15,000

The construction of asset was completed on 31 December 2018.


Required:
Calculate the general weighted average borrowing rate and eligible borrowing cost.
Answer:
Borrowing costs eligible for capitalisation Rs.
30,000 x 9.89% x 12/12 2,967
20,000 x 9.89% x 8/12 1,318
15,000 x 9.89% x 3/12 371
Borrowing costs to be capitalised 4,656

(70,000 x 10%) + (60,000 x 8%) + (50,000 x 12%)


Capitalisation rate = X 100 = 9.89%
70,000 + 60,000 + 50,000

Example 05: LOONEY


Question: Looney has recently finished building a new item of plant for its own use. The item is a
press for use in the manufacture of industrial diamonds. Looney commenced construction of the
asset on 1st April 2013 and completed it on 1st April 2015.
1st January 2013, Looney took out a loan to finance the construction of the asset. Interest is
charged on the loan at the rate of 5% per annum. The annual interest must be paid in four equal
instalments at the end of each quarter. Looney capitalises interest on manufactured assets in
accordance with the rules in IAS 23 Borrowing costs.
The costs (excluding finance costs) of manufacturing the asset were Rs. 28 million.

© Emile Woolf International 127 The Institute of Chartered Accountants of Pakistan


Financial accounting and reporting I

Required:
State the IAS 23 rules on the capitalisation of borrowing costs, calculate the cost of the asset on
initial recognition and explain the amount of borrowing cost capitalised.

Answer:
IAS 23 should be applied in accounting for borrowing costs. Borrowing costs are recognised as an
expense in the period in which they are incurred unless they are capitalised in accordance with IAS
23 which says that borrowing costs that are directly attributable to the acquisition, construction or
production of a qualifying asset can be capitalised as part of the cost of that asset.
 A qualifying asset is an asset that necessarily takes a substantial period of time to get ready
for its intended use or sale.
 Borrowing costs that are directly attributable to acquisition, construction or production are
taken to mean those borrowing costs that would have been avoided if the expenditure on the
qualifying asset had not been made.

When an enterprise borrows specifically for the purpose of funding an asset, the identification of
the borrowing costs presents no problem as the amount capitalised is the actual borrowing costs
net of any income earned on the temporary investment of those borrowings.
If funds are borrowed, generally, the amount of borrowing costs eligible for capitalisation is
determined by applying a capitalisation rate to the expenditures on that asset calculated as the
weighted average of the borrowing costs applicable to general borrowings.
IAS 23 also contains rules on commencement of capitalisation, suspension of capitalisation and
cessation of capitalisation.

Amount capitalised Rs.000


Cost of manufacture 28,000
Interest capitalised (Rs.28m × 5% × 2 years) 2,800
30,800

Example 06: GOOGLY INDUSTRIES LIMITED


Question: On 1 January 20X6 Googly Industries Limited (GIL) borrowed Rs.15 million to finance the
production of two assets, both of which were expected to take a year to build. Work started during
20X6. The loan facility was drawn down and incurred on 1 January 20X6, and was utilised as
follows, with the remaining funds invested temporarily.

Asset A Asset B
----------- Rs. in million ---------
1 January 20X6 2.5 5
1 July 20X6 2.5 5

The loan rate was 9% and GIL can invest surplus funds at 7%.
Required:
Calculate the borrowing costs which may be capitalised for each of the assets and consequently
the cost of each asset as at 31 December 20X6.

© Emile Woolf International 128 The Institute of Chartered Accountants of Pakistan


Chapter 3: Non-current assets: sundry standards

Answer:
The borrowing cost to be capitalized & cost of assets are as under:
Cost of asset A Rs.
Expenditure 5,000,000

Borrowing costs incurred 5m x 9% x 12/12 450,000


Less: Temporary investment income 2.5m x 7% x 6/12 (87,500)
362,500
5,362,000

Cost of asset B Rs.


Expenditure 10,000,000

Borrowing costs incurred 10m x 9% x 12/12 900,000


Less: Temporary investment income 5m x 7% x 6/12 (175,000)
725,000
10,725,000
.

Example 07: KHAN LIMITED


Question: Khan Limited (KL) has the following loan arrangements as at 1 January 2020:

Rs. in million
7% Debentures 55
8% Loan notes 110
12% Line of credit 85
10% Running finance arrangement 150
On the 1 January 2020, KL commenced the construction of a new factory. The construction of the
factory will cost Rs.100 million and the company funded the construction with the existing
borrowings. The factory was completed on 31 August 2020 but was not available for use until 31
January 2021 as a result of minor modification. During the construction period, active work was
interrupted and the building construction was stopped for two months as a result of adverse weather
conditions.
Required:
Calculate the borrowing cost to be capitalised and the cost of the building to be recognised upon
initial recognition.

Answer:
The borrowing cost to be capitalized & cost of assets are as under:
Rs.
Expenditure 100,000,000
Borrowing costs to be capitalised 100,000,000 x 9.46% x 6/12 4,730,000
104,730,000

January to August are 8 months – two months suspension = 6 months

(55 x 7%) + (110 x 8%) +(85 x 12%) + (150 x 10%)


Capitalisation rate = X 100 = 9.46%
55+110+85+150

© Emile Woolf International 129 The Institute of Chartered Accountants of Pakistan


Financial accounting and reporting I

Example 08: SPIN INDUSTRIES LIMITED


Question: On September 1, 2015, Spin Industries Limited (SIL) started construction of its new
office building and completed it on May 31, 2016. The payments made to the contractor were as
follows:

Date of Payment Rs. in million

September 1, 2015 10

December 1, 2015 15

February 1, 2016 12

June 1, 2016 9

In addition to the above payments, SIL paid a fee of Rs. 8 million on September 1, 2015 for
obtaining a permit allowing the construction of the building.
The project was financed through the following sources:
(i) On August 1, 2015 a medium term loan of Rs. 25 million was obtained specifically for the
construction of the building. The loan carried mark up of 12% per annum payable semi-
annually. A commitment fee @ 0.5% of the amount of loan was charged by the bank.
Surplus funds were invested in savings account @ 8% per annum. On February 1, 2016 SIL
paid the six monthly interest plus Rs. 5 million towards the principal.
(ii) Existing running finance facilities of SIL
 Running finance facility of Rs. 28 million from Bank A carrying mark up of 13%
payable annually. The average outstanding balance during the period of construction
was Rs. 25 million.
 Running finance facility of Rs. 25 million from Bank B. The mark up accrued during
the period of construction was Rs. 3 million and the average running finance balance
during that period was Rs. 20 million.
Required:
Calculate the amount of borrowing costs to be capitalised on June 30, 2016 in accordance with the
requirements of International Accounting Standards. (Borrowing cost calculations should be based
on number of months).

Answer:
The detailed calculation is as under:

Borrowing costs to be capitalized 2016

Rs.

Commitment fee [Rs.25m x 0.5%] 125,000

Interest on specific borrowings [25m x 12% x 5/12] 1,250,000

[20m x 12% x 4/12] 800,000

Temporary Investment income [Working below] (137,500)

Interest on general borrowings [218,156 + 1,416,338] 1,634,494

3,671,994

© Emile Woolf International 130 The Institute of Chartered Accountants of Pakistan


Chapter 3: Non-current assets: sundry standards

Date Particulars Rs. m Working


01-09-15 Specific loan 25,000,000
(125,000)
Permit (8,000,000)
1st payment (10,000,000)
6,875,000 x 8% x 3/12 = Rs. 137,500
(temporary investment income)
01-12-15 2nd payment (15,000,000)
(8,125,000) x 16.11% x 2/12 = Rs. 218,156
(Interest on general borrowings)
01-02-16 Loan interest (1,250,000)
Loan Principal (5,000,000)
3rd payment (12,000,000)
(26,375,000) x 16.11% x 4/12 = Rs. 1,416,338
(interest on general borrowings)
31-05-16 Completion

(25m x 13%) + (3m x 12/9)


Capitalisation rate = X 100 = 16.11%
25m + 20m
Alternative calculation:
(25m x 13% x 9/12) + (20m x 20% x 9/12)
Capitalisation rate = X 100 = 16.11%
25m x 9/12 + 20m x 9/12

Example 09: Monday Limited


Question: The following information pertains to Monday Limited (ML):
(i) The balances of property, plant and equipment as on 1 January 2018:

Cost/Revalued Accumulated
Assets amount depreciation
----------- Rs. in million -----------
Office building 240 36
Equipment 190 60
Revaluation surplus related to the office building as at 1 January 2018 amounted to Rs.
8.5 million.
(ii) On 1 September 2018, a new equipment was acquired by making payment of Rs. 70 million
to the supplier. An old equipment was also given in exchange to the supplier. The fair values
of the old and new equipment were assessed at Rs. 21 million and Rs. 93 million
respectively. The old equipment had been acquired at a cost of Rs. 40 million on 1 July
2016. Cost incurred on installing the new equipment amounted to Rs. 5 million.
(iii) On 1 January 2018, ML commenced construction of a manufacturing plant. The whole
process of assembling and installation was completed on 31 October 2018. However, the
work was stopped from 16 to 31 August 2018 due to unexpected rains.
The total cost of Rs. 660 million incurred on the plant was paid as under:
Description Payment date Rs. in million
1st payment 1 February 2018 140
2nd payment 1 April 2018 214
3rd payment 1 September 2018 146
4th payment 1 December 2018 160

© Emile Woolf International 131 The Institute of Chartered Accountants of Pakistan


Financial accounting and reporting I

The plant was financed through a bank loan of Rs. 500 million obtained on 1 March 2018.
The loan carries a mark-up of 18% payable annually. The surplus funds available from the
loan were invested in a saving account and earned Rs. 17 million during capitalization
period.
(iv) On 31 December 2018, the revalued amount of office building was assessed at Rs. 178
million by Precise Valuers, an independent valuation firm. Value in use of the office building
as at 31 December 2018 was estimated at Rs. 186 million.
(v) Other relevant details are as follows:

Depreciation Subsequent
Assets Life/rate
method measurement
Office building Straight line 20 years* Revaluation
Equipment Reducing balance 20% Cost
Manufacturing plant Straight line 15 years Cost
* Remaining life at the date of last revaluation
ML accounts for revaluation on net replacement value method and transfers the maximum
possible amount from revaluation surplus to retained earnings on an annual basis.
Required:
In accordance with IFRSs, prepare a note on ‘Property plant and equipment’ for inclusion in ML’s
financial statements for the year ended 31 December 2018. (Comparatives figures and column for
total are not required)
Answer:
Monday Limited
Notes to the financial statement
For the year ended 31 December 2018
Property, plant and equipment: Office building Equipment Plant
------------------ Rs. in million ------------------
Gross carrying amount – opening 240.00 190.00 -
Accumulated depreciation (36.00) (60.00) -
Opening carrying amount 204.00 130.00 -
Additions 96.00 699.25
(70+21+5) (W-3)
Depreciation for the year (12.00) (30.48) (7.77)
(240÷20) (W-2) (699.25÷15×2÷ 12)
Disposal (24.96)
(W-1)
Revaluation
- Surplus (8.00)
8.5(8.5÷17)
- P&L (6.00)
Closing carrying amount 178.00 170.56 691.48

Gross carrying amount – closing 178.00 246.00 699.25


Accumulated depreciation - (75.44) (7.77)
Closing carrying amount 178.00 170.56 691.48

© Emile Woolf International 132 The Institute of Chartered Accountants of Pakistan


Chapter 3: Non-current assets: sundry standards

Office building Equipment Plant

Measurement base Revaluation model Cost model Cost model

Useful life (years)/depreciation rate


20 years 20% 15 years
%

Reducing
Straight line Straight line
Depreciation method balance

The last revaluation was performed on 31 December 2018 by Precise Valuers, an independent firm
of valuers.

Carrying value of building, had the cost model been used instead (178+6) 184

W-1: Disposal Rs. in million

2016 40×20%×6÷12 4.00

2017 36×20% 7.20

2018 28.8×20%×8÷12 3.84

Accumulated depreciation 15.04

Book value (4015.04) 24.96

W-2: Depreciation for the year – equipment Rs. in million


Disposal 28.8×20%×8÷12 3.84
Others (13028.8)×20% 20.24
Addition 96×20%×4÷12 6.40
30.48

W-3: Cost of plant Rs. in million


Payments 660.00
Borrowing cost:
Interest cost 500×18%×7.5÷12 56.25
Investment income (17.00)
699.25
.

© Emile Woolf International 133 The Institute of Chartered Accountants of Pakistan


Financial accounting and reporting I

3 IAS 40: INVESTMENT PROPERTY

Section overview

 Scope & Definitions


 Accounting treatment of investment property
 Why investment properties are treated differently from other properties
 Transfers and disposals of investment property
 Disclosure requirements

3.1 Scope & Definitions


IAS 40: Investment Property defines and sets out the rules on accounting for investment properties.

Definition: Investment property


An investment property is property (land or a building, part of a building or both) held to earn
rentals or for capital appreciation or both.

Investment property differs from other property, which is:


 used in the production or supply of goods, or for administrative purposes (which is covered
by IAS 16; Property, plant and equipment); or
 Held for sale in the ordinary course of business (which is covered by IAS 2: Inventories).
Examples of investment property
The following are examples of investment property:
 Land held for long-term capital appreciation rather than for short-term sale in the ordinary
course of business.
 Land held for a currently undetermined future use. (If an entity has not determined that it
will use the land as owner-occupied property or for short-term sale in the ordinary course
of business),
 Property that is being constructed or developed for future use as investment property.
Not investment property
The following are examples of items that are not investment property:
 property intended for sale in the ordinary course of business;
 owner-occupied property including (among other things),
i. property held for future use as owner-occupied property,
ii. property held for future development and subsequent use as owner-occupied property,
iii. property occupied by employees (whether or not the employees pay rent at market
rates) and
iv. owner-occupied property awaiting disposal;
Partly occupied buildings
An entity might use part of a property for the production or supply of goods or services or for
administrative purposes and hold another part of the same property to earn rentals or for capital
appreciation. In other words, part of a property might be owner occupied and part held as an
investment. The two parts are accounted for separately if they could be sold separately (or leased
out separately under a finance lease).

© Emile Woolf International 134 The Institute of Chartered Accountants of Pakistan


Chapter 3: Non-current assets: sundry standards

If this is not the case, the property is investment property only if an insignificant portion is held for
use in the production or supply of goods or services or administrative purpose.

3.2 Accounting treatment of investment property


The recognition criteria for investment property are the same as for property, plant and equipment
under IAS 16. An owned investment property should be recognized as an asset only when:
 it is probable that future economic benefits associated with the property will flow to the
entity; and
 The cost of the property can be measured reliably.

Measurement at recognition
Owned investment property should be measured initially at cost plus any directly attributable
expenditure (e.g. legal fees, property transfer taxes and other transaction costs) incurred to acquire
the property.
The cost of an investment property is not increased by:
 start-up costs (unless necessary to bring the property to the condition necessary for it to be
capable of operating in the manner intended by management);
 operating losses incurred before the investment property achieves the planned level of
occupancy; or
 Abnormal waste incurred in constructing or developing the property.

Measurement after recognition


After initial recognition an entity may choose as its accounting policy:
 the fair value model; or
 The cost model.
The chosen policy must be applied to all the investment property of the entity.
Once a policy has been chosen it cannot be changed unless the change will result in a more
appropriate presentation. IAS 40 states that a change from the fair value model to the cost model
is unlikely to result in a more appropriate presentation.

Fair value model for investment property

Definition: Fair value


Fair value is the price that would be received to sell an asset or paid to transfer a liability in an
orderly transaction between market participants at the measurement date.

Under the fair value model the entity should:


 revalue all its investment property to ‘fair value’ (open market value) at the end of each
financial year; and
 Recognize any resulting gain or loss in profit or loss for the period.
The property would not be depreciated.
This is different to the revaluation model of IAS 16, where gains are reported as other
comprehensive income and accumulated as a revaluation surplus.
If an entity’s policy is to measure investment properties at fair value but its fair value cannot be
measured reliably such investment property shall be measured at cost for example an investment
property under construction. However, if the entity expects the fair value of the investment property
under construction to be reliably measured when construction is complete it shall measure that

© Emile Woolf International 135 The Institute of Chartered Accountants of Pakistan


Financial accounting and reporting I

investment property under construction at cost until either its fair value becomes reliably measured
or construction is completed (whichever is earlier).
If it is not possible to arrive at a reliable fair value figure then the cost model should be adopted for
that property using the cost model in accordance with IAS 16 for owned assets or IFRS 16 for
investment property held by a lessee as a right-of-use asset. This is an exception to the rule that
all investment property must be valued under either one model or the other.

Cost model for investment property


The cost model follows the provisions of IAS 16. The property is measured at cost less
accumulated depreciation (related to the non-land element) and less impairment loss if any.

Illustration 10: Accounting for investment property


On 1 January Year 1 Entity P purchased a building for its investment potential. The building cost
Rs. 1 million with transaction costs of Rs. 10,000.
The depreciable amount of the building component of the property at this date was Rs. 300,000.
The property has a useful life of 50 years.
At the end of Year 1 the property’s fair value had risen to Rs. 1.3 million.
The amounts which would be included in the financial statements of Entity P at 31 December
Year 1, under the cost model are as follows:

Cost model
The property will be included in the statement of financial position as follows:
Rs.
Cost (1,000,000 + 10,000) 1,010,000
Accumulated depreciation (300,000 ÷ 50 years) (6,000)
Carrying amount 1,004,000
The statement of profit or loss will include depreciation of Rs. 6,000.
The amounts which would be included in the financial statements of Entity P at 31 December
Year 1, under the fair value model are as follows:
Fair value model
The property will be included in the statement of financial position at its fair value of Rs.
1,300,000.
The statement of profit or loss will include a gain of Rs. 290,000 (Rs. 1,300,000 – Rs. 1,010,000)
in respect of the fair value adjustment.

3.3 Why investment properties are treated differently from other properties
Most properties are held to be used directly or indirectly in the entity’s business. For example, a
factory, plant and equipment which is used to produce goods for sale. The property is being
consumed and it is appropriate to depreciate it over its useful life.
An investment property is held primarily because it is expected to increase in value over time
(capital appreciation) or it is held to earn rentals. It generates economic benefits for the entity
because it might earn regular stream of income in the form of rentals or might be sold at a profit.
An investment property also differs from owner-occupied properties (IAS 16) because it generates
cash flows that are largely independently of other assets held by an entity.
The most relevant information about an investment property is its fair value (the amount for which
it could be sold). Depreciation is largely irrelevant. Therefore it is appropriate to re-measure an
investment property to fair value each year and to recognize gains and losses in profit or loss for
the period.

© Emile Woolf International 136 The Institute of Chartered Accountants of Pakistan


Chapter 3: Non-current assets: sundry standards

3.4 Transfers and disposals of investment property


If a property is transferred into or out of this category it must be reclassified as an investment
property or as no longer being an investment property. A transfer of investment property can only
be made where there is a change in use (when the property meets or ceases to meet, the definition
of investment property) as illustrated below.

Circumstance for
Transfer from/to Deemed transfer value
a change in use
Commencement Transfer from investment Fair value at the date of change of use
of or development property to owner-occupied becomes the deemed cost for future
with a view to property accounting purposes
owner-occupation
End of owner- Transfer from owner- Where investment properties are measured
occupation occupied property to at fair value, revalue in accordance with IAS
investment property 16 prior to the transfer
Commencement Transfer from investment Fair value at the date of change of use
of development property to inventories becomes the deemed cost for future
with a view to sale accounting purposes
Inception of an Transfer from inventories to Fair value at the date of the transfer, and
operating lease to investment property any difference compared to previous
another party carrying amount is recognised in profit or
loss

Gain or loss on disposal


Gains or losses on disposals of investment properties are included in profit or loss in the period in
which the disposal occurs.

Illustration 11: Disposal of investment property


The investment property in the previous illustration was sold early in Year 2 for Rs. 1,550,000,
Selling costs were Rs. 50,000.
The amount that would be included in the statement of profit or loss for Year 2 in respect of this
disposal under the cost model is as follows:

Cost model Rs.


Sale value 1,550,000
Selling costs (50,000)
Net disposal proceeds 1,500,000
Minus: Carrying amount (1,004,000)
Gain on disposal 496,000
The amount that would be included in the statement of profit or loss for Year 2 in respect of this
disposal under the fair value model is as follows:
(Fair value model Rs.
Sale value 1,550,000
Selling costs (50,000)
Net disposal proceeds 1,500,000
Minus: Carrying amount (1,300,000)
Gain on disposal 200,000

© Emile Woolf International 137 The Institute of Chartered Accountants of Pakistan


Financial accounting and reporting I

3.5 Disclosure requirements


The following disclosures are required by IAS 40 in the notes to the accounts.

Disclosure requirements applicable to both the fair value model and the cost model
 whether the fair value model or the cost model is used
 the methods and assumptions applied in arriving at fair values
 the extent to which the fair value of investment property was based on a valuation by a
qualified, independent value with relevant, recent experience
 amounts recognized as income or expense in the statement of profit or loss for:
 rental income from investment property
 operating expenses in relation to investment property
 details of any restrictions on the ability to realize investment property or any restrictions on
the remittance of income or disposal proceeds
 The existence of any contractual obligation to purchase, construct or develop investment
property or for repairs, maintenance or enhancements.

Disclosure requirements applicable to the fair value model only


There must be reconciliation, in a note to the financial statements, between opening and closing
values for investment property, showing:
 additions during the year
 assets classified as held for sale in accordance with IFRS 5
 net gains or losses from fair value adjustments
 acquisitions through business combinations
This reconciliation should show separately any amounts in respect of investment properties
included at cost because their fair values cannot be estimated reliably.
For investment properties included at cost because fair values cannot be estimated reliably, the
following should also be disclosed:
 a description of the property
 an explanation as to why fair values cannot be determined reliably
 If possible, the range within which the property’s fair value is likely to lie.

Disclosure requirements applicable to the cost model only


 the depreciation methods used
 the useful lives or depreciation rates used
 gross carrying amounts and accumulated depreciation at the beginning and at the end of
the period
 A reconciliation between opening and closing values showing:
 additions
 depreciation
 assets classified as held for sale in accordance with IFRS 5
 acquisitions through business combinations
 impairment losses

© Emile Woolf International 138 The Institute of Chartered Accountants of Pakistan


Chapter 3: Non-current assets: sundry standards

 transfers
When the cost model is used, the fair value of investment property should also be disclosed. If the
fair value cannot be estimated reliably, the same additional disclosures should be made as under
the fair value model.

Example 10: VICTORIA


Question: Victoria owns several properties and has a year end of 31 December. Wherever possible,
Victoria carries investment properties under the fair value model.
Property 1 was acquired on 1 January Year 1. It had a cost of Rs. 1 million, comprising Rs. 500,000
for land and Rs. 500,000 for buildings. The buildings have a useful life of 40 years. Victoria uses this
property as its head office.
Property 2 was acquired many years ago for Rs. 1.5 million for its investment potential. On 31
December Year 7 it had a fair value of Rs. 2.3 million. By 31 December Year 8 its fair value had
risen to Rs. 2.7 million. This property has a useful life of 40 years.
Property 3 was acquired on 30 June Year 2 for Rs. 2 million for its investment potential. The directors
believe that the fair value of this property was Rs. 3 million on 31 December Year 7 and Rs. 3.5
million on 31 December Year 8. However, due to the specialised nature of this property, these figures
cannot be corroborated. This property has a useful life of 50 years.
Required:
For each of the above properties briefly state how it would be treated in the financial statements of
Victoria for the year ended 31 December Year 8, identifying any impact on profit or loss and produce
an analysis of non-current assets for Victoria for the year ended 31 December Year 8, showing each
of the above properties separately.
Answer:
Treatment in the financial statements for the year ended 31 December Year 8
Property 1
This is used by Victoria as its head office and therefore cannot be treated as an investment property.
It will be stated at cost minus accumulated depreciation in the statement of financial position. The
depreciation for the year will be charged in the statement of profit or loss.
Property 2
This is held for its investment potential and should be treated as an investment property. It will be
carried at fair value, Victoria’s policy of choice for investment properties. It will be revalued to fair
value at each year end and any resultant gain or loss taken to the statement of profit or loss (Rs.
400,000 gain in Year 8).
Property 3
This is held for its investment potential and should be treated as an investment property. However,
since its fair value cannot be arrived at reliably it will be held at cost minus accumulated depreciation
in the statement of financial position. The depreciation for the year will be an expense in the
statement of profit or loss.
This situation provides the exception to the rule whereby all investment properties must be held
under either the fair value model, or the cost model.
Based on above analysis the property plant and equipment for the year ended 31 December year
8 is as under:
Other land and Investment Investment
buildings property held property held Total
(W1) at fair value at cost (W2)

Rs. Rs. Rs. Rs.


Cost/valuation
On 1 January Year 8 1,000,000 2,300,000 2,000,000 5,300,000
Revaluation - 400,000 - 400,000
---------------------- ---------------------- ---------------------- ----------------------
On 31 December Year 8 1,000,000 2,700,000 2,000,000 5,700,000
---------------------- ---------------------- ---------------------- ----------------------

© Emile Woolf International 139 The Institute of Chartered Accountants of Pakistan


Financial accounting and reporting I

Other land and Investment Investment


buildings property held property held Total
(W1) at fair value at cost (W2)

Rs. Rs. Rs. Rs.


Accumulated depreciation
On 1 January Year 8 87,500 - 220,000 307,500
Charge for the year (W1) 12,500 - 40,000 52,500
---------------------- ---------------------- ---------------------- ----------------------
On 31 December Year 8 100,000 - 260,000 360,000
---------------------- ---------------------- ---------------------- ----------------------
Carrying amount
On 31 December Year 7 912,500 2,300,000 1,780,000 4,992,500
---------------------- ---------------------- ---------------------- ----------------------
On 31 December Year 8 900,000 2,700,000 1,740,000 5,340,000
---------------------- ---------------------- ---------------------- ----------------------
Tutorial note
In practice, with a more complex property, plant and equipment table the investment properties
would be included within the land and buildings column with the required disclosures being given
separately in a note to the table.
Workings
(1) Depreciation on Property 1

Rs.
Brought forward (500,000 ÷ 40  7) 87,500
Year 8 (500,000 ÷ 40) 12,500

(2) Depreciation on Property 3

Rs.
Brought forward (2,000,000 ÷ 50  5.5) 220,000
Year 8 (2,000,000 ÷ 50) 40,000

Example 11: Distaghil Limited


Question: Following information pertains to non-current assets of Distaghil Limited (DL):
(i) DL purchased specialised vehicles for Rs. 370 million on 1 July 2017. The vehicles have
an estimated useful life of 10 years with residual value of Rs. 30 million.
The revalued amounts of the vehicle as at 31 December 2018 and 2019 were
determined at Rs. 302 million and Rs. 290 million respectively. There was no change in
useful life or residual value.
(ii) DL setup a manufacturing plant in a remote area at a cost of Rs. 280 million. The plant
had a useful life of 8 years. The plant was purchased on 1 January 2018 and was
available for use on 1 April 2018. The commercial production started on 1 June 2018.
On 1 July 2018, DL received a government grant of Rs. 120 million towards the cost of
the plant. The sanction letter states that if DL ceases to use the plant in the remote
area before 31 December 2021, DL would be required to repay the grant in full.

© Emile Woolf International 140 The Institute of Chartered Accountants of Pakistan


Chapter 3: Non-current assets: sundry standards

(iii) A warehouse was given on rent on 1 January 2018. Previously, the warehouse was in
use of DL.
On 1 January 2018, carrying value and remaining useful life of the warehouse was Rs.
80 million and 16 years respectively. Fair value of the warehouse on various dates are
as follows:

Rs. in million
01 January 2018 104
31 December 2018 96
31 December 2019 115

Other information:
 DL uses cost model for subsequent measurement of property, plant and equipment
except for specialised vehicles for which revaluation model is used.
 DL transfers the maximum possible amount from the revaluation surplus to retained
earnings on an annual basis.
 Government grant is recorded as deferred income and a part of it is transferred to income
each year.
 Investment property is carried at fair value model.
Required:
Prepare relevant extracts from DL’s statement of profit or loss and other comprehensive income
for the year ended 31 December 2019 and statement of financial position as on that date.
(Show comparative figures)

Answer:

Distaghil Limited
Extracts from statement of financial position as on 31 December 2019
2019 2018
---- Rs. in million ----
Non-current assets:
Property, plant and equipment:
 Vehicles (W-1) 290.00 302.00
 Plant (W-2) 218.75 253.75

Investment property 115.00 96.00

Share capital and reserves:


Revaluation surplus:
 Vehicles 5.00 -
 Warehouse[2019: 22.5–(24÷16)], 21.00 22.50
[2018: 24–(24÷16)]

Non-current liabilities:
Deferred government grant (W-2) 93.75 108.75

© Emile Woolf International 141 The Institute of Chartered Accountants of Pakistan


Financial accounting and reporting I

Extracts from statement of profit or loss and other comprehensive income


For the year ended 31 December 2019
2019 2018
------ Rs. in million ------
Profit or loss:
Depreciation:
 Vehicles (W-1) (32.00) (34.00)
 Plant (W-2) (35.00) (26.25)

Impairment / Revaluation loss – vehicles (W-1) - (17.00)


Revaluation gain - vehicles (W-1) 15.00
Grant income – Plant (W-2) 15.00 11.25
Change in fair value of investment property - warehouse
(2019: 115–96), (2018: 96–104) 19.00 (8.00)

Other comprehensive income:


Revaluation surplus:
 Vehicles (W-1) 5.00
 Warehouse (104–80) - 24.00

W-1: Vehicles Rs. in million


Purchased on 1 July 2017 370
Depreciation for 2017 (370–30)÷10×(6÷12) (17)
Book value on 31 December 2017 353
Depreciation for 2018 (370–30)÷10 (34)
Book value on 31 December 2018 before revaluation 319
Revaluation loss Balancing (17)
Revalued amount on 31 December 2018 302
Depreciation for 2019 (302–30)÷8.5 (32)
Book value on 31 December 2019 before revaluation 270
Revaluation gain to P&L 17×(7.5÷8.5) 15
Revaluation surplus Balancing 5
Revalued amount on 31 December 2019 290

W-2: Plant and government grant Plant Gov. grant


-------- Rs. in million --------
Initial balance 280.00 120.00
Depreciation for 2018 (280÷8)×(9÷12) (26.25)
Grant income for 2018 (120÷8)×(9÷12) (11.25)
Balance on 31 December 2018 253.75 108.75
Depreciation for 2019 280÷8 (35.00)
Grant income for 2019 120÷8 (15.00)
Balance on 31 December 2019 218.75 93.75
.

© Emile Woolf International 142 The Institute of Chartered Accountants of Pakistan


Chapter 3: Non-current assets: sundry standards

4 OBJECTIVE BASED QUESTIONS


01. On 1 January 2021 Aim Limited (AL) received Rs. 1,000,000 from the local government on the
condition that they employ at least 150 persons each year for the next 4 years.
Due to an economic downturn and reduced consumer demand on 1 January 2022, AL no longer
needed to employ any more staff and the conditions of the grant required full repayment.
What should be recorded in the financial statements on 1 January 2022?

(a) Reduce deferred income balance by Rs. 750,000


(b) Reduce deferred income by Rs. 750,000 and recognize a loss of Rs. 250,000
(c) Reduce deferred income by Rs. 1,000,000
(d) Reduce deferred income by Rs. 1,000,000 and recognize a gain of Rs. 250,000

02. Which of the following are acceptable methods of accounting for a government grant relating
to an asset in accordance with IAS 20 Accounting for Government Grants and Disclosure of
Government Assistance?
(i) Set up the grant as deferred income
(ii) Credit the amount received to profit or loss
(iii) Deduct the grant from the carrying amount of the asset
(iv) Add the grant to the carrying amount of the asset

(a) (i) and (ii)


(b) (ii) and (iv)
(c) (i) and (iii)
(d) (iii) and (iv)

03. On 1 January 2019, Boom Limited (BL) received Rs. 2,000,000 from the local government on
the condition that they employ at least 200 staff each year for the next 4 years. On this date, it
was virtually certain that BL would meet these requirements.
However, on 1 January 2022, due to an economic downturn and reduced consumer demand,
BL no longer needed to employ 100 staff. The conditions of the grant required half repayment.
What should be recorded in the financial statements on 1 January 2022 for repayment of grant?

(a) Debit Deferred grant by Rs. 500,000 and PL by Rs. 500,000


(b) Debit Deferred grant by Rs. 250,000 and PL by Rs. 250,000
(c) Debit Deferred grant by Rs. 1,500,000 and PL by Rs. 500,000
(d) Debit Deferred grant by Rs. 500,000 and PL by Rs. 1,500,000

04. Which TWO of the following statements about IAS 20 Accounting for Government Grants and
Disclosure of Government Assistance are true?
(a) A government grant related to the purchase of an asset must be deducted from the
carrying amount of the asset in the statement of financial position.
(b) A government grant related to the purchase of an asset should be recognised in profit
or loss over the life of the asset.

© Emile Woolf International 143 The Institute of Chartered Accountants of Pakistan


Financial accounting and reporting I

(c) Free marketing advice provided by a government department is excluded from the
definition of government grants.
(d) Any required repayment of a government grant received in an earlier reporting period is
treated as prior period adjustment.

05. Which TWO of the statements below regarding IAS 23 Borrowing Costs are correct?
(a) Borrowing costs must be capitalised if they are directly attributable to qualifying assets
(b) Borrowing costs should cease to be capitalised once the related asset is substantially
complete
(c) Borrowing costs must be capitalised if they are directly attributable to non-current assets
(d) Borrowing costs may be capitalised if they are directly attributable to qualifying assets

06. Fine Limited (FL) received a Rs.10 million loan at 7.5% on 1 April 2017. The loan was
specifically issued to finance the building of a new store.
Construction of the store commenced on 1 May 2017 and it was completed and ready for use
on 28 February 2018 but did not open for trading until 1 April 2018.
How much should be recorded as finance costs in the statement of profit or loss for the year
ended 31 March 2018?
(a) Rs. 250,000
(b) Rs. 750,000
(c) Rs. 125,000
(d) Rs. 625,000

07. Fine Limited (FL) received a Rs.10 million loan at 7.5% on 1 April 2017. The loan was
specifically issued to finance the building of a new store.
Construction of the store commenced on 1 May 2017 and it was completed and ready for use
on 28 February 2018 but did not open for trading until 1 April 2018.
How much interest should be capitalised as part of property, plant and equipment as at 31
March 2018?
(a) Rs. 250,000
(b) Rs. 750,000
(c) Rs. 125,000
(d) Rs. 625,000

08. An entity decided that not all of the funds raised were needed immediately and temporarily
invested some of the funds for one month before the construction started, earning Rs.40, 000
interest.
How should the Rs. 40,000 be accounted for in the financial statements?

(a) Net off the amount capitalised in property, plant and equipment
(b) Taken to the statement of profit or loss as investment income
(c) Taken as other comprehensive income
(d) Deducted from the outstanding loan amount in the statement of financial position

© Emile Woolf International 144 The Institute of Chartered Accountants of Pakistan


Chapter 3: Non-current assets: sundry standards

09. Shine Limited (SL) had the following bank loans outstanding during the whole of 2018:

Rs. m
9% loan repayable 2019 15
11% loan repayable 2022 24
SL began construction of a qualifying asset on 1 April 2018 and withdrew funds of Rs. 6 million
on that date to fund construction. On 1 August 2018 an additional Rs. 2 million was withdrawn
for the same purpose.
Calculate the borrowing costs which can be capitalised in respect of this project for the year
ended 31 December 2018.

(a) Rs. 545,600


(b) Rs. 472,350
(c) Rs. 750,600
(d) Rs. 350,350

10. Jazz Limited (JL) has borrowed Rs. 24 million to finance the building of a factory. Construction
is expected to take two years.
The loan was drawn down and incurred on 1 January 2019 and work began on 1 March 2019.
Rs. 10 million of the loan was not utilized until 1 July 2019 so JL was able to invest it until
needed. JL is paying 8% on the loan and can invest surplus funds at 6%.
Calculate the borrowing costs to be capitalised for the year ended 31 December 2019 in respect
of this project.

(a) Rs. 1,400,000


(b) Rs. 1,920,000
(c) Rs. 1,300,000
(d) Rs. 1,620,000

11. A company has the following loans in place throughout the year ended 31 December 2018.

Rs. m
10% bank loan 140
8% bank loan 200
On 1 July 2018 Rs. 50 million was drawn down for construction of a qualifying asset which was
completed during 2019.
What amount should be capitalised as borrowing costs at 31 December 2018 in respect of this
asset?

(a) Rs. 5.6 million


(b) Rs. 2.8 million
(c) Rs. 4.4 million
(d) Rs. 2.2 million

© Emile Woolf International 145 The Institute of Chartered Accountants of Pakistan


Financial accounting and reporting I

12. An entity purchased an investment property on 1 January 2013 for a cost of Rs. 35m. The
property had an estimated useful life of 50 years, with no residual value, and at 31 December
2015 had a fair value of Rs. 42m.
On 1 January 2016 the property was sold for net proceeds of Rs. 40m.
Calculate the profit or (loss) on disposal under both the cost and fair value (FV) model.

(a) Cost model: Rs. 7.1 m and FV model: (Rs. 2.0 m)


(b) Cost model: Rs. 2.0 m and FV model: Rs. 2.0 m
(c) Cost model: Rs. 5.0 m and FV model: (Rs. 2.0 m)
(d) Cost model: Rs. 7.1 m and FV model: Rs. 5.0 m

13. An investment property with a useful life of 10 years was purchased by Akram Limited on 1
January 2019 for Rs. 200 million. By 31 December 2019 the fair value of the property had risen
to Rs. 300 million. Akram Limited measures its investment properties under the fair value
model.
What values would go through the statement of profit or loss in the year?

(a) Gain: Rs. 100 million and Depreciation Rs. 30 million


(b) Gain: Rs. 0 and Depreciation of Rs. 30 million
(c) Gain: Rs. 100 million and Depreciation of 0
(d) Gain: Rs. 120 million and Depreciation of Rs. 20 million

14. Which of the following properties owned by an entity would be classified as an investment
property?
(a) A property that had been leased to a tenant, but which is no longer required and is now
being held for resale
(b) Land purchased for its investment potential. Planning permission has not been obtained
for building construction of any kind
(c) A new office building used as entity’s head office, purchased specifically in order to
exploit its capital gains potential
(d) A bungalow used for executive training

15. Sarfraz Limited (SL) uses fair value accounting where possible and has an office building used
by SL for administrative purposes. At 1 April 2012 it had a carrying amount of Rs. 20 million
and a remaining life of 20 years. On 1 October 2012, the property was let to a third party and
reclassified as an investment property. The property had a fair value of Rs. 23 million at 1
October 2012, and Rs. 23.4 million at 31 March 2013.
What is the correct treatment when the above property is reclassified as an investment
property?

(a) Take Rs. 3,500,000 gain to other comprehensive income


(b) Take Rs. 3,500,000 gain to the statement of profit or loss
(c) Take Rs. 4,000,000 gain to other comprehensive income
(d) Take Rs. 4,000,000 gain to the statement of profit or loss

© Emile Woolf International 146 The Institute of Chartered Accountants of Pakistan


Chapter 3: Non-current assets: sundry standards

16. A manufacturing entity receives a grant of Rs. 1,000,000 towards the purchase of a machine
on 1 January 2013. The grant will be repayable if the entity sells the asset within 4 years, which
it does not intend to do. The asset has a useful life of 5 years.
What is the deferred income liability balance at 30 June 2013?

Rs. ___________

17. A company receives a government grant of Rs. 500,000 on 1 April 2017 to facilitate purchase
on the same day of an asset which costs Rs. 750,000. The asset has a five-year useful life and
is depreciated on a 30% reducing balance basis. Company policy is to account for all grants
received as deferred income.
What amount of income will be recognized in respect of the grant in the year to 31 March 2019?

Rs. ___________

18. A manufacturing entity is entitled to a grant of Rs. 3 million for creating 50 jobs and maintaining
them for three years. Rs. 1.5m is received when the jobs are created and the remaining Rs.
1.5m is receivable after three years, provided that the 50 jobs are still in existence. The entity
creates 50 jobs at the beginning of year one and there is reasonable assurance that this level
of employment will be maintained.
What is the deferred income balance at the end of the first year?

Rs. ___________

19. An entity uses funds from its general borrowings to build a new production facility. Details of
the entity's borrowings are shown below:
Rs.10 million 6% loan
Rs.6 million 8% loan
The entity used Rs.12 million of these funds to construct the facility, which was under
construction for the entire year.
How much interest should be capitalised as part of the cost of the asset?

Rs. ___________

20. Cool Limited acquired a building with a 40-year life for its investment potential for Rs. 8 million
on 1 January 2013. At 31 December 2013, the fair value of the property was estimated at Rs.
9 million with costs to sell estimated at Rs. 200,000.
If Cool Limited uses the fair value model for investment properties, what gain should be
recorded in the statement of profit or loss for the year ended 31 December 2013?

Rs. ___________

21. If a government grant must be repaid, then it is;


(a) An error
(b) A change in accounting policy
(c) A change in accounting estimate
(d) A new transaction

© Emile Woolf International 147 The Institute of Chartered Accountants of Pakistan


Financial accounting and reporting I

22. If an entity receives a non-monetary asset as a grant, this is accounted for at the;
(a) Market value
(b) Fair value
(c) Net realizable value
(d) Present value

23. Which of the following is not covered by IAS 20 – Government Grants?


(a) Tax breaks
(b) Employment grants
(c) Subsidized loans
(d) Forgivable loans

24. Which of the following is not a correct treatment of government grants related to an asset?

(a) Deferred income


(b) Credit to income in period received
(c) Deducting the grant from the carrying amount of the asset
(d) None of the above

25. Which of the following is not a correct treatment of government grants related to income?
(a) Present as. Other income
(b) Deduct from the related expense
(c) Deduct from the cost of the asset
(d) None of the above

26. Which of the following is not considered a “borrowing cost” under IAS 23?
(a) Interest expense calculated by the effective interest method
(b) Finance charges in respect of loan
(c) Exchange differences arising from foreign currency borrowings to the extent that they
are regarded as an adjustment to interest costs
(d) Principal repayments on a loan for property, plant and equipment

27. When activities to prepare an asset for its sale or use are suspended, borrowing costs must
be?
(a) Capitalized
(b) Expensed
(c) Ignored
(d) Charged to equity

© Emile Woolf International 148 The Institute of Chartered Accountants of Pakistan


Chapter 3: Non-current assets: sundry standards

28. Which of the following is not a condition to commence capitalisation of borrowing costs?
(a) Expenditures are being incurred
(b) Borrowing costs are being incurred
(c) Repayment of borrowings has commenced
(d) Activities to produce the asset for its intended use or sale have commenced

29. Ghazi Limited (GL) is constructing an office building and is capitalising borrowing costs in
accordance with IAS 23. The office is almost complete; the only remaining work is to install
furniture. Is GL allowed to continue capitalising the borrowing costs?
(a) Yes
(b) No
(c) Don’t know
(d) None of the above

30. Which of the following is not a “qualifying asset” under IAS 23?
(a) Mass produced inventory
(b) Manufacturing plants
(c) Made to order inventory
(d) Investment property

31. Under IAS 40 – Investment Property, where should a gain or loss on disposal be recognized?
(a) Statement of Financial Position
(b) Profit and loss statement
(c) Statement of changes in equity
(d) None

32. If an entity uses part of a building for their own use, and rents the remainder. How should this
be treated?
(a) All as investment property under IAS 40 – Investment Property
(b) All under IAS 16 – Property, Plant and Equipment
(c) Account for separately under ‘IAS - 16 Property, Plant and Equipment’ and ‘IAS - 40
Investment Property’
(d) None of these

33. An investment property should initially be measured at?


(a) Cost
(b) Fair value
(c) Market value
(d) Net realizable value

© Emile Woolf International 149 The Institute of Chartered Accountants of Pakistan


Financial accounting and reporting I

34. If an entity wishes to change from a cost model to fair value model under IAS 40 – Investment
Property, when may it do so?
(a) When the board of directors approves a change
(b) When the value of the assets will improve with a revised model
(c) When a change will result in a more appropriate presentation
(d) When the market for these properties is fluctuation

35. Which two of the following properties fall under the definition of investment property and
therefore within the scope of IAS 40?
(a) Property occupied by an employee paying market rent
(b) A building owned by an entity and leased out under an operating lease
(c) Property being constructed on behalf of 3rd parties
(d) Land held for long term appreciation

36. Which of the following can NOT be a ‘qualifying asset’ under IAS 23 Borrowing Costs?
(a) Inventories
(b) Manufacturing plants
(c) Assets that are ready for their intended use when acquired
(d) Investment property

37. Afternoon Limited (AL) uses cost model for its property, plant and equipment and fair value
model for its investment property. AL has an office building which was being used for
administrative purposes. At 1 July 2018, the building had a carrying amount of Rs. 20 million.
On that date, the building was let out to a third party and therefore reclassified as an investment
property. The building had a fair value of Rs. 23 million on 1 July 2018 and Rs. 23.4 million on
30 June 2019.

What would be the increase in the profit or loss and other comprehensive income for the year
ended 30 June 2019?
Profit or loss Other comprehensive income
(a) Nil Rs. 3.4 million
(b) Rs. 0.4 million Rs. 3 million
(c) Rs. 3.4 million Nil
(d) Rs. 3 million Rs. 0.4 million

38. Which TWO of the following fall under the definition of investment property?
(a) Property occupied by an employee
(b) A building owned by an entity and leased out under an operating lease
(c) Property being constructed on behalf of third party
(d) Land held for long term appreciation

© Emile Woolf International 150 The Institute of Chartered Accountants of Pakistan


Chapter 3: Non-current assets: sundry standards

39. Capitalisation of borrowing costs should be suspended:


(a) when substantially all the activities necessary to prepare a qualifying asset for its
intended use or sale are complete
(b) during a temporary delay which is a necessary part of the process of getting an asset
ready for its intended use or sale
(c) during extended periods in which active development of a qualifying asset is interrupted
(d) all of the above

40. Under IAS 40 ‘Investment property’, which of the following disclosures is NOT required to be
made under cost model?
(a) Fair value of the property
(b) Depreciation method
(c) Reconciliation of carrying amounts at the beginning and end of a period
(d) Residual value of the property

41. Which of the following statements is correct in the context of capitalisation of borrowing costs?
(a) If funds have been arranged from various general borrowings, the amount to be
capitalised is based on the weighted average cost of borrowings
(b) Capitalisation always commences as soon as expenditure for the asset is incurred
(c) Capitalisation always continues until the asset is brought into use
(d) Capitalisation always commences as soon as borrowing costs are incurred

© Emile Woolf International 151 The Institute of Chartered Accountants of Pakistan


Financial accounting and reporting I

4 OBJECTIVE BASED ANSWERS


01. (b) This is a grant related to income and would therefore be released to the
statement of profit or loss over the 4 year life. By the end of year one, Rs.
250,000 would have been credited to the statement of profit or loss, leaving
Rs. 750,000 held in deferred income. At this point the amount is repaid,
meaning that the deferred income is removed, as well as the Rs. 250,000
income previously recorded.

02. (c) The grant can be treated as deferred income or deducted from the carrying
amount of the asset. It cannot be credited directly to profit or loss.

03. (a) Half repayment is Rs. 1,000,000 (Rs. 2,000,000 x 50%)


At the date of repayment, the balance in deferred grant would be Rs. 500,000
and additional Rs. 500,000 shall be charged to profit or loss.

04. (b, c) Item a is incorrect as the deferred income method can be used.
Item d is incorrect as any repayment is corrected in the current period, not
retrospectively.

05. (a, b) Borrowing costs must be capitalised if they are directly attributable to
qualifying assets, which are assets that take a substantial time to complete.
Capitalization should cease once substantially all the activities to prepare the
asset are complete.

06. (c) Rs. 10 million x 7.5% x 2/12 = Rs. 125,000

07. (d) Rs. 10 million x 7.5% x 10/12 = Rs. 625,000

08. (b) Temporary investment income earned during the construction period should
be netted off the amount capitalised.
However, the interest was earned prior to the period of construction. Therefore
the investment income earned should be taken to the statement of profit or
loss as investment income.

09. (a) (9% × 15𝑚) + (11% × 24𝑚)


𝐶𝑎𝑝𝑖𝑡𝑎𝑙𝑖𝑠𝑎𝑡𝑖𝑜𝑛 𝑟𝑎𝑡𝑒 = = 10.23%
15 + 24
Rs. 6m × 10.23% × 9/12 = Rs.460,350
Rs. 2m × 10.23% × 5/12 = Rs. 85,250
Total Rs. 545,600

10. (a) Rs.

March – December (Rs. 24m × 8% × 10/ 12) 1,600,000

Less investment income (Rs. 10m × 6% × 4/12) (200,000)

1,400,000

Temporary investment income before commencement would be recognized


as finance income in profit or loss.

© Emile Woolf International 152 The Institute of Chartered Accountants of Pakistan


Chapter 3: Non-current assets: sundry standards

11. (d) (10% × 140𝑚) + (8% × 200𝑚)


𝐶𝑎𝑝𝑖𝑡𝑎𝑙𝑖𝑠𝑎𝑡𝑖𝑜𝑛 𝑟𝑎𝑡𝑒 = = 8.8%
140 + 200
Rs. 50 million × 8.8% × 6/12 = Rs. 2.2 million

12. (a) Under the cost model the property will be depreciated over 50 years for 3
years up to the date of disposal. Therefore, at the disposal date the carrying
value would have been Rs. 35m – (Rs. 35m/50 × 3 years) = Rs. 32.9m and
the profit on disposal Rs. 7.1m (Rs. 40m – Rs. 32.9).
Under the fair value model the property will not be depreciated hence the loss
on disposal would be Rs. 2m (Rs. 40m – Rs. 42m).

13. (c) Under the fair value model the property will not be depreciated hence the gain
on valuation would be Rs. 100 million (Rs. 300 million – Rs. 200 million).

14. (b) Asset A would be classed as a non-current asset held for sale under IFRS 5.
Assets C and D would both be classified as property, plant and equipment
under IAS 16.

15. (a) As SL uses the fair value model for investment properties, the asset should
be revalued to fair value before being classed as an investment property. The
gain on revaluation should be taken to other comprehensive income, as the
asset is being revalued while held as property, plant and equipment.
At 1 October, the carrying amount of the asset is Rs. 19.5 million, being Rs.
20 million less 6 months’ depreciation. As the fair value at 1 October is Rs. 23
million, this leads to a Rs. 3,500,000 gain which will be recorded in other
comprehensive income.

16. Rs. 900,000 The grant should be released over the useful life, not based on the possibility
of the item being repaid. Therefore, the Rs. 1m should be released over 5
years, being a release of Rs. 200,000 a year. At 30 June 2013, 6 months
should be released, meaning Rs. 100,000 has been released (6/12 × Rs.
200,000). This leaves Rs. 900,000 in deferred income.

17. Rs. 105,000 Rs.

Grant received 1.4.17 500,000

Recognized year to 31.3.18 (500,000 × 30%) (150,000)

Balance 31.3.18 350,000

Recognized year to 31.3.19 (350,000 × 30%) 105,000

18. Rs. 500,000 The total grant income is Rs. 3m, to be recognized over a three-year period.
Annual income is therefore Rs. 1m. At the end of the first year the entity has
received Rs. 1.5m of which Rs. 1m has been recognized in the statement of
profit or loss, leaving Rs. 500,000 deferred into future periods.

19. Rs. 810,000 Rs.12m × 6.75% = Rs. 810,000


Capitalisation rate
= ((Rs.10m × 6%) + (Rs.6m × 8%))/Rs.16m = 6.75%.

© Emile Woolf International 153 The Institute of Chartered Accountants of Pakistan


Financial accounting and reporting I

20. Rs. The fair value gain of Rs. 1 million (Rs. 9m – Rs. 8m) should be taken to the
1,000,000 statement of profit or loss. Costs to sell are ignored and, since entity uses the
fair value model, no depreciation will be charged on the building.

21. (c)

22. (b)

23. (a)

24. (b)

25. (c)

26. (d)

27. (b)

28. (c)

29. (b)

30. (a)

31. (b)

32. (c)

33. (a)

34. (c)

35. (b) & (d)

36. (c) Assets that are ready for their intended use when acquired

37. (b) Profit or loss Rs. 0.4 million and Other comprehensive income Rs. 3 million

38. (b) & (d) A building owned by an entity and leased out under an operating lease & Land
held for long term appreciation

39. (c) during extended periods in which active development of a qualifying asset is
interrupted

40. (d) Residual value of the property

41. (a) If funds have been arranged from various general borrowings, the amount to
be capitalised is based on the weighted average cost of borrowings

© Emile Woolf International 154 The Institute of Chartered Accountants of Pakistan


Certificate in Accounting and Finance

4
Financial accounting and reporting I

CHAPTER
IAS 36: Impairment of assets

Contents
1 Impairment of assets

2 Objective based questions and answers

* The student must refer original handbook of IFRS.

© Emile Woolf International 155 The Institute of Chartered Accountants of Pakistan


Financial accounting and reporting I

1 IMPAIRMENT OF ASSETS

Section overview

 Objective and scope of IAS 36


 Definitions
 Stages in accounting for an impairment loss
 Indications of impairment
 Measuring recoverable amount
 Recognition & Accounting for impairment
 Summary of the approach

1.1 Objective and scope of IAS 36


An asset is said to be impaired when its recoverable amount is less than its carrying amount in the
statement of financial position. From time to time an asset may have a carrying value that is greater
than its fair value but this is not necessarily impairment as the situation might change in the future
as the reporting date approaches.
The objective of IAS 36 Impairment of assets is to ensure that assets are ‘carried’ (valued) in the
financial statements at no more than their recoverable amount.
Scope of IAS 36
IAS 36 applies to accounting for impairment of all assets except the following:
 inventories (IAS 2: Inventories);
 Assets arising from contracts with customers that are recognised in accordance with IFRS
15: Revenue from contracts with customers.
 investment property that is measured at fair value (IAS 40):
The requirements of IAS 36 also do not apply to IFRSs like IAS 12, IAS 27, IAS 28, IAS 38, IAS
41, IFRS 4, IFRS 5, IFRS 10, IFRS 11, which are out of syllabus at this level.

1.2 Definitions

Definitions
The recoverable amount of an asset is defined as the higher of its fair value minus costs of
disposal, and its value in use.
Fair value is the price that would be received to sell an asset or paid to transfer a liability in an
orderly transaction between market participants at the measurement date.
Value in use is the present value of future cash flows from using an asset, including its eventual
disposal.
Impairment loss is the amount by which the carrying amount of an asset (or a cash-generating
unit) exceeds its recoverable amount.

1.3 Stages in accounting for an impairment loss


There are various stages in accounting for an impairment loss:
Stage 1: Establish whether there is an indication of impairment.
Stage 2: If so, assess the recoverable amount.

© Emile Woolf International 156 The Institute of Chartered Accountants of Pakistan


Chapter 4: IAS 36: Impairment of assets

Stage 3: Write down the affected asset (by the amount of the impairment) to its recoverable
amount.
Each of these stages will be considered in turn.

1.4 Indications of impairment


An entity must carry out an impairment review when there is evidence or an indication that
impairment may have occurred. At the end of each reporting period, an entity should assess
whether there is any indication that impairment might have occurred. If such an indication exists,
the entity must estimate the recoverable amount of the asset, in order to establish whether
impairment has occurred and if so, the amount of the impairment.
The following are given by IAS 36 as possible indicators of impairment. These may be indicators
outside the entity itself (external indicators), such as market factors and changes in the market.
Alternatively, they may be internal indicators relating to the actual condition of the asset or the
conditions of the entity’s business operations.
When assessing whether there is an indication of impairment, IAS 36 requires that, at a minimum,
the following sources are considered:

External sources Internal sources

An unexpected decline in the asset’s market value. Evidence that the asset is damaged
or no longer of use to the entity.

Significant changes in technology, markets, economic There are plans to discontinue or


factors or laws and regulations that have an adverse restructure the operation for which
effect on the company. the asset is currently used.

An increase in interest rates, affecting the value in use of There is a reduction in the asset’s
the asset. expected remaining useful life.

The company’s net assets have a higher carrying value There is evidence that the entity’s
than the company’s market capitalisation (which expected performance is worse
suggests that the assets are over-valued in the statement than expected.
of financial position).

Internal indicators for impairment are generally refers to items under control of management while
external indicators are outside the control of management.
If there is an indication that an asset is impaired then it is tested for impairment. This involves the
calculating the recoverable amount of the item in question and comparing this to its carrying
amount.

1.5 Measuring recoverable amount


It has been explained that recoverable amount is the higher of an asset’s:
 fair value less costs of disposal; and
 its value in use.
If either of these amounts is higher than the carrying value of the asset, there has been no
impairment.
IAS 36 sets out the requirements for measuring ‘fair value less costs of disposal’ and ‘value in use’.

© Emile Woolf International 157 The Institute of Chartered Accountants of Pakistan


Financial accounting and reporting I

Measuring fair value less costs of disposal


Fair value of an asset at a particular date is normally its current market value. If no active market
exists, it may be possible to estimate the amount that the entity could obtain from the disposal.
Direct selling costs normally include legal costs, taxes and costs necessary to bring the asset into
a condition to be sold. However, redundancy and similar costs (for example, where a business is
reorganized following the disposal of an asset) are not direct selling costs.
Calculating value in use
Value in use represents the present value of the expected future cash flows from use of the asset,
discounted at a suitable discount rate or cost of capital.
The following elements should be reflected in the calculation of an asset’s value in use:
 An estimate of the future cash flows the entity expects to derive from the asset
 Expectations about possible variations in the amount or timing of those future cash flows
 The time value of money (represented by the current market risk-free rate of interest)
 The price for bearing the uncertainty inherent in the asset
 Other factors that market participants would reflect in pricing the future cash flows the entity
expects to derive from the asset.
Estimates of future cash flows should be based on reasonable and supportable assumptions that
represent management’s best estimate of the economic conditions that will exist over the
remaining useful life of the asset.
Estimates of future cash flows must include:
 cash inflows from the continuing use of the asset;
 cash outflows that will be necessarily incurred to generate the cash inflows from continuing
use of the asset; and
 net disposal proceeds at the end of the asset’s useful life.
Estimates of future cash flows must not include:
 cash inflows or outflows from financing activities; or
 income tax receipts or payments.
Also note that future cash flows are estimated for the asset in its current condition. Therefore,
any estimate of future cash flows should not include estimated future cash flows that are
expected to arise from:
 a future restructuring to which an entity is not yet committed; or
 improving or enhancing the asset’s performance.
The discount rate must be a pre-tax rate that reflects current market assessments of:
 the time value of money; and
 the risks specific to the asset for which the future cash flow estimates have not been
adjusted.
However, both the expected future cash flows and the discount rate might be adjusted to allow for
uncertainty about the future – such as the business risk associated with the asset and expectations
of possible variations in the amount or timing of expected future cash benefits from using the asset.

© Emile Woolf International 158 The Institute of Chartered Accountants of Pakistan


Chapter 4: IAS 36: Impairment of assets

Illustration 01: Measurement of recoverable amount


A company has a machine in its statement of financial position at a carrying amount of
Rs.300,000.
The machine is used to manufacture the company’s best-selling product range, but the entry of a
new competitor to the market has severely affected sales.
As a result, the company believes that the future sales of the product over the next three years will
be only Rs.150,000, Rs.100,000 and Rs.50,000. The asset will then be sold for Rs.25, 000.
An offer has been received to buy the machine immediately for Rs.240,000, but the company
would have to pay shipping costs of Rs.5, 000.The risk-free market rate of interest is 10%.
Market changes indicate that the asset may be impaired and so the recoverable amount for the
asset must be calculated.
Fair value less costs of disposal Rs.
Fair value 240,000
Costs of disposal (5,000)
235,000
Year Cash flow (Rs.000) Discount factor Present value
1 150,000 1/1.1 136,364
2 100,000 1/1.12 82,645
3 50,000 + 25,000 1/1.13 56,349
Value in use 275,358
The recoverable amount is the higher of Rs.235, 000 and Rs.275, 358, i.e. Rs.275, 358.
The asset must be valued at the lower of carrying value and recoverable amount.
The asset has a carrying value of Rs.300, 000, which is higher than the recoverable amount from
using the asset.
It must therefore be written down to the recoverable amount, and an impairment of Rs.24, 642
(Rs.300, 000 – Rs.275, 358) must be recognized.

1.6 Recognition & Accounting for impairment


The impairment loss is normally recognized immediately in profit or loss.

Illustration 02: Measurement of recoverable amount


A company has a machine in its statement of financial position at a carrying amount of
Rs.300,000.
The machine has been tested for impairment and found to have recoverable amount of Rs.275,358
meaning that the company must recognize an impairment loss of Rs.24,642.
This is accounted for as follows:

Debit Credit

Statement of profit or loss 24,642

Accumulated impairment loss 24,642

(Property, plant and equipment would be


presented net of the balance on this account
on the face of the statement of financial
position).

© Emile Woolf International 159 The Institute of Chartered Accountants of Pakistan


Financial accounting and reporting I

Illustration 03:
On 1 January Year 1 Entity Q purchased for Rs.240,000 a machine with an estimated useful life of
20 years and an estimated zero residual value.
Depreciation is on a straight-line basis.
On 1 January Year 4 an impairment review showed the machine’s recoverable amount to be
Rs.100,000 and its remaining useful life to be 10 years.
a) The carrying amount of the machine on 31 December Year 3 (immediately before the
impairment).
The carrying amount of machinery on 31st December year three immediately before the
impairment is calculated as under;

Carrying amount of the machine on 31 December Year 3 Rs.


Cost 240,000
Accumulated depreciation (3 × (240,000 ÷ 20 years)) (36,000)
Carrying amount 204,000

b) The impairment loss recognized in the year to 31 December Year 4 is calculated as under:

Impairment loss at the beginning of Year 4 of Rs.104,000 (Rs.204,000 – Rs.100,000).


This is charged to profit or loss.

c) The depreciation charge in the year to 31 December Year 4.is calculated as under:

Depreciation charge in Year 4 of Rs.10,000 (= Rs.100,000 ÷ 10). The depreciation charge


is based on the recoverable amount of the asset.

Assets having revaluation surplus


However, an impairment loss recognized in respect of an asset carried at a previously recognized
revaluation surplus is recognized in other comprehensive income to the extent that it is covered by
that surplus. Thus it is treated in the same way as a downward revaluation, reducing the revaluation
reserve balance relating to that asset.
Impairment not covered by a previously recognized surplus on the same asset is recognized in
profit or loss.

Illustration 04: Measurement of recoverable amount


A company has a machine in its statement of financial position at a carrying amount of Rs.300,
000 including a previously recognized surplus of Rs.20,000.
The machine has been tested for impairment and found to have recoverable amount of
Rs.275,358 meaning that the company must recognize an impairment loss of Rs.24,642.

This is accounted for as follows:

Debit Credit
Statement of profit or loss 4,642
Other comprehensive income 20,000
Property, plant and equipment 24,642

© Emile Woolf International 160 The Institute of Chartered Accountants of Pakistan


Chapter 4: IAS 36: Impairment of assets

Depreciation of impaired assets


After the recognition of the impairment, the future depreciation of the asset must be based on the
revised carrying amount, minus the residual value, over the remaining useful life.

Illustration 05:
On 1 January Year 1 Entity Q purchased for Rs.240, 000 a machine with an estimated useful life
of 20 years and an estimated zero residual value.
Depreciation is on a straight-line basis.
The asset had been re-valued on 1 January Year 3 to Rs.250, 000, but with no change in useful
life at that date.
On 1 January Year 4 an impairment review showed the machine’s recoverable amount to be
Rs.100, 000 and its remaining useful life to be 10 years.
a) The carrying amount of the machine on 31 December Year 2 and hence the revaluation
surplus arising on 1 January Year 3 is calculated as under:

Carrying amount on Rs.


Cost 240,000
Accumulated depreciation at 1 January Year 3 (2 years × (240,000 ÷ 20)) (24,000)
Carrying amount 216,000
Valuation at 1 January Year 3 250,000
Revaluation surplus 34,000

b) The carrying amount of the machine on 31 December Year 3 (immediately before the
impairment) is calculated as under:
When the asset is revalued on 1 January Year 3, depreciation is charged on the revalued
amount over its remaining expected useful life.
On 31 December Year 3 the machine was therefore stated at:

Rs.
Valuation at 1 January (re-valued amount) 250,000
Accumulated depreciation in Year 3 (= Rs.250,000 ÷ 18)) (13,889)
Carrying amount 236,111

c) The impairment loss recognised in the year to 31 December Year 4 is calculated as under:
On 1 January Year 4 the impairment review shows an impairment loss of Rs.136,111
(Rs.236,111 – Rs.100,000).

An impairment loss of Rs.32,111 (Rs.34,000 Rs.1,889) will be taken to other


comprehensive income (reducing the revaluation surplus for the asset to zero).

The remaining impairment loss of Rs.104,000 (Rs.136,111 Rs.32,111) is recognised in the


statement of profit or loss for Year 4.
d) The depreciation charge in the year to 31 December Year 4 is calculated as under:
Year 4 depreciation charge is Rs.10,000 (Rs.100,000 ÷ 10 years).

© Emile Woolf International 161 The Institute of Chartered Accountants of Pakistan


Financial accounting and reporting I

1.7 Summary of the approach


Impairment of an asset should be identified and accounted for as follows:
(1) At the end of each reporting period, the entity should assess whether there are any
indications that an asset may be impaired.
(2) If there are such indications, the entity should estimate the asset’s recoverable amount.
(3) When the recoverable amount is less than the carrying value of the asset, the entity should
reduce the asset’s carrying value to its recoverable amount. The amount by which the value
of the asset is written down is an impairment loss.
(4) This impairment loss is recognized as a loss for the period.
(5) However, if the impairment loss relates to an asset that has previously been re-valued
upwards, it is first offset against any remaining revaluation surplus for that asset. When this
happens it is reported as other comprehensive income for the period (a negative value) and
not charged against profit.
(6) Depreciation charges for the impaired asset in future periods should be adjusted to allocate
the asset’s revised carrying amount, minus any residual value, over its remaining useful life
(revised if necessary).

EXAMPLE 01: ABA LIMITED


Question: Aba Limited conducts its activities from two properties, a head office in the city centre
and a property in the countryside where staff training is conducted. Both properties were acquired
on 1 April 2013 and had estimated lives of 25 years with no residual value.
The company has a policy of carrying its land and buildings at current values. However, until
recently property prices had not changed for some years.
On 1 October 2015 the properties were revalued by a firm of surveyors. Details of this and the
original costs are:

Land Buildings
Rs. Rs.
Head office – cost 1 April 2013 500,000 1,200,000
– revalued 1 October 2015 700,000 1,350,000
Training premises – cost 1 April 2013 300,000 900,000
– revalued 1 October 2015 350,000 600,000

The fall in the value of the training premises is due mainly to damage done by the use of heavy
equipment during training. The surveyors have also reported that the expected life of the training
property in its current use will only be a further 10 years from the date of valuation. The estimated
life of the head office remained unaltered.
Note: Aba Limited treats its land and its buildings as separate assets. Depreciation is based on
the straight-line method from the date of purchase or subsequent revaluation.

Required:
Prepare extracts of the financial statements of Aba Limited in respect of the above properties for
the year to 31 March 2016.

© Emile Woolf International 162 The Institute of Chartered Accountants of Pakistan


Chapter 4: IAS 36: Impairment of assets

Answer:

ABA Limited
Extracts (Year to 31 March 2016) Rs.

Statement of Profit or loss


Depreciation Expense [24,000 + 18,000 + 30,000 + 30,000] (102,000)
Impairment loss (210,000)

Other comprehensive income


Gain on revaluation [200,000 + 50,000 + 270,000] 520,000

Changes in equity
Revaluation surplus [30,000 – 24,000] (6,000)
Retained earnings 6,000

Statement of financial Position


Land 1,050,000
Building 1,890,000

Revaluation surplus [520,000 – 6,000] 514,000

Land Buildings
Total
Working 1 HO TP HO TP
Rs.
Cost 1 April 2013 500,000 300,000 1,200,000 900,000 2,900,000
2014 Depreciation (48,000) (36,000) (84,000)
2015 Depreciation (48,000) (36,000) (84,000)
1 April 2015 500,000 300,000 1,104,000 828,000 2,732,000
Depreciation (six months) (24,000) (18,000) (42,000)
1 October 2015 (pre-revaluation) 500,000 300,000 1,080,000 810,000 2,690,000
Revaluation gain (loss) 200,000 50,000 270,000 (210,000) 310,000
1 October 2015 (fair value) 700,000 350,000 1,350,000 600,000 3,000,000
Depreciation (six months) (30,000) (30,000) (60,000)
31 March 2016 700,000 350,000 1,320,000 570,000 2,940,000

Depreciation (2014)
Head office Rs. 1,200,000 / 25 years = Rs. 48,000
Training premises Rs. 900,000 / 25 years = Rs. 36,000

Depreciation (1 Apr 2015 to 30 Sep 2015)


Head office Rs. 48,000 x 6/12 = Rs. 24,000
Training premises Rs. 36,000 x 6/12 = Rs. 18,000

Depreciation (1 Oct 2015 to 31 Mar 2016)


Head office Rs. 1,350,000 / 22.5 years x 6/12 = Rs. 30,000
Training premises Rs. 600,000 / 10 years x 6/12 = Rs. 30,000
.

© Emile Woolf International 163 The Institute of Chartered Accountants of Pakistan


Financial accounting and reporting I

EXAMPLE 02: HUSSAIN ASSOCIATES LTD


Question: The assistant financial controller of the Hussain Associates Ltd group has identified the
matters below which she believes may indicate impairment of one or more assets:
Hussain Associates Ltd owns and operates an item of plant that cost Rs. 640,000 and had
accumulated depreciation of Rs. 400,000 at 1 October 2015. It is being depreciated at 12½% on
cost.
On 1 April 2016 (exactly half way through the year) the plant was damaged when a factory vehicle
collided into it. Due to the unavailability of replacement parts, it is not possible to repair the plant,
but it still operates, albeit at a reduced capacity. It is also expected that as a result of the damage
the remaining life of the plant from the date of the damage will be only two years.
Based on its reduced capacity, the estimated present value of the plant in use is Rs. 150,000. The
plant has a current disposal value of Rs. 20,000 (which will be nil in two years’ time), but Hussain
Associates Ltd has been offered a trade-in value of Rs. 180,000 against a replacement machine
which has a cost of Rs. 1 million (there would be no disposal costs for the replaced plant). Hussain
Associates Ltd is reluctant to replace the plant as it is worried about the long-term demand for the
product produced by the plant. The trade-in value is only available if the plant is replaced.
Required:
Prepare extracts from the statement of financial position and statement of profit or loss of Hussain
Associates Ltd in respect of the plant for the year ended 30 September 2016. Your answer should
explain how you arrived at your figures.

Answer:
 The plant had a carrying amount of Rs. 240,000 on 1 October 2015. The accident may have caused
impairment occurred on 1 April 2016. However, as per IFRS 36, the entity will do an impairment
test at the end of the reporting period, i.e., 30 September 2016.
 The depreciation on the plant from 1 October 2015 to 30 Sept 2016 would be Rs. 80,000 (640,000
x 12.5% giving a carrying amount of Rs. 160,000 at the date of impairment. An impairment test
requires the plant’s carrying amount to be compared with its recoverable amount. The recoverable
amount of the plant is the higher of its value in use of Rs. 150,000 or its fair value less costs to sell.
 If Hussain Associates Ltd trades in the plant it would receive Rs. 180,000 by way of a part exchange,
but this is conditional on buying new plant which Hussain Associates Ltd. is reluctant to do. A more
realistic amount of the fair value of the plant is its current disposal value of only Rs. 20,000.
Thus the recoverable amount would be its value in use of Rs. 150,000 giving an impairment loss of
Rs. 10,000 (Rs. 160,000 – Rs. 150,000). Thus extracts from the financial statements for the year
ended 30 September 2016 would be:

Statement of financial position


Non-current assets Rs.
Plant 150,000
Statement of profit or loss
Plant depreciation 80,000
Plant impairment loss 10,000

© Emile Woolf International 164 The Institute of Chartered Accountants of Pakistan


Chapter 4: IAS 36: Impairment of assets

EXAMPLE 03: SKY-LINE LTD (SL)


Question: Sky-Line Limited (SL) operates a 4 Star Hotel facility in Murree. The hotel was
constructed at a cost of Rs.300 million, 5 years back and it is depreciated on a straight-line basis
(total useful life of 15 years and residual value of 20%). There are indications that the property is
not performing as expected due to;

(a) opening of a competing hotel nearby,


(b) a significant drop in number of tourists to the area because of terrorism.
There is a 40% probability that the hotel will generate net cash flows of Rs.40 million per annum
and 60% probability that the cash flows would only be Rs.20 million per annum.
The property’s net operating income is Rs.30 million which is at the rate of 15%. 5% of the
proceeds from sale would be expended in closing the deal.
Required:
Calculate the impairment loss if the appropriate discount rate is 10%.

Answer:

Carrying value of asset = Rs.300 million – [5 x (300 – 60) / 15 years]


= Rs.220 million
Recoverable amount is the higher of fair value less cost to sell and value is use
Fair value less cost to sell = Net operating income / capitalization rate (since no active market) x
(1 - disposal process)
= Rs.30 m / 15% =200 million
= Rs.200 million – 200 million (5%)=190 million
Value in use = Present value of net cash flows discounted at 10% for 10 years
= (Rs.40 m x 0.4 + Rs.20 m x 0.6) x [1- (1.10)-10]/ 10%]
= Rs.28 m x 6.145
= Rs.172 m
Recoverable amount = Rs.190 million
Impairment loss = Rs.30 million

© Emile Woolf International 165 The Institute of Chartered Accountants of Pakistan


Financial accounting and reporting I

EXAMPLE 04: PREMIER LIMITED (PL)


Question: Premier Limited (PL) owns a plant which has a carrying amount of Rs.248 million as
at 1 April 2019. It is being depreciated at 12½% per annum on a reducing balance basis.
The plant is used to manufacture a specific product which has been suffering a decline in sales
due to obsolescence.
PL has estimated that the plant will be retired from use on 31 March 2023.
The estimated net cash flows from the use of the plant and their present values are:

Net cash Present


flows values

Rs. In million

Year to 31 March 2021 120 109.2

Year to 31 March 2022 80 66.4

Year to 31 March 2023 52 39

252 214.6

On 1 April 2020, PL had an alternative offer from the competitor to purchase the plant for Rs.200
million.
Required:
Calculate the impairment loss.

Answer:
At 31 March 2021

Recoverable amount is the higher of value in use [PV of future net cash flows (Rs.214.6 million)
and fair value less costs of disposal (Rs.200 million)].

Carrying amount = Rs.217 million [248 m – (248 m x 12·5%)]

Impairment loss = Carrying amount – Recoverable amount

= Rs.217 million – Rs.214.6 million

= Rs.2.4 million

© Emile Woolf International 166 The Institute of Chartered Accountants of Pakistan


Chapter 4: IAS 36: Impairment of assets

EXAMPLE 05: NAVEED LIMITED


Question: Naveed Limited has an item of plant which has a carrying value of Rs.1,800,000 as at
the end of the year December 2020. It has undergone an impairment review and the following
estimates were produced:
Fair value of plant = Rs.1,400,000
Costs to sell 2% of selling price
Revenue and associated costs per annum for remaining useful life:
(assume all cash flows occur at the end of the year).
Revenue Costs
2021 Rs.960,000 Rs.240,000
2022 Rs.880,000 Rs.220,000
2023 Rs.700,000 Rs.290,000
The plant has an estimated residual value of Rs.50,000.
A discount rate of 10% is applicable to investments equivalent in risk to this plant.
Required:
Calculate the impairment loss if the appropriate discount rate is 10%.

Answer:
Cash flows 2021 2022 2023
--------------- Amount in Rs. --------------
Revenue 960,000 880,000 700,000
Costs (240,000) (220,000) (290,000)
Net Cash Inflow 720,000 660,000 410,000
Discount factor 0.909 0.826 0.751
Present Value 654,545 545,457 308,037

Residual Value (50,000 x 0.751) = 37,566


Value in use = 654,545 + 545,457 + 308,037 + 37,566 = Rs.1,545,605
Fair value less cost to sell = Rs.1,400,000 – 2% of Rs.1.4 million = Rs.1,372,000
Recoverable amount = Rs. 1,545,605
Carrying value = Rs.1,800,000
Impairment loss = Rs.254,395

Example 06: Indus Pharma Limited (IPL)


Question: On 1 July 2014, Indus Pharma Limited (IPL) received a government grant of Rs. 280
million to setup a plant in an under-developed rural area. The grant is repayable in full if the
conditions attached to the grant are not met for a period of five years from the date of
commencement of the production. At the inception, it was highly probable that IPL would comply
with the conditions for the required period.

© Emile Woolf International 167 The Institute of Chartered Accountants of Pakistan


Financial accounting and reporting I

IPL incurred total cost of Rs. 630 million on plant and it started production on1 January 2015.
Useful life of the plant was estimated at 7 years. IPL deducted government grant in arriving at the
carrying amount of the asset.
In January 2019, IPL showed its inability to comply with the conditions attached to the grant and
regulatory authority issued a notice to IPL for repayment of the grant in full. Accordingly, the grant
was repaid by IPL.
In view of repayment of the grant, IPL carried out an impairment review of the plant on 31 December
2019. Net annual cash inflows for the remaining life of the plant have been estimated at Rs. 90
million and Rs. 80 million for 2020 and 2021 respectively. These cash inflows are net of annual
interest and maintenance cost of Rs. 10 million and Rs. 6 million respectively for both years.
Applicable discount rate is 12%.
On the date of impairment review, the existing plant can be sold in the local market for Rs. 160
million. Estimated cost of disposal would be Rs. 5 million.
Required:
Prepare journal entries for the year ended 31 December 2019 in respect of the above information.
(Show all necessary workings. Narrations are not required)

Answer:
Indus Pharma Limited
General Journal
Debit Credit
Date Description
Rs. in million
Jan. 2019 Plant 280
Cash/Bank 280

Jan. 2019/ Depreciation expense/Profit or loss 280÷7×4 160


31-12-2019 Accumulated depreciation - Plant 160

31-12-2019 Depreciation expense 630÷7 90


Accumulated depreciation - Plant 90

31-12-2019 Impairment loss (W-1) 19


Accumulated impairment - Plant 19

W-1: Impairment review as on 31 December 2019 Rs. in million


Year Net inflows Discounting at 12% Present value
2020 (90+10) 100 0.8929 89
2021 (80+10) 90 0.7972 72
Value in use 161

Fair value less cost to sell 160–5 155

Recoverable amount (Higher of both) 161

WDV of the plant as on 31 December 2019 630÷7×2 180


Impairment loss 180–161 19
.

© Emile Woolf International 168 The Institute of Chartered Accountants of Pakistan


Chapter 4: IAS 36: Impairment of assets

Example 07: Harappa Industries


Question: Following information pertain to property, plant and equipment of Harappa Industries
Limited (HIL) for the year ended 30 June 2020:
(i) Balance as on 30 June 2019
Assets Cost/revalued Accumulated Revaluation Depreciation Useful
amount depreciation surplus method life/rate

----------- Rs. in '000 -----------


Land* 100,000 - - - Infinite
Buildings 70,000 14,000 16,000 Straight line 20 years
Plant 180,000 60,000 - Straight line 15 years
Vehicles 8,800 4,000 - Reducing balance 20%

*An amount of Rs. 12 million had been charged to profit or loss upon previous revaluation

(ii) On 30 June 2020, the revalued amounts of the land and buildings were assessed by
Smart Consultant at Rs. 120 million and Rs. 35 million respectively.
(iii) Setting up of a new plant was commenced on 1 July 2019 and substantially completed
on 29 February 2020. The plant was available for use on 1 April 2020 and immediately
put into use. Useful life of the plant was estimated at 10 years. Details of the cost
incurred are as under:
Description Payment date Rs. in '000
1st payment 1 August 2019 12,000

2nd payment 1 October 2019 48,000

3rd payment 29 February 2020 48,000

4th payment 31 July 2020 12,000

120,000

The cost of the plant was financed through an existing running finance facility with a
limit of Rs. 200 million carrying mark-up of 12% per annum. A government grant of Rs.
20 million related to the plant was received on 1 January 2020. The grant amount was
used for repayment of the running facility.
(iv) One of the vehicles had an engine failure on 1 January 2020 and its engine had to be
sold as scrap for Rs. 0.1 million. The vehicle had been acquired on 1 January 2018 at
a cost of Rs. 2.5 million. 40% of the cost is attributable to its engine. Though the engine
of similar capacity was available at a cost of Rs. 1.2 million, the old engine was replaced
on 1 January 2020 with a higher capacity engine at a cost of Rs. 1.8 million.
(v) HIL uses cost model for subsequent measurement of property, plant and equipment
except for land and buildings.
(vi) HIL accounts for revaluation on net replacement value method and transfers the
maximum possible amount from revaluation surplus to retained earnings on an annual
basis.
(vii) HIL deducts government grant in arriving at the carrying amount of the asset.

Required:
In accordance with IFRSs, prepare a note on ‘Property, plant and equipment’ for inclusion in
HIL’s financial statements for the year ended 30 June 2020.(Comparatives figures and column
for total are not required).

© Emile Woolf International 169 The Institute of Chartered Accountants of Pakistan


Financial accounting and reporting I

Answer:
Harappa Industries Limited
Notes to the financial statements for the year ended 30 June 2020

1 Property, plant and equipment:


Land Buildings Plant Vehicles
------------------ Rs. in '000 ------------------
Gross carrying amount - opening 100,000 70,000 180,000 8, 800
Accumulated depreciation - (14,000) (60,000) (4,000)
Opening carrying amount 100,000 56,000 120,000 4,800
(W-
Additions - - 1)102,840 1,800
Depreciation for the year - (3,500) (14,571) (1,068)
(70,000÷20) (W-2) (W-4)
Disposals - - - (W-3) (648)
Revaluation
- Surplus (Bal.) 8,000 (W-5) (15,000) - -
- P&L 12,000 (Bal.) (2,500) - -
Closing carrying amount 120,000 35,000 208,269 4,884

Gross carrying amount - closing 120,000 35,000 282,840 9,600


Accumulated depreciation - - (74,571) (4,716)
Closing carrying amount 120,000 35,000 208,269 4,884

1.1 Land Buildings Plant Vehicles


Cost Cost
Measurement base Revaluation Revaluation model model
15/10
Useful life /depreciation rate Infinite 15 Years years 20%
Straight Straight
Depreciation method - Reducing bal.
line line

1.2 The last revaluation was performed on 30 June 2020 by Smart Consultants, an independent
firm of valuers.

1.3 Had revaluations not made, the carrying value of the land and buildings as on 30 June 2020
would have been Rs. 112 million (100+12) and Rs. 37.5 million (35,000+2,500) respectively.

W-1: Cost - Plant Rs. in '000


Cost 120,000
Government grant (20,000)
Capitalisation of borrowing cost:
1 August - 1 October 2019 12,000×12%×2÷12 240
1 October - 31 December 2019 60,000×12%×3÷12 1,800
1 January - 29 February 2020 (60,000–20,000)×12%×2÷12 800
2,840
102,840

© Emile Woolf International 170 The Institute of Chartered Accountants of Pakistan


Chapter 4: IAS 36: Impairment of assets

W-2: Depreciation – Plant Rs. in '000


On opening balance 180,000÷15 12,000
On the new plant 102,840(W-1)÷10×3÷12 2,571
14,571

W-3: Written down value - Engine disposed off Rs. in '000


Cost 2,500×40% 1,000
Accumulated depreciation:
For the six months ended 30 June 2018 1,000×20%×6÷12 100
For 2018-2019 (1,000–100)×20% 180
(1,000–
For the six months ended 31 December 2019 280)×20%×6÷12 72
352
648

W-4: Depreciation – Vehicles Rs. in '000


On disposal of old engine (W-3) 72
On remaining opening balance [(8,800–1,000)–(4,000–280)]×20% 816
On addition of new engine 1,800×20%×6÷12 180
1,068

W-5: Revaluation surplus – Buildings Rs. in '000


Opening balance 16,000
Incremental depreciation 16,000÷16[(56,000÷70,000)×20] (1,000)
15,000
.

© Emile Woolf International 171 The Institute of Chartered Accountants of Pakistan


Financial accounting and reporting I

2 OBJECTIVE BASED QUESTIONS


01. If the fair value less costs to sell cannot be determined

(a) The asset is not impaired.

(b) The recoverable amount is the value-in-use.

(c) The net realizable value is used.

(d) The carrying value of the asset remains the same.

02. Which TWO of the following could be an indication that an asset may be impaired according
to IAS 36 Impairment of Assets?

(a) Decrease in market interest rates

(b) Increase in market values for the asset

(c) Damage caused to the asset

(d) Management intention to reorganise the business

03. IAS 36 Impairment of Assets contains a number of examples of internal and external events
which may indicate the impairment of an asset.
In accordance with IAS 36, which of the following would definitely NOT be an indicator of the
potential impairment of an asset (or group of assets)?

(a) An unexpected fall in the market value of one or more assets


(b) Adverse changes in the economic performance of one or more assets
(c) A significant change in the technological environment in which an asset is employed
making its software effectively obsolete
(d) The carrying amount of an entity’s net assets being below the entity’s market
capitalisation

04. A fire at the factory on 1 October 2016 damaged the machine, leaving it with a lower
operating capacity. The accountant considers that entity will need to recognise an impairment
loss in relation to this damage. The accountant has ascertained the following information at 1
October 2016:
 The carrying amount of the machine is Rs.60,750.
 An equivalent new machine would cost Rs.90,000.
 The machine could be sold in its current condition for a gross amount of Rs.45,000.
Dismantling costs would amount to Rs.2,000.
 In its current condition, the machine could operate for three more years which gives it
a value in use figure of Rs.38,685.
What is the total impairment loss associated with the above machine at 1 October 2016?

(a) Rs. Nil

(b) Rs.17,750

(c) Rs.22,065

(d) Rs.15,750

© Emile Woolf International 172 The Institute of Chartered Accountants of Pakistan


Chapter 4: IAS 36: Impairment of assets

05. Which of the following is NOT an indicator of impairment?


(a) Advances in the technological environment in which an asset is employed have an
adverse impact on its future use.
(b) An increase in interest rates which increases the discount rate an entity uses.
(c) The carrying amount of an entity’s net assets is higher than the entity’s number of
shares in issue multiplied by its share price.
(d) The estimated net realisable value of inventory has been reduced due to fire damage
although this value is greater than its carrying amount.

06. Cost of disposal are

(a) Incremental costs, directly attributable to the disposal of an asset, excluding finance
costs and income tax expense

(b) Incremental costs, directly attributable to the disposal of an asset, plus finance costs,
but excluding income tax expense

(c) Incremental costs, directly attributable to the disposal of an asset, plus finance costs
and income tax expense

(d) Incremental costs, directly attributable to the disposal of an asset, plus tax expense,
but excluding finance costs

07. An asset is impaired if:

(a) Its carrying amount equals the amount to be recovered through use (or sale) of the
asset

(b) Its carrying amount exceeds the amount to be recovered through use (or sale) of the
asset

(c) The amount to be recovered through use (or sale) of the asset exceeds its carrying
amount

(d) If it has been damaged

08. Value in use is:

(a) The market value

(b) The discounted present value of future cash flows arising from use of the asset and
from its disposal.

(c) The higher of an asset’s fair value less cost to sell and its market value.

(d) The amount at which an asset is recognized in the statement of financial position.

09. IAS 36 applied to which of the following assets:


(a) Inventories.
(b) Financial assets including property plant and equipment and intangible assets
(c) Assets held for sale.
(d) Property, plant, and equipment and intangible assets

© Emile Woolf International 173 The Institute of Chartered Accountants of Pakistan


Financial accounting and reporting I

10. In accordance with IAS 36 Impairment of Assets which of the following statements are true?
1. An impairment review must be carried out annually on all intangible assets.
2. If the fair value less costs to sell of an asset exceed the carrying amount there is no
need to calculate a value in use.
3. Impairment is charged to the statement of profit or loss unless it reverses a gain that
has been recognised in equity in which case it is offset against the revaluation
surplus.
(a) All three

(b) 1 and 2 only

(c) 1 and 3 only

(d) 2 & 3 only

11. What is the recoverable amount of an asset?

(a) Its current market value less costs of disposal

(b) The lower of carrying amount and value in use

(c) The higher of fair value less costs of disposal and value in use

(d) The higher of carrying amount and market value

12. A machine has a carrying amount of Rs. 850,000 at the year end of 31 March 2019. Its market
value is Rs. 780,000 and costs of disposal are estimated at Rs. 25,000. A new machine would
cost Rs. 1,500,000. The company which owns the machine expects it to produce net cash flows
of Rs. 300,000 per annum for the next three years. The company has a cost of capital of 8%.

What is the impairment loss on the machine to be recognised in the financial statements at 31
March 2019?

(a) Rs. 76,870

(b) Rs. 95,000

(c) Rs. 1,66,700

(d) Rs. 220,000

13. IAS 36 Impairment of Assets suggests how indications of impairment might be recognised.
Which TWO of the following would be external indicators that one or more of an entity's
assets may be impaired?

(a) An unusually significant fall in the market value of one or more assets

(b) Evidence of obsolescence of one or more assets

(c) A decline in the economic performance of one or more assets

(d) An increase in market interest rates used to calculate value in use of the assets

© Emile Woolf International 174 The Institute of Chartered Accountants of Pakistan


Chapter 4: IAS 36: Impairment of assets

14. The following information relates to an item of plant.


 Its carrying amount in the statement of the financial position is Rs. 3 million.
 The company has received an offer of Rs. 2.7 million from a company in Karachi
interested in buying the plant.
 The present value of the estimated cash flows from continued use of the plant is Rs.
2.6 million.
 The estimated cost of transport the plant to Karachi is Rs. 50,000.
What is the amount of the impairment loss that should be recognised on the plant?

(a) Rs. 300,000

(b) Rs. 400,000

(c) Rs. 350,000

(d) Rs. 250,000

15. When calculating the estimates of the future cash flows, which of the following cash flows
should not be included?

(a) Cash flows from disposal.

(b) Income tax payments.

(c) Cash flows from the sale of assets produced by the asset.

(d) Cash outflows on the maintenance of the asset.

16. The following information relates to three assets held by a company:


Asset A Asset B Asset C
Rs. m Rs. m Rs. m
Carrying amount 200 100 80
Value in use 160 120 70
Fair value less cost to sell 180 130 60

What is the total impairment loss?

Rs. ___________

17. The following information relates to four assets held by the company:
A B C D
Rs. m Rs. m Rs. m Rs. m
Carrying amount 240 60 80 140
Value in use 160 140 160 40
Fair value less costs to sell 180 80 140 60

What is the total impairment loss?

Rs. ___________

© Emile Woolf International 175 The Institute of Chartered Accountants of Pakistan


Financial accounting and reporting I

18. A vehicle was involved in an accident exactly halfway through the year. The vehicle cost Rs.
10 million and had a remaining life of 10 years at the start of the year. Following the accident,
the expected present value of cash flows associated with the vehicle was Rs. 3.4 million and
the fair value less costs to sell was Rs. 6.5 million.
What is the recoverable amount of the vehicle following the accident?

Rs. ___________

19. Radium Limited (RL) acquired a non-current asset on 1 October 2019 at a cost of Rs. 100
million which had a useful life of ten years and a nil residual value. The asset had been correctly
depreciated up to 30 September 2024.
At that date the asset was damaged and an impairment review was performed. On 30
September 2024, the fair value of the asset less costs to sell was Rs. 30 million and the
expected future cash flows were Rs. 8.5 million per annum for the next five years.
The current cost of capital is 10% and a five year annuity of Rs. 1 per annum at 10% would
have a present value of Rs. 3.79.
What amount would be charged to profit or loss for the impairment of this asset for the year
ended 30 September 2024?

Rs. ___________

20. Metal Limited (ML) owns an item of plant which has a carrying amount of Rs. 248 million as at
1 April 2013. It is being depreciated at 12.5% per annum on a reducing balance basis.
The plant is used to manufacture a specific product which has been suffering a slow decline in
sales. ML has estimated that the plant will be retired from use on 31 March 2017.
The estimated net cash flows from the use of the plant and their present values are:

Net cash flows Present


values

Rs.000 Rs.000

Year to 31 March 2015 120,000 109,200

Year to 31 March 2016 80,000 66,400

Year to 31 March 2017 52,000 39,000

252,000 214,600

On 1 April 2014, Metric had an offer from a rival to purchase the plant for Rs. 200 million
At what value should the plant appear in Metric’s statement of financial position as at 31
March 2014?

Rs. ___________

21. Which of the following is covered by IAS 36 – Impairment?

(a) Non-current assets held for sale

(b) Investment property carried at cost

(c) Investment property carried at fair value

(d) Inventories

© Emile Woolf International 176 The Institute of Chartered Accountants of Pakistan


Chapter 4: IAS 36: Impairment of assets

22. Which of the following is not covered by IAS 36 – Impairment?


(a) Goodwill
(b) Investment property carried at cost
(c) Investment property carried at fair value
(d) Intangible assets

23. When should an impairment loss be recognised?


(a) Immediately
(b) Over a number of accounting periods
(c) At management’s discretion
(d) When requested by the entity’s auditors

24. Value in use is?


(a) The undiscounted present value of future cash flows expected to arise from continuing
use of asset, and from its disposal at the end of its useful life.
(b) The undiscounted future value of present cash flows expected to arise from continuing
use of asset, and from its disposal at the end of its useful life.
(c) The discounted present value of future cash flows expected to arise from continuing
use of asset, and from its disposal at the end of its useful life.
(d) The discounted present value of historical cash flows expected to arise from
continuing use of asset, and from its disposal at the end of its useful life.

25. Which of the following element is not considered while computing value in use?
(a) expectations about possible variations in the amount or timing of those future cash
flows
(b) the time value of money, represented by the current market risk-free rate of interest
(c) the price for bearing the uncertainty inherent in the asset
(d) estimated future restructuring cost

26. In measuring value in use, the discount rate used for discounting the cash flows should be
the?
(a) Pre-tax rate that reflects the market assessment of time value of money and risks
specific to the asset
(b) Pre-tax rate that reflects the market assessment of time value of money and risks
specific to the entity’s competitors
(c) Post-tax rate that reflects the entity’s assessment of time value of money and risks
specific to the asset
(d) Pre-tax rate that reflects the entity’s assessment of time value of money and risks
specific to the asset

© Emile Woolf International 177 The Institute of Chartered Accountants of Pakistan


Financial accounting and reporting I

27. When the recoverable amount of an asset is less than its carrying value in the Statement of
Financial Position, the asset is?

(a) in a revaluation deficit

(b) Flawed

(c) In negative equity

(d) Impaired

28. Which of the following is an internal indication of impairment?

(a) Decline in market value

(b) Worse economic performance than expected

(c) Increase in market interest rates

(d) Technological obsolescence

29. Which of the following is an external indication of impairment?

(a) Physical damage

(b) Worse economic performance than expected

(c) Increase in market interest rates

(d) Asset is part of a restructuring program

30. Under IAS 36, what is the recoverable amount of an asset?

(a) The lower of its cost and net realisable value

(b) The higher of fair value less costs of disposal and value in use

(c) The lower of net present value and cost

(d) The higher of net present value and cost

31. Which of the following is not permitted as a cost to sell under IAS 36?

(a) Cost to dismantle machine

(b) Auctioneers fees

(c) Standard wages for employees

(d) Transport costs for machine

© Emile Woolf International 178 The Institute of Chartered Accountants of Pakistan


Chapter 4: IAS 36: Impairment of assets

32. If the fair value less costs to sell for an asset cannot be determined, then recoverable amount
is equal to its?

(a) Market value

(b) Fair value

(c) Value in use

(d) Replacement value

33. Which of the following is the best evidence of an asset's fair value less costs to sell?

(a) The carrying value of the asset

(b) The price in a binding sale agreement

(c) The disposal value of the asset in an arm`s length transaction

(d) An asset that is traded in an active market

34. When calculating the estimates of future cash flows which of the following cash flows should
not be included?

(a) Cash out flows on the maintenance of the asset

(b) Cash flows from disposal

(c) Cash flows from the sale of inventory produced by the asset

(d) Benefits from future restructuring

35. Under IAS 36 Impairment of Assets, if the fair value less costs to sell of an asset cannot be
determined then:

(a) the asset is not impaired

(b) the recoverable amount is the value in use

(c) the net realizable value is used

(d) the carrying value of the asset remains the same

36. Which TWO of the following would be external indicators that one or more of an entity's assets
may be impaired?

(a) An unusually significant fall in the market value of one or more assets

(b) Evidence of obsolescence of one or more assets

(c) A decline in the economic performance of one or more assets

(d) An increase in market interest rates used to calculate value in use of the assets

© Emile Woolf International 179 The Institute of Chartered Accountants of Pakistan


Financial accounting and reporting I

37. Which of the following future cash flows should NOT be included in the calculation of value in
use of an asset?

(a) Cash flows from disposal

(b) Income tax payments

(c) Cash flows from the sale of inventory produced by the asset

(d) Cash outflows on the maintenance of the asset

38. A plant has a carrying amount of Rs. 1,500,000 as at 31 December 2019. Its fair value is Rs.
900,000 and costs of disposal are estimated at Rs. 50,000. A new plant would cost Rs.
2,500,000. Cash flows from the plant for the next four years are estimated at Rs. 350,000 per
annum. Applicable discount rate is 10%.
What is the approximate impairment loss on the plant to be recognised in the financial
statements as at 31 December 2019?

(a) Rs. 650,000

(b) Rs. 390,000

(c) Rs. 1,000,000

(d) Nil

39. In measuring value in use, the discount rate used for discounting the cash flows should be the:

(a) pre-tax rate that reflects the market assessment of time value of money and risks
specific to the asset

(b) pre-tax rate that reflects the market assessment of time value of money and risks
specific to the entity

(c) post-tax rate that reflects the entity’s assessment of time value of money and risks
specific to the asset

(d) pre-tax rate that reflects the entity’s assessment of time value of money and risks
specific to the asset

© Emile Woolf International 180 The Institute of Chartered Accountants of Pakistan


Chapter 4: IAS 36: Impairment of assets

2 OBJECTIVE BASED ANSWERS


01. (b) The recoverable amount is higher of value in use and fair value less cost
to sell and in case fair value cannot be measured reliably, the recoverable
amount is value in use.

02. (c) & (d) A decrease in interest rates would reduce the discount applied to future
cash flows in calculating the value in use, therefore increasing the value
in use. An increase in market values will lead to the asset value
increasing rather than being impaired.

03. (d) The entity’s market capitalisation would not be reflected within the values
on the statement of financial position.

04. (b) Value in use of Rs.38,685 is lower than fair value less costs to sell of
Rs.43,000, so recoverable amount is Rs.43,000 and impairment is
Rs.60,750 – Rs.43,000 = Rs.17,750.

05. (d) Although the estimated net realisable value is lower than it was (due to
fire damage), the entity will still make a profit on the inventory and thus it
is not an indicator of impairment.

06. (a) Tax and finance costs are not cost of disposal.

07. (b) Asset may not be impaired even after damage. Impairment loss is excess
of carrying amount over recoverable amount.

08. (b) This is definition of value in use

09. (d) (a), (b) and (c) are excluded from scope of IAS 36 as the prudence
mechanism is already incorporated in the relevant standards of these
items.

10. (d) Item 1 is untrue. An annual impairment review is only required for
intangible assets with an indefinite life.

11. (c) The higher of fair value less costs of disposal and value in use.

12. (a)

Fair value – costs of disposal


(780,000 – 25,000) Rs. 755,000

Value in use:

300,000 × 1 / 1.08 277,780

300,000 × 1 / 1.082 257,200

300,000 × 1 / 1.083 238,150

Rs. 773,130

Recoverable amount is Rs. 773,130 and carrying amount is Rs. 850,000,


so impairment is Rs. 76,870.

© Emile Woolf International 181 The Institute of Chartered Accountants of Pakistan


Financial accounting and reporting I

13. (a & d) The other options are internal indicators of impairment.

14. (c)
Rs.

Fair value less costs of disposal (2.7m – 50,000) 2,650,000

Value in use 2,600,000

Recoverable amount is therefore: 2,650,000

Impairment loss (balancing figure) 350,000

Carrying amount 3,000,000

15. (b) Cash flows related to taxations are ignored while calculating value in use.

16. Rs. 30 million 20 + Nil + 10 = Rs. 30 million

17. Rs. 140 million 60 + Nil + Nil +80 = Rs. 140 million

18. Rs. 6.5 million The recoverable amount of an asset is the higher of its value in use
(being the present value of future cash flows) and fair value less costs to
sell. Therefore the recoverable amount is Rs. 6.5 million.

19. Rs. 17.785


million Rs. m

Cost 1 October 2019 100

Depreciation (100 /10 x 5 years) (50)

Carrying amount 50

The recoverable amount is the higher of fair value less costs to sell (Rs.
30 million) and the value in use (Rs. 8.,5 x 3.79 = Rs. 32.215).
Recoverable amount is therefore Rs. 32.215.

Rs. m

Carrying amount 50

Recoverable amount (32.215)

Impairment to statement of profit or loss 17.785

20. Rs. Is the lower of its carrying amount (Rs. 217 million) and recoverable
214,600,000 amount (Rs. 214.6 million) at 31 March 2015.
Recoverable amount is the higher of value in use (Rs. 214.6 million) and
fair value less costs to (Rs. 200 million).
Carrying amount = Rs. 217 million (248 million – (248 million × 12.5%))
Value in use is based on present values = Rs. 214.6 million

21. (b)

© Emile Woolf International 182 The Institute of Chartered Accountants of Pakistan


Chapter 4: IAS 36: Impairment of assets

22. (c)

23. (a)

24. (c)

25. (d)

26. (a)

27. (d)

28. (b)

29. (c)

30. (b)

31. (c)

32. (c)

33. (b)

34. (d)

35. (b) the recoverable amount is the value in use

36. (a) & (d) An unusually significant fall in the market value of one or more assets & An
increase in market interest rates used to calculate value in use of the
assets

37. (b) Income tax payments

38. (b) Rs. 390,000

39. (a) pre-tax rate that reflects the market assessment of time value of money
and risks specific to the asset

© Emile Woolf International 183 The Institute of Chartered Accountants of Pakistan


Financial accounting and reporting I

© Emile Woolf International 184 The Institute of Chartered Accountants of Pakistan


Certificate in Accounting and Finance

CHAPTER
Financial accounting and reporting I

Statement of changes in equity

Contents
1 Statement of changes in Equity
2 Objective based questions and answers

* The student must refer original handbook of IFRS.

© Emile Woolf International 185 The Institute of Chartered Accountants of Pakistan


Financial accounting and reporting I

1 STATEMENT OF CHANGES IN EQUITY (SOCIE)


Section overview

 IAS 1 Preparation of Financial Statements


 The Statement of Changes in Equity (SOCIE)
 Purpose and Importance
 Elements of Statement of Changes in Equity
 Journal Entries
 Presentation of Statement of Changes in Equity

1.1 IAS 1 Preparation of Financial Statements


IAS 1 prescribes the basis for presentation of financial statements for the comparability of
company’s financial position to last year’s performance or with other entities. It sets the
requirement, guidelines for structure and presentation for the preparation of following statements.
A complete set of financial statements consists of:
 A statement of financial position (balance sheet) as at the end of the period;
 A statement of comprehensive income for the period;
 A statement of changes in equity for the period;
 A statement of cash flows for the period
Additionally, notes to these statements, consisting of a summary of significant accounting policies
used by the entity and other explanatory information; and comparative information also form part
of the financial statements.
IAS 1 - Presentation of Financial Statements specifies what published ‘general-purpose’ financial
statements should include, and provides a format for a statement of financial position, statement
of comprehensive income, and statement of changes in equity.
In your earlier studies you have studied about statement of financial position and statement of
comprehensive income. This chapter focuses on the preparation of a statement of changes in
equity.

1.2 The Statement of Changes in Equity (SOCIE)


A change in equity is simply the increase or decrease in the net assets of the entity.
The statement of changes in equity separates owner and non-owner changes in equity in the
following manner:
 Transactions with owners; and
 Non-owner changes in equity, referred to as total comprehensive income.

1.3 Purpose and Importance


The statement of changes in equity actually tells the users about the status of owner’s equity at the
beginning of the financial period, how it has changed during the year and the status of equity at
the end of the period.
It is important to note that the Statement of financial position provides information about the
financial position of the business by presenting total assets, liabilities and equity. Income
statement on the other hand provides information about how business has performed during the
year and how much income has been generated against the expenses and liabilities incurred.
But shareholders are interested in knowing how the business’ financial position and financial
performance has impacted their interest in the business. And this is not particularly addressed by
Statement of Financial Position or Income Statement.

© Emile Woolf International 186 The Institute of Chartered Accountants of Pakistan


Chapter 5: Statement of changes in equity

Thus the statement of changes in equity helps users of financial statement to identify the factors
that cause a change in the owners' equity over the accounting periods.
Therefore, through Statement of Changes in Equity users, especially shareholders can get great
insights about the effects of business operations and related factors on the wealth of the owners
invested in the business.
Examples of the information provided in the statement of changes in equity include share capital
issue and redemption during the period, gains and losses recognised outside profit or loss,
dividends, right shares and bonus shares issued during the period.

1.4 Elements of Statement of Changes in Equity


A statement of changes in equity shows for each component of equity the amount at the beginning
of the period, changes during the period, and its amount at the end of the period that includes:

A). Transactions with owners


The detail of transactions with owners is as under:
 Share capital
 Share premium
 Redemption
 Dividend
 Bonus shares
 Right issue
Share capital
Share capital is the sum of all funds raised by the company in the form of ordinary shares and
(irredeemable) preference shares. Preference shares confer preferential rights for their holders
such entitlement to a dividend out of profits before ordinary shareholders. Once the preference
dividend has been paid, the remaining profit 'belongs' to the ordinary shareholders at the discretion
of the board of directors of the company. The directors may decide to retain some profits (retained
earnings) within the company or declare the dividend.
Preference Shares
Preference shares are of two types: redeemable and irredeemable. Only irredeemable preference
shares are treated as share capital.
Redeemable Preference shares which the company is entitled to redeem in future are treated as
non-current liabilities
Types of Share Capital
Following are the different types of share capital:
Authorized Share Capital
It is the maximum amount of share capital that a company is authorized to raise. A company
does not usually issue the full amount of its authorized share capital. Instead, some of it is held in
reserve by the company for possible future use.
Issued Share Capital
A company may elect to only issue a portion of the total share capital with the plan of issuing more
shares at a later date. Issued share capital is the total value of the shares a company sells. The
issued share capital of a company is the par value of the shares that have actually been issued to
shareholders.
Subscribed Share Capital
When a company issues its authorised share capital, some part of it may not be subscribed by the
investors. Subscribed share capital is the monetary value of all the shares that the investors have
committed to buy.

© Emile Woolf International 187 The Institute of Chartered Accountants of Pakistan


Financial accounting and reporting I

Paid up Share Capital


Paid-up share capital is the amount of money a company has received from shareholders in
exchange for its shares.

Share premium
The difference between the par value of a company’s shares and the total amount a company
received for shares is called Share premium. It is a type of capital reserve. Example: if a Rs. 10
share is sold for Rs, 12 then Rs. 2 is the share premium.

Redemption
It is the reacquisition of the entity’s own equity instruments. A company may redeem its shares for
a number of reasons such as to buy out certain shareholders or to provide an exit strategy to a
third party investor.

Dividend
It is the distribution of profits to shareholders. Many companies pay dividends in two stages during
the course of their accounting year.
(a) In mid-year, after the half-year financial results are published, the company might pay an
interim dividend.
(b) At the end of the year, the company might propose a further final dividend.
According to financial reporting framework the dividends are incorporated as under:
(a) In case of interim dividend, it must be incorporated in the statement of changes in equity of
same year.
(b) In case of final dividend, the final dividend of current year shall be incorporated in the statement
of changes of the next year provided that it has been declared by Directors and approved by
shareholders in annual general meeting. In the current year only disclosure note is given for
final dividend.
The dividend on preference shares may be cumulative or non-cumulative. Any unpaid cumulative
dividends pass to future years and have to be paid out before dividends for ordinary shareholders.
The accounting treatment is as follows:
(a) If preference dividend is cumulative, such dividend for the period need to be taken into account
irrespective of whether declared or not.
(b) If preference dividend is non-cumulative, such dividend for the period need to be taken into
account only for the amount of dividend declared for the period.

Bonus shares
These shares are distributed by a company to its current shareholders free of charge. A bonus
issue does not involve any cash inflow. The company converts some of its reserves (share
premium or retained earnings or both) into new fully-paid share capital issued at its par value.

Right issue
It is an invitation to existing shareholders to purchase additional new shares in proportion to their
shareholding in the company at a discount to the market price on a stated future date.
Say a company with a paid up capital of 10 million shares raises funds by issuing 2 million new
shares. It can offer the new shares to existing shareholder in a '1 for 5' rights issue: each existing
shareholder is offered one new share for every five shares currently held (10 million/2 million = 5).

B). Total comprehensive income


The detail of total comprehensive income is as under:
 Profit for the year & Retained Earnings
 Revaluation Surplus

© Emile Woolf International 188 The Institute of Chartered Accountants of Pakistan


Chapter 5: Statement of changes in equity

Profit for the Year & Retained Earnings


Retained earnings comprise the income (profits and gains less losses) that the company retains
within the business, i.e., income that has not been paid out as dividends or transferred to any other
reserve. A company might hold retained earnings that it has no intention of distributing to owners
as a dividend at any time in the future in a general reserve rather than in retained earnings.
A debit balance on the retained earnings account indicates that the company has accumulated
losses.
Revaluation surplus
The result of an upward revaluation of a non-current asset is a 'revaluation surplus'. The amount
accumulated in revaluation surplus is non-distributable, as it represents un-realised profits on the
revalued assets. It is another capital reserve. The revaluation surplus may diminish if an asset
which had previously been revalued upwards is devalued later. Revaluation surplus is transferred
to retain earnings in case the asset is sold. If due to upward revaluation the incremental
depreciation occurred, then that incremental balance shell be transferred to retained earnings from
the revaluation surplus.
General Reserves
General Reserves are non-statutory reserves consisting of profits which are distributable as
dividends at the discretion of the company. These are also called revenue reserves. Profits are
transferred to these reserves by making an appropriation out of profits, usually profits for the year.

Illustration 01:
Profit after taxation 300,000
Appropriations of profit:
Dividend (100,000)
Transfer to general reserve (50,000)
(150,000)
Retained earnings for the year 150,000
Retained earnings b/f 500,000
Retained earnings c/f 650,000

1.5 Journal Entries


Following journal entries are made in order to record the effects of changes in elements of equity
during the period.
Ref Account / Description Debit Credit
1 Bank X
Share Capital X
Issue of new shares/ right shares at par value
2 Bank X
Share capital X
Share premium X
Issue of new shares/ right shares at premium
3 Retained earnings X
Share capital X
Issue of bonus shares
4 Share premium X
Share capital X
Issue of bonus share (If share premium balance is available)
5 Retained earnings X
Dividend payable X
On approval of interim / final dividend

© Emile Woolf International 189 The Institute of Chartered Accountants of Pakistan


Financial accounting and reporting I

Ref Account / Description Debit Credit


6 Dividend payable X
Bank X
Payment of interim/final dividend
7 Retained earnings X
General reserves X
Transfer of balance to general reserves
8 Revaluation surplus X
Retained earnings X
Transfer of balance in case of incremental depreciation of
revalued assets
9 Revaluation surplus X
Retained earnings X
Transfer of balance in case of disposal of revalued asset
10 Share capital (par value) X
Bank X
Payment of cash for redemption (and cancellation) of shares at par
11 Share capital (par value) X
Share premium / Retained earnings (excess of par value) X
Bank X
Payment of cash for redemption (and cancellation) of shares at premium
12 Share capital (par value) X
Capital repurchase reserve account (discount) X
Bank X
Payment of cash for redemption (and cancellation) of shares at discount
13 Retained earnings X
Capital repurchase reserve account X
Transfer from distributable reserves to capital repurchase reserve account on
redemption
Transfer equal to par value (if redemption is at par value or at premium)
Transfer equal to par value less discount (if redemption is at discount)
Accounting for treasury shares is not examinable at this level.

1.6 Presentation of Statement of Changes in Equity


In accordance with the requirements of IAS 1, an entity is required to present a statement of
changes in equity which includes the following information:
(a) total comprehensive income for the period, showing separately the total amounts attributable
to owners of the parent and to non-controlling interests;
(b) for each component of equity, a reconciliation between the carrying amount at the beginning
and the end of period separately disclosing changes resulting from:
 profit or loss;
 other comprehensive income; and
 transactions with owners in their capacity as owners showing separately
contributions by and distributions to owners

© Emile Woolf International 190 The Institute of Chartered Accountants of Pakistan


Chapter 5: Statement of changes in equity

Format of Statement of Changes in Equity


The format is explained through following illustrations

Illustration 02:
Statement of changes in equity for the year ended 31 December 2019
Share Share Retained Revaluation
Total
Capital Premium earnings Surplus

------------------------- Rs. in million ------------------------


Balance as at 1 January 2019 100 10 60 5 175
Effect of change in policy (see IAS 8) (15) (15)
Effect of correction of error (see IAS 8) 25 25

Balance as at 1 January 2019 (restated) 100 10 70 5 185


Profit after tax for the year - - 10 - 10
Revaluation loss - - - (3) (3)

Balance as at 31 December 2019 100 10 80 2 192

Illustration 03:
Statement of changes in equity for the year ended 31 December 2019 considering there is
redemption, bonus and right issues along with dividends paid during the year
Capital
Share Share Retained
repurchase Total
Capital Premium earnings
reserve
---------- Rs. in million -------
Balance as at 1 January 2019 100 10 - 60 170
Profit after tax for the year - - - 20 20
Bonus issue 10 - - (10) -
Right issue 5 - - - 5
Redemption (10) - 10 (10) (10)
Dividends paid during the year - - - (20) (20)
Balance as at 31 December 2019 105 10 10 40 165

Example 01: SMS Limited


Question: The equity of SMS Ltd as on December 31, 2019 is as follows:
Rs. in million
Total equity at the beginning of the year:
- Share Capital (@ Rs. 10 fully paid ordinary shares) 3,000
- Share premium 1,900
- Retained earnings 4,500
Profit for the year 400
Dividends declared and paid (300)
Total equity at the end of the year 9,500

The company made a bonus issue of 2 to 1.

© Emile Woolf International 191 The Institute of Chartered Accountants of Pakistan


Financial accounting and reporting I

Required:
The Statement of Changes in Equity of SMS Ltd for the year ended on December 31, 2019.
Answer:

Statement of Changes in equity for the year ended on December 31, 2019
Share Share Retained
Total
Capital Premium earnings
------------------------- Rs. in million ------------------------
Balance as at 1 January 2019 3,000 1,900 4,500 9,400
Profit after tax for the year - - 400 400
Bonus shares 6,000 (1,900) (4,100) -
Dividends - - (300) (300)
Balance as at 31 December 2019 9,000 - 500 9,500
.

Example 02: Pak Ocean Limited (POL)


Question: The following information pertains to draft financial statements of Pak Ocean Limited
(POL) for the year ended 31 December 2018.

2018

Shareholders' equity as at 1 January 2018 was as follows: Rs in million

Share capital (Rs. 100 each) 200

Retained earnings 45

Profit after tax 78

Other comprehensive income (gain on revaluation) 12

Incremental depreciation on revaluation of property, plant and 1.5


equipment

On 30 November 2018, POL issued 25% right shares to its ordinary shareholders at Rs. 120
per share.

Cash dividend @ 18% for the year ending 2017 was declared in January 2018

Bonus issue declared during the year ended 31 December 2018:

 Interim(declared with half yearly accounts) 10%

 Final 25%

Required:
Prepare the statement of changes in equity for the year ended December 31, 2018 for Pak Ocean
Limited.

© Emile Woolf International 192 The Institute of Chartered Accountants of Pakistan


Chapter 5: Statement of changes in equity

Answer:

Statement of Changes in Equity for the year ended 31 December 2018


Share Share Retained Revaluation
Total
capital premium earnings Surplus
------------- Rupees in million -------------
Balance as at 1 January 2018 200 - 45 - 245
Total comprehensive income 78 12 90
Transfer from surplus on
revaluation of incremental
depreciation for the period 1.5 (1.5) -
Final cash dividend at 18% for
the year ended 31 December
2017 (200 × 18%) (36) (36)
Interim bonus issue at 10% for
the year ended 31 December
2018 (200 × 10%) 20 - (20) -
25% Right issue at a premium
of Rs. 20 per share (200 + 20)
× 25% 55 11 - 66
Balance as at 31 December
2018 275 11 68.5 10.5 365

Example 03: Daffodil Limited (DL)


Question: For the purpose of preparation of statement of changes in equity for the year ended 31
December 2017, Daffodil Limited (DL) has extracted the following information:

2017 2016 2015


Draft Audited Audited
-------------- Rs. in million -------------
Net profit 650 318 214
Transfer to general reserves 112 - 141
Transfer of incremental depreciation - 49 55
Final cash dividend - - 7.5%

(i) Details of share issues:


 25% right shares were issued on 1 May 2016 at Rs. 18 per share. The market price per
share immediately before the entitlement date was also Rs. 18 per share.
 A bonus issue of 10% was made on 1 April 2017 as final dividend for 2016.
 50 million right shares were issued on 1 July 2017 at Rs. 15 per share. The market price
per share immediately before the entitlement date was Rs. 25 per share.
 A bonus issue of 15% was made on 1 September 2017 as interim dividend.
 The asset which was revalued upward in previous years has been sold on 15 October, 2017.

© Emile Woolf International 193 The Institute of Chartered Accountants of Pakistan


Financial accounting and reporting I

(ii) Share capital and reserves as at 31 December:

2015 2014

------ Rs. in million ------

Ordinary share capital (Rs. 10 each) 1,600 1,600

General reserves 1,850 1,709

Retained earnings 1,430 1,302

Revaluation surplus 75 -

Required:
Prepare DL’s statement of changes in equity for the year ended 31 December 2017 along with
comparative figures.

Answer:
Statement of changes in equity for the year ended 31 December 2017
Ordinary Share General Retained Revaluation
Total
share capital premium reserves earnings Surplus
-------------------- Rs. in million --------------------

Balance as at 31 December 2015 1,600 1,850 1,430 75 4,955


Final cash dividend @ 7.5% - 2015 (120) (120)
(1,600×7.5%)

Right issue @ 25% 400 320 720


(1,600×25%) (160×25%×8)

Net profit – 2016 318 318


Transfer of Incremental (49)
49 0
depreciation

Balance as at 31 December 2016 2,000 320 1,850 1,677 26 5,873


Final bonus dividend @ 10% - 2016 200 (200) -
(2,000×10%)

Right issue 500 250.00 750.00


(50×10) (50×5)
Interim bonus dividend @ 15% - 400 (405) -
2017 (2,700×15%)
Net profit – 2017 650 650
Transfer of revaluation surplus due 26 (26) 0
to disposal of asset
Transfer to general reserves 112 (112) -
Balance as at 31 December 2017 3,105 570.00 1,962 1,636 0 7,273

© Emile Woolf International 194 The Institute of Chartered Accountants of Pakistan


Chapter 5: Statement of changes in equity

Example 04: MK Corporation Limited


Question: MK Corporation Limited, an entity listed in Pakistan Stock Exchange is in the business of
manufacturing and sale of yarn products. Company year-end is December. Below is the relevant
information given:

Opening balances as at January 01, 2018 Rs.

Share Capital (at par value of Rs. 10 per share) 25,000,000

Share Premium 7,500,000

General Reserves 750,000

Retained Earnings 18,250,000

Revaluation Surplus 1,500,000

Following events have taken place in year 2018 and 2019:


1. On March 31, 2018, Company issued Right shares for Rs. 20 per share. Right shares were
issued in the proportion of 1 right share against 5 ordinary shares held.
2. Board of Directors of the Company approved Interim dividend of Rs. 2.25 per share for the half
year ended June 30, 2018.
3. Annual profit for the year ended December 31, 2018 is Rs. 10,250,000.
4. The Board of Directors of the Company recommended annual dividend of Rs. 4.25 per share
on February 15, 2019, which was duly approved by the Shareholders on March 21, 2019.
5. The Board of Directors approved Bonus Shares of 20% of the outstanding shares on June 30,
2019 which were duly credited in Shareholders account on August 31, 2019.
6. Board of Directors of the Company approved Interim dividend of Rs. 1.25 per share for the
third quarter ended September 30, 2019.
7. Annual profit for the year ended December 31, 2019 is Rs. 12,500,000.
8. The Board of Directors of the Company recommended annual dividend of Rs. 5 per share on
February 15, 2020, which was duly approved by the Shareholders on March 21, 2020.
9. The Board of Director approved a transfer 5% of the Annual Profit to General Reserves.
10. Company revalued Fixed assets on December 31, 2017 resulting in Revaluation Surplus of Rs.
1,500,000. Remaining useful life of the Asset is 10 years and Company has a straight line
method for Depreciation.
Required:
Prepare statement of changes in equity for the year ended December 31, 2018 and 2019.

© Emile Woolf International 195 The Institute of Chartered Accountants of Pakistan


Financial accounting and reporting I

Answer:
Statement of changes in equity
Share Share General Retained Revaluation
Description Total
Capital Premium Reserves Earnings Surplus
Rupees
Balance as at Jan
01, 2018 25,000,000 7,500,000 750,000 18,250,000 1,500,000 53,000,000
Issuance of Right
Shares 5,000,000 5,000,000 - - - 10,000,000
Interim Dividend (6,750,000) (6,750,000)
Profit for the year 10,250,000 10,250,000
Transfer 512,500 (512,500) -
Transfer of Revaluation
Surplus to Retained
Earnings 150,000 (150,000) -
Balance at the end of
the year 30,000,000 12,500,000 1,262,500 21,387,500 1,350,000 66,500,000
Annual Dividend (12,750,000) (12,750,000)
Bonus Shares 6,000,000 (6,000,000) - -
Interim Dividend (4,500,000) (4,500,000)
Profit for the year 12,500,000 12,500,000
Transfer 625,000 (625,000) -
Transfer of Revaluation
Surplus to Retained
Earnings 150,000 (150,000) -
Balance at the end of
the year 36,000,000 6,500,000 1,887,500 16,162,500 1,200,000 61,750,000

Example 05: ABC Corporation Limited 1


Question: ABC Corporation Limited, an entity listed in Pakistan Stock Exchange is in the business of
manufacturing and sale of Cars. Company year-end is December. Below is the relevant information
given:

Opening balances as at January 01, 2018: Rs.


Share Capital (at par value of Rs. 10 per share) 100,000,000
Share Premium 50,000,000
General Reserves 5,000,000
Retained Earnings 55,000,000

Following events have taken place in year 2018 and 2019:


1) On February 28, 2018, the Company issued Initial Public Offering, thereby offering 5 million
shares for Rs. 25 each. All shares were subscribed and company received the subscription
money by March 31, 2018.

© Emile Woolf International 196 The Institute of Chartered Accountants of Pakistan


Chapter 5: Statement of changes in equity

2) The Board of Directors of the Company recommended annual dividend for the year ended
December 31, 2017 of Rs. 5 per share on February 15, 2018, whereas Shareholders only
approved Rs. 4 per share on March 31, 2018.
3) Board of Directors of the Company approved Interim dividend of Rs. 1 per share for the third
quarter ended September 30, 2018.
4) Annual profit for the year ended December 31, 2018 is Rs. 130,250,000.
5) The Board of Directors of the Company recommended annual dividend of Rs. 4.25 per share
on February 15, 2019, whereas Shareholders approved Rs. 6 per share on March 31, 2019.
6) The Board of Directors approved Bonus Shares of 10% of the Outstanding shares on June 30,
2019.
7) Board of Directors of the Company approved Interim dividend of Rs. 1.5 per share for the third
quarter ended September 30, 2019.
8) Annual profit for the year ended December 31, 2019 is Rs. 175,000,000.
9) The Board of Directors of the Company recommended annual dividend of Rs. 5.5 per share
on February 15, 2020, which was duly approved by the Shareholders on March 31, 2020.
10) The Board of Directors approved a transfer 5% of the Annual Profit to General Reserves.
Required:
Prepare statement of changes in equity for the year ended December 31, 2018 and 2019.

Answer:
The statement of changes in equity as on December 31, 2018-2019 is as under:

Share Share General Retained


Description Total
Capital Premium Reserves Earnings

Rupees
Balance as at Jan 01,
2018 100,000,000 50,000,000 5,000,000 55,000,000 210,000,000
Issuance of Shares under
IPO 50,000,000 75,000,000 - - 125,000,000
Annual Dividend 17 (40,000,000) (40,000,000)
Interim Dividend (15,000,000) (15,000,000)
Profit for the year 130,250,000 130,250,000
Transfer 6,512,500 (6,512,500) -
Balance at the end of the
year 150,000,000 125,000,000 11,512,500 123,737,500 410,250,000
Annual Dividend (63,750,000) (63,750,000)
Bonus Shares 15,000,000 (15,000,000) -
Interim Dividend (24,750,000) (24,750,000)
Profit for the year 175,000,000 175,000,000
Transfer 8,750,000 (8,750,000) -
Balance at the end of the
year 165,000,000 110,000,000 20,262,500 201,487,500 496,750,000

© Emile Woolf International 197 The Institute of Chartered Accountants of Pakistan


Financial accounting and reporting I

Example 06: ABC Corporation Limited 2


Question: ABC Corporation Limited, an entity listed in Pakistan Stock Exchange is in the business of
manufacturing and sale of Cars. Company year-end is December. Below is the relevant information
given:

Opening balances as at January 01, 2018 Rs.


Share Capital (at par value of Rs. 10 per share) 100,000,000
Share Premium 50,000,000
General Reserves 5,000,000
Retained Earnings 55,000,000

Following events taken place in year 2018 and 2019:


1) On February 28, 2018, the Company issued Initial Public Offering, thereby offering 5 million
shares for Rs. 25 each. Only 75% shares were subscribed and company received the
subscription money by March 31, 2018.
2) The Board of Directors of the Company recommended annual dividend for the year ended
December 31, 2017 of Rs. 5 per share on February 15, 2018, whereas Shareholders only
approved Rs. 4 per share on March 31, 2018.
3) Board of Directors of the Company approved Interim dividend of Rs. 1 per share for the third
quarter ended September 30, 2018.
4) Annual profit for the year ended December 31, 2018 is Rs. 175,000,000.
5) The Board of directors has declared a final dividend for the year ended December, 31,2018
at the rate of 1.65 per share on February 15, 2019, which was duly approved by the
shareholders in annual general meeting on March 21, 2019
6) The Board of Directors approved Bonus Shares of 10% of the Outstanding shares on June 30,
2019 which were duly credited in Shareholders account on August 31, 2019.
7) Board of Directors of the Company approved Interim dividend of Rs. 1.5 per share for the third
quarter ended September 30, 2019.
8) Annual profit for the year ended December 31, 2019 is Rs. 185,250,000.
9) The Board of Directors of the Company has approved a transfer 5% of the Annual Profit to
General Reserves.

Required:
Prepare statement of changes in equity for the year ended December 31, 2018 and 2019.

© Emile Woolf International 198 The Institute of Chartered Accountants of Pakistan


Chapter 5: Statement of changes in equity

Answer:

The statement of changes in equity as on December 31, 2018-2019 is as under:

Share Share General Retained


Description Total
Capital Premium Reserves Earnings
Rupees
Opening Balance 100,000,000 50,000,000 5,000,000 55,000,000 210,000,000
Issuance of Shares under
IPO 37,500,000 56,250,000 - - 93,750,000
Annual Dividend 17 (40,000,000) (40,000,000)
Interim Dividend (13,750,000) (13,750,000)
Profit for the year 175,000,000 175,000,000
Transfer 8,750,000 (8,750,000) -
Balance at the end of the
year 137,500,000 106,250,000 13,750,000 167,500,000 425,000,000
Annual Dividend (22,687,500) (22,687,500)
Bonus Shares 13,750,000 (13,750,000) -
Interim Dividend (22,687,500) (22,687,500)
Profit for the year 185,250,000 185,250,000
Transfer 9,262,500 (9,262,500) -
Balance at the end of the
year 151,250,000 92,500,000 23,012,500 298,112,500 564,875,000
.

Example 07: Wednesday Limited


Question: The following information pertains to Wednesday Limited (WL) for the year ended 30 June
2019:
(i) Shareholders' equity as at 1 July 2018:

Rs. in million
Share capital (Rs. 100 each) 200
Share premium 85
Retained earnings 124
Revaluation surplus 65
(ii) On 30 November 2018, WL issued 30% right shares at a premium of Rs. 120 per share.
(iii) Cash dividend and bonus shares for the last two years:

Final dividend *Interim dividend


For the year ended
Cash Bonus Cash Bonus
30 June 2018 18% - 20% -
30 June 2019 - 25% - 10%
*Declared with half yearly accounts
(iv) Profit for the year amounted to Rs. 95 million.

© Emile Woolf International 199 The Institute of Chartered Accountants of Pakistan


Financial accounting and reporting I

(iv) Revaluation surplus arising during the year amounted to Rs. 35 million whereas transfer of
incremental depreciation for the year was Rs. 9 million.
Required:
Prepare WL’s Statement of Changes in Equity for the year ended 30 June 2019. (Column for total
and comparative figures are not required)

Answer:
Wednesday Limited
Statement of changes in equity
For the year ended 30 June 2019

Share Share Retained Revaluation


capital premium earnings surplus

--------------- Rs. in million ---------------


Balance as at 1 July 2018 (As given) 200 85 124 65
Final cash dividend 200 x 18% (36)
Right issue @ 30% 60 72
Interim bonus dividend 260 x 10% 26 (26)
Total comprehensive income 95 35
Transfer of incremental depreciation 9 (9)
Balance as at 30 June 2019 286 157 166 91
.

© Emile Woolf International 200 The Institute of Chartered Accountants of Pakistan


Chapter 5: Statement of changes in equity

2 OBJECTIVE BASED QUESTIONS


01. Which TWO of the following are separately identified in statement of changes in equity?
(a) Profit for the year
(b) Transactions with owners
(c) Other comprehensive income
(d) Non-owner changes in equity

02. Which of the following is not considered transaction with owners with reference to statement of
changes in equity?
(a) Share capital
(b) Redemption of equity shares
(c) Profit for the year
(d) Bonus issue of shares (no cash received from owners)

03. The maximum amount of share capital that a company is authorized to raise is called:
(a) Authorized share capital
(b) Issued share capital
(c) Subscribed share capital
(d) Paid up share capital

04. The total value of shares a company offers to subscribe is called:


(a) Authorized share capital
(b) Issued share capital
(c) Subscribed share capital
(d) Paid up share capital

05. The monetary value of all the shares that the investors have committed to buy is called:
(a) Authorized share capital
(b) Issued share capital
(c) Subscribed share capital
(d) Paid up share capital

06. The amount of money a company has received from shareholders in exchange for its shares is
called:
(a) Authorized share capital
(b) Issued share capital
(c) Subscribed share capital
(d) Paid up share capital

© Emile Woolf International 201 The Institute of Chartered Accountants of Pakistan


Financial accounting and reporting I

07. Which of the following issued by an entity is treated as liability?


(a) Ordinary share capital
(b) Redeemable preference share capital
(c) Irredeemable preference share capital
(d) None of above

08. A debit balance on the retained earnings account indicates that:


(a) The company has made more dividend payments than the profit earned.
(b) the company has accumulated losses
(c) the company has redeemed some of its share capital
(d) the company has issued bonus shares

09. A company has profit after tax of Rs. 80 million for the financial year ended on 30 June 2019. It
has share capital of Rs. 500 million. During the year company has declared interim dividend of
10%.
How this dividend shall be presented in financial statements for the year ended 30 June 2019?
(a) Rs. 8 million deducted from retained earnings in statement of changes in equity
(b) Rs. 50 million deducted from retained earnings in statement of changes in equity
(c) Rs. 50 million deducted from profit or loss as finance cost
(d) It shall not be recorded, only disclosure shall be made.

10. A company has profit after tax of Rs. 80 million for the financial year ended on 30 June 2019. It
has share capital of Rs. 500 million. The board of directors proposed a final dividend of 10% just
after the year end, for the year ended 30 June 2019
How this dividend shall be presented in financial statements for the year ended 30 June 2019?
(a) Rs. 8 million deducted from retained earnings in statement of changes in equity
(b) Rs. 50 million deducted from retained earnings in statement of changes in equity
(c) Rs. 50 million deducted from profit or loss as finance cost
(d) It shall not be recorded, only disclosure shall be made.

11. Which TWO of the following are usually shown in statement of changes in equity when right issue
of shares is made?
(a) Increase in share capital
(b) Decrease in share premium
(c) Increase in share premium
(d) Increase in retained earnings

12. Which TWO of the following are usually shown in statement of changes in equity when bonus
issue of shares is made?
(a) Increase in share capital
(b) Decrease in share premium
(c) Increase in share premium
(d) Increase in retained earnings

© Emile Woolf International 202 The Institute of Chartered Accountants of Pakistan


Chapter 5: Statement of changes in equity

13. Transaction costs relating to issue of shares are usually debited to:
(a) Profit or loss
(b) Share capital
(c) Share premium
(d) Revaluation surplus

14. If there is no balance in share premium account, transaction costs relating to issue of shares are
usually debited to:
(a) Profit or loss
(b) Share capital
(c) Retained earnings
(d) Revaluation surplus

15. Incremental depreciation has following effects on statement of changes in equity:


(a) Increase in revaluation surplus and decrease in retained earnings
(b) Decrease in revaluation surplus and increase in retained earnings
(c) Decrease in revaluation surplus and decrease in retained earnings
(d) No effect

16. A company has following balances on 1 January 2019:


Rs. m
Share capital (Rs. 100 each) 100
Share premium 30
Revaluation surplus 20
Retained earnings 35
The company made a right issue of 1 for 5 shares already held at Rs. 145 per share.
What amount of share capital shall be presented in statement of changes in equity as at 31
December 2019?

Rs. ___________

17. A company has following balances on 1 January 2019:


Rs. m
Share capital (Rs. 100 each) 100
Share premium 30
Revaluation surplus 20
Retained earnings 35
The company made a right issue of 1 for 5 shares already held at Rs. 145 per share.
What amount of share premium shall be presented in statement of changes in equity as at 31
December 2019?

Rs. ___________

© Emile Woolf International 203 The Institute of Chartered Accountants of Pakistan


Financial accounting and reporting I

18. A company has following balances on 1 January 2019:


Rs. m
Share capital (Rs. 100 each) 100
Share premium 30
Revaluation surplus 20
Retained earnings 35

The company made a bonus issue of 2 for 5 shares already held.


What amount of share capital shall be presented in statement of changes in equity as at 31
December 2019?

Rs. ___________

19. A company has following balances on 1 January 2019:


Rs. m
Share capital (Rs. 100 each) 100
Share premium 30
Revaluation surplus 20
Retained earnings 35

The company made a bonus issue of 2 for 5 shares already held.


What amount of share premium shall be presented in statement of changes in equity as at 31
December 2019?

Rs. ___________

20. A company has following balances on 1 January 2019:


Rs. m
Share capital (Rs. 100 each) 100
Share premium 30
Revaluation surplus 20
Retained earnings 35

On 2 January 2019, all the revalued assets were disposed of for Rs. 90 million.
Profit for the year ended was Rs. 32 million.
Interim dividend of 5% was paid in July 2019 and final dividend of 8% has been proposed by
directors.
What amount of retained earnings shall be presented in statement of changes in equity as at 31
December 2019?

Rs. ___________

21. Which of the following does not appear in statement of changes in equity?
(a) Share premium
(b) Retained earning
(c) Goodwill
(d) Revaluation surplus

© Emile Woolf International 204 The Institute of Chartered Accountants of Pakistan


Chapter 5: Statement of changes in equity

22. Which of the following statements is likely to be true, for a company making profits?
(a) The operating profit will be less than the profit for the year.
(b) The profit for the year will be greater than the gross profit.
(c) Retained profits at the year-end will be greater than shareholders' equity.
(d) Retained profits at the year-end will be greater than retained profits at the beginning of
the year.

23. Which of the following is NOT a component of the statement of changes in equity?
(a) Total comprehensive income for the period
(b) The revaluation gain
(c) The amount of cash that the company has on hand
(d) Dividends paid to shareholders during the period

24. Which of the following statements is not true about preferred stock?
(a) The rate of dividend is usually fixed
(b) Shareholders always have a voting right
(c) Shareholders' usually have a preference as to assets upon liquidation of the corporation
(d) Shareholders' usually have a preference as to dividends

25. Redeemable preferred shares is required to be reported as:


(a) Liability
(b) Equity
(c) Asset
(d) None of the above

26. Any unpaid dividend is carried forward to the future periods for which type of stock?
(a) Ordinary shares
(b) Cumulative preferred shares
(c) Non-cumulative preferred shares
(d) All of the above

27. What is the impact of dividend payments to shareholders on the statement of changes in equity?
(a) It increases the retained earnings balance
(b) It decreases the retained earnings balance
(c) It increases the share capital balance
(d) It decreases the share capital balance

28. What is the impact of an additional share issue on the statement of changes in equity?
(a) It increases the share capital balance
(b) It increases the retained earnings balance
(c) It decreases the share capital balance
(d) It decreases the retained earnings balance

© Emile Woolf International 205 The Institute of Chartered Accountants of Pakistan


Financial accounting and reporting I

29. Xavier Limited issued 5,000 shares of its Rs.10 par value to its shareholder. These shares were
issued at a premium at a price of Rs.25 per share.
The correct journal entry to record this transaction is:
(a) Cash Rs.125,000 (Debit); Share capital Rs.125,000 (Credit)
(b) Cash Rs.50,000 (Debit); Share capital Rs.50,000 (Credit)
(c) Share capital Rs.50,000 (Debit); Share premium Rs.75,000 (Debit); Cash Rs.125,000
(Credit)
(d) Cash Rs.125,000 (Debit); Share capital Rs.50,000 (Credit); Share premium Rs.75,000
(Credit)

30. Dynasty Limited issues 1 million, Rs.10 shares at Rs.50 for each share. Which of the following
statements is true?
(a) Ordinary share capital will increase by Rs.10 million and share premium will increase by
Rs.50 million.
(b) Ordinary share capital will increase by Rs.10 million and share premium will increase by
Rs.40 million.
(c) Ordinary share capital will increase by Rs.20 million and share premium will increase by
Rs.50 million.
(d) Ordinary share capital will increase by Rs.10 million and share premium will increase by
Rs.30 million.

31. Handsome Limited statement of financial position shows ordinary share capital of Rs.150 million
and share premium of Rs.50 million at the beginning of a financial year. If the ordinary share
capital is Rs.250 million and share premium is Rs.120 million at the end of the financial year, how
much did the ordinary share with share premium issue raise?
(a) Rs.100 million
(b) Rs.150 million
(c) Rs.160 million
(d) Rs.170 million

32. Gigantic Limited opening retained earning balance was Rs.150 million. It made a net profit for the
year ended 31 March 2020 of Rs.30 million. During that year, an ordinary dividend of Rs.50 paisa
per share was paid on 40 million ordinary shares. What was the retained profit for the year ended
31 March 2020?
(a) Rs.150 million
(b) Rs.160 million
(c) Rs.165 million
(d) Rs.170 million

© Emile Woolf International 206 The Institute of Chartered Accountants of Pakistan


Chapter 5: Statement of changes in equity

33. SK Limited paid Rs.10 million in debenture interest and an ordinary dividend of 10 paisa per share
on Rs.50 million ordinary shares. The retained profit was Rs.120 million. What was SK Limited
profit for the year?
(a) Rs. 125 million
(b) Rs.135 million
(c) Rs. 130 million
(d) Rs.140 million

34. Which of the following would be an entry in the statement of changes in equity?
(a) Taxation
(b) Long term loans
(c) Revaluation gain
(d) Revaluation reserve

35. During the year ended 30 June 2021, a company's revaluation reserve increased from Rs.
300,000 to Rs. 380,000 as a result of a property (land) revaluation. At the start of that financial
year, the company's property had been valued at Rs. 810,000. Assuming that no property was
disposed of during the year, which of the following statements is true?
(a) The property's revalued amount was Rs.890, 000.
(b) The property's revalued amount was Rs.1, 190,000.
(c) The property's revalued amount was Rs.380, 000.
(d) The property's revalued amount was Rs.1,310,000

36. A debit balance on the retained earnings account indicates that the company has:
(a) made more dividend payments than the profit earned
(b) redeemed some of its share capital
(c) accumulated losses
(d) issued bonus shares

© Emile Woolf International 207 The Institute of Chartered Accountants of Pakistan


Financial accounting and reporting I

2 OBJECTIVE BASED ANSWERS


01. (b) & (d)

02. (c)

03. (a)

04. (b)

05. (c)

06. (d)

07. (b)

08. (b)

09. (b) Rs. 500 million x 10% = Rs. 50 million to be recognized in statement of
changes in equity.

10. (d) This dividend shall be recognized next year. This year the proposed dividend
shall be disclosed only.

11. (a) & (c)

12. (a) & (b)

13. (c)

14. (c)

15. (b)

16. Rs. 120 million Rs. 100 million +


Rs. 100 million / Rs. 100 x 1/5 x Rs. 100] = Rs. 120 million

17. Rs. 39 million Rs. 30 million +


Rs. 100 million / Rs. 100 x 1/5 x Rs. 45] = Rs. 39 million

18. Rs. 140 million Rs. 100 million +


Rs. 100 million / Rs. 100 x 2/5 x Rs. 100] = Rs. 140 million

19. Rs. Nil Rs. 100 million / Rs. 100 x 2/5 x Rs. 100] = Rs. 40 million shares issued
Rs. 30 million from share premium and remaining Rs. 10 million from
retained earnings.

20. Rs. 82 million Rs. 35 million + Rs. 20 million from revaluation surplus + Profit of Rs. 32
million – Rs. 5 million dividends = Rs. 82 million
Proposed dividend shall be disclosed only.

21. (c)

22. (d)

© Emile Woolf International 208 The Institute of Chartered Accountants of Pakistan


Chapter 5: Statement of changes in equity

23. (c)

24. (b)

25. (a)

26. (b)

27. (b)

28. (a)

29. (d)

30. (b)

31. (d)

32. (b)

33. (a)

34. (c)

35. (a)

36. (c) Accumulated losses

© Emile Woolf International 209 The Institute of Chartered Accountants of Pakistan


Financial accounting and reporting I

© Emile Woolf International 210 The Institute of Chartered Accountants of Pakistan


Certificate in Accounting and Finance

6
Financial accounting and reporting I

CHAPTER
IAS 8: Accounting Policies, Changes in
Accounting Estimates and Errors

Contents
1 Key Definitions
2 Accounting policies
3 Accounting estimates
4 Errors
5 Objective based questions and answers

* The student must refer original handbook of IFRS.

© Emile Woolf International 211 The Institute of Chartered Accountants of Pakistan


Financial accounting and reporting I

1 KEY DEFINITIONS
Section overview

Key definitions

Accounting policies are Specific principles, bases, conventions, rules and practices applied by an entity
in preparing and presenting financial statements.
A change in accounting estimate is an adjustment of the carrying amount of an asset or a liability, or
the amount of the periodic consumption of an asset, that results from the assessment of the present
status of, and expected future benefits and obligations associated with, assets and liabilities. Changes
in accounting estimates result from new information or new developments and, accordingly, are not
corrections of errors.
Prior period errors are omissions from, and misstatements in, the entity's financial statements for one
or more prior periods arising from a failure to use, or misuse of, reliable information that:
(a) was available when financial statements for those periods were authorised for issue; and
(b) could reasonably be expected to have been obtained and taken into account in the preparation
and presentation of those financial statements.
Such errors include the effects of mathematical mistakes, mistakes in applying accounting policies,
oversights or misinterpretations of facts, and fraud.
Retrospective application: Applying a new accounting policy to transactions, other events and
conditions as if that policy had always been applied.
Prospective application of a change in accounting policy and of recognising the effect of a change in
an accounting estimate, respectively, are:
(a) applying the new accounting policy to transactions, other events and conditions occurring after the
date as at which the policy is changed; and
(b) recognising the effect of the change in the accounting estimate in the current and future periods
affected by the change.
Retrospective restatement is correcting the recognition, measurement and disclosure of amounts of
elements of financial statements as if a prior period error had never occurred.
Information is material if omitting, misstating or obscuring it could reasonably be expected to influence
decisions that the primary users of general purpose financial statements make on the basis of those
financial statements, which provide financial information about a specific reporting entity.
Materiality depends on the nature or magnitude of information, or both. An entity assesses whether
information, either individually or in combination with other information, is material in the context of its
financial statements taken as a whole.
Applying a requirement is impracticable when the entity cannot apply it after making every reasonable
effort to do so. For a particular prior period, it is impracticable to apply a change in an accounting policy
retrospectively or to make a retrospective restatement to correct an error if:
(a) the effects of the retrospective application or retrospective restatement are not determinable;
(b) the retrospective application or retrospective restatement requires assumptions about what
management's intent would have been in that period; or
(c) the retrospective application or retrospective restatement requires significant estimates of amounts
and it is impossible to distinguish objectively information about those estimates that:
(i) provides evidence of circumstances that existed on the date(s) as at which those amounts
are to be recognised, measured or disclosed; and
(ii) would have been available when the financial statements for that prior period were
authorised for issue from other information.

© Emile Woolf International 212 The Institute of Chartered Accountants of Pakistan


Chapter 6: IAS 8: Accounting policies, changes in accounting estimates and errors

2 ACCOUNTING POLICIES
Section overview

 Introduction to IAS 8
 Accounting policies
 Selection of accounting policies
 Changes in accounting policies
 Retrospective application of a change in accounting policy
 Limitation on retrospective application
 Disclosure of a change in accounting policy

2.1 Introduction to IAS 8


The aim of IAS 8: Accounting policies, changes in accounting estimates and errors is to enhance
comparability of the entity’s financial statements to previous periods and to the financial statements
of other entities.
It does this by establishing:
 the criteria for selecting accounting policies; and,
 the accounting treatment and disclosure of:
 changes in accounting policies;
 changes in accounting estimates; and
 errors.
Much of IAS 8 is concerned with how changes or corrections should be reported in the financial
statements.
2.2 Accounting policies
IFRSs set out accounting policies that result in financial statements containing relevant and reliable
information about the transactions, other events and conditions to which they apply. Those policies
need not be applied when the effect of applying them is immaterial.
2.3 Selection of accounting policies
Selection of accounting policies – Areas covered by IFRS
If an IFRS (or an Interpretation) applies to an item in the financial statements, the accounting policy
or policies applied to that item must be determined by applying the Standard or Interpretation and
any relevant implementation guidance issued.
Selection of accounting policies – Area not covered by IFRS
If there is no rule in IFRS that specifically applies to an item in the financial statements,
management must use its judgement to develop and apply an accounting policy that results in
information that is:
 relevant to the economic decision-making needs of users; and
 reliable in that the financial statements:
- represent faithfully the financial performance, financial position and cashflows of the
entity;
- reflect the economic substance of transactions and other events, and not merely the
legal form;
- are neutral, i.e. free from bias;
- are prudent; and
- are complete in all material respects.
In making the judgement management must consider the following sources in descending order:
 the requirements and guidance in IFRS dealing with similar and related issues;
 the definitions, recognition criteria and measurement concepts for assets, liabilities, income
and expenses set out in the “Framework”.

© Emile Woolf International 213 The Institute of Chartered Accountants of Pakistan


Financial accounting and reporting I

Management may also consider the most recent pronouncements of other standard-setting bodies
that use a similar conceptual framework to the extent that these do not conflict with the above
sources.
Professional Judgement
Sometime, it may be difficult to understand how an accounting policy would be applied and you
may need to use your professional judgement. Professional judgement is required usually when
you want to develop your own accounting policy for certain transaction.
The professional judgement should be based on:
a) standards and interpretations on similar issues;
b) the definitions, recognition criteria and measurement concepts in the Conceptual Framework;
and on condition that they do not conflict with the standards and interpretations on similar issues
and the Conceptual Framework, we are also allowed to consider recent pronouncements from
other standard setting bodies, other accounting literature and industry accepted practices.
Consistency of accounting policies
An entity must apply consistent accounting policies in a period to deal with similar transactions,
and other events and circumstances, unless IFRS specifically requires or permits categorisation of
items for which different policies may be appropriate.
Illustration 01: Consistency
IAS 16: Property, plant and equipment allows the use of the cost model or the revaluation model
for measurement after recognition.
This is an example of where IFRS permits categorisation of items for which different policies may
be appropriate.
If chosen, each model must be applied to an entire class of assets. Each model must be applied
consistently within each class that has been identified.

Illustration 02:
Company A has 2 machines, costing Rs. 1 million and 5 million respectively. Company A decides to
use Cost model for 1 Machine and Revaluation Model for 1 Machine. Are they allowed to do this as
per IAS 8?
No, Accounting policy for a specific class of assets shall be consistent. Therefore, Company A may
either use Cost Model or Revaluation Model for all assets under the category of Machines.

Illustration 03:
Company A has Land costing Rs. 100 million, Plant & Machinery Costing Rs. 500 million, Vehicles
costing Rs. 10 million and Electrical equipment costing Rs. 5 million. Company wish to apply Cost
Model for Plant & Machinery, Vehicles and Electrical equipment and Revaluation Model for Land.
Whether Company A is permitted under IAS 8 to do this?
Yes, as the Company is applying same policy for a category of asset, it is allowed to apply different
policies for different class / category of assets.

2.4 Changes in accounting policies


Users of financial statements need to be able to compare financial statements of an entity over
time, so that they can identify trends in its financial performance or financial position. Frequent
changes in accounting policies are therefore undesirable because they make comparisons with
previous periods more difficult.
The same accounting policies must be applied within each period and from one period to the next
unless a change in accounting policy meets one of the following criteria. A change in accounting
policy is permitted only if the change is:
 required by IFRS; or
 results in the financial statements providing reliable and more relevant financial
information.
A new or revised standard usually include specific transitional provisions’ to explain how the
change required by the new rules should be introduced.

© Emile Woolf International 214 The Institute of Chartered Accountants of Pakistan


Chapter 6: IAS 8: Accounting policies, changes in accounting estimates and errors

In the absence of specific transitional provisions, a change in policy should be applied


retrospectively. This is explained shortly.
If at least one of these criteria is changed, then there is a change in accounting policy.
Illustration 04: Changes in accounting policies
 Valuation of inventory using FIFO or weighted average cost method
 Measurement of financial assets and liabilities
 Method used to measure non-current assets such as historical cost or revaluation model
 Accruals basis of preparation of financial statements
 Presentation (e.g. an entity changes from presenting a classified statement of financial
position (current and non-current assets and current and non-current liabilities shown as
separate classifications) to a liquidity presentation (items presented in order of liquidity
without current/non-current classification)

Illustration 05:
IAS 23 requires the capitalisation of borrowing costs directly attributable to the acquisition,
construction or production of a qualifying asset.
Previously, IAS 23 allowed companies to expense or capitalise borrowing costs.
The revision to IAS 23 led to a change in accounting policy for some companies as it affected:
 recognition – the interest cost previously recognised as an expense had to be recognised as
an asset; and
 presentation – the interest cost previously presented in the statement of profit or loss had
to be presented in the statement of financial position.
IAS 8 specifies that the application of a new accounting policy to transactions or events that did
not occur previously or differ in substance from those that occurred previously, is not a change of
accounting policy. It is simply the application of a suitable accounting policy to a new type of
transaction.
Important Note:
Usually a question is being asked as to why change from Cost Model to Revaluation Model is not
accounted for under IAS 8, the answer is:
The initial application of a policy to revalue assets in accordance with IAS 16 Property, Plant and
Equipment or IAS 38 Intangible Assets is a change in an accounting policy. However, it is
accounted for in accordance guidance in those standards rather than in accordance with IAS 8.
IAS 8 specifically mentions that when there is a change in accounting policy, the accounting
standard which was applicable on that element of financial statements be looked into first and if
the treatment with regard to its accounting is given then such treatment will be used, in case no
treatment of such transaction is given under relevant accounting standard, then IAS 8 will be
followed, as per which in case of change in Accounting Policy, there is a retrospective application.

2.5 Retrospective application of a change in accounting policy


When a change in accounting policy is required, and there are no transitional provisions relating
to the introduction of a new accounting standard, the change in policy should be applied
retrospectively.
The entity should adjust the opening balance for each item of equity affected by the change, for
the earliest prior period presented, and the other comparative amounts for each prior period
presented, as if the new accounting policy had always been applied.
IAS 1: Presentation of Financial Statements requires a statement of financial position at the
beginning of the earliest comparative period when a new accounting policy is applied
retrospectively.
Illustration 06:
A company presents comparatives for the previous year only.
During the year ended 31 December 2016 it changes an accounting policy and this change must
be applied retrospectively.
If there were no change in accounting policy the company would present statements of financial
position as at December 2016 and December 2015 only.

© Emile Woolf International 215 The Institute of Chartered Accountants of Pakistan


Financial accounting and reporting I

However, because there is a change in policy the company must also present a statement of
financial position as at 1 January 2015 (the beginning of the earliest comparative period).
The change in accounting policy is applied retrospectively. This means that the change should be
applied to the balances at as at 1 January 2015 as if the new policy had always been applied.
Similarly, any other comparative amounts in previous periods should be adjusted as if the new
accounting policy had always been applied.

If this is impracticable, retrospective application should be applied from the earliest date that is
practicable.
2.6 Limitation on retrospective application
It might be impracticable to retrospectively apply an accounting policy. This could be because the
information necessary for the application of the policy to earlier periods is not available because it
had not been collected then.
There are different degrees of impracticability.
Period specific effect
It might be impracticable to determine the effect of changing an accounting policy on comparative
information for one or more prior periods presented. For example, it might be impracticable to
determine the impact on profit for the prior year.
In this case a company must apply the new accounting policy to the carrying amounts of assets
and liabilities (and therefore equity) as at the beginning of the earliest period for which retrospective
application is practicable. This may be the current period.
Cumulative effect
It might be impracticable to determine the cumulative effect, at the beginning of the current period,
of applying a new accounting policy to all prior periods,
In this case a company must adjust the comparative information to apply the new accounting policy
prospectively from the earliest date practicable.
When the cumulative effect of applying the policy to all prior periods cannot be determined, a
company must apply the new policy prospectively from the start of the earliest period practicable.
This means that it would disregard the portion of the cumulative adjustment to assets, liabilities
and equity arising before that date.
Example 01: ASIF ENGINEERING LIMITED
Question: The following information has been taken from the financial statements of Asif
Engineering Limited (AEL) for the year ended 31 December 2015:
2015 (draft) 2014 2013
---------- Rs. in million ----------
Property, plant equipment 2,430 2,402 2,105
Stores and spares 73 80 70
Retained earnings as at 31 December 353 224 101
Net profit 129 123 112

In the above financial statements, AEL has recognised consumption of spare parts as expense. AEL
has now decided to change its above policy and classify consumption of spares having useful life
of more than one year as capital spares under property, plant and equipment.

Following information pertains to capital spares consumed during the past three years:
Parts issued during the year Useful life of the issued
Year ended
Rs. in million parts
31 December 2013 55 5 years
31 December 2014 39 3 years
31 December 2015 44 4 years

Depreciation on these parts is to be charged using straight line method over its useful life.

© Emile Woolf International 216 The Institute of Chartered Accountants of Pakistan


Chapter 6: IAS 8: Accounting policies, changes in accounting estimates and errors

Required:
In accordance with the requirements of International Financial Reporting Standards, prepare the
revised extracts (including comparative figures) of the following:
(a) statement of financial position as at 31 December 2015
(b) statement of comprehensive income for the year ended 31 December 2015
(c) statement of changes in equity for the year ended 31 December 2015
(ignore taxation)

Answer:
(a) Statement of financial position as at 31 December 2015
2015 2014 2013
(Restated) (Restated)
Extracts from statement of financial position --------- Rs. in million ---------
Property, plant & equipment (W-2) 2,498 2,461 2,149
Stores and spares 73 80 70
Retained earnings 421 283 145

(b) Statement of comprehensive income for the year ended 31 December 2015
2015 2014 (Restated)
Extracts from statement of comprehensive income --------- Rs. in million ---------
Net profit (W-1) 138 138

(c) Statement of changes in equity for the year ended 31 December 2015 (ignore taxation)
Retained earnings
Extracts from statement of changes in equity Rs. in million
Balance as at 1 January 2014 101
Effect of retrospective change in accounting policy (W-1) 44
Balance at 1 January 2014 – restated 145
Total comprehensive income – 2014 (W-1) 138
Balance as at 1 January 2015 – (restated) 283
Total comprehensive income – 2015 138
Balance as at 31 December 2015 421

Depreciation expense for the year


2013 2014 2015
W-1: Computation of net profit ---------- Rs. in million ----------
Depreciation for 2013 (55÷5) 11 11 11
Depreciation for 2014 (39÷3) - 13 13
Depreciation for 2015 (44÷4) - - 11
11 24 35
Less: Amount already charged 55 39 44
Adjustment to be made in net profit 44 15 9
Profit for the year 123 129
Adjusted profit for the year 138 138

© Emile Woolf International 217 The Institute of Chartered Accountants of Pakistan


Financial accounting and reporting I

W-2: Property, plant and equipment


As given 2,105 2,402 2,430
Add: Stores issued 2013 55 55 55
Add: Stores issued 2014 0 39 39
Add: Stores issued 2015 0 0 44
Less: Accumulated depreciation as
calculated above
2014: 11 + 24; 2015: 11 + 24 + 35 (11) (35) (70)
Revised carrying value 2,149 2,461 2,498

2.7 Disclosure of a change in accounting policy


When a change in accounting policy has an effect on the current period or any prior period (or
would have an affected that period except that it is impracticable to determine the amount of the
adjustment) or might have an effect on future periods the following must be disclosed:

Disclosure: Change due Voluntary


to IFRS change

The title of the IFRS 

The nature of the change in accounting policy  

When applicable, that the change in accounting policy is



made in accordance with its transitional provisions

A description of any transitional provisions 

When applicable, the transitional provisions that might have



an effect on future periods

The reason why the new accounting policy provides reliable



and more relevant information

For the current and previous period(s), to the extent


practicable, the amount of the adjustment to each item in the
 
financial statements and if IAS 33 Earnings per Share applies
to the entity, for basic and diluted earnings per share

To the extent practicable, the adjustment relating to


accounting periods before those presented in the financial  
statements

If retrospective application is impracticable, an explanation of


 
how the accounting policy change has been applied

© Emile Woolf International 218 The Institute of Chartered Accountants of Pakistan


Chapter 6: IAS 8: Accounting policies, changes in accounting estimates and errors

3 ACCOUNTING ESTIMATES
Section overview

 Accounting estimates
 Changes in accounting estimates
 Disclosures

3.1 Accounting estimates


An accounting estimate is made for an item in the financial statements when the item cannot be
measured with precision, and there is some uncertainty about it.
An estimate is therefore based, to some extent, on management’s judgement. Management
estimates might be required, for example, for the following items:
 bad debts;
 inventory obsolescence;
 the fair value of financial assets or liabilities;
 the useful lives of non-current assets;
 the most appropriate depreciation pattern (depreciation method, for example straight line or
reducing balance) for a category of non-current assets;
 measurement of warranty provisions.
The use of reasonable estimates is an essential part of the preparation of financial statements and
does not undermine their reliability.
Accounting policy vs accounting estimate
It is important to distinguish between an accounting policy and an accounting estimate.
Sometimes it can be difficult to distinguish between changes in accounting policy from changes in
accounting estimate. In such cases any change is treated as a change in accounting estimate.

Illustration 07:
Accounting policy: Depreciating plant and equipment over its useful life.
Accounting estimate: How to apply the policy. For example whether to use the straight line method
of depreciation or the reducing balance method is a choice of accounting estimate.

A change in the measurement basis applied is a change in an accounting policy and is not a
change in an accounting estimate.

Illustration 08:
IAS 16: Property, plant and equipment allows the use of the cost model or the revaluation model
for measurement after recognition.
This is a choice of accounting policy.

3.2 Changes in accounting estimates


A change in accounting estimate may be needed if changes occur in the circumstances on which
the estimate was based, or if new information becomes available. A change in estimate is not the
result of discovering an error in the way an item has been accounted for in the past and it is not a
correction of an error.
IAS 8 requires a change in an accounting policy to be accounted for retrospectively whereas a
change in an accounting estimate is normally recognised from the current period.

© Emile Woolf International 219 The Institute of Chartered Accountants of Pakistan


Financial accounting and reporting I

The effect of a change in accounting estimate should be recognised prospectively, by including it:
 in profit or loss for the period in which the change is made, if the change affects that period
only, or
 in profit or loss for the period of change and future periods, if the change affects both.
To the extent that a change in estimate results in a change in assets and liabilities, it should be
recognised by adjusting the carrying amount of the affected assets or liabilities in the period of
change.

Illustration 09:
A non-current asset was purchased for Rs. 200,000 two years ago, when its expected economic life
was ten years and its expected residual value was nil. The asset is being depreciated by the straight-
line method.
A review of the non-current assets at the end of year 2 revealed that due to technological change,
the useful life of the asset is only six years in total, and the asset therefore has a remaining useful
life of four years.
The original depreciation charge was Rs. 20,000 per year (Rs. 200,000/10 years) and at the beginning of
Year 2, its carrying value was Rs. 180,000 (Rs. 200,000 - Rs. 20,000).
The change in the estimate occurs in Year 2. The change in estimate should be applied
prospectively, for years 2 onwards (years 2 – 6). From the beginning of year 2, the asset has a
revised useful remaining life of five years.
The annual charge for depreciation for year 2 (the current year) and for the future years 3 – 6 will
be changed from Rs. 20,000 to Rs. 36,000 ( Rs. 180,000/5 years).

Illustration 10:
A generator was purchased in Year 2018 at the cost of Rs. 50 million. It was estimated in Year
2018 that it will run for 80,000 hours. Company estimates that each year, it will run 8,000 hours
i.e. 10% of estimated generation. The Company intends to use Straight line method for
depreciation.
In year 1, Generator ran for 8,000 hours, however, in year 2, it ran for 7,200 hours, decline of 10%.
On re assessment, the management concluded that their initial assessment of running of 8,000
hour per year for generator needs revision, and each year, the generator will run for 90% of the
hours it ran in last year, i.e. it will give more benefit in year 1 and the benefit will get reduced by
10% each year. Therefore, now Company intends to apply reducing balance / diminishing balance
method of the remaining carrying amount of this asset.

3.3 Disclosures
The following information must be disclosed:
 The nature and amount of a change in an accounting estimate that has an effect in the
current period or is expected to have an effect in future periods, except for the effect on
future periods when it is impracticable to estimate that effect.
 The fact that the effect in future periods is not disclosed because estimating it is
impracticable (if this is the case).

© Emile Woolf International 220 The Institute of Chartered Accountants of Pakistan


Chapter 6: IAS 8: Accounting policies, changes in accounting estimates and errors

4 ERRORS
Section overview

 Errors
 Correction of prior period errors
 Limitation on retrospective restatement
 Disclosure of prior period errors

4.1 Errors
Errors can arise in respect of the recognition, measurement, presentation or disclosure of elements
of financial statements. Financial statements do not comply with IFRSs if they contain either
material errors or immaterial errors made intentionally to achieve a particular presentation of an
entity’s financial position, financial performance or cash flows. . If they are discovered quickly, they
are corrected before the financial statements are published. When this happens, the correction of
the error is of no significance for the purpose of financial reporting.
A problem arises, when an error is discovered that relates to a prior accounting period. For
example, in preparing the financial statements for Year 3, an error may be discovered affecting the
financial statements for Year 2, or even Year 1.

4.2 Correction of prior period errors


All material prior period errors should be corrected retrospectively in the first set of financial
statements following the discovery of the error.
Comparative amounts for the previous period should be re-stated at their corrected amount.
If the error occurred before the previous year, the opening balances of assets, liabilities and equity
for the previous period should be re-stated at their corrected amount unless that is impracticable.
The correction of a prior period error is excluded from profit or loss in the period when the error
was discovered.

Illustration 11:
In preparing its financial statements for 31 December 2017 Company A discovers an error affecting
the 31 December 2016 financial statements.
The error should be corrected in the 31 December 2017 financial statements by restating the
comparative figures for 31 December 2016 at their correct amount.
If the error had occurred in 31 December 2015, the comparative opening balances for the
beginning of 31 December 2016 should be restated at their correct amount.
The reported profit for 31 December 2017 is not affected.

Illustration 12:
DEF is preparing its financial statements for 2017.
The draft statement of changes in equity is as follows:
Share Share Retained Total
capital premium earnings
Rs.000 Rs.000 Rs.000 Rs.000
Balance at 31/12/15 500 50 90 640
Profit for the year - - 150 150
Balance at 31/12/16 500 50 240 790

© Emile Woolf International 221 The Institute of Chartered Accountants of Pakistan


Financial accounting and reporting I

2017

Dividends - - (100) (100)

Profit for the year - - 385 385

Balance at 31/12/17 500 50 525 1,075

DEF has now discovered an error in its inventory valuation. Inventory was overstated by Rs. 70,000
at 31 December 2017 and by Rs. 60,000 at 31 December 2016. (Ignore taxation)
The error in 2017 is corrected against the current year profit.
The error in 2016 is corrected against the prior year profit. (Note that the 2016 closing inventory is
the opening inventory in 2017 so the 2016 adjustment will impact both periods statements
comprehensive income.

Profit adjustments: 2017 2016

Rs.000 Rs.000

Profit (2017 draft and 2016 actual) 385 150

Deduct error in closing inventory (70) (60)

Add error in opening inventory 60

(10) (60)

Adjusted profit 375 90

The statement of changes in equity as published in 2017 becomes:

Share Share Retained Total


capital premium earnings

Rs.000 Rs.000 Rs.000 Rs.000

Balance at 31/12/15 500 50 90 640

Profit for the year (restated) - - 90 90

Balance at 31/12/16 500 50 180 730

2017

Dividends - - (100) (100)

Profit for the year - - 375 375

Balance at 31/12/17 500 50 455 1,005

Example 02: Mohani Manufacturing Limited


Question: Mohani Manufacturing Limited is engaged in manufacturing of spare parts for motor
car assemblers. The audited financial statements for the year ended December 31, 2014 disclosed
that the profit and retained earnings were Rs. 21 million and Rs. 89 million respectively. The draft
financial statements for the year show a profit of Rs. 15 million. However, following adjustments
are required to be made:

© Emile Woolf International 222 The Institute of Chartered Accountants of Pakistan


Chapter 6: IAS 8: Accounting policies, changes in accounting estimates and errors

(i) The management of the company has decided to change the method for valuation of raw
materials from FIFO to weighted average. The value of inventory under each method is as
follows:

FIFO Weighted Average

Rs. m Rs. m

December 31, 2013 37.0 35.5

December 31, 2014 42.3 44.5

December 31, 2015 58.4 54.4

(ii) In 2014, the company purchased a plant for Rs. 100 million. Depreciation on plant was
recorded at Rs. 25 million instead of Rs. 10 million. This error was discovered after the
publication of financial statements for the year ended December 31, 2014. The error is
considered to be material.
Required:
Produce an extract showing the movement in retained earnings, as would appear in the
statement of changes in equity for the year ended December 31, 2015.
Answer:

Mohani Manufacturing (Private) Limited Retained


Statement of changes in equity Earnings
For the year ended December 31, 2015 Rs. in million
Balance at December 31, 2013 as previously reported (Rs. 89m –
Rs. 21m) 68.00
Effect of change in accounting policy (Rs. 37m - Rs. 35.5m) (1.50)
Balance at December 31, 2013 – restated 66.50
Profit for the year ended December 31, 2014 - restated (W1) 39.70
Balance at December 31, 2014 – restated 106.20
Profit for the year ended December 31, 2015 (W2) 8.80
Balance at December 31, 2015 115.00

W1: Profit for the year ended December 31, 2014 (as restated)
Profit as previously reported 21.00
Incorrect recording of depreciation (Rs. 25 million – Rs. 10 million) 15.00

Reversal of FIFO method


Opening inventory 37.00
Closing inventory (42.30)
(5.30)

Application of weighted average method


Opening inventory (35.50)
Closing inventory 44.50
9.00
39.70

© Emile Woolf International 223 The Institute of Chartered Accountants of Pakistan


Financial accounting and reporting I

W2: Adjusted profit for year ended June 30, 2015


Profit as per draft financial statements 15.00
Adjustment in Opening Inventory
FIFO 42.30
Weighted average (44.50)
(2.20)
Adjustment in Closing Inventory
FIFO (58.40)
Weighted average 54.40
(4.00)
Adjusted profit 8.80

4.3 Limitation on retrospective restatement


A prior period error must be corrected by retrospective restatement except to the extent that it is
impracticable to determine either the period-specific effects or the cumulative effect of the error.
Period specific effect
It might be impracticable to determine the effect of correcting an error in comparative information
for one or more prior periods presented. For example, it might be impracticable to determine the
impact on profit for the prior year.
In this case a company must restate the carrying amounts of assets and liabilities (and therefore
equity) as at the beginning of the earliest period for which retrospective restatement is practicable.
This may be the current period.
Cumulative effect
It might be impracticable to determine the cumulative effect, at the beginning of the current period,
of correcting an error in all prior periods.
In this case a company must correct the error prospectively from the earliest date practicable.

4.4 Disclosure of prior period errors


The following information must be disclosed:
 the nature of the prior period error;
 for each period presented in the financial statements, and to the extent practicable, the
amount of the correction for each financial statement item and the change to basic and fully
diluted earnings per share;
 the amount of the correction at the beginning of the earliest prior period in the statements
(typically, at the start of the previous year);
 if retrospective re-statement is not practicable for a prior period, an explanation of how and
when the error has been corrected.
IAS 8 therefore requires that a note to the financial statements should disclose details of the prior
year error, and the effect that the correction has had on ‘line items’ in the prior year.

© Emile Woolf International 224 The Institute of Chartered Accountants of Pakistan


Chapter 6: IAS 8: Accounting policies, changes in accounting estimates and errors

Illustration 13 (continued from Illustration 12):


The following note would be needed to the financial statements for the year to 31 December 2017
to explain the adjustments made to figures previously published for the year to 31 December 2016.

Note about statement of profit or loss Rs.000

(Increase) in cost of goods sold (60)


(Decrease) in profit (60)

Note about statement of financial position

(Decrease) in closing inventory (60)


(Decrease) in equity (60)

Example 03: WONDER LIMITED


Wonder Limited (WL) is engaged in the manufacturing and sale of textile machinery. Following are
the draft extracts of the statement of financial position and the statement of profit or loss for the
year ended 30 June 2015:

Statement of Financial Position 2015 2014


Rs. m Rs. m
Property, plant and equipment 189 130
Retained earnings 198 108

Statement of profit or loss 2015 2014


Rs. m Rs. m
Profit for the year 90 78

Following additional information has not been taken into account in the preparation of the
above financial statements:
(i) Cost of repairs amounting to Rs. 20 million was erroneously debited to the machinery
account on 1 October 2013. The estimated useful life of the machine is 10 years.
(ii) On 1 July 2014, WL reviewed the estimated useful life of its plant and revised it from 5
years to 8 years. The plant was purchased on 1 July 2013 at a cost of Rs. 70 million.
Depreciation is provided under the straight line method. Ignore taxation.
Required:
Prepare the relevant extracts (including comparative figures) for the year ended 30 June 2015
related to statement of financial position, statement of profit or loss and statement of changes in
equity alongwith the correction of error note.
Answer:
Wonder Limited 2014
2015
Statement of Financial Position (Extracts) (Restated)
As at 30 June 2015 Rs. m Rs. m

Property, plant and equipment W1 178.5 111.5


Retained Earnings 187.5 89.5

© Emile Woolf International 225 The Institute of Chartered Accountants of Pakistan


Financial accounting and reporting I

2014
2015
Statement of Comprehensive income (Extracts) (Restated)
For the year ended 30 June 2015 Rs. m Rs. m
Profit for the year W2 98 59.5

2014
2015
Statement of changes in equity (Retained Earnings) (Restated)
For the year ended 30 June 2015 Rs. m Rs. m

Balance as at 1 January [108 – 78 Profit] 89.5 30


Total Comprehensive Income 98 59.5
Balance as at 31 December 187.5 89.5

Notes to the financial statements (Extracts)


For the year ended 30 June 2015

Change in estimate (Note)


The remaining useful life estimate of a plant was revised from 4 to 7 years resulting in reduction of
Depreciation Rs. 6 million in current period and for remaining life of the plant.

Correction of error
During the year ended June 30, 2014, the repair work was erroneously included in machine
account. The effect of correction of this error is as follows:
2014
Effect on statement of financial position Rs. m
Decrease in Property, plant & equipment W1 (18.5)
Decrease in Retained earnings (18.5)

Effect on statement of profit or loss


Increase in repair expense W2 (20)
Decrease in depreciation expense 1.5
Decrease in profit (18.5)

2015 2014
W1: Property, Plant & Equipment
Rs. m Rs. m
As given 189 130
Correction of error
Less: Repair & Maintenance (20) (20)
Add: Depreciation 2014 [20 x 10% x 9/12] 1.5 1.5
Add: Depreciation 2015 [20 x 10%] 2 -
(16.5) (18.5)
Change in estimate
Add: Lower Depreciation 6 -
178.5 111.5

Change in estimate effect Rs. m


Cost 70
Less: Depreciation [70 / 5 years] (14)
Net Carrying amount 56

© Emile Woolf International 226 The Institute of Chartered Accountants of Pakistan


Chapter 6: IAS 8: Accounting policies, changes in accounting estimates and errors

Rs. m
Depreciation (Previous estimate) [56 / 4 years] 14
Depreciation (New estimate) [56 / 7 years] 8
Decrease 6

2015 2014
W2: Profit for the year
Rs. m Rs. m
As given 90 78
Correction of error
Less: Repair & Maintenance - (20)
Add: Depreciation 2014 [20 x 10% x 9/12] - 1.5
Add: Depreciation 2015 [20 x 10%] 2 -
2 (18.5)
Change in estimate
Add: Lower Depreciation 6 -
98 59.5

Example 04: MARVELLOUS Limited


Question: Following information has been extracted from the draft financial statements of
Marvellous Limited (ML) for the year ended 30 June 2017:
Statement of financial position

2017 2016
Rs. in million
Property, plant and equipment 700 612
Retained earnings 275 240

Statement of profit or loss

Profit for the year 65 85

The following matters are under consideration of the management:


 It was identified that ML’s had incorrectly charged Rs. 36.75 million as maintenance expense,
incurred on installation of the plant. The plant was available for use on 1 July 2014 and had
been depreciated on straight line basis over a useful life of four years.
 In view of significant change in the expected pattern of economic benefits from an item of the
equipment, it has been decided to change the depreciation method from reducing balance to
straight line. The equipment was purchased on 1 July 2015 at a cost of Rs. 80 million having
estimated useful life of 5 years and residual value of Rs. 16 million. The depreciation at the rate
of 27.5% on reducing balance method is included in the above draft financial statements.
The following balances pertain to ML’s statement of financial position as on 30 June 2015:

Rs. in million
Property, plant and equipment 650
Retained earnings 180

Ignore tax.
Required:
Prepare extracts from the statement of financial position, statement of profit or loss and correction
of error note (including comparative figures) for the year ended 30 June 2017.

© Emile Woolf International 227 The Institute of Chartered Accountants of Pakistan


Financial accounting and reporting I

Answer:
Marvellous Limited 2016 2015
2017
Statement of Financial Position (Extracts) (Restated) (Restated)
As at 30 June 2017 Rs. m Rs. m Rs. m

Property, plant and equipment W1 714.64 630.37 677.56


Retained Earnings W3 289.64 258.37 207.56

Part (b) 2016


2017
Statement of Comprehensive income (Extracts) (Restated)
For the year ended 30 June 2017 Rs. m Rs. m

Profit for the year W2 61.26 75.81

Part (c)
Notes to the financial statements (Extracts)
For the year ended 30 June 2017

Change in estimate (Note)


The depreciation method for equipment was revised from reducing balance method to straight line
method resulting in reduction in depreciation expense by Rs. 5.45 in the current year. The
depreciation expense over remaining life would also change due to this change of estimate.
Correction of error
It was identified in current year that installation costs of Rs. 36.75 million to be capitalised was
incorrectly charged as maintenance expense on 1 July 2014. The effect of correcting this error are
as follows:
2016 2015
Effect on statement of financial position Rs. m Rs. m
Increase in Property, plant & equipment W1 18.37 27.56
Increase in Retained earnings W3 18.37 27.56

Effect on statement of profit or loss


Increase in depreciation expense W2 9.19
Decrease in profit (9.19)

2017 2016 2015


W1: Property, Plant & Equipment
Rs. m Rs. m Rs. m
As given 700 612 650
Correction of error
Add: Reversal of maintenance costs 36.75 36.75 36.75
Less: Depreciation effect [36.75 / 4 years] (27.56) (18.38) (9.19)
9.19 18.37 27.56
Change in estimate
Add: Lower Depreciation 5.45 - -
714.64 630.37 677.56

© Emile Woolf International 228 The Institute of Chartered Accountants of Pakistan


Chapter 6: IAS 8: Accounting policies, changes in accounting estimates and errors

Change in estimate effect Rs. m

Cost 80

Less: Depreciation [80 x 27.5%] (22)

Net Carrying amount 58

Depreciation (Previous estimate) [58 x 27.5%] 15.95

Depreciation (New estimate) [(58-16)/4 years] 10.50

Decrease 5.45

2017 2016 2015


W2: Profit before Tax
Rs. m Rs. m Rs. m

As given 65 85

Correction of error

Add: Reversal of Maintenance expense 36.75

Less: Depreciation expense (9.19) (9.19) (9.19)

Change in estimate

Add: Lower Depreciation 5.45

61.26 75.81 27.56

2017 2016 2015


W3: Retained Earnings
Rs. m Rs. m Rs. m

As given 275 240 180

Correction of error

Reversal of Maintenance expense 36.75 36.75 36.75

Depreciation on plant (27.56) (18.38) (9.19)

9.19 18.37 27.56

Change in estimate

Add: Effect of change in estimate 5.45

289.64 258.37 207.56

© Emile Woolf International 229 The Institute of Chartered Accountants of Pakistan


Financial accounting and reporting I

Example 05: Chand paints Limited


Question: Chand Paints Limited (CPL) is engaged in the manufacturing of chemicals and paints. In
April 2016 it was discovered that certain errors had been made in the financial statements for the
year ended 30 June 2015. The errors were corrected in 2016. The details are as follows:
2015
2016 2015
After correction
(Draft) Audited
of errors
---------- Rs. in million -----------
Statement of comprehensive income
Sales tax, commission and discounts (7,939) (8,246) (7,916)
Cost of sales (45,508) (44,606) (44,633)
Selling and distribution expenses (2,940) (2,635) (2,441)
Administration expenses (2,356) (2,254) (2,149)
Other operating charges (495) (467) (515)
Other operating income 920 427 509
Profit for the year 4,089 3,723 4,359
Statement of financial position
Trade and other receivables 1,839 1,613 2,025
Trade and other payables 11,600 8,894 8,670
The share capital and un-appropriated profit of CPL as on 1 July 2014 was Rs. 10,400 million and
Rs. 19,089 million respectively.
The details of dividend declared are as follows:
2016 2015
Cash dividend – Interim 10% 5%
– Final 15% 10%
Required:
(a) Prepare a correction of error note, to be included in the financial statements for the year
ended 30 June 2016. (Ignore earnings per share and taxation)
(b) Prepare the statement of changes in equity for the year ended 30 June 2016
Answer:
Chand Paints Limited
Notes to the financial statements for the year ended 30 June 2016
The effect of retrospective restatement on statement of comprehensive income is tabulated below:

Increase / (decrease) in income 2015


Rs. in million
Increase in sales tax, commission and discounts (7,916 – 8,246) (330)
Decrease in cost of sales (44,633 – 44,606) 27

Increase in selling and distribution expenses (2,441 – 2,635) (194)


Increase in administration expenses (2,149 – 2,254) (105)
Decrease in operating income (602)
Decrease in other operating charges (515 – 467) 48
Decrease in other operating income ( 509 – 427 ) (82)
Decrease in profit for the year (636)

© Emile Woolf International 230 The Institute of Chartered Accountants of Pakistan


Chapter 6: IAS 8: Accounting policies, changes in accounting estimates and errors

The effect of retrospective restatement on statement of financial position for 2015 is tabulated
below:

Decrease in trade debts (2,025 – 1,613) (412)


Increase in trade and other payables (8,894 – 8,670) 224
Decrease in un-appropriated profit (636)
(b) The statement of changes in equity for the year ended 30 June 2016 shall be
presented as follows:
Statement of changes in equity for the year ended 30 June 2016
Share Retained
*Total
Description capital earnings
---------- Rs. in million ----------
Balance as on 1 July 2014 10,400 19,089 29,489
Interim dividend for the year ended 30 June (520) (520)
2015 (10,400×5%)
Total comprehensive income for the year 3,723 3,723
2015 - restated
Balance as at 30 June 2015 restated 10,400 22,292 32,692
Final dividend for the year ended 30 June (1,040) (1,040)
2015 (10,400×10%)
Interim cash dividend for the year 2016 (1,040) (1,040)
(10,400×10%)
Total comprehensive income for the year 4,089 4,089
ended 30 June 2016
Balance as at 30 June 2016 10,400 24,301 34,701

Example 06: DAFFODIL LIMITED


Question: For the purpose of preparation of statement of changes in equity for the year ended 31
December 2017, Daffodil Limited (DL) has extracted the following information:

2017 2016 2015


Draft Audited Audited
--------- Rs. in million ---------
Net profit 650 318 214
Transfer to general reserves 112 - 141
Transfer of incremental depreciation - 49 55
Final cash dividend - - 7.5%

Additional information:
(i) Details of share issues:
 25% right shares were issued on 1 May 2016 at Rs. 18 per share. The market price per
share immediately before the entitlement date was also Rs. 18 per share.
 A bonus issue of 10% was made on 1 April 2017 as final dividend for 2016.
 50 million right shares were issued on 1 July 2017 at Rs. 15 per share. The market price
per share immediately before the entitlement date was Rs. 25 per share.
 A bonus issue of 15% was made on 1 September 2017 as interim dividend.

© Emile Woolf International 231 The Institute of Chartered Accountants of Pakistan


Financial accounting and reporting I

(ii) After preparing draft financial statements, it was discovered that depreciation on a plant costing
Rs. 700 million has been charged @ 25% under reducing balance method, from the date of
commencement of manufacturing i.e. 1 July 2014. However, the plant was available for use on
1 February 2014.
(iii) Share capital and reserves as at 31 December:
2015 2014
--------- Rs. in million ---------
Ordinary share capital (Rs. 10 each) 1,600 1,600
General reserves 1,850 1,709
Retained earnings 1,430 1,302
Required:
Prepare DL’s statement of changes in equity for the year ended 31 December 2017 along with
comparative figures (ignore taxation).
Answer:
Statement of changes in equity
For the year ended 31 December 2017
Ordinary Share General Retained
Total
share capital premium reserves earnings
-------------------- Rs. in million --------------------
Balance as at 31 December 2015 1,600.00 1,850.00 1,430.00 4,880.00
(As given)
Effect of correction of error (W-1) (54.69) (54.69)
Balance as at 31 December 2015 1,600.00 1,850.00 1,375.31 4,825.31
– Restated
Final cash dividend @ 7.5% - 2015 (120.00) (120.00)
(1,600×7.5%)
Right issue @ 25% 400.00 320.00 720.00
(1,600×25%) (160×25%×8)
Net profit - 2016 – Restated 331.67 331.67
[318+13.67(W-1)]
Transfer of incremental 49.00 49.00
depreciation
Balance as at 31 December 2016 2,000.00 320.00 1,850.00 1,635.98 5,805.98
- Restated
Final bonus dividend @ 10% - 200.00 (200.00) -
2016 (2,000×10%)
Right issue 500.00 250.00 750.00
(50×10) (50×5)
Interim bonus dividend @ 15% - 405.00 (405.00) -
2017 (2,700×15%)
Net profit - 2017 [650 + 10.25 660.25 660.25
(W-1)]
Transfer to general reserves 112.00 (112.00) -
Balance as at 31 December 2017 3,105.00 570.00 1,962.00 1,579.23 7,216.23

© Emile Woolf International 232 The Institute of Chartered Accountants of Pakistan


Chapter 6: IAS 8: Accounting policies, changes in accounting estimates and errors

W-1: Correction of error

Wrong depreciation @ Increase/(decrease)


Correct depreciation @ 25%
25% in depreciation
-------------------------- Rs. in million --------------------------
Cost 700 700
2014 160.42 87.50 72.92
(700 × 25% × 11 ÷ 12) (700 × 25% × 6 ÷ 12)
2015 134.90 153.13 (18.23)
(700 – 160.42) × 25% (700 – 87.50) × 25%
54.69
2016 101.17 114.84 (13.67)
(134.90 × 75%) (153.13 × 75%)
2017 75.88 86.13 (10.25)
(101.17 × 75%) (114.84 × 75%)

Example 07: COAL LIMITED


Question: During the year, it was discovered that due to some calculation error in excel sheet, fair
value of Coal Limited’s (CL’s) office building was taken incorrectly as Rs. 460 million instead of Rs.
360 million. Resultantly, the building was recorded based on incorrect revaluation amount in CL’s
financial statements for the year ended 30 June 2017.
This building was acquired on 1 July 2015 for Rs. 500 million and then revalued for the first time
on 30 June 2017. CL follows revaluation model for subsequent measurement of its building
classified as property, plant and equipment and charges depreciation over its useful life of 10 years
using straight line method. CL accounts for revaluation on net replacement value method and
transfers the maximum possible amount from the revaluation surplus to retained earnings on an
annual basis.
As on 30 June 2019, the revalued amount of building has been determined at Rs. 320 million.
Required:
Prepare extracts from CL’s statement of financial position and related notes to the financial
statements for the year ended 30 June 2019 alongwith comparative figures for the above. (Note on
Property, plant and equipment is not required)

Answer:
Coal Limited 2018 2017
2019
Statement of Financial Position (Extracts) (Restated) (Restated)
As at 30 June 2019 Rs. m Rs. m Rs. m

Non-current assets: Building W1 320 315 360


Equity: Revaluation Surplus W1 20 - -
Coal Limited
Notes to the financial statements (Extracts) for the year ended 30 June 2019
Correction of error note
It was identified in current year that revalued amount of one of buildings was taken as Rs. 460 million
instead of Rs. 360 million in 2017's financial statements.

© Emile Woolf International 233 The Institute of Chartered Accountants of Pakistan


Financial accounting and reporting I

2018 2017
Effect on statement of financial position Rs. m Rs. m
Decrease in PPE W1 (87.5) (100)
Decrease in Revaluation Surplus W2 (52.5) (60)
Decrease in Retained Earnings W3 (35) (40)

Effect on statement of profit or loss


Decrease in depreciation expense W1 (12.5)

W1: Building Error Corrected


Rs. m Rs. m
Cost 1 July 2015 500 500
Depreciation 2016 [Over 10 years] (50) (50)
Depreciation 2017 [Same] (50) (50)
CA at 30 Jun 2017 (pre revaluation) 400 400
Revaluation gain (loss) [Balancing] 60 (40)
CA at 30 Jun 2017 (Revalued) 460 360
Depreciation 2018 [Over 8 years] (57.5) (45)
CA at 30 June 2018 402.5 315
Depreciation 2019 [Over 7 years] (45)
CA at 30 June 2019 (pre revaluation) 270
Reversal of loss (net of depreciation) [40 - 5 - 5] 30
Revaluation surplus (OCI) [Balancing] 20
CA at 30 June 2019 (Revalued) 320

Error Corrected
W2: Revaluation surplus
Rs. m Rs. m
At 30 June 2017 60 -
Transfer to RE [60 / 8 years] (7.5) -
At 30 June 2018 52.5 -

Error Corrected
W3: Impact on Retained Earnings
Rs. m Rs. m
At 30 June 2017 - (40)
Transfer from RS 7.5 -
Depreciation Expense (57.5) (45)
At 30 June 2018 (50) (85)

© Emile Woolf International 234 The Institute of Chartered Accountants of Pakistan


Chapter 6: IAS 8: Accounting policies, changes in accounting estimates and errors

Reclassification of items of Statement of Profit and Loss Statement and Statement of Financial
Position:
Sometime, an item is wrongly classified in a wrong category, for example, an expense that has to
be accounted for under Cost of Sales was inadvertently categorized in Administrative expenses or
an item of Administrative expenses was wrongly classified in Cost of Sales. Such wrong
reclassification is corrected once this is identified, even though have to change last year’s financial
statements figures, but since there will be no impact on the overall financial statements, this is
regarded as reclassification and not restatement. If such case a rise, where the impact is Nil on the
Financial Statements and if the item is material, a note is given in Financial Statement describing
the reclassification.
A illustrative note is given below:

Description Head of Account Earlier Head of Account Now Amount in PKR


Vehicle running and Cost of Sales Administrative 1,000,000
Maintenance Expenses

© Emile Woolf International 235 The Institute of Chartered Accountants of Pakistan


Financial accounting and reporting I

5 OBJECTIVE BASED QUESTIONS


01. Which TWO of the following situations would not require a prior year adjustment as per IAS 8
Accounting Policies, Changes in Accounting Estimates and Errors?
(a) In last year's financial statements, inventories were understated by a material
amount due to system error
(b) A company has changed its allowance for irrecoverable receivables from 10% of
outstanding debt to everything over 120 days old
(c) A new accounting standard has been issued that requires a company to change its
accounting policy but gives no guidance on the specific application of the change
itself
(d) A company has decided to move from charging depreciation on the straight line
basis to the reducing balance basis

02. In accordance with IAS 8 Accounting Policies, Changes in Accounting Estimates and Errors
how is a change in accounting estimate accounted for?
(a) By changing the current year figures but not the previous years' figures
(b) By changing the current year figures and the previous years' figures
(c) No alteration of any figures but disclosure in the notes
(d) Neither alteration of any figures nor disclosure in the notes

03. According to IAS 8 Accounting Policies, Changes in Accounting Estimates and Errors, how
should a material error in the previous financial reporting period be accounted for in the current
period?
(a) By making an adjustment in the financial statements of the current period through
the statement of profit or loss, and disclosing the nature of the error in a note.
(b) By making an adjustment in the financial statements of the current period as a
movement on reserves, and disclosing the nature of the error in a note.
(c) By restating the comparative amounts for the previous period at their correct value,
and disclosing the nature of the error in a note.
(d) By restating the comparative amounts for the previous period at their correct value,
but without the requirement for a disclosure of the nature of the error in a note.

04. Which of these changes would be classified as ‘a change in accounting policy’ as determined
by IAS 8 Accounting Policies, Changes in Accounting Estimates and Errors?
(a) Increased the allowance for irrecoverable receivables from 5% to 10% of
outstanding debts
(b) Changed the method of valuing inventory from FIFO to average cost
(c) Changed the depreciation of plant and equipment from straight line depreciation to
reducing balance depreciation
(d) Changed the useful life of motor vehicles from six years to four years

05. In which TWO of the following situations can a change in accounting policy be made by an
entity?
(a) If the change is required by an IFRS
(b) If the entity thinks that a new accounting policy would be easier to report
(c) If a new accounting policy would show more favourable results
(d) If a new accounting policy results in more reliable and relevant presentation of
events or transactions

© Emile Woolf International 236 The Institute of Chartered Accountants of Pakistan


Chapter 6: IAS 8: Accounting policies, changes in accounting estimates and errors

06. Which one of the following would be treated under IAS 8 Accounting policies, changes in
accounting estimates and errors as a change of accounting policy?
(a) A change in valuation of inventory from a weighted average to a FIFO basis
(b) A change of depreciation method from straight line to reducing balance
(c) Adoption of the revaluation model for non-current assets previously held at cost
(d) Capitalisation of borrowing costs which have arisen for the first time

07. Which of the following would be a change in accounting policy in accordance with IAS 8
Accounting policies, changes in accounting estimates and errors?
(a) Adjusting the residual value amount based on latest information received
(b) A change in reporting depreciation charges as cost of sales rather than as
administrative expenses
(c) Depreciation charged on reducing balance method rather than straight line
(d) Reducing the value of inventory from cost to net realisable value

08. Which of the following items is a change of accounting policy under IAS 8 Accounting policies,
changes in accounting estimates and errors?
(a) Classifying commission earned as revenue in the statement of profit or loss, having
previously classified it as other operating income
(b) Switching to hiring plants on leases from a previous policy of purchasing plants for
cash
(c) Reversal of write down to NRV of inventory after sales prices increased significantly
this year
(d) Revising the remaining useful life of a depreciable asset

09. The directors of Tom Limited are disappointed by the draft profit for the year ended 30
September 2013. The company's assistant accountant, Jerry, has suggested following:
A major item of plant that cost Rs. 20 million to purchase and install on 1 October 2010 is being
depreciated on a straight-line basis over a five-year period. On 1 October 2012, the production
manager believed that the plant was likely to last eight years in total (i.e. from the date of its
purchase).
Jerry believes that as the useful life estimate has increased, the previous years’ depreciation
was overstated and it depreciation expense should be reversed in current year leading to
increased profit.
What is the nature of the change being proposed by Jerry and how should it be applied?
(a) Change of accounting policy : Retrospective application
(b) Change of accounting policy : Prospective application
(c) Change of accounting estimate : Retrospective application
(d) Change of accounting estimate : Prospective application

10. If it is impractical to make a retrospective application to a period:


(a) Make the change only to the current period
(b) Apply the change to the earliest period that is practical
(c) Do not make the change at all
(d) Make the change in next year

© Emile Woolf International 237 The Institute of Chartered Accountants of Pakistan


Financial accounting and reporting I

11. Which TWO of the following would be treated as a change of accounting policy?
(a) Entity has received its first government grant and is applying the deferred income
method.
(b) Entity has revalued its properties. Up to now they had all been carried at historical
cost.
(c) Entity has reclassified certain expense from other operating expenses to cost of
sales.
(d) Entity has increased its irrecoverable debt allowance from 10% to 12%.

12. Correcting the recognition, measurement and disclosure of amounts in financial statements as
if a prior-period error had never occurred. This is:
(a) Retrospective restatement
(b) Retrospective application
(c) Change in accounting estimate
(d) Prospective restatement

13. Specific principles bases conventions rules and practices applied in presenting financial
statements. This defines:
(a) Accounting estimates
(b) Accounting policies
(c) Prospective application
(d) Accounting method

14. Adjustment of the carrying amount of an asset or a liability or the consumption of an asset as a
result of change in assessment. This defines:
(a) A change in accounting estimate
(b) Accounting policies
(c) Misstatements
(d) Correction of error

15. Applying a new policy to transactions as if that policy had always been applied. This is:
(a) Retrospective restatement
(b) Retrospective application
(c) Change in accounting estimate
(d) Prospective application

16. The directors of Tom Limited are disappointed by the draft profit for the year ended 30
September 2013. The company's assistant accountant, Jerry, has suggested following:
A major item of plant that cost Rs. 20 million to purchase and install on 1 October 2010 is being
depreciated on a straight-line basis over a five-year period. On 1 October 2012, the production
manager believed that the plant was likely to last eight years in total (i.e. from the date of its
purchase).
Jerry believes that as the useful life estimate has increased, the previous years’ depreciation
was overstated and it depreciation expense should be reversed in current year leading to
increased profit.

© Emile Woolf International 238 The Institute of Chartered Accountants of Pakistan


Chapter 6: IAS 8: Accounting policies, changes in accounting estimates and errors

Adjusting for the change of useful life correctly, what will be the carrying amount of the plant at
30 September 2013?

Rs. ___________

17. Imad Textile Limited (ITL) purchased a plant on January 01, 2011 for Rs. 1,120,000. At this date
the useful life of the asset was estimated at 10 years after which it can be sold for Rs. 120,000.
However, during 2013 ITL estimates the remaining useful life of this plant as 6 years and expects
to fetch residual value of Rs. 170,000. ITL uses straight line method for depreciating such plants.
Calculate the amount of depreciation for the year ended on 31 December 2018.

Rs. ___________

18. A company is preparing its financial statements for the year ended 31 December 2019 and
discovered that in previous years following amounts were incorrectly capitalised in an intangible
asset with indefinite useful life.
Year Rs. m
2018 5
2017 4
2016 4
2015 3
The applicable tax rate is 30%.
What amount should be deducted from retained earnings in statement of changes in equity on
1 January 2018 for correction of above error?

Rs. ___________

19. A company is preparing its financial statements for the year ended 31 December 2019 and
discovered that in previous years following amounts were incorrectly capitalised in an intangible
asset with indefinite useful life.
Year Rs. m
2018 5
2017 4
2016 4
2015 3
The applicable tax rate is 30%.
Calculate the effect on profit after tax for the year ended 31 December 2018 correction of above
error.

Rs. ___________

20. Most of entity’s competitors value their inventory using the average cost (AVCO) basis, whereas
the entity uses the first in first out (FIFO) basis.
The value of inventory at 30 September 2013 (on the FIFO basis) is Rs. 20 million, however on
the AVCO basis it would be valued at Rs. 18 million. By adopting the same method (AVCO) as
its competitors. The inventory at 30 September 2012 was reported as Rs. 15 million, however
on the AVCO basis it would have been reported as Rs. 13.4 million.
What will be the effect of the change on profits for the year ended 30 September 2013?

Rs. ___________

© Emile Woolf International 239 The Institute of Chartered Accountants of Pakistan


Financial accounting and reporting I

21. Disclosure requirements of IAS 8 in respect of change in accounting policy are NOT applicable
in case of :
(a) change in method for inventory valuation from FIFO to weighted average
(b) initial adoption of revaluation model for property, plant and equipment
(c) change in revenue recognition policy
(d) none of the above

© Emile Woolf International 240 The Institute of Chartered Accountants of Pakistan


Chapter 6: IAS 8: Accounting policies, changes in accounting estimates and errors

5 OBJECTIVE BASED ANSWERS


01. (b) & (d) A change in the calculation of the allowance for irrecoverable receivables,
and a change in the depreciation method, are changes in accounting
estimate so therefore require prospective adjustment only.

02. (a) Change in accounting estimates results in alteration of figures but not
retrospectively. The change is made prospectively.

03. (c) The prior period error is corrected by restating the comparative amounts
for the previous period at their correct value. A note to the accounts should
disclose the nature of the error, together with other details.

04. (b) A change in the method of inventory valuation would be classed as a


change in accounting policy under IAS 8. The allowance for receivables,
useful life and depreciation method are all accounting estimates.

05. (a) & (d) A change in accounting policy may be made firstly if this is required by an
IFRS Standard. If there is no requirement, an entity can choose to change
their accounting policy if they believe a new accounting policy would result
in a more reliable and relevant presentation of events and transactions.
Entities cannot change their accounting policies simply to make financial
reporting easier, or to try and show a more favourable picture of results.

06. (a) A change of depreciation method is treated as a change of accounting


estimate. Adoption of the revaluation method is dealt with under IAS 16.
Application of a new accounting policy (such as capitalisation of borrowing
costs) for transactions that did not previously occur is not a change in
accounting policy according to IAS 8.

07. (b) This is a change in presentation which will affect calculation of gross profit
and will be retrospectively adjusted when presenting comparatives. (a(
and (d) are simply adjustments made during preparation of the financial
statements, (c) is a change of accounting estimate.

08. (a) This is a change in presentation so qualifies as a change in accounting


policy.

09. (d) This is a change of accounting estimate so does not need to be


retrospectively applied.

10. (b) In this situation, change is applied to the earliest period possible.

11. (b) & (c) This is change in measurement basis, so it is a change in accounting
policy.
This is a change in presentation, so it is a change of accounting policy.

12. (a) Correction of error in previous period is called retrospective restatement.

13. (b) Specific principles bases conventions rules and practices applied in
presenting financial statements are accounting policies.

14. (a) Change in assessment is change in estimate.

© Emile Woolf International 241 The Institute of Chartered Accountants of Pakistan


Financial accounting and reporting I

15. (b) Retrospective application is applying a policy as if it had always been


applied.

16. Rs. 10 million


Rs. m
Original cost 1 October 2010 20
Two years depreciation ((20/5) × 2) (8)
Carrying amount at 1 October 2012 12
Depreciation to 30 September 2013 (12 / 6) (2)
Carrying amount at 30 September 2013 10

17. Rs. 125,000


Per year depreciation Rs.
Year 2011 (Rs. 1,120,000 – 120,000) / 10 years 100,000
Year 2012 100,000

Year 2013 ((Rs. 920,000 – 170,000) / 6 years 125,000

18. Rs. 7.7 million Adjustment in opening balance of retained earnings (net of tax)
Rs. 4m + 4m + 3m = Rs. 11m x 70% = Rs. 7.7 million

19. Rs. 3.5 million Effect on profit for the year ended 31 December 2018 (net of tax)
Rs. 5m x 70% = Rs. 3.5 million

20. Rs. (400,000)


FIFO AVCO Profit
Rs. m Rs. m Rs. m
Year to 30 September 2012 15 13.4 (1.6)
B/f 1 October 2012 1.6
Year to 30 September 2013 20 18 ( 2.0)
At 30 September 2013 (0.4)

The net effect at 30 September 2013 of this will be to reduce current year
profits by Rs. 400,000.

21. (b) Initial adoption of revaluation model for property, plant and equipment

© Emile Woolf International 242 The Institute of Chartered Accountants of Pakistan


Certificate in Accounting and Finance

CHAPTER
Financial accounting and reporting I

IAS 33: Earnings per share

Contents
1 Introduction
2 Basic EPS
3 Diluted EPS
4 Presentation and disclosure
5 Usefulness and limitations
6 Objective based questions and answers

* The student must refer original handbook of IFRS.

© Emile Woolf International 243 The Institute of Chartered Accountants of Pakistan


Financial accounting and reporting I

1 INTRODUCTION
Section overview

 Concept
 Scope
 Comparability
 Impact of different types of shares on EPS

1.1 Concept
As its name implies, Earnings per share (EPS) is calculated as reported earnings divided by the
number of ordinary shares in issue.
The concept of EPS is quite straightforward. It is simply the profit for the year (adjusted for a few
things) divided by the weighted average number of ordinary shares in that year.
IAS 33 specifies the profit figure that should be used and explains how to calculate the appropriate
number of shares when there have been changes in share capital during the period under review.
The standard also describes the concept of dilution which is caused by the existence of potential
ordinary shares.
An entity is required to present basic and diluted earnings per share, even if the amounts are
negative (i.e. a loss per share).

1.2 Scope
IAS 33 applies to entities whose ordinary shares are publicly traded (listed companies) or are in
the process of being issued in public markets.
Some publicly-traded entities prepare consolidated financial statements as well as individual
financial statements, when this is the case, IAS 33 requires disclosure only of EPS based on figures
in the consolidated financial statements.

1.3 Comparability
EPS is widely accepted as one of the important indicator of entity’s financial performance. EPS is
a useful measure of profitability and when compared with EPS of other similar entities, it gives a
view of the comparative earning power of the entities. EPS when calculated over a number of years
indicates whether the earning power of an entity has improved or deteriorated.
EPS is used by investors as a measure of performance of entities in which they invest, or might
possibly invest. Investors are usually interested in changes in an entity’s EPS over time (trend
analysis) and also in the size of EPS relative to the current market price of the entity’s shares.
It is important for users of financial statements to be able to compare the EPS of different entities
and of the same entity over the years.
IAS 33 achieves the objective of comparability by:
 Defining earnings
 Prescribing methods to determine number of shares
 Prescribing standardised presentation and disclosure

1.4 Impact of different types of shares on EPS


Ordinary share

Definition
An ordinary share is an equity instrument that is subordinate to all other classes of equity
instruments.

© Emile Woolf International 244 The Institute of Chartered Accountants of Pakistan


Chapter 7: IAS 33: Earnings per share

The ordinary shares used in the EPS calculation are those entitled to the residual profits of the
entity, after dividends relating to all other shares have been paid.
Redeemable preference shares
These shares are classified as liabilities. Any dividend relating to that share is recognised as a
finance cost in the statement of profit or loss. It is already deducted from the profit or loss from
continuing operations and no further adjustment is required.
Irredeemable preference shares
These shares are classified as equity and the dividend relating to them is disclosed in the statement
of changes in equity. This dividend must be deducted from profit for the year to arrive at profit
attributable to the ordinary shareholders.
Potential ordinary share

Definition
A potential ordinary share is a financial instrument or other contract that may entitle its holder to
ordinary shares (at some time in the future).

Potential ordinary shares do not impact calculation of basic EPS but diluted EPS might differ from
basic EPS when there are potential ordinary shares in existence.

© Emile Woolf International 245 The Institute of Chartered Accountants of Pakistan


Financial accounting and reporting I

2 BASIC EPS
Section overview

 General concept
 Issue of shares at full market price
 Issue of shares for no consideration (Bonus issue)
 Right issue
 Share split
 Share consolidation
 Retrospective adjustments

2.1 General concept


Basic earnings per share is calculated by dividing the profit or loss on continuing operations by
the weighted average number of ordinary shares in issue during the period.
The calculation of the basic EPS is as follows:

Formula: Basic EPS

Earnings attributable to ordinary shareholders


weighted average number of ordinary shares in issue during the period

Earnings
 Earnings attributable to ordinary shareholders usually means profit after tax less preference
dividend.
 If preference dividend is cumulative, such dividend for the period need to be taken into
account irrespective of whether declared or not.
 If preference dividend is non-cumulative, such dividend for the period need to be taken into
account only for the amount of dividend declared for the period.
 In case of consolidated financial statements, share of profit of non-controlling interest is also
excluded.
 Earnings from discontinued operations are dealt with separately. An EPS from any
discontinued operations must also be disclosed, but this does not have to be disclosed on
the face of the statement of profit or loss. Instead, it may be shown in a note to the financial
statements.

Example 01: Ghalib Limited


Question: In the year ended 31 December 20X1, Ghalib Limited (GL) made profit after tax of Rs.
3,500,000. Of this, Rs. 3,000,000 was from continuing operations and Rs. 500,000 from
discontinued operations.
GL paid ordinary dividends of Rs. 150,000 and preference dividends of Rs. 65,000. The preference
shares were correctly classified as liabilities in accordance with IAS 32.
GL had 1 million ordinary shares of Rs. 10 each in issue throughout the year.

Required:
Calculate basic EPS for the year ended 31 December 20X1.

© Emile Woolf International 246 The Institute of Chartered Accountants of Pakistan


Chapter 7: IAS 33: Earnings per share

Answer:

Basic EPS Rs. 3,000,000


= = Rs. 3 per share
(continued operations) 1,000,000 shares

Basic EPS Rs. 500,000


Rs. 0.5 per
= =
(discontinued operations) share
1,000,000 shares
.

Example 02: Jazib Limited


Question: In the year ended 31 December 20X1, Jazib Limited (JL) made profit after tax of Rs.
3,500,000. Of this, Rs. 3,000,000 was from continuing operations and Rs. 500,000 from
discontinued operations.
JL paid ordinary dividends of Rs. 150,000 and preference dividends of Rs. 65,000. The preference
shares were correctly classified as equity in accordance with IAS 32.
JL had 1 million ordinary shares of Rs. 10 each in issue throughout the year.

Required:
Calculate basic EPS for the year ended 31 December 20X1.

Answer:

Basic EPS Rs. 3,000,000 – 65,000


Rs. 2.94 per
= =
(continued operations) share
1,000,000 shares

Basic EPS Rs. 500,000


Rs. 0.5 per
= =
(discontinued operations) share
1,000,000 shares
.

Example 03: Aqeel Limited


Question: In the year ended 31 December 20X1, Aqeel Limited (AL) made profit after tax of Rs.
3,500,000.
AL has Rs. 1,000,000 10% cumulative preference shares in issue. This would entitle the investors
to receive a dividend of Rs. 100,000 (10% of Rs. 1,000,000) if declared).
AL had 1 million ordinary shares of Rs. 10 each in issue throughout the year.

Required:
Calculate basic EPS for the year ended 31 December 20X1.

Answer:

Rs. 3,500,000 – 100,000


Basic EPS = = Rs. 3.4 per share
1,000,000 shares

Note that the Rs. 100,000 deducted above would be deducted irrespective whether a dividend has
been declared or not.

© Emile Woolf International 247 The Institute of Chartered Accountants of Pakistan


Financial accounting and reporting I

Example 04: Adeel Limited


Question: In the year ended 31 December 20X1, Adeel Limited (AL) made profit after tax of Rs.
3,500,000.
AL has Rs. 1,000,000 10% non-cumulative preference shares in issue. This would entitle the
investors to receive a dividend of Rs. 100,000 (10% of Rs. 1,000,000) if declared). The preference
dividend was declared during the year ended 31 December 20X1.
AL had 1 million ordinary shares of Rs. 10 each in issue throughout the year.

Required:
Calculate basic EPS for the year ended 31 December 20X1.

Answer:

Rs. 3,500,000 – 100,000


Basic EPS = = Rs. 3.4 per share
1,000,000 shares
.

Example 05: Kashif Limited


Question: In the year ended 31 December 20X1, Kashif Limited (KL) made profit after tax of Rs.
3,500,000.
KL has Rs. 1,000,000 10% non-cumulative preference shares in issue. This would entitle the
investors to receive a dividend of Rs. 100,000 (10% of Rs. 1,000,000) if declared). The preference
dividend was not declared during the year ended 31 December 20X1.
KL had 1 million ordinary shares of Rs. 10 each in issue throughout the year.

Required:
Calculate basic EPS for the year ended 31 December 20X1.

Answer:

Rs. 3,500,000
Basic EPS = = Rs. 3.5 per share
1,000,000 shares
.
Weighted average number of shares
The number of shares to be used in the calculation being the weighted average number of shares
in issue during the period. Changes in share capital during a period must be taken into account in
arriving at this number. IAS 33 provides guidance on how to do this.
There are different ways in which the number of shares may change:
 Issue of shares at full market price
 Issue of shares for no consideration (Bonus issue)
 Right issue
 Share split (where one share is split into several others)
 Share consolidation (where two or more shares are consolidated into one)

2.2 Issue of shares at full market price


The weighted average ordinary shares are to be considered taking into account of the date any
new shares are issued during the year since the shares issues during the year provided additional
resources only for part of the year. Therefore, new shares issued during the period are taken into
account by applying a time weighting factor.

© Emile Woolf International 248 The Institute of Chartered Accountants of Pakistan


Chapter 7: IAS 33: Earnings per share

There is no adjustment to comparatives resulting from an issue at full price.

Example 06: Friday Limited


Question: Friday Limited (FL) has a financial year ending 31 December.
On 1 January 20X1, there were 6,000,000 ordinary shares (Rs. 10 each) in issue. On 1 April 20X1,
it issued 1,000,000 new shares at full market price.
Total earnings for the year ended 31 December 20X1 were Rs. 27,000,000.

Required:
Calculate basic EPS for the year ended 31 December 20X1.

Answer:

Rs. 27,000,000
Basic EPS = = Rs. 4 per share
6,750,000 shares

Number of Time Weighted


Date
shares factor average number
1 January to 31 March 6,000,000 x 3/12 1,500,000
New issue on 1 April 1,000,000
1 April to 31 December 7,000,000 x 9/12 5,250,000
6,750,000
.

Example 07: Sunday Limited


Question: Sunday Limited (SL) has a financial year ending 31 December.
On 1 January 20X3, there were 9,000,000 ordinary shares in issue. On 1 May 20X3, SL issued
1,200,000 new shares at full market price. On 1 October 20X3, SL issued a further 1,800,000
shares, also at full market price.
Total earnings in 20X3 were Rs.36,900,000.

Required:
Calculate basic EPS for the year ended 31 December 20X3.

Answer:

Rs. 36,900,000
Basic EPS = = Rs. 3.6 per share
10,250,000 shares

Number of Time Weighted


Date
shares factor average number
1 January to 30 April 9,000,000 x 4/12 3,000,000
New issue on 1 May 1,200,000
1 May to 30 September 10,200,000 x 5/12 4,250,000
New issue on 1 October 1,800,000
1 October to 31 December 12,000,000 x 3/12 3,000,000
10,250,000
.

© Emile Woolf International 249 The Institute of Chartered Accountants of Pakistan


Financial accounting and reporting I

2.3 Issue of shares for no consideration (Bonus issue)


A bonus issue of shares (also called a scrip issue or a capitalisation issue) is an issue of new
shares to existing shareholders, in proportion to their existing shareholding, for no consideration.
The new shares are created by converting equity reserves (e.g. share premium, retained earnings,
etc.) into ordinary share capital.
No cash is raised from a bonus issue and there is no earnings boost from the issue. Bonus issue
shares are treated as if they have always been in issue. This impact can be achieved by applying
a ‘bonus issue fraction’ to ordinary shares already in issue.

Formula: Bonus issue fraction

Number of shares in holding after the bonus issue


Number of shares in holding before the bonus issue

In order to ensure that EPS in the year of the bonus issue is comparable with previous year’s EPS,
the comparative EPS is restated on the same basis. This may be achieved by either:
 Applying bonus issue fraction to number of shares in issue in comparative period; or
 Applying reverse bonus issue fraction to the reported EPS in comparative period.

Example 08: Pink Limited


Question: Pink Limited (PL) has a 31 December financial year end.
On 1 January 20X4 it has 4,000,000 shares in issue. There were no share issues in Year 20X4. On
1 July 20X5 it made a 1 for 4 bonus issue.

Basic EPS reported in 20X4 was: Rs. 20,000,000/4,000,000 shares = Rs. 5 per share

Earning attributable to ordinary shareholders for the year 20X5 are Rs. 24,000,000

Required:
Calculate basic EPS for the year ended 31 December 20X5 (including comparative for 20X4).

Answer:
Bonus issue fraction = 5 / 4

Basic EPS Rs. 24,000,000


= = Rs. 4.8 per share
(20X5) 5,000,000 shares

Number of Time Weighted


Date Fraction
shares factor average number
1 January to 30 June 4,000,000 x 6/12 x 5/4 2,500,000
New issue on 1 July 1,000,000
1 July to 31 December 5,000,000 x 6/12 2,500,000
5,000,000

Basic EPS Rs. 20,000,000


= = Rs. 4 per share
(20X4 restated) 4,000,000 x 5/4 shares

Alternatively, Basic EPS (20X4 restated) = Rs. 5 x 4/5 = Rs. 4 per share

© Emile Woolf International 250 The Institute of Chartered Accountants of Pakistan


Chapter 7: IAS 33: Earnings per share

Example 09: Red Limited


Question: Red Limited (RL) has a 31 December financial year-end and had 2,000,000 ordinary
shares in issue on 1 January 20X2.
On 31 March 20X2, it issued 500,000 ordinary shares, at full market price.
On 1 July 20X2, RL made a 1 for 2 bonus issue.

In Year 20X1, the EPS had been calculated as Rs. 30 per share.
In Year 20X2, total earnings were Rs. 85,500,000.

Required:
Calculate the EPS for the year to 31 December 20X2, and the comparative EPS figure for 20X1.
Answer:
Bonus issue fraction = 3 / 2

Basic EPS Rs. 85,500,000


= = Rs. 24 per share
(20X2) 3,562,500 shares

Number of Time Weighted


Date Fraction
shares factor average number
1 January to 31 March 2,000,000 x 3/12 x 3/2 750,000
Issue at full price on 31 March 500,000
1 April to 30 June 2,500,000 x 3/12 x 3/2 937,500
Bonus issue on 1 July 1,250,000
1 July to 31 December 3,750,000 x 6/12 1,875,000
3,562,500

Basic EPS (20X1 restated) = Rs. 30 x 2/3 = Rs. 20 per share

2.4 Right issue


A rights issue of shares is an issue of new shares for cash, where the new shares are offered
initially to current shareholders in proportion to their existing shareholdings.
The issue price of the new shares in a rights issue is always below the current market price for the
shares already in issue. This means that they include a bonus element which must be taken into
account in the calculation of the weighted average number of shares. This impact can be achieved
by applying a ‘right issue bonus fraction’.

Formula: Rights issue bonus fraction

Actual cum-rights price


Theoretical ex-rights price

The actual cum-rights price is the market price of the shares before the rights issue.
The theoretical ex-rights price is the price that the shares ought to be, in theory, after the rights
issue. It is a weighted average price of the shares before the rights issue and the new shares in
the rights issue.

Formula: Theoretical ex-rights price

(Shares before right issue x Actual cum-rights price) + (Right issue shares x Issue price)
Shares before right issue + Right issue shares

© Emile Woolf International 251 The Institute of Chartered Accountants of Pakistan


Financial accounting and reporting I

The comparative EPS is also restated using the same basis as discussed in bonus issue.

Example 10: Peach Limited


Question: Peach Limited (PL) has a 31 December financial year-end.
PL had 3,600,000 shares in issue on 1 January 20X2.
It made a 1 for 4 rights issue on 1 June 20X2, at a price of Rs. 40 per share. (After the rights issue,
there will be 1 new share for every 4 shares previously in issue). The share price just before the
rights issue was Rs. 50.
Total earnings in the financial year to 31 December 20X2 were Rs. 25,125,000.
The reported EPS in Year 20X1 was Rs. 6.4 per share.
Required:
Calculate the EPS for the year to 31 December 20X2, and the comparative EPS figure for 20X1.

Answer:

(4 shares x Rs. 50) + (1 share x Rs. 40)


Theoretical ex-
= = Rs. 48 per share
rights price
4 shares + 1 share

Right issue bonus fraction = 50 / 48

Basic EPS Rs. 25,125,000


= = Rs. 6 per share
(20X2) 4,187,500 shares

Number of Time Weighted


Date Fraction
shares factor average number

1 January to 31 May 3,600,000 x 5/12 x 50/48 1,562,500

Right issue on 1 June 900,000

1 June to 31 December 4,500,000 x 7/12 2,625,000

4,187,500

Basic EPS (20X1 restated) = Rs. 6.4 x 48/50 = Rs. 6.14 per share

Example 11: Grapes Limited


Question: Grapes Limited (GL) has a 31 December financial year-end.
GL had 3 million ordinary shares in issue on 1 January 20X7.
On 1 April 20X7, it made a 1 for 2 rights issue of 1,500,000 ordinary shares at Rs. 20 per share.
The market price of the shares prior to the rights issue was Rs. 50.
An issue of 400,000 shares at full market price was then made on 1 August 20X7.
In the year to 31 December 20X7, total earnings were Rs. 17,468,750.
In Year 20X6, EPS had been reported as Rs. 3.5 per share
Required:
Calculate the EPS for the year to 31 December 20X7, and the adjusted EPS for 20X6 for
comparative purposes.

© Emile Woolf International 252 The Institute of Chartered Accountants of Pakistan


Chapter 7: IAS 33: Earnings per share

Answer:

(2 shares x Rs. 50) + (1 share x Rs. 20)


Theoretical ex-
= = Rs. 40 per share
rights price
2 shares + 1 share

Right issue bonus fraction = 50 / 40

Basic EPS Rs. 17,468,750


= = Rs. 3.9 per share
(20X7) 4,479,167 shares

Number of Time Weighted


Date Fraction
shares factor average number
1 January to 31 March 3,000,000 x 3/12 x 50/40 937,500
Right issue on 1 April 1,500,000
1 April to 31 July 4,500,000 x 4/12 1,500,000
Issue at full price on 1 August 400,000
1 August to 31 December 4,900,000 x 5/12 2,041,667
4,479,167
Basic EPS (20X6 restated) = Rs. 3.5 x 40/50 = Rs. 2.8 per share

2.5 Share split


Share split (usually to achieve greater marketability of shares) is the division of the existing issued
share capital of the company into a larger number of smaller denominations shares. The overall
capital remains the same.
For example, a shareholder had 100 ordinary shares of Rs. 50 par value per share, that is total Rs.
5,000. The company made a 5 for 1 share split. Now the shareholder would have 500 shares of
Rs. 10 par value per share, that is same total of Rs. 5,000.
Since there is no change in actual resources, the calculation perspective is same as that of bonus
issue and a ‘share split fraction’ is applied to ordinary shares already in issue.

Formula: Share split fraction

Number of shares in holding after share split


Number of shares in holding before share split

The comparative EPS is also restated using the same basis as discussed in bonus issue.

Example 12: Pacific Limited


Question: Pacific Limited (PL) has a 31 December financial year end.
On 1 January 20X4 it has 4,000,000 shares in issue. There were no share issues or other changes
in Year 20X4. On 1 July 20X5 it made a 5 for 2 share split.
Basic EPS reported in 20X4 was: Rs. 20,000,000/4,000,000 shares = Rs. 5 per share
Earning attributable to ordinary shareholders for the year 20X5 are Rs. 24,000,000
Required:
Calculate basic EPS for the year ended 31 December 20X5 (including comparative for 20X4).

© Emile Woolf International 253 The Institute of Chartered Accountants of Pakistan


Financial accounting and reporting I

Answer:
Share split fraction = 5 / 2

Basic EPS Rs. 24,000,000


= = Rs. 2.4 per share
(20X5) 10,000,000 shares

Number of Time Weighted


Date Fraction
shares factor average number
1 January to 30 June 4,000,000 x 6/12 x 5/2 5,000,000
Increase due to share split 6,000,000
1 July to 31 December 10,000,000 x 6/12 5,000,000
10,000,000

Basic EPS Rs. 20,000,000


= = Rs. 2 per share
(20X4 restated) 4,000,000 x 5/2 shares

Alternatively, Basic EPS (20X4 restated) = Rs. 5 x 2/5 = Rs. 2 per share

2.6 Share consolidation


Share consolidation transactions, also known as reverse share split arrangements, results in
smaller number of larger denominations shares of an entity without affecting the total value of
share capital. Share consolidation arrangements are rare in practice and may be undertaken by
companies to reduce the number of shareholders in order to drive ownership of the company
towards institutional investors.
For example, a shareholder had 500 ordinary shares of Rs. 10 par value per share, that is total Rs.
5,000. The company made a 1 for 2 share consolidation. Now the shareholder would have 100
shares of Rs. 50 par value per share, that is same total of Rs. 5,000.
Since there is no change in actual resources, the calculation perspective is same as that of bonus
issue and a ‘share consolidation fraction’ is applied to ordinary shares already in issue.

Formula: Share consolidation fraction

Number of shares in holding after share consolidation


Number of shares in holding before share consolidation

The comparative EPS is also restated using the same basis as discussed in bonus issue.

Example 13: Atlantic Limited


Question: Atlantic Limited (AL) has a 31 December financial year end.
On 1 January 20X4 it has 4,000,000 shares in issue. There were no share issues or other changes
in Year 20X4. On 1 July 20X5 it made a 2 for 5 share consolidation.
Basic EPS reported in 20X4 was: Rs. 20,000,000/4,000,000 shares = Rs. 5 per share
Earning attributable to ordinary shareholders for the year 20X5 are Rs. 24,000,000
Required:
Calculate basic EPS for the year ended 31 December 20X5 (including comparative for 20X4).

© Emile Woolf International 254 The Institute of Chartered Accountants of Pakistan


Chapter 7: IAS 33: Earnings per share

Answer:
Share consolidation fraction = 2 / 5

Basic EPS Rs. 24,000,000


= = Rs. 15 per share
(20X5) 1,600,000 shares

Number of Time Weighted


Date Fraction
shares factor average number
1 January to 30 June 4,000,000 x 6/12 x 2/5 800,000
Reduction: share consolidation (2,400,000)
1 July to 31 December 1,600,000 x 6/12 800,000
1,600,000
Basic EPS Rs. 20,000,000
= = Rs. 12.5 per share
(20X4 restated) 4,000,000 x 2/5 shares

Alternatively, Basic EPS (20X4 restated) = Rs. 5 x 5/2 = Rs. 12.5 per share

2.7 Retrospective adjustments


If the number of ordinary (or potential ordinary shares for diluted EPS) outstanding increases as a
result of a capitalisation, bonus issue or share split, or decreases as a result of a reverse share
split, the calculation of basic (and diluted) EPS for all periods presented shall be adjusted
retrospectively.
If these changes occur after the reporting period but before the financial statements are authorised
for issue, the per share calculations for those and any prior period financial statements presented
shall be based on the new number of shares.
The fact that per share calculations reflect such changes in the number of shares shall be
disclosed.
In addition, basic (and diluted) earnings per share of all periods presented shall be adjusted for the
effects of errors and adjustments resulting from changes in accounting policies accounted for
retrospectively.

Example 14: Home Dynamics Limited


Question: The following information pertains to the financial statements of Home Dynamics Limited
(HDL), a listed company, for the year ended 31 December 20X6:

(i) Profit after tax for the year: Rs. in million


Profit from continuing operations – net of tax 765
Profit from discontinued operations – net of tax 155
Profit after tax 920

(ii) Shareholders’ equity as on 1 January 20X6 comprised of:


 10 million ordinary shares of Rs. 10 each, having market value of Rs. 25 each.
 4 million cumulative preference shares of Rs. 10 each entitled to a cumulative dividend
at 10%.
(iii) On 31 March 20X6, HDL announced 40% right shares to its ordinary shareholders at Rs. 25
per share. The entitlement date of right shares was 31 May 20X6. The market price per
share immediately before the announcement date and entitlement date was Rs. 28 and
Rs. 32 respectively.

© Emile Woolf International 255 The Institute of Chartered Accountants of Pakistan


Financial accounting and reporting I

(iv) On 2 August 20X6, HDL announced 20% bonus issue. The entitlement date of bonus shares
was 31 August 20X6.
(v) On 1 February 20X7, the board of directors announced 20% cash dividend and 10% bonus
issue being the final dividend to the ordinary shareholders and 10% cash dividend for
preference shareholders.
Required:
Calculate basic earnings per share for inclusion in HDL’s financial statements for the year ended
31 December 20X6. Show all relevant calculations.
Answer:

(10m shares x Rs. 32) + (4m share x Rs. 25)


Theoretical ex-
= = Rs. 30 per share
rights price
10m shares + 4m share

31st May Right issue bonus fraction = 32 / 30


31st August Bonus issue fraction = 120 / 100 = 1.2
1st February (20X7) Bonus issue fraction = 110 / 100 = 1.1
Preference dividend = 4 million x Rs. 10 x 10% = Rs. 4 million

Basic EPS (20X6) Rs. 765,000,000 – 4,000,000


Rs. 45.71 per
= =
Continuing operations share
16,646,667 shares

Basic EPS (20X6) Rs. 155,000,000


Rs. 9.31 per
= =
Discontinued operations share
16,646,667 shares

Number of Time Weighted


Date Fraction(s)
shares factor average
1 January to 31 May 10,000,000 x 5/12 x 32/30 x 1.2 x 1.1 5,866,667
Right issue on 31 May 4,000,000
1 June to 31 August 14,000,000 x 3/12 x 1.2 x 1.1 4,620,000
Bonus issue on 31 August 2,800,000
1 September to 31 December 16,800,000 x 4/12 x 1.1 6,160,000
Bonus issue on 1 February 1,680,000
After the year-end 18,480,000 x 0/12 0
16,646,667
.

© Emile Woolf International 256 The Institute of Chartered Accountants of Pakistan


Chapter 7: IAS 33: Earnings per share

3 DILUTED EPS
Section overview

 General concept
 When potential ordinary shares are anti-dilutive
 Convertible instruments
 Share options or warrants
 Order of dilution

3.1 General concept


An entity might have potential ordinary shares in issue. There is a possibility that these will become
actual ordinary shares at some time in the future. Potential ordinary shares do not impact
calculation of basic EPS but diluted EPS might differ from basic EPS when there are potential
ordinary shares in existence.
Examples of potential ordinary shares include:
 Convertible bonds (debentures)
 Convertible preference shares
 Share options or warrants
Definition
Options, warrants and their equivalents are financial instruments that give the holder the right to
purchase ordinary shares.

If potential shares become actual ordinary shares, the earnings figure will be shared with a larger
number of ordinary shares. This might dilute the EPS. The literal meaning of ‘dilution’ is ‘watering
down’ or ‘reduction in strength. IAS 33 defines ‘dilution’ as follows:
Definition
Dilution is a reduction in earnings per share or an increase in loss per share resulting from the
assumption that convertible instruments are converted, that options or warrants are exercised, or
that ordinary shares are issued upon the satisfaction of specified conditions.

The objective of diluted EPS is consistent with that of basic earnings per share, that is, to provide
a measure of the performance of each ordinary share taking into account dilutive potential ordinary
shares outstanding during the period.
If potential ordinary shares become actual ordinary shares, not only there would be change in
number of shares but earnings may also change as entity will no longer be paying interest on
convertible bonds or dividend on convertible preference shares.
For the purpose of calculating diluted EPS, an entity shall adjust profit or loss attributable to
ordinary equity holders, and the weighted average number of shares outstanding, for the effects of
all dilutive potential ordinary shares.
Not all potential ordinary shares are dilutive, though, and may be anti-dilutive.
Definition
Anti-dilution is an increase in earnings per share or a reduction in loss per share resulting from the
assumption that convertible instruments are converted, that options or warrants are exercised, or
that ordinary shares are issued upon the satisfaction of specified conditions.

3.2 When potential ordinary shares are anti-dilutive


When potential ordinary shares are anti-dilutive, they are disregarded in the calculation of diluted
EPS. The following summary is helpful:

© Emile Woolf International 257 The Institute of Chartered Accountants of Pakistan


Financial accounting and reporting I

Convertible bonds
Incremental EPS = Interest (net of tax) / ordinary shares obtainable on conversion
Anti-dilutive, when:
 Incremental EPS > basic earnings per share
 Incremental EPS < basic loss per share
Convertible preference shares
Incremental EPS = Dividend for current period / ordinary shares obtainable on conversion
Anti-dilutive, when:
 Incremental EPS > basic earnings per share
 Incremental EPS < basic loss per share
Options and warrants
Anti-dilutive, when:
 Options are ‘out of money’ in case of basic earnings per share
 There is basic loss per share
Options are ‘out of money’ when exercise price of the option exceeds share price. Nobody would
pay an exercise price of Rs. 100 for something worth only Rs. 80.

3.3 Convertible instruments


When there are convertible bonds or convertible preference shares, diluted EPS is calculated as
follows, by making adjustments to total earnings and the number of shares in issue used in the
basic EPS calculation.
Earnings
Earnings are adjusted because the entity would not have to pay the dividend or interest on the
convertible securities.
 For convertible preference shares, add back the preference dividend paid in the year. Total
earnings will be increased by the preference dividend saved.
 For convertible bonds, add back the interest charge on the bonds in the year less the tax
relief relating to that interest. Total earnings will increase by the interest saved less tax.
Number of shares
The weighted average number of shares is increased, by adding the maximum number of new
shares that would be created if all the potential ordinary shares were converted into actual ordinary
shares.
The additional number of shares is calculated on the assumption that they were in issue from the
beginning of the year or from the date of issue whichever is later.
IAS 33 provides for use of most dilutive option when multiple conversion options are available.

Example 15: Gold Limited


Question: Gold Limited (GL) has 12,000,000 ordinary shares and Rs. 4,000,000 5% convertible
bonds in issue as at 31 December 20X2, there have been no new issues of shares or bonds for
several years.
The bonds are convertible into ordinary shares in 20X3 or 20X4, at the following rates:
 At 30 shares for every Rs. 100 of bonds if converted at 31 December 20X3
 At 25 shares for every Rs. 100 of bonds if converted at 31 December 20X4
Total earnings for the year to 31 December 20X2 were Rs. 36,000,000. Tax is payable at a rate of
30% on profits.
Required:
Calculate basic EPS and diluted EPS for the year ended 31 December 20X2.

© Emile Woolf International 258 The Institute of Chartered Accountants of Pakistan


Chapter 7: IAS 33: Earnings per share

Answer:

Basic EPS Rs. 36,000,000


= = Rs. 3 per share
(20X2) 12,000,000 shares

Convertible bonds
Interest saving = Rs. 4,000,000 x 5% = Rs. 200,000
Tax effect = Rs. 200,000 x 30% = Rs. 60,000
Incremental earnings = Rs. 200,000 – 60,000 = Rs. 140,000
Incremental shares (maximum) = Rs. 4,000,000 / 100 x 30 shares = 1,200,000 shares
Incremental EPS = Rs. 140,000 / 1,200,000 shares = Rs. 0.1167
These are potentially dilutive since incremental EPS is less than basic EPS.

Diluted EPS Rs. 36,000,000 + 140,000


Rs. 2.74 per
= =
(20X2) share
12,000,000 + 1,200,000 shares
.

If new convertibles are issued during the course of the year, the additional number of shares and
the earnings adjustment are included only from the time that the convertibles were issued.

Example 16: Silver Limited


Question: Silver Limited (SL) has 10,000,000 ordinary shares in issue on 1 January 20X5. There has
been no new issue of shares for several years. However, the company issued Rs. 2,000,000 of
convertible 6% convertible preference shares on 1 April 20X5.
These are convertible into ordinary shares at the following rates:
 On 31 March 20X8: 25 shares for every Rs. 100 of convertible preference shares
 On 31 March 20X9: 20 shares for every Rs. 100 of convertible preference shares
In the financial year to 31 December 20X5 total earnings were Rs. 40,870,000. Tax is at the rate of
30%, however, preference dividend is not deductible for tax purposes.
Required:
Calculate basic EPS and diluted EPS for the year ended 31 December 20X5.

Answer:

Basic EPS Rs. 40,870,000


Rs. 4.09 per
= =
(20X5) share
10,000,000 shares

Convertible Preference shares


Preference dividend saving = Rs. 2,000,000 x 6% x 9/12 = Rs. 90,000
Tax effect = Nil since there is no tax relief on preference dividend
Incremental earnings = Rs. 90,000
Incremental shares (maximum) = Rs. 2,000,000 / 100 x 25 shares x 9/12 = 375,000 shares
Incremental EPS = Rs. 90,000 / 375,000 shares = Rs. 0.24
These are potentially dilutive since incremental EPS is less than basic EPS.

Diluted EPS Rs. 40,870,000 + 90,000


Rs. 3.95 per
= =
(20X5) share
10,000,000 + 375,000 shares
.

© Emile Woolf International 259 The Institute of Chartered Accountants of Pakistan


Financial accounting and reporting I

3.4 Share options or warrants


If the option holder exercises this right the number of shares would increase and the company
would receive the cash paid for the shares and this would be available to invest in the business
and in turn this would be expected to boost its earnings. However, it is impossible to predict how
total earnings will be affected when the cash is eventually received.
This presents a problem. Including the shares in the diluted EPS calculation without adjusting the
earnings would be inconsistent but it is not possible to adjust the earnings.
IAS 33 solves this problem in quite a neat way. The amount that would be received on exercise of
the options is treated as cash received from selling shares at full price with the remaining shares
having been given away. The shares sold at full price are not considered to be dilutive as any cash
would be invested to earn the same return as earned in the period. It is only the free shares that
are dilutive.
Free shares can be calculated as follows:

Formula: Free shares

Fair value* – Exercise price


Number of options in issue X
Fair value*
*Average share price during the period

Diluted EPS can be calculated by adding free shares in weighted average number of shares used
in basic EPS calculation and not change in the amount of earnings is required.

Example 17: Bronze Limited


Question: Bronze Limited (BL) had total earnings during Year 20X3 of Rs. 25,000,000. It has
5,000,000 ordinary shares in issue.
There are outstanding share options on 400,000 shares, which can be exercised at a future date,
at an exercise price of Rs. 25 per share.
The average market price of shares in BL during Year 20X3 was Rs. 40.
Required:
Calculate basic EPS and diluted EPS for the year ended 31 December 20X3.

Answer:

Basic EPS Rs. 25,000,000


= = Rs. 5 per share
(20X3) 5,000,000 shares

Share options
These are dilutive since exercise price of Rs. 25 is less than share market price of Rs. 40 i.e. in the
money options.
Free shares = 400,000 options x (40 – 25) / 40 = 150,000 shares

Diluted EPS Rs. 25,000,000


Rs. 4.85 per
= =
(20X3) share
5,000,000 + 150,000 shares
.

3.5 Order of dilution


When there are several types of potential ordinary share in issue, they should be ranked in order
of dilution, with the most dilutive potential ordinary shares ranked first. In order to carry out the
ranking the earnings per incremental share is found for each potential ordinary share. This is the
earnings adjustment that would be necessarily divided by the number of shares that would come
into being if the share were included in the calculation of diluted EPS.

© Emile Woolf International 260 The Institute of Chartered Accountants of Pakistan


Chapter 7: IAS 33: Earnings per share

Note that ‘in the money’ options always rank first as they increase the number of shares in the
calculation without affecting the earnings.

Example 18: Olympics Limited


Question: The following information relates to Olympics Limited (OL) for the year ended 31
December 20X5.
Number of ordinary shares in issue 2,000,000
Reported earnings in the year Rs. 6,000,000
Average market price of shares during the year Rs. 80

Potential ordinary shares:

Options 600,000 options, with an exercise price of Rs. 60

1,000,000 7% convertible Each preference share is convertible in 20X8 into ordinary shares
preference shares of Rs. 10 at the rate of 3 ordinary share for every 10 preference shares
each

4% convertible bond: Each bond is convertible in 20X9 into ordinary shares at the rate
of 20 new shares for every Rs. 100 of bonds.
Rs. 5,000,000

Tax rate is 30%.


Required:
Calculate basic EPS and diluted EPS for the year ended 31 December 20X5.

Answer:

Basic EPS Rs. 6,000,000


= = Rs. 3 per share
(20X5) 2,000,000 shares

Ranking order

Options Rank
Incremental earnings Nil
Incremental shares 600,000 x (80 – 60) / 80 150,000
Incremental EPS Nil 1

Convertible preference shares


Incremental earnings 1,000,000 x Rs. 10 x 7% Rs. 700,000
Incremental shares 1,000,000 / 10 x 3 shares 300,000
Incremental EPS Rs. 2.33 3

Convertible bond
Incremental earnings Rs. 5,000,000 x 4% x 70% Rs. 140,000
Incremental shares Rs. 5,000,000 / 100 x 20 shares 1,000,000
Incremental EPS Rs. 0.14 2

© Emile Woolf International 261 The Institute of Chartered Accountants of Pakistan


Financial accounting and reporting I

Although individually all of the above are dilutive, we must consider each item one by one to
consider dilution.

Earnings Rs. Shares EPS Rs. Impact


Basic EPS 6,000,000 2,000,000 3.00
Options (Rank 1) 0 150,000
6,000,000 2,150,000 2.79 Dilutive
Convertible bonds (Rank 2) 140,000 1,000,000
6,140,000 3,150,000 1.94 Dilutive
Convertible pref. shares (Rank 3) 700,000 300,000
6,840,000 3,450,000 1.98 Anti-dilutive

The convertible preference shares are not dilutive, and the reported diluted EPS should be Rs. 1.94
(and not Rs. 1.98).

© Emile Woolf International 262 The Institute of Chartered Accountants of Pakistan


Chapter 7: IAS 33: Earnings per share

4 PRESENTATION AND DISCLOSURE


Section overview

 Presentation requirements
 Disclosure requirements
 Additional measure of EPS

4.1 Presentation requirements


An entity should present in the statement of profit or loss:
 the basic EPS and
 the diluted EPS
for the profit or loss from continuing operations.
For consolidated accounts, this is the EPS and diluted EPS attributable to the owners of the parent
company.
The basic EPS and diluted EPS should be presented with equal prominence for all the periods
presented (the current year and the previous year). These figures are presented at the end of the
statement of profit or loss.
If the entity presents a separate statement of profit or loss:
 the EPS and diluted EPS should be shown in this statement, and
 not in the statement of comprehensive income.
If there is a discontinued operation, the basic EPS and diluted EPS from discontinued operation
should be shown either on the face of the statement of profit or loss or in a note to the financial
statements.
The basic and the diluted EPS should be presented, even if it is a negative figure (even if it is a
loss per share).
If diluted earnings per share is reported for at least one period, it shall be reported for all periods
presented, even if it equals basic earnings per share. If basic and diluted earnings per share are
equal, dual presentation can be accomplished in one line in the statement of comprehensive
income.

4.2 Disclosure requirements


IAS 33 also requires disclosure in a note to the financial statements of the following:
 The total amounts used as the numerators (total earnings figures) to calculate the basic EPS
and diluted EPS, and a reconciliation of these numerator figures to the profit or loss for the
period
 The total amounts used in the denominators (weighted average number of shares) to
calculate the basic EPS and diluted EPS, and a reconciliation of these two denominator
figures to each other.

4.3 Additional measure of EPS


An entity may disclose, in addition to basic and diluted earnings per share, amounts per share
using a reported component of profit other than one required by IAS 33, for example, EPS based
on operating profit or profit from major segment of the business. In such case, the following
requirements apply:
 such amounts shall be calculated using the weighted average number of ordinary shares
determined in accordance with IAS 33.

© Emile Woolf International 263 The Institute of Chartered Accountants of Pakistan


Financial accounting and reporting I

 Basic and diluted amounts per share relating to such a component shall be disclosed with
equal prominence and presented in the notes (not on face).
 An entity shall indicate the basis on which the numerator(s) is (are) determined, including
whether amounts per share are before tax or after tax.
 If a component of profit is used that is not reported as a line item in the statement of profit or
loss and other comprehensive income, a reconciliation shall be provided.

© Emile Woolf International 264 The Institute of Chartered Accountants of Pakistan


Chapter 7: IAS 33: Earnings per share

5 USEFULNESS AND LIMITATIONS


Section overview

 Usefulness of EPS as financial information


 Limitations of EPS

5.1 Usefulness of EPS as financial information


EPS is an important component of determining an entity’s P/E ratio i.e. calculated as market value
per share divided by earnings per share. P/E ratio is often used by investor in making stock market
decisions. The reliable EPS means reliable P/E ratio leading to better and informed decisions by
investors.
Investors and their advisers pay close attention to an entity’s net profit for the period. However,
profit for the period can include large and unusual items and also the results of discontinued
operations. This may make it volatile i.e. liable to fluctuate rapidly up and down. Users can then
find it difficult to assess trends in the profit figure or to use the current year’s profit to predict an
entity’s performance in future years.
The trend (improvement or deterioration) in an entity’s published EPS figure can sometimes be a
more reliable indicator of future performance. There are a number of reasons for this.
 The standard version of both basic and diluted EPS is based on profit from continuing
operations. This means that the results of discontinued operations (which may distort total
profit) are excluded.
 An entity may also choose to present one or more alternative versions of EPS. These
normally exclude large or unusual items so that EPS is based on ‘normal’ recurring earnings.
 EPS measures an entity’s performance from the viewpoint of investors. It shows the amount
of earnings available to each ordinary shareholder. This means that EPS takes the effect of
preference dividends (if any) into account. It also takes share issues into account.
 Diluted EPS can provide an ‘early warning’ of any changes to an investor’s potential return
on their investment due to future share issues.

5.2 Limitations of EPS


EPS is probably the single most important indicator of an entity’s performance. It is a very useful
measure when it is used as the starting point for a more detailed analysis of an entity’s
performance.
However, EPS can have serious limitations:
 Not all entities use the same accounting policies. It may not always be possible to make
meaningful comparisons between the EPS of different entities.
 EPS does not take account of inflation, so that growth in EPS over time might be misleading.
 EPS measures an entity’s profitability, but this is only part of an entity’s overall performance.
An entity’s cash flow can be just as important as its profit (and more essential to its
immediate survival). Changes in the value of assets (holding gains) can also be an important
part of performance for some entities.
 Diluted EPS is often described as an ‘early warning’ to investors that the return on their
investment may fall sometime in the future. However, diluted EPS is based on current
earnings, not forecast earnings. This means that it may not be a reliable predictor of future
EPS.
One of the main problems with EPS can be the way that it is used by investors and others. Users
often rely on EPS as the main or only measure of an entity’s performance. Management know this
and try to make EPS appear as high as possible. They may attempt to manipulate the figure by
using ‘creative accounting’. They may also make decisions which increase EPS in the short term
but which damage the entity in the longer term.

© Emile Woolf International 265 The Institute of Chartered Accountants of Pakistan


Financial accounting and reporting I

5 OBJECTIVE BASED QUESTIONS


01. Anum Limited had 1 million ordinary shares of Rs. 10 each in issue throughout the year ended 30
June 20X3. On 1 July 20X2 it had issued Rs. 2 million of 6% convertible debentures, each Rs.
100 of debenture is convertible into 8 ordinary shares on 1 July 20X6 at the option of the holder.
Anum Limited had profit after tax for the year ended 30 June 20X3 of Rs.1,850,000. It pays tax
on profits at 30%.
What was diluted EPS for the year ended 30 June 20X3?

(a) Rs. 1.67


(b) Rs. 1.85
(c) Rs. 1.61
(d) Rs. 1.70

02. Zahra Limited’s basic EPS for the year ended 30 June 20X1 was Rs. 6 per share. Which of the
following issue during the year during the year ended 30 June 20X2 would result in restated
comparative basic EPS (for the year ended 20 June 20X1) of more than Rs. 6?
(a) Right issue
(b) Bonus issue
(c) Share split
(d) Share consolidation / Reverse share split

03. Hania Limited has correctly calculated its basic earnings per share (EPS) for the current year.
Which of the following items need to be additionally considered when calculating the diluted EPS
of Hania Limited for the year?
(i) A 1 for 5 rights issue of equity shares during the year at Rs. 12 when market price of
equity shares was Rs. 18.
(ii) The issue during the year of a convertible debentures
(iii) Issue of directors' share options exercisable in three years' time
(iv) Equity shares issued during year as the purchase consideration for acquisition of plant &
machinery
(a) All four
(b) (i) and (ii) only
(c) (ii) and (iii) only
(d) (iii) and (iv) only

04. Many analyst believe that the trend of earnings per share (EPS) is a more reliable indicator of
underlying performance than the trend of net profit for the year.
Which of the following statements supports this view?

(a) Net profit can be manipulated by the choice of accounting policies but EPS cannot be
manipulated in this way.
(b) EPS takes into account the additional resources made available to earn profit when new
shares are issued for cash, whereas net profit does not.
(c) The disclosure of a diluted EPS figure is a forecast of the future trend of profit.
(d) The comparative EPS is restated where a change of accounting policy affects the
previous year's profits.

© Emile Woolf International 266 The Institute of Chartered Accountants of Pakistan


Chapter 7: IAS 33: Earnings per share

05. At 1 January 20X8 Maria Limited had 5 million Rs. 10 equity shares in issue. On 1 June 20X8 it
made a 1 for 5 rights issue at a price of Rs. 15. The market price of the shares before right issue
was Rs. 18. Total earnings for the year ended 31 December 20X8 was Rs. 7.6 million.
What is basic EPS for the year ended 31 December 20X8?

(a) Rs.1.35
(b) Rs.1.36
(c) Rs.1.27
(d) Rs.1.06

06. Gulfishan Limited (GL) had profits after tax of Rs. 30 million in the year ended 31 December 20X7.
On 1 January 20X7, GL had 2.4 million ordinary shares in issue. On 1 April 20X7 it made a 1 for
2 rights issue at a price of Rs. 14 when the market price of Garfish’s shares was Rs. 20 before
right issue.
What is the basic EPS for the year ended 31 December 20X7?

(a) Rs. 4.95


(b) Rs. 8.90
(c) Rs. 9.12
(d) Rs. 9.26

07. On 1st January 20X4, Sameen Limited had 3 million ordinary shares in issue. On 1st June 20X4,
Sameen Limited made a 1 for 3 bonus issue. On 30th September 20X4, Sameen Limited then
issued a further 1 million shares at full market price. Sameen Limited had profits attributable to
ordinary shareholders of Rs. 20 million for the year ended 31st December 20X4.
What is the basic EPS for the year ended 31st December 20X4?

(a) Rs. 4.71


(b) Rs. 4.29
(c) Rs. 4.93
(d) Rs. 5.22

08. During the year, Mariam Limited made a 1 for 3 rights issue at Rs. 16 when the market price was
Rs. 22.The previous year’s financial statements showed an earnings per share figure of Rs. 4.
There were no other issues of shares during the year.
What will the restated earnings per share figure be for comparative purposes in the current year
financial statements?

(a) Insufficient data for calculation


(b) Rs. 3.73
(c) Rs. 4.00
(d) Rs. 4.29

09. Saba Limited has net profit for the year ended 30 June 20X5 of Rs. 10.5 million. Saba Limited
has had 6 million shares in issue for many years. In the current year, Saba Limited has issued a
convertible bond. It was issued at its nominal value of Rs. 2.5 million and carries an effective
interest rate of 8%.

© Emile Woolf International 267 The Institute of Chartered Accountants of Pakistan


Financial accounting and reporting I

The bond is convertible in five years in 50 shares for each Rs. 100 of the bond.
Saba Limited pays tax at a rate of 30%
What is the Basic EPS and Diluted EPS for the year ended 30 June 20X5?

(a) Basic EPS Rs. 1.70 and Diluted EPS Rs. 1.50
(b) Basic EPS Rs. 1.75 and Diluted EPS Rs. 1.70
(c) Basic EPS Rs. 1.70 and Diluted EPS Rs. 1.55
(d) Basic EPS Rs. 1.75 and Diluted EPS Rs. 1.47

10. Alia Limited’s (AL) financial statements show a profit for the year of Rs. 20 million. On 1 January
20X5, AL had 4 million shares in issue.
There were no new issues of shares in the year, but there were 1 million outstanding options to
buy shares for Rs. 30 each. For the year to 31 December 20X5, the average market value of AL’s
shares was Rs. 50.
What is AL’s Diluted EPS for the year ended 31 December 20X5?

(a) Rs. 3.00


(b) Rs. 4.35
(c) Rs. 4.55
(d) Rs. 4.00

11. Which of the following statements in relation to the term 'dilution' is/are true or false, as per IAS
33 Earnings per share?
1) A reduction in earnings per share is an example of dilution.
2) A reduction in loss per share is an example of dilution.

(a) Both statements are false


(b) Statement 2 is true and statement 1 is false
(c) Statement 1 is true and statement 2 is false
(d) Both statements are true

12. Which TWO of the following items must be disclosed, as per IAS33 Earnings per share?
(a) Forecast earnings per share for the following financial year
(b) A five-year trend analysis of earnings per share
(c) The weighted average number of ordinary shares used to calculate earnings per share
(d) The earnings figures used in calculating basic and diluted earnings per share

13. Which of the following statements is/are true or false, according to IAS 33 Earnings per share?
1) Earnings per share amounts should not be presented if they are negative, i.e. losses per
share.
2) Earnings per share amounts calculated for discontinued operations should be presented.
(a) Both statements are false
(b) Statement 1 is false and statement 2 is true
(c) Statement 1 is true and statement 2 is false
(d) Both statements are true

© Emile Woolf International 268 The Institute of Chartered Accountants of Pakistan


Chapter 7: IAS 33: Earnings per share

14. Pareesa Limited issued new ordinary shares for cash at full market price and also made a 1 for 8
bonus issue.
Are the following statements true or false, according to IAS 33 Earnings per share?
1) New shares issued as a result of bonus issue should be time apportioned from their date
of issue.
2) New shares issued for cash at full market price should be time apportioned from their
date of issue.
(a) Both statements are false
(b) Statement 1 is false and statement 2 is true
(c) Statement 1 is true and statement 2 is false
(d) Both statements are true

15. Saima Limited (SL) is a company listed on a Pakistan Stock Exchange. Given below is an extract
from its statement of comprehensive income for the year ended 31 December 20X7.

Rs.
Profit before tax 5,700,000
Tax expense (1,500,000)
Profit after tax 4,200,000

SL paid during the year an ordinary dividend of Rs. 400,000 and a dividend on its redeemable
preference shares of Rs. 500,000. These have been correctly accounted for in the financial
statements.
SL had ordinary share capital of Rs. 2,000,000 (Rs. 10 each) throughout the year and authorised
share capital of Rs. 10,000,000 (Rs. 10 each).
What is basic EPS for the year ended 31 December 20X7?

(a) Rs. 21
(b) Rs. 19
(c) Rs. 38
(d) Rs. 42

16. Shaista Limited is a company listed on Pakistan Stock Exchange. Its financial statements for the
year ended 31 December 20X7 showed EPS of Rs. 8.5 per share.
On 1 July 20X8 Shaista Limited made a 1 for 3 bonus issue.
What figure for the 20X7 earnings per share will be shown as comparative information in the
financial statements for the year ended 31 December 20X8?

(a) Rs. 2.13


(b) Rs. 2.55
(c) Rs. 6.38
(d) Rs. 2.83

© Emile Woolf International 269 The Institute of Chartered Accountants of Pakistan


Financial accounting and reporting I

17. Simba Limited is a listed company. At 31 December 20X7, it had ordinary share capital of Rs.
2,000,000 (Rs. 10 each). Profit before tax for the year was Rs. 500,000 and the tax charge was
Rs. 125,000.
What is Simba Limited’s basic EPS for the year?

(a) Rs. 2.50


(b) Rs. 3.75
(c) Rs. 2.58
(d) Rs. 1.88

18. Muntaha Limited (ML) is listed on Pakistan Stock Exchange. During the year ended 31 December
20X7, the company had 5 million ordinary shares of Rs.10 each and 500,000 6% irredeemable
preference shares of Rs. 10 in issue. The profit after tax for the year 20X7 was Rs. 6,000,000.
What is ML's basic EPS for the year?

(a) Rs. 1.14


(b) Rs. 1.09
(c) Rs. 1.60
(d) Rs. 1.20

19. Mansha Limited had 100,000 equity shares in issue on 1 January 20X7. On 1 July 20X7 it issued
20,000 new shares by way of a 1 for 5 bonus issue. On 1 October 20X7 it issued 28,000 new
shares for cash at full market price.
What is weighted average number of shares for calculation of basic EPS?

(a) 100,000 shares


(b) 117,000 shares
(c) 148,000 shares
(d) 127,000 shares

20. Which of the following need not be adjusted in profit after tax, for calculation of basic EPS?
(a) Redeemable preference share dividends
(b) Irredeemable preference share dividends
(c) Ordinary (final) dividends
(d) Ordinary (interim) dividends

© Emile Woolf International 270 The Institute of Chartered Accountants of Pakistan


Chapter 7: IAS 33: Earnings per share

5 OBJECTIVE BASED ANSWERS


01. (a) Basic EPS = Rs. 1,850,000 / 1,000,000 shares = Rs. 1.85
Incremental earnings = Rs. 2,000,000 x 6% x 70% = Rs. 84,000
Incremental shares = Rs. 2,000,000 / 100 x 8 shares = 160,000 shares
Diluted EPS = [Rs. 1,850,000 + 84,000] / [1,000,000 + 160,000 shares] = Rs. 1.67

02. (d) Share consolidation/ reverse share split would result in lower number of denominator
(shares) leading to higher comparative EPS than reported earlier.
All other issues would result in higher number of shares (denominator) resulting in
lower comparative EPS than reported earlier.

03. (c) The convertible loan note and the share options should be taken into account when
calculating diluted EPS. Other items would have been incorporated in calculation of
basic EPS already.

04. (b) EPS takes into account the additional resources made available to earn profit when
new shares are issued for cash, whereas net profit does not.

05. (a) TERP = [(5x18) + (1x15)] / (5+1) = Rs. 17.5 per share
Right issue bonus fraction = 18 / 17.5

Weighted average shares


January to May 5 million x 5/12 x 18 /17.5 = 2.14 million
June to December 6 million x 7/12 = 3.50 million
Total = 5.64 million

Basic EPS = Rs. 7.6 million / 5.64 million shares = Rs. 1.35

06. (b) TERP = [(2x20) + (1x14)] / (2+1) = Rs. 18 per share


Right issue bonus fraction = 20 / 18

Weighted average shares


January to March 2.4 million x 3/12 x 20 /18 = 0.67 million
April to December 3.6 million x 9/12 = 2.70 million
Total = 3.37 million

Basic EPS = Rs. 30 million / 3.37 million shares = Rs. 8.90

07. (a) Bonus issue fraction = 4 / 3

Weighted average shares


January to May 3 million x 5/12 x 4 /3 = 1.67 million
June to September 4 million x 4/12 = 1.33 million
October to December 5 million x 3/12 = 1.25 million
Total = 4.25 million

Basic EPS = Rs. 20 million / 4.25 million shares = Rs. 4.71

© Emile Woolf International 271 The Institute of Chartered Accountants of Pakistan


Financial accounting and reporting I

08. (b) TERP = [(3x22) + (1x16)] / (3+1) = Rs. 20.5 per share
Right issue bonus fraction = 22 / 20.5
Basic EPS (restated comparative) = Rs. 4 x 20.5 / 22 = Rs. 3.73

09. (d) Basic EPS = Rs. 10.5 million / 6 million shares = Rs. 1.75
Incremental earnings = Rs. 2.5 million x 8% x 70% = Rs. 0.14 million
Incremental shares = Rs. 2.5 million / 100 x 50 = 1.25 million shares
Diluted EPS = (Rs. 10.5m + 0.14m) / (6m + 1.25m shares) = Rs. 1.47

10. (c) Free shares = 1,000,000 x (50 – 30) / 50 = 400,000 shares


Diluted EPS = Rs. 20,000,000 / (4,000,000 + 400,000 shares) = Rs. 4.55

11. (c) A reduction in earnings per share is an example of dilution, but a reduction in loss
per share is an example of anti-dilution.

12. (c) & (d) The earnings figures and the weighted average number of shares are required to
be disclosed. Future forecasts and trend analysis are not required.

13. (b) EPS is to be presented even if negative (i.e. loss per share), so statement 1 is
false.
EPS for discontinued operations should be presented separately as well, so
statement 2 is true.

14. (b) Bonus shares provide no additional consideration to the issuer, so they are not
time-apportioned.

15. (a) The redeemable preference share dividend is recognised as a finance cost and
deducted at arriving at profit before tax.
Basic EPS = Rs. 4,200,000 / 200,000 shares = Rs. 21

16. (c) Bonus issue fraction = 4 / 3


Rs. 8.5 per share x 3 / 4 = Rs. 6.38 per share

17. (d) Basic EPS = (Rs. 500,000 – Rs. 125,000) / 200,000 shares = Rs. 1.88

18. (a) Basic EPS = (Rs. 6,000,000 – (6% × Rs. 5,000,000)) / 5,000,000 shares = Rs. 1.14

19. (d) Bonus issue is not time-apportioned while issue at full market price is time-
apportioned.
The weighted average is 100,000 + 20,000 + (28,000 × 3/12) = 127,000.

20. (a) Redeemable preference dividends will already have been removed from net profit
when arriving at this figure in a profit or loss account. Therefore, this adjustment is
not necessary.

© Emile Woolf International 272 The Institute of Chartered Accountants of Pakistan


Certificate in Accounting and Finance

CHAPTER
Financial accounting and reporting I

IAS 7: Statement of cash flows

Contents
1 Introduction
2 Cash flows from operating activities: The indirect method
3 Cash flows from operating activities: The direct method
4 Cash flows from investing activities
5 Cash flows from financing activities
6 Objective based questions and answers

* The student must refer original handbook of IFRS.

© Emile Woolf International 273 The Institute of Chartered Accountants of Pakistan


Financial accounting and reporting I

1 INTRODUCTION
Section overview

 Cash Flows
 Purpose & Importance of cash flow for business
 Cash Flows & Financial Statements
 Statement of cash flows
 The sections of a statement of cash flows

1.1 Cash Flows


Generating positive, sustainable cash flow is critical for an organisation's long-term success.
Keeping track of cash flows is particularly important for management to project the financial health
of the organisation to potential investors. Analysing the cash flow statement is extremely valuable
because it provides a reconciliation of the beginning and ending cash on the balance sheet.
Cash flows are inflows and outflows of cash and cash equivalents. Cash comprises cash on hand
and demand deposits. Cash equivalents are short-term, highly liquid investments that are readily
convertible to known amounts of cash and which are subject to an insignificant risk of changes in
value.
Cash is one of the major lubricants of business activity, but there are certain things that are not
reflected in cash flows. For example, profit earned after tax during a period because profitability is
composed also of things that are not cash based. Therefore, the overall financial well-being of the
company is not indicated in cash flows. Furthermore, accounts receivable and accounts payable
are also not reflected in the cash flow statement.

1.2 Purpose & Importance of cash flow for business


Monitoring the cash flows is one of the most pressing management tasks for any business. The
most common outflow of cash includes payments of salaries and to suppliers. The inflow includes
the receipt from customers and investors.
Businesses must have sufficient cash; otherwise they cannot survive.
 A business can make a loss but still survive if it has sufficient cash or access to liquidity
(cash, assets that can be quickly turned into cash and new sources of borrowing).
 On the other hand, a business that is profitable cannot survive if it cannot pay its obligations
when they fall due, because it does not have enough cash or access to other sources of
liquidity.
Cash flow is therefore extremely important, and it is appropriate that entities should present a
statement of cash flows as a financial statement.
The purpose of a statement of cash flows is to show what the cash flows of the entity have been.
It can also be used to make assessments of what the cash flows of the entity might be in the future.
In other words, the cash flow statement is a compressed version of the company's check book that
includes a few other items that affect cash, like the financing section, which shows how much the
company spent or collected from the repurchase or sale of shares, the amount of issuance or
retirement of debt and the amount the company paid out in dividends.
IAS 1 states that a statement of cash flows is a part of a complete set of the financial statements
of an entity. It provides information about:
 the cash flows of the entity during the reporting period, and
 The changes in cash and cash equivalents during the period.

© Emile Woolf International 274 The Institute of Chartered Accountants of Pakistan


Chapter 8: IAS 7: Statement of cash flows

IAS 7: Statements of cash flows sets out the benefits of cash flow information to users of financial
statements.
 A statement of cash flows provides information that helps users to evaluate changes in the
net assets of an entity and in its financial structure (including its liquidity and solvency).
 It provides information that helps users to assess the ability of the entity to affect the amount
and timing of its cash flows in order to adapt to changing circumstances and unexpected
opportunities.
 It is useful in assessing the ability of the entity to generate cash and cash equivalents.
 It helps users of accounts to compare the performance of different entities because unlike
profits, comparisons of cash flows are not affected by the different accounting policies used
by different entities.
 Historical cash flows are often a fairly reliable indicator of the amount, timing and certainty
of future cash flows.
1.2.1 Comparison of usefulness of cash flow information with profit or loss
Each financial statement, individually and in combination with other financial statements and other
information, provides useful information that helps users of financial statements to make informed
decisions. A balance between profitability and liquidity (cash balance) is required, a huge cash
balance does not usually indicate good management as this could have been invested to earn
more profits. In particular, following points should be considered:
 The amount and composition of net assets of an entity changes due to income and expenses
(statement of profit or loss) and cash flows (statement of cash flows). Both statements are
relevant but provide different aspects of information.
 Statement of cash flows additionally provides information that helps understand the
relationship between profits earned and cash flows generated by an entity (indirect method),
thus it indicates the quality of the profit earned;
 Many decision making models and valuation models rely on present value of the future cash
flows generated by an entity e.g. NPV and IRR. Historical cash flow information can be
useful to check the accuracy of past assessments and development of future assessments.
 Profitability is an important performance measure and this information is provided by
statement of profit or loss, liquidity information is also important and this information is
provided by statement of cash flows in conjunction with statement of financial position.
 Cash flows are necessary to survive in short term but in long term business must be
profitable to survive. Entities often forego short term benefits for long term major benefits
e.g. sales on credit usually earns higher profit margin as compared to cash sales.
 Cash flows information cannot be manipulated easily, as compared to profit or loss, the
margin of manipulation is significantly less. The cash flow is not affected by different
accounting policies and estimates and this makes cash flow information more comparable.
1.3 Cash Flows & Financial Statements
When a business makes a profit of Rs. 1,000, this does not mean that it receives Rs. 1,000 more
in cash than it has spent. Profit and cash flow are different, for several reasons:
There are items of cost in the statement of comprehensive income that do not represent a cash
flow. Examples are:
 depreciation and amortisation charges; and
 The gain or loss on the disposal of non-current assets.
There are items of cash flow that do not appear in the statement of comprehensive income.
Examples are:
 Cash flows relating to the acquisition or disposal of investments, such as the purchase of
new non-current assets, and cash from the sale of non-current assets. (The statement of
comprehensive income includes gains or losses on the disposal of non-current assets, but
this is not the same as the cash proceeds from the sale.)

© Emile Woolf International 275 The Institute of Chartered Accountants of Pakistan


Financial accounting and reporting I

 Cash flows relating to financial transactions, such as obtaining cash by issuing shares or
obtaining loans, the repayment of loans and the payment of dividends to ordinary
shareholders.
 Theoretically this could be done by analysing every entry in and out of the cash account(s)
over the course of a period. However, the cash account is often the busiest account in the
general ledger with potentially many thousands of entries. Documents that summarise the
transactions are needed.
 These documents already exist. They are the other financial statements (statement of
financial position and statement of profit or loss and other comprehensive income).

Illustration 01:
A business might buy 100 new non-current assets over the year. There would be 100 different
entries for these in the cash account.
However, it should be easy to estimate the additions figure from comparing the opening and closing
balances for non-current assets and isolating any other causes of movement.
For example, if we know that property plant and equipment has increased by Rs. 100,000 and that
the only other cause of movement was depreciation of Rs. 15,000 then additions must have been
Rs.115,000.

 A lot of the numbers in cash flow statements are derived from comparing opening and
closing positions of line items in the statement of financial position. Other causes of
movement can then be identified leaving the cash double entry as a balancing figure.

1.4 Statement of cash flows


A statement of cash flows provides information about where a business obtained its cash during
the financial period, and how it made use of its cash.
A statement of cash flows groups inflows and outflows of cash fewer than three broad headings:
 cash generated from or (used in) operating activities
 cash obtained from or (used in) investing activities
 Cash received from or paid in financing activities.
A statement of cash flows reports the change in the amount of cash and cash equivalents held by
the entity during the financial period.

Definition: Cash, cash equivalents and cash flows


Cash comprises cash on hand and demand deposits.
Cash equivalents are short-term, highly liquid investments that are readily convertible to known
amounts of cash and which are subject to an insignificant risk of changes in value.
Cash flows are inflows and outflows of cash and cash equivalents.

For the purpose of a statement of cash flows, cash and cash equivalents are treated as being the
same thing. This means that cash flows between cash and cash equivalent balances are not shown
in the statement of cash flows. These components are part of the cash management of an entity
rather than part of its operating, investing and financing activities.
Cash and cash equivalents are held in order to meet short-term cash commitments, rather than for
investment purposes or other purposes.
Examples of cash equivalents are:
 A bank deposit where some notice of withdrawal is required
 Short-term investments with a maturity of three months or less from the date of acquisition
(e.g. Government bills).

© Emile Woolf International 276 The Institute of Chartered Accountants of Pakistan


Chapter 8: IAS 7: Statement of cash flows

Bank borrowings are generally considered to be financing activities. In that case they would be
held outside cash and cash equivalents and movements on the bank borrowings would be shown
under financing activities as a cash inflow if borrowing increase or as a cash outflow if borrowings
fell.
Sometimes, bank overdrafts which are repayable on demand form an integral part of an entity's
cash management. In these circumstances, bank overdrafts are included as a component of cash
and cash equivalents.
It also shows whether there was an increase or a decrease in the amount of cash held by the entity
between the beginning and the end of the period

Illustration 02:
Cash generated from or (used in) operating activities X/(X)
Cash obtained from or (used in) investing activities X/(X)
Cash received from or (paid in) financing activities. X/(X)
Net cash inflow (or outflow) during the period X/(X)
Cash and cash equivalents at the beginning of the period X/(X)
Cash and cash equivalents at the end of the period X/(X)

1.5 The sections of a statement of cash flows


The content and format of statements of cash flows are specified by IAS 7 Statements of cash
flows. IAS 7 does not specify what the exact format of a statement of cash flows should be, but it
provides suggested layouts in an appendix.
Entities are required by IAS 7 to report cash flows for the period under three headings:
 Cash flows from operating activities
 Cash flows from investing activities
 Cash flows from financing activities.
All cash flows (except for changes from cash to cash equivalents or from cash equivalents to cash)
can be included in one of these three categories.
Together, the cash flows arising from these three categories of activity explain the increase or
decrease in cash and cash equivalents during the financial period.
The cash flows for each category might be positive or negative. The total of the cash flows for all
three categories together explains the overall increase or decrease in cash and cash equivalents
during the period.
A single transaction might include more than one type of cash flow. For example, a cash repayment
of a loan might include both interest and capital. In this case the interest element might be classified
as an operating activity and the capital element as a financing activity.

1.5.1 Cash flows from operating activities


Operating activities are the normal trading activities of the entity. Cash flows from operating
activities are the cash inflows or cash outflows arising in normal trading activities.
Operating activities normally provide an operating profit before tax. However, profit is not the same
as cash flow, and the cash flows from operating activities are different from profit.

A statement of cash flows normally makes a distinction between:


 Cash generated from operations, which is the cash from sales less the cash payments for
operating costs, and

© Emile Woolf International 277 The Institute of Chartered Accountants of Pakistan


Financial accounting and reporting I

 Net cash from operating activities, which is the cash generated from operations, less
interest payments and tax paid on profits.
Cash flows from operating activities are primarily derived from the principal revenue-producing
activities of the entity. Therefore, they generally result from the transactions and other events that
enter into the determination of profit or loss.
Examples of cash flows from operating activities are:
 Cash receipts from the sale of goods and the rendering of services;
 Cash receipts from royalties, fees, commissions and other revenue;
 Cash payments to suppliers for goods and services;
 Cash payments to and on behalf of employees;
 Cash receipts and cash payments of an insurance entity for premiums and claims, annuities
and other policy benefits;
 Cash payments or refunds of income taxes unless they can be specifically identified with
financing and investing activities; and
 Cash receipts and payments from contracts held for dealing or trading purposes.
Some transactions result in the recognition of a gain or loss in profit or loss (e.g. sale of an item of
plant). However, the cash flows relating to such transactions are cash flows from investing
activities.
Cash payments to manufacture or acquire assets held for rental to others and subsequently held
for sale are cash flows from operating activities. The cash receipts from rents and subsequent
sales of such assets are also cash flows from operating activities.
The amount of cash flows arising from operating activities is a key indicator of the extent to which
the operations of the entity have generated sufficient cash flows to function without recourse to
external sources of financing. In addition, it forms a basis for forecasting future operating cash
flows.

1.5.2 Cash flows from investing activities


The second section of a statement of cash flows shows cash flows from investing activities.
Investing activities are defined by IAS 7 as ‘the acquisition and disposal of long-term assets and
other investments not included in cash equivalents’. It generally refers to money made or spent
on long-term assets the company has purchased or sold. Investing transactions generate cash
outflows, such as capital expenditures for plant, property and equipment, business acquisitions
and the purchase of investment securities. Inflows come from the sale of assets, businesses and
investment securities. For investors, the most important item in this category is capital
expenditures, made to ensure the proper maintenance of, and additions to, a company's physical
assets to support its efficient operation and competitiveness.
Cash flows from investing activities might also include cash received from investments, such as
interest or dividends received.
The separate disclosure of cash flows arising from investing activities is important because the
cash flows represent the extent to which expenditures have been made for resources intended to
generate future income and cash flows.
Examples of cash flows arising from investing activities are:
 cash payments to acquire property, plant and equipment, intangibles and other long-term
assets (including those relating to capitalised development costs and self-constructed
tangible assets);
 cash receipts from sales of property, plant and equipment, intangibles and other long-term
assets;
 cash payments to acquire equity or debt instruments;
 cash receipts from sales of equity or debt instruments of other entities;

© Emile Woolf International 278 The Institute of Chartered Accountants of Pakistan


Chapter 8: IAS 7: Statement of cash flows

 cash advances and loans made to other parties (other than advances and loans made by a
financial institution which would be an operating activity);
 cash receipts from the repayment of advances and loans made to other parties (other than
advances and loans of a financial institution);

1.5.3 Cash flow from financing activities


The third section of the statement of cash flows shows the cash flows from financing activities.
These activities are defined by IAS 7 as ‘activities that result in changes in the size and composition
of the contributed equity and borrowings of the entity.’ It measures the flow of cash between a firm
and its owners and creditors. Companies often borrow money to fund their operations, acquire
another company or make other major purchases. Here again for investors, the most important
item is cash dividends paid.
Examples of cash flows arising from financing activities are:
 cash proceeds from issuing shares or other equity instruments;
 cash payments to owners to acquire or redeem the entity's shares;
 cash proceeds from issuing debentures, loans, notes, bonds, mortgages and other short-
term or long-term borrowings;
 cash repayments of amounts borrowed; and
 Cash payments by a lessee for the reduction of the outstanding liability relating to a finance
lease.
The separate disclosure of cash flows arising from financing activities is important because it is
useful in predicting claims on future cash flows by providers of capital to the entity.

© Emile Woolf International 279 The Institute of Chartered Accountants of Pakistan


Financial accounting and reporting I

2 CASH FLOWS FROM OPERATING ACTIVITIES: THE INDIRECT METHOD


Section overview

 Operating Cash flows


 Profit before taxation
 Non-cash items
 Accruals based figures
 Taxation
 Dividends
 Presentation of interest, taxation and dividends cash flows
 Working capital
 Changes in trade and other receivables
 Possible complication: Allowances for doubtful debts
 Changes in inventory
 Changes in trade payables
 Lack of detail

2.1 Operating cash flows


The operations of the business are probably the most significant source of cash.
IAS 7 allows two approaches to identifying the cash flows from operating activities:
 Indirect method; and
 Direct method
The indirect method identifies the cash flows from operating activities by adjusting the profit before
tax figure. It arrives at the cash from operating activities figure indirectly by reconciling a profit figure
to a cash figure. The adjustments remove the impact of accruals and non-cash items and also
relocate some figures to other positions in the statement of cash flows.
The following illustration shows the net cash flow from operating activities arrived.
Illustration 03:
Cash flows from operating activities
Profit before taxation 80,000
Adjustments for:
Depreciation and amortization charges 20,000
Interest charge in the statement of comprehensive income 2,300
Gains on disposal of non-current assets (6,000)
Losses on disposal of non-current assets 4,500
100,800
Increase/decrease in:
Increase in trade and other receivables (7,000)
Decrease in inventories 2,000
Increase in trade payables 3,000
Cash generated from operations 98,800
Taxation paid (tax on profits) (21,000)
Interest charges paid (2,500)
Net cash flow from operating activities 75,300

© Emile Woolf International 280 The Institute of Chartered Accountants of Pakistan


Chapter 8: IAS 7: Statement of cash flows

2.2 Profit before taxation


The starting point for the statement of cash flows for a company is the operating profit after
deducting interest but before taxation.
This profit figure is adjusted to calculate the amount of cash received by the business or the amount
of cash paid out as a consequence of its trading operations.
The adjustments are to remove the effect of:
 Non-cash items, for example:
 Depreciation and amortisation;
 Gain or loss on disposal of non-current assets;
 Doubtful debts;
 Provision for obsolete inventory; and
 Accruals based figures, for example:
 Interest expense or income;
 Movement on working capital items (receivables, payables and inventory).

2.3 Non-cash items


Depreciation and amortisation
Depreciation and amortisation charges are not cash flows. They are expenses in the statement of
comprehensive income, but do not represent payments of cash.
In order to obtain a figure for cash flow from the figure for profit, charges for depreciation and
amortisation must therefore be added back to the profit figure.
Gains or losses on disposal of non-current assets
Gains or losses on the disposal of non-current assets are not cash flows. The gain or loss is
calculated as the difference between:
 the net cash received from the disposal; and
 The carrying amount (net book value) of the asset at the date of disposal.
The effect of the gain/loss on disposal (a non-cash item) from the operating profit is removed by:
 deducting gain on disposal; and
 adding back losses on disposal.
The relevant cash flow is the net cash received from the sale. This is included in cash flows from
investing activities as the net cash flows received from the disposal of non-current assets.

Illustration 04:
A company disposed of an item of equipment for Rs. 40,000. The equipment had originally cost
Rs. 60,000 and the accumulated depreciation charged up to the date of disposal was Rs. 32,000.
Rs.
Cost 60,000
Accumulated depreciation (32,000)
Carrying value at date of disposal 28,000
Cash proceeds from sale (40,000)
Gain on disposal 12,000

In the statement of cash flows, the gain on disposal of Rs. 12,000 is deducted as an adjustment to
the operating profit. The cash proceeds of Rs. 40,000 are included as a cash inflow under the
heading: ‘Cash flows from investing activities’.

© Emile Woolf International 281 The Institute of Chartered Accountants of Pakistan


Financial accounting and reporting I

2.4 Accruals based figures


Interest
The accruals concept is applied in accounting.
Interest charge in the income statement is an accrual based figure. It is added back to profit and
the actual cash interest paid is deducted further down the cash flow statements.
The final items in the operating cash flows part of a statement of cash flows are the amount of
interest paid and the amount of tax paid (see later).
This figure must be calculated as follows:

Illustration 05:
Rs.
Interest liability at the beginning of the year X
Interest charge for the year (income statement figure) X
Total amount of interest payable in the year X
Interest liability at the end of the year (X)
Interest paid in the year (cash) X

The same approach is used to calculate other figures.


The interest liability at the start of the year and the interest charge during the year is the most the
business would pay. If the business had paid nothing it would owe this figure. The difference
between this amount and the liability at the end of the year must be the amount that the business
has paid.

Example 01:
Question: A company had liabilities in its statement of financial position at the beginning and at the
end of 2017, as follows:
Interest payable (Rs.)
Beginning of 2017 4,000
End of 2017 22,000

During the year, interest charges in the income statement were Rs. 22,000.
Required: Calculate the amount of interest paid during 2017.

Answer: The interest payment for inclusion in the statement of cash flows can be calculated as
follows:
Rs.
Liability at the start of the year 4,000
Charge for the year 22,000
Total amount payable in the year 26,000
Liability at the end of the year (22,000)
Cash paid 4,000

Note that this approach would work to find the cash paid in respect of any liability for which expense
was recognised in the statement of profit or loss.
It would not matter if you did not know anything about the type of liability as long as you are told
that there is a movement and you are given the amount recognised in the statement of profit or
loss. For example, instead of the above example being about interest it could be about warranty
provision, gratuity, retirement benefit, health insurance, bonus, and so on.

© Emile Woolf International 282 The Institute of Chartered Accountants of Pakistan


Chapter 8: IAS 7: Statement of cash flows

2.5 Taxation
The tax paid is the last figure in the operating cash flow calculation.
There is no adjustment to profit in respect of tax. This is because the profit figure that we start with
is profit before tax; therefore, tax is not included in it to be adjusted!
However, there is a tax payment and this must be recognised as a cash flow. It is calculated in the
same way as shown above.

Example 02:
Question: A company had liabilities in its statement of financial position at the beginning and at
the end of 2017, as follows:
Taxation (Rs.)
Beginning of 2017 53,000
End of 2017 61,000

During the year, taxation on profits was Rs. 77,000.

Required: Calculate the amount of tax paid during 2017.

Answer: The tax payment (cash flows) for inclusion in the statement of cash flows can be calculated
as follows:
Rs.
Taxation liability at the start of the year 53,000
Charge for the year 77,000
Total amount payable 130,000
Taxation liability at the end of the year (61,000)
Cash paid 69,000

2.6 Dividends
A question might require the calculation of cash paid out as dividends in the year.
This is calculated in the usual way remembering that the dividend charge is a debit in the statement
of changes in equity.

Illustration 06:
Rs.
Dividend liability at the beginning of the year X
Dividend charge for the year X
Total amount of dividend payable in the year X
Dividend liability at the end of the year (X)
Dividend paid in the year (cash) X

Pakistan
Typically, in Pakistan a company will pay a dividend once a year. Dividend payments in Pakistan
must be approved by the members in a general meeting and this usually takes place after the year
end. This means that the dividend expensed in any one year is the previous year’s dividend (which
could not be recognised last year as it had not yet been approved in the general meeting).

© Emile Woolf International 283 The Institute of Chartered Accountants of Pakistan


Financial accounting and reporting I

Listed companies often pay an interim dividend part way through a year and a final dividend after
the year end. The actual dividend payment recognised in any one year would then be that year’s
interim dividend and the previous year’s final dividend (which could not be recognised last year as
it had not yet been approved in the general meeting).
A question may tell you that a dividend was declared at just before or just after the year end but
the company is not allowed to recognise that dividend until it is approved. Last year’s figure is
needed.

Example 03:
Question: A company had liabilities in its statement of financial position at the beginning and at
the end of 2017, as follows:
Dividends (Rs.)
Beginning of 2017 65,000
End of 2017 71,000
The company had share capital of Rs. 1,000,000. The directors recommended a dividend of
20% (2016: 18%) on 25th December 2017. The company AGM is held in March each year.

Required: Calculate the amount of dividend paid during 2017.

Answer: The dividend payment (cash flows) for inclusion in the statement of cash flows can be
calculated as follows:
Rs.
Dividend liability at the start of the year 65,000
Dividend in the year (18% of 1,000,000) 180,000
Total amount payable 245,000
Dividend liability at the end of the year (71,000)
Cash paid 171,000

2.7 Presentation of interest, taxation and dividends cash flows


IAS 7 allows some variations in the way that cash flows for interest and dividends are presented
in a statement of cash flows, although the following should be shown separately:
 interest received
 dividends received
 interest paid
 Dividends paid.

Interest payments
IAS 7 states that there is no consensus about how to treat interest payments by an entity, other
than a financial institution such as a bank. Interest payments may be classified as either:
 an operating cash flow, because they are deducted when calculating operating profit before
taxation, or
 A financing cash flow, because they are costs of obtaining finance.
In examples of statements of cash flows in the appendix to IAS 7, interest paid is shown as a
separate line item within cash flows from operating activities. This approach is therefore used here.

© Emile Woolf International 284 The Institute of Chartered Accountants of Pakistan


Chapter 8: IAS 7: Statement of cash flows

Interest and dividends received


Interest and dividends received may be classified as either:
 an operating cash flow, because they are added when calculating operating profit before
taxation, or
 An investing cash flow, because they represent returns on investment.
In examples of statements of cash flows in the appendix to IAS 7, interest received and dividend
received are shown as separate items within cash flows from investing activities. This approach is
therefore used here.

Dividends paid
IAS 7 allows dividend payments to be treated as either:
 a financing cash flow because they are a cost of obtaining financial resources, or
 A component of the cash flows from operating activities, in order to assist users to determine
the ability of the entity to pay dividends out of its operating cash flows.
In examples of statements of cash flows in the appendix to IAS 7, dividends paid are shown as a
line item within cash flows from financing activities. This approach is therefore used here.

Taxes on profits
Cash flows arising from taxation on income should normally be classified as a cash flow from
operating activities (unless the tax payments or refunds can be specifically associated with an
investing or financing activity).
The examples of statements of cash flows in this chapter therefore show both interest paid and tax
paid as cash flow items, to get from the figure for cash generated from operations to the figure for
‘net cash from operating activities’.

2.8 Working capital

Definition
Working capital is current assets less current liabilities.

The previous section showed that taxation and interest cash flows can be calculated by using a
figure from the statement of comprehensive income and adjusting it by the movement on the
equivalent balances in the statement of financial position.
This section shows how this approach is extended to identify the cash generated from operations
by making adjustments for the movements between the start and end of the year for:
 trade receivables and prepayments;
 inventories; and
 Trade payables and accruals.
Assuming that the calculation of the cash flow from operating activities starts with a profit (rather
than a loss) the adjustments are as follows:

Increase in balance from start to Decrease in balance from start to the


Balance
the end of the year end of the year
Receivables Subtract from profit before tax Add back to profit before tax
Inventory Subtract from profit before tax Add back to profit before tax
Payables Add back to profit before tax Subtract from profit before tax

These are known as the working capital adjustments and are explained in more detail in the rest
of this section.

© Emile Woolf International 285 The Institute of Chartered Accountants of Pakistan


Financial accounting and reporting I

Working capital is made up of the following balances:

Illustration 07:
Rs.
Inventory X
Trade and other receivables X
Cash X
Trade payables (X)
Working capital X

Trade and other receivables include any prepayments.


Trade payables include accrued expenses, provided the accrued expenses do not relate to other
items dealt with separately in the statement of cash flows, in particular:
 accrued interest charges; and
 Taxation payable.
Interest charges and payments for interest are presented separately in the statement of cash flows,
and so accrued interest charges should be excluded from the calculation of changes in trade
payables and accruals.
Similarly, taxation payable is dealt with separately; therefore, taxation payable is excluded from the
calculation of working capital changes.
Accrued interest and accrued tax payable must therefore be deducted from the total amount for
accruals, and the net accruals (after making these deductions) should be included with trade
payables.

Changes in working capital and the effect on cash flow


When working capital increases, the cash flows from operations are less than the operating profit,
by the amount of the increase.
Similarly, when working capital is reduced, the cash flows from operations are more than the
operating profit, by the amount of the reduction.
This important point will be explained with several simple illustrations and examples.

2.9 Changes in trade and other receivables


Sales revenue in a period differs from the amount of cash received from sales by the amount of
the increase or decrease in receivables during the period.
When trade and other receivables go up during the year, cash flows from operations are less than
operating profit by the amount of the increase.
When trade and other receivables go down during the year, cash flows from operations are more
than operating profit by the amount of the reduction.
In a statement of cash flows presented using the indirect method, the adjustment for receivables is
therefore:
 subtract the increase in receivables during the period (the amount by which closing
receivables exceed opening receivables); or
 Add the reduction in receivables during the period (the amount by which opening receivables
exceed closing receivables).
Prepayments in the opening and closing statement of financial position should be included in the
total amount of receivables and therefore does not show separately.

© Emile Woolf International 286 The Institute of Chartered Accountants of Pakistan


Chapter 8: IAS 7: Statement of cash flows

Illustration 08: A company had receivables at the beginning of the year of Rs. 6,000 and at the end
of the year receivables were Rs. 9,000.
During the year, sales were Rs. 50,000 in total. Purchases were Rs. 30,000, all paid in cash.
The company holds no inventories. The profit before tax for the year was Rs. 20,000 (Rs. 50,000 –
Rs. 30,000).
The cash flow from operations is calculated as follows:
Rs.
Profit before tax 20,000
Adjustments for:
Increase in receivables (9,000 – 6,000) (3,000)
17,000

Illustration 09:
Proof
Cash flow from operations can be calculated as follows:
Rs.
Receivables at the beginning of the year 6,000
Sales in the year 50,000
56,000
Receivables at end of the year (9,000)
Cash received 47,000
Cash paid (purchases) (30,000)
Cash flow from operations 17,000

2.10 Possible complication: Allowances for doubtful debts


A question might provide information on the allowance for doubtful debts at the start and end of
the year.
There are two ways of dealing with this:
 Adjust the profit for the movement on the allowance as a non-cash item and adjust the profit
figure for the movement in receivables using the gross amounts (i.e. the balances before
any deduction of the allowance for doubtful debts); or
 Make no adjustments for the movement on receivables as a non-cash item adjust the profit
figure for the movement in receivables using the net amounts (i.e. the balances after the
deduction of the allowance for doubtful debts).

Illustration 10: The following information is available:

2016 2017
(Rs. m) (Rs. m)
Receivables 5,000 7,100
Allowance for doubtful debts (500) (600)
Net-amount 4,500 6,500

© Emile Woolf International 287 The Institute of Chartered Accountants of Pakistan


Financial accounting and reporting I

Rs. m or Rs. m
Profit before taxation 10,000 10,000
Adjustments for non- cash items:
Increase in allowance for doubtful debts 100 
10,100 10,000
Increase in receivables:
Gross amounts: (7,100  5,000) (2,100)
Net amounts: (6,500  4,500) (2,000)
8,000 8,000

2.11 Changes in inventory


Purchases in a period differ from the cost of sales by the amount of the increase or decrease in
inventories during the period.
If all purchases were paid for in cash, this means that cash payments and the cost of sales (and
profit) would differ by the amount of the increase or decrease in inventories.
When the value of inventory goes up between the beginning and end of the year, cash flows from
operations are less than operating profit by the amount of the increase.
When the value of inventory goes down between the beginning and end of the year, cash flows
from operations are more than operating profit by the amount of the reduction.
In a statement of cash flows presented using the indirect method, the adjustment for inventories
is therefore:
 subtract the increase in inventories during the period (the amount by which closing inventory
exceeds opening inventory); or
 Add the reduction in inventories during the period (the amount by which opening inventory
exceeds closing inventory).

Illustration 11:
A company had inventory at the beginning of the year of Rs. 5,000 and at the end of the year the
inventory was valued at Rs. 3,000.
During the year, sales were Rs. 50,000 and there were no receivables at the beginning or end of
the year.
Purchases were Rs. 28,000, all paid in cash.
The operating profit for the year was Rs. 20,000, calculated as follows:
Rs.
Sales 50,000
Opening inventory 5,000
Purchases in the year (all paid in cash) 28,000
33,000
Closing inventory (3,000)
Cost of sales (30,000)
Profit before tax 20,000

© Emile Woolf International 288 The Institute of Chartered Accountants of Pakistan


Chapter 8: IAS 7: Statement of cash flows

Rs.
Profit before tax 20,000
Adjustments for:
decrease in inventory (5,000 – 3,000) 2,000
22,000

Proof: The cash flow from operations is calculated as follows:


Rs.
Cash from sales in the year 50,000
Purchases paid in cash (28,000)
Cash flow from operations 22,000

2.12 Changes in trade payables


Payments for purchases in a period differ from purchases by the amount of increase or decrease
in trade payables during the period.
When trade payables go up between the beginning and end of the year, cash flows from
operations are more than operating profit by the amount of the increase.
When trade payables go down between the beginning and end of the year, cash flows from
operations are less than operating profit by the amount of the reduction.
In a statement of cash flows presented using the indirect method, the adjustment for trade
payables is therefore:
 add the increase in trade payables during the period (the amount by which closing trade
payables exceed opening trade payables); or
 Subtract the reduction in trade payables during the period (the amount by which opening
trade payables exceed closing trade payables).
Accruals in the opening and closing statement of financial position should be included in the total
amount of trade payables.
However, deduct interest payable and tax payable from opening and closing payables, if the total
for payables includes these items.

Illustration 12:
A company had no inventory and no receivables at the beginning and end of the year. All its sales
are for cash, and sales in the year were Rs. 50,000.
Its purchases are all on credit. During the year, its purchases were Rs. 30,000.
Trade payables at the beginning of the year were Rs. 4,000 and trade payables at the end of the
year were Rs. 6,500.
The operating profit for the year was Rs. 20,000 (Rs. 50,000 – Rs. 30,000)

Rs.
Profit before tax 20,000
Adjustments for:
Increase in payables (6,500 – 4,000) 2,500
22,500

© Emile Woolf International 289 The Institute of Chartered Accountants of Pakistan


Financial accounting and reporting I

Proof: The cash flow from operations is calculated as follows:


Rs.
Trade payables at the beginning of the year 4,000
Purchases in the year 30,000
34,000
Trade payables at the end of the year (6,500)
Cash paid to suppliers 27,500
Cash from sales (50,000)
Cash flow from operations 22,500

The cash flow is Rs. 2,500 more than the operating profit, because trade payables were increased
during the year by Rs. 2,500.

Illustration 13:
A company made an operating profit before tax of Rs. 16,000 in the year just ended.
Depreciation charges were Rs. 15,000.
There was a gain of Rs. 5,000 on disposals of non-current assets and there were no interest charges.
Values of working capital items at the beginning and end of the year were:

Receivables Inventory Trade payables

Beginning of the year Rs. 9,000 Rs. 3,000 Rs. 4,000

End of the year Rs. 6,000 Rs. 5,000 Rs. 6,500

Taxation paid was Rs. 4,800.


The calculation is as under:

Rs. Rs.

Cash flows from operating activities

Profit before taxation 16,000

Adjustments for:

Depreciation and amortisation charges 15,000

Gains on disposal of non-current assets (5,000)

26,000

Decrease in trade and other receivables 3,000

Increase in inventories (2,000)

Increase in trade payables 2,500

Cash generated from operations 29,500

Taxation paid (tax on profits) (4,800)

Net cash flow from operating activities 24,700

© Emile Woolf International 290 The Institute of Chartered Accountants of Pakistan


Chapter 8: IAS 7: Statement of cash flows

2.13 Lack of detail


A question might not provide all the detail needed to split out working capital into all of its
component parts. If this is the case the adjustment must be made using whatever totals are
available in the question.

Illustration 14:
A company made an operating profit before tax of Rs. 16,000 in the year just ended.
Depreciation charges were Rs. 15,000.
There was a gain of Rs. 5,000 on disposals of non-current assets and there were no interest charges.
Values of working capital items at the beginning and end of the year were:
Current assets Trade payables
Beginning of the year Rs. 12,000 Rs. 4,000
End of the year Rs. 11,000 Rs. 6,500
Taxation paid was Rs. 4,800.
The calculation is as under:
Rs. Rs.
Cash flows from operating activities
Profit before taxation 16,000
Adjustments for:
Depreciation and amortisation charges 15,000
Gains on disposal of non-current assets (5,000)
26,000
Decrease in current assets 1,000
Increase in trade payables 2,500
Cash generated from operations 29,500
Taxation paid (tax on profits) (4,800)
Net cash flow from operating activities 24,700

Example 04:
Question: The following information has been extracted from the financial statements of Hopper
Company for the year ended 31 December 2017.
Rs.
Sales 1,280,000
Cost of sales (400,000)
Gross profit 880,000
Wages and salaries (290,000)
Other expenses (including depreciation Rs. 25,000) (350,000)
240,000
Interest charges (50,000)
Profit before tax 190,000
Taxation (40,000)
Profit after tax 150,000

© Emile Woolf International 291 The Institute of Chartered Accountants of Pakistan


Financial accounting and reporting I

Extracts from the statement of financial position:

At 1 At 31
January December
2017 2017
Rs. Rs.
Trade receivables 233,000 219,000
Inventory 118,000 124,000
Trade payables 102,000 125,000
Accrued wages and salaries 8,000 5,000
Accrued interest charges 30,000 45,000
Tax payable 52,000 43,000

Required: Using the above information, prepare cash flows from operating activities section of
statement of cash flows for Hopper Limited using indirect method.
Answer: The cash flows from operating activities using the indirect method is as under:
Statement of cash flows Rs.
Cash flows from operating activities
Profit before taxation 190,000
Adjustments for:
Depreciation charges 25,000
Interest expense 50,000
265,000
Decrease in trade receivables (233,000 – 219,000) 14,000
Increase in inventories (124,000 – 118,000) (6,000)
Increase in trade and other payables 20,000
(125,000 + 5,000) – (102,000 + 8,000)
Cash generated from operations 293,000
Taxation paid (W1) (49,000)
Interest paid (W1) (35,000)
Net cash flow from operating activities 209,000
Workings
(W1) Interest and tax payments Tax Interest
Rs. Rs.
Liability at the beginning of the year 52,000 30,000
Taxation charge/interest charge for the year 40,000 50,000
92,000 80,000
Liability at the end of the year (43,000) (45,000)
Tax paid/interest paid during the year 49,000 35,000

© Emile Woolf International 292 The Institute of Chartered Accountants of Pakistan


Chapter 8: IAS 7: Statement of cash flows

3 CASH FLOWS FROM OPERATING ACTIVITIES: THE DIRECT METHOD


Section overview

 Cash from sales


 Cash paid for materials
 Cash paid for wages and salaries
 Cash paid for other expenses

3.1 Cash from sales


The format for the direct method of presenting a statement of cash flows is as follows:

Illustration 15:
Statement of cash flows: direct method Rs.
Cash flows from operating activities
Cash receipts from customers 348,800
Cash payments to suppliers (70,000)
Cash payments to employees (150,000)
Cash paid for other operating expenses (30,000)
Cash generated from operations 98,800
Taxation paid (tax on profits) (21,000)
Interest charges paid (2,500)
Net cash flow from operating activities 75,300

The task is therefore to establish the amounts for cash receipts and cash payments. In an
examination, you might be expected to calculate any of these cash flows from figures in the opening
and closing statements of financial position, and the statement of profit or loss.
The cash receipts from sales during a financial period can be calculated as follows:

Illustration 16:

Rs.
Trade receivables at the beginning of the year X
Sales in the year X
X
Trade receivables at the end of the year (X)
Cash from sales during the year X

A T account could also be used to calculate the cash receipt


Receivables
Balance b/f X
Sales X Cash (balancing figure) X
Balance c/f X
X X

© Emile Woolf International 293 The Institute of Chartered Accountants of Pakistan


Financial accounting and reporting I

3.2 Cash paid for materials


To calculate the amount of cash paid to suppliers, you might need to calculate first the amount of
material purchases during the period.

Illustration 17: Calculation of purchases in the year


Rs.
Closing inventory at the end of the year X
Cost of sales X
X
Opening inventory at the beginning of the year (X)
Purchases in the year X

Having calculated purchases from the cost of sales, the amount of cash payments for purchases
may be calculated from purchases and opening and closing trade payables.

Illustration 18:
Rs.
Trade payables at the beginning of the year X
Purchases in the year (as above) X
X
Trade payables at the end of the year (X)
Cash paid for materials X

A T account could also be used to calculate the cash paid


Payables
Balance b/f X
Cash (balancing figure) X Purchases X
Balance c/f X
X X

Note that if the business had paid for goods in advance at the start or end of the year they would
have an opening or closing receivable but this situation would be quite unusual.

3.3 Cash paid for wages and salaries


Cash payments for wages and salaries can be calculated in a similar way.
Illustration 19:
Rs.
Accrued wages and salaries at the beginning of the year X
Wages and salaries expenses in the year X
X
Accrued wages and salaries at the end of the year (X)
Cash paid for wages and salaries X
A T account could also be used to calculate the cash paid
Payables
Balance b/f X
Cash (balancing figure) X Purchases X
Balance c/f X
X X

© Emile Woolf International 294 The Institute of Chartered Accountants of Pakistan


Chapter 8: IAS 7: Statement of cash flows

If wages and salaries had been paid in advance the business would have a receivable and the
workings would change to the following.

Illustration 20:

Rs.

Wages and salaries paid in advance at the beginning of the year (X)

Wages and salaries expenses in the year X

Wages and salaries paid in advance at the end of the year X

Cash paid for wages and salaries X

A T account could also be used to calculate the cash paid

Payables

Balance b/f X

Cash (balancing figure) X Purchases X

Balance c/f X

X X

3.4 Cash paid for other expenses


Other expenses in the statement of profit or loss usually include depreciation charges, which are
not cash flows. Depreciation charges should therefore be excluded from other expenses when
calculating cash payments.
Cash payments for other expenses can be calculated as follows.

Illustration 21:

Rs.

Payables for other expenses at the beginning of the year X

Other expenses in the year, excluding depreciation and amortisation X

Payables for other expenses at the end of the year (X)

Cash paid for other expenses X

Payables for other expenses should exclude accrued wages and salaries, accrued interest
charges and taxation payable.

© Emile Woolf International 295 The Institute of Chartered Accountants of Pakistan


Financial accounting and reporting I

Example 05:
Question: The following information has been extracted from the financial statements of Hopper
Company for the year ended 31 December 2015.
Rs.
Sales 1,280,000
Cost of sales (400,000)
Gross profit 880,000
Wages and salaries (290,000)
Other expenses (including depreciation Rs. 25,000) (350,000)
240,000
Interest charges (50,000)
Profit before tax 190,000
Tax on profit (40,000)
Profit after tax 150,000
Extracts from the statement of financial position:
At 1 January At 31 December
2015 2015
Rs. Rs.
Trade receivables 233,000 219,000
Inventory 118,000 124,000
Trade payables 102,000 125,000
Accrued wages and salaries 8,000 5,000
Accrued interest charges 30,000 45,000
Tax payable 52,000 43,000

Required: Using the above information, prepare cash flows from operating activities section of
statement of cash flows for Hopper Limited using direct method.

Answer: The calculation is as under:

Direct method
Statement of cash flows: direct method Rs.
Cash flows from operating activities
Cash receipts from customers(W1) 1,294,000
Cash payments to suppliers(W3) (383,000)
Cash payments to employees(W4) (293,000)
Cash paid for other operating expenses (325,000)
Cash generated from operations 293,000
Taxation paid (tax on profits)(W5) (49,000)
Interest charges paid(W5) (35,000)
Net cash flow from operating activities 209,000
.

© Emile Woolf International 296 The Institute of Chartered Accountants of Pakistan


Chapter 8: IAS 7: Statement of cash flows

Workings
(W1) Cash from sales Rs.
Trade receivables at 1 January 2015 233,000
Sales in the year 1,280,000
1,513,000
Trade receivables at 31 December 2015 (219,000)
Cash from sales during the year 1,294,000

(W2) Purchases Rs.


Closing inventory at 31 December 2015 124,000
Cost of sales 400,000
524,000
Opening inventory at 1 January 2015 (118,000)
Purchases in the year 406,000

(W3) Cash paid for materials supplies Rs.


Trade payables at 1 January 2015 102,000
Purchases in the year (W2) 406,000
508,000
Trade payables at 31 December 2015 (125,000)
Cash paid for materials 383,000

(W4) Cash paid for wages and salaries Rs.


Accrued wages and salaries at 1 January 2015 8,000
Wages and salaries expenses in the year 290,000
298,000
Accrued wages and salaries at 31 December 2015 (5,000)
Cash paid for wages and salaries 293,000

(W5) Interest and tax payments Tax Interest


Rs. Rs.
Liability at the beginning of the year 52,000 30,000
Taxation charge/interest charge for the year 40,000 50,000
92,000 80,000
Liability at the end of the year (43,000) (45,000)
Tax paid/interest paid during the year 49,000 35,000

© Emile Woolf International 297 The Institute of Chartered Accountants of Pakistan


Financial accounting and reporting I

4 CASH FLOWS FROM INVESTING ACTIVITIES


Section overview

 Cash paid for the purchase of property, plant and equipment


 Cash from disposals of property, plant and equipment
 Cash paid for the purchase of investments and cash received from the sale of investments
 Non-cash purchases

4.1 Cash paid for the purchase of property, plant and equipment
This is the second part of a statement of cash flows, after cash flows from operating activities.
The most important items in this part of the statement are cash paid to purchase non-current assets
and cash received from the sale or disposal of non-current assets but it also includes interest
received and dividends received on investments.
It is useful to remember the following relationship:

Illustration 22: Movement on non-current assets


Rs.
Carrying amount at the start of the year X
Depreciation (X)
Disposals (X)
Additions X
Revaluation X/(X)
Carrying amount at the end of the year X

When there are no disposals or revaluations during the year


When there are no disposals or revaluations of non-current assets during the year, purchases of
non-current assets (normally assumed to be the amount of cash paid for these purchases) may be
calculated as follows:

Illustration 23:
Using cost: Rs.
Non-current assets at the end of the year at cost X
Non-current assets at the beginning of the year at cost (X)
Additions to non-current assets X

Alternatively carrying amount (NBV) can be used Rs.


Non-current assets at the end of the year at NBV X
Non-current assets at the beginning of the year at NBV (X)
X
Depreciation X
Additions X

© Emile Woolf International 298 The Institute of Chartered Accountants of Pakistan


Chapter 8: IAS 7: Statement of cash flows

Illustration 24: The plant and equipment of PM Company at the beginning and the end of its
financial year were as follows:

At cost Accumulated depreciation Net book value


Rs. Rs. Rs.
Beginning of the year 180,000 (30,000) 150,000
End of the year 240,000 (50,000) 190,000
There were no disposals of plant and equipment during the year.
The cash paid for plant and equipment in the year (additions) may be calculated in either of the
following ways.
Rs. Rs.
At cost at the end of the year Carrying amount (NBV) at the
240,000 end of the year 190,000
At cost at the beginning of the Carrying amount (NBV at the
year 180,000 beginning of the year 150,000

Additions 60,000 Increase in NBV 40,000

Depreciation charge for the year


(50,000 – 30,000) 20,000

Additions 60,000

Note that in the above illustration it is assumed that the purchases have been made for cash.
This might not be the case. If the purchases are on credit the figure must be adjusted for any
amounts outstanding at the year end.

Illustration 25: PM company has purchased various items of property, plant and equipment on
credit during the year. The total purchased was Rs. 60,000.
The statements of financial position of PM company at the beginning and end of 2017 include the
following information:

2016 2017
(Rs. m) (Rs. m)

Payables:

Suppliers of non-current assets 4,000 12,000

The cash paid to buy property, plant and equipment in the year can be calculated as follows:

Rs. m

Additions 60,000

Less: increase in payables that relate to these items (8,000)

Cash paid in the year 52,000

This can be thought of as the payment of the Rs. 4,000 owed at the start and a payment of Rs.
48,000 towards this year’s purchases.

If the payables had decreased the movement would be added to the additions figure to find the
cash outflow.

© Emile Woolf International 299 The Institute of Chartered Accountants of Pakistan


Financial accounting and reporting I

Illustration 26: PM company has purchased various items of property, plant and equipment on
credit during the year. The total purchased was Rs. 60,000.
The statements of financial position of PM company at the beginning and end of 2017 include the
following information:

2016 2017
(Rs. m) (Rs. m)

Payables:
Suppliers of non-current assets 14,000 4,000

The cash paid to buy property, plant and equipment in the year can be calculated as follows:
Rs. m
Additions 60,000
Less: increase in payables that relate to
these items 10,000
Cash paid in the year 70,000

This can be thought of as the payment of the Rs. 14,000 owed at the start and a payment of
Rs. 56,000 towards this year’s purchases.

When there are disposals during the year


When there are disposals of non-current assets during the year, the purchases of non-current
assets may be calculated as follows:

Illustration 27: Movement on non-current assets

Rs.
Assets at cost at the beginning of the year X
Disposals during the year (cost) (X)

X
Additions to non-current assets (balancing figure) X

Assets at cost at the end of the year X

Alternatively carrying amount (NBV) can be used Rs.


Non-current assets at the beginning of the year at NBV X
Depreciation (X)
Disposals during the year (NBV) (X)

X
Additions to non-current assets (balancing figure) X

Non-current assets at the end of the year at NBV (X)

© Emile Woolf International 300 The Institute of Chartered Accountants of Pakistan


Chapter 8: IAS 7: Statement of cash flows

Example 06:
Question: The motor vehicles of PM Company at the beginning and the end of its financial year
were as follows:
Accumulated Carrying
At cost depreciation amount
Rs. Rs. Rs.
Beginning of the year 150,000 (105,000) 45,000
End of the year 180,000 (88,000) 92,000
During the year a vehicle was disposed of for a gain of Rs. 3,000. The original cost of this asset was
Rs. 60,000. Accumulated depreciation on the asset was Rs. 45,000.

Required: Calculate the cash paid for acquisition of motor vehicles.


Answer:
Cost NBV
Balance at the start of the year 150,000 45,000
Disposals during the year:
At cost (60,000)
At carrying amount: (60,000 – 45,000) (15,000)
Depreciation (88,000 – (105,000 – 45,000) (28,000)
90,000 2,000
Additions (balancing figure) 90,000 90,000
Balance at the end of the year 180,000 92,000

Rs.
Assets at cost at the end of the year 180,000
Assets at cost at the beginning of the year 150,000
30,000
Disposals during the year: original asset cost 60,000
Purchases 90,000

Alternatively using carrying amount (NBV): Rs.


Assets at carrying amount (NBV) at the end of the year 92,000
Assets at carrying amount (NBV) at the beginning of the year 45,000
47,000
Disposals during the year (carrying amount): (60,000 – 45,000) 15,000
Depreciation (88,000 – (105,000 – 45,000) 28,000
Purchases 90,000

When there are revaluations during the year


When there are revaluations of non-current assets during the year, the purchases of non-current
assets should be calculated as follows.

© Emile Woolf International 301 The Institute of Chartered Accountants of Pakistan


Financial accounting and reporting I

Illustration 28: Movement on non-current assets


Rs.
At cost or valuation, at the beginning of the year X
Disposals during the year (cost) (X)
Upward/(downward) revaluation during the year X/(X)
X
Additions to non-current assets (balancing figure) X
At cost or valuation, at the end of the year X

Alternatively carrying amount (NBV) can be used Rs.


Non-current assets at the beginning of the year at NBV X
Depreciation (X)
Disposals during the year (NBV) (X)
Upward/(downward) revaluation during the year X/(X)
X
Additions to non-current assets (balancing figure) X
Non-current assets at the end of the year at NBV (X)

Example 07:
Question: The statements of financial position of Grand Company at the beginning and end of 2017
include the following information:

Property, plant and equipment 2016 2017


Rs. Rs.
At cost/re-valued amount 1,400,000 1,900,000
Accumulated depreciation 350,000 375,000
Carrying value 1,050,000 1,525,000

During the year, some property was re-valued upwards by Rs. 200,000. An item of equipment was
disposed of during the year at a profit of Rs. 25,000. This equipment had an original cost of Rs.
260,000 and accumulated depreciation of Rs. 240,000 at the date of disposal.
Depreciation charged in the year was Rs. 265,000.
Required: Calculate the cash paid for acquisition of property, plant and equipment.
Answer:

Rs.
At cost/re-valued amount, at the end of the year 1,900,000
At cost/re-valued amount, at the beginning of the year 1,400,000
500,000
Add: Cost of assets disposed of in the year 260,000
Subtract: Asset revaluation during the year (200,000)
Purchases during the year 560,000

© Emile Woolf International 302 The Institute of Chartered Accountants of Pakistan


Chapter 8: IAS 7: Statement of cash flows

Alternatively using carrying amount (NBV): Rs.


Assets at carrying amount (NBV) at the end of the year 1,525,000
Assets at carrying amount (NBV) at the beginning of the year 1,050,000
475,000
Revaluation during the year (200,000)
Carrying amount of assets disposed of in the year
(260,000 – 240,000) 20,000
Depreciation charged during the year 265,000
Purchases during the year 560,000

When there are other additions during the year


The above example showed the need to take revaluation into account when reconciling the opening
and closing balances on non-current assets so as to find the additions figure as a balancing
amount.
This applies to other additions too:
 Transfers from capital work in progress
 These are assets constructed by a company for its own use.
 During the course of construction costs are accumulated in a capital work in progress
account and these are transferred into the relevant category of non-current asset on
completion.
 The cash consequence of capital work in progress is estimated as a separate
exercise.
 Transfers into the relevant category of non-current asset on completion show as an
addition and so must be taken into account when trying to estimate the cash additions.
 Assets acquired under finance leases.
 A finance lease is capitalised on the statement of financial position as an assets and
as a liability.
 The asset side of the entry will show as an addition into non-current assets and so
must be taken into account when trying to estimate the cash additions.
 The liability is a form of loan. Movements on the liability represent new amounts
borrowed (additions to non-current assets) and repayments of capital.

Example 08:
Question: The statements of financial position of Grand Company at the beginning and end of 2017
include the following information:
Property, plant and equipment 2016 2017
Rs. Rs.
At cost/re-valued amount 1,400,000 1,900,000
Accumulated depreciation 350,000 375,000
Carrying value 1,050,000 1,525,000

Capital work in progress 600,000 620,000

© Emile Woolf International 303 The Institute of Chartered Accountants of Pakistan


Financial accounting and reporting I

During the year:


Property was revalued upwards by Rs. 200,000.
An item of equipment was disposed of at a profit of Rs. 25,000. This equipment had an original
cost of Rs. 260,000 and accumulated depreciation of Rs. 240,000 at the date of disposal.
Depreciation charged in the year was Rs. 265,000.
The company capitalised Rs. 200,000 as capital work in progress.
Required: Calculate the cash paid for acquisition (additions) of property, plant and equipment.
Answer:

Cost NBV
Balance at the start of the year 1,400,000 1,050,000
Disposals during the year:
At cost (260,000)
At carrying amount: (260,000 – 240,000) (20,000)
Depreciation (265,000)
Revaluation 200,000 200,000
Additions – Transfer from capital WIP
(200,000 – (620,000 – 600,000)) 180,000 180,000

1,520,000 1,145,000
Additions (balancing figure) 380,000 380,000

Balance at the end of the year 1,900,000 1,525,000

4.2 Cash from disposals of property, plant and equipment


A statement of cash flows should include the net cash received from any disposals of non-current
assets during the period.
This might have to be calculated from the gain or loss on disposal and the carrying amount of the
asset at the time of its disposal.

Illustration 29: Disposal of property, plant and equipment

Rs.

At cost (or revalued amount at the time of disposal) X

Accumulated depreciation, at the time of disposal (X)

Net book value/carrying amount at the time of disposal X

Gain or (loss) on disposal X

Net disposal value (= assumed cash flow) X

If there is a gain on disposal, the net cash from the disposal is more than the net book value.
If there is a loss on disposal the net cash from the disposal is less than the net book value.

© Emile Woolf International 304 The Institute of Chartered Accountants of Pakistan


Chapter 8: IAS 7: Statement of cash flows

Illustration 30:
During an accounting period, an entity disposed of some equipment and made a gain on disposal
of Rs. 6,000.
The equipment originally cost Rs. 70,000 and at the time of its disposal, the accumulated
depreciation on the equipment was Rs. 56,000.
What was the amount of cash obtained from the disposal of the asset?
Disposal of equipment Rs.

At cost 70,000

Accumulated depreciation, at the time of disposal (56,000)

Net book value/carrying amount at the time of disposal 14,000

Gain on disposal 6,000

Net disposal value (assumed cash flow) 20,000

This cash flow would be included in the cash flows from investing activities.

Note that in the above illustration it is assumed that the cash received for the disposal has been
received. This might not be the case. If the disposal was on credit the figure must be adjusted for
any amounts outstanding at the year end.

4.3 Cash paid for the purchase of investments and cash received from the sale of
investments
A statement of cash flows should include the net cash paid to buy investments in the period and
the cash received from the sale of investment in the period.
It is useful to remember the following relationship:

Illustration 31: Movement on investments

Rs.

Carrying amount at the beginning of the year X

Disposals (X)

Additions X

Revaluation X/(X)

Carrying amount at the end of the year X

The issues to be considered in calculating cash paid for investments or cash received on the sale
of investments are very similar to those for the purchase and sale of property, plant and equipment
except for the absence of depreciation.

Illustration 32:
The statements of financial position of Grand Company at the beginning and end of 2017 include
the following information:
2016 2017
(Rs. m) (Rs. m)
Non-current asset investments 1,000 1,500

© Emile Woolf International 305 The Institute of Chartered Accountants of Pakistan


Financial accounting and reporting I

Additional information:
The investments were revalued upwards during the year. A revaluation gain of Rs. 150m has been
recognised.
Investments sold for Rs. 250m resulted in a profit on the sale (measured as the difference
between sale proceeds and carrying amount at the date of sale) of Rs. 50m
The cash paid to buy investments in the period can be calculated as a balancing figure as follows:

Rs. m
Investments at the start of the year (given) 1,000
Disposal (carrying amount of investments sold = Rs. 250m – Rs. 50m) (200)
Revaluation gains (given) 150
950
Additions (as balancing figure): 550
Investments at the end of the year (given) 1,500

4.4 Non-cash purchases


IAS 7 states that investing and financing transactions that do not require the use of cash must be
excluded from the statement of cash flows, but that details of these transactions should be
disclosed somewhere in the financial statements, possibly as a note to the financial statements.
An example of a non-cash transaction is the acquisition of non-current assets under a finance lease
arrangement. The assets are included in the financial statements at cost, but the lessee has not
paid the purchase price.
IAS 7 therefore suggests that there should be a disclosure, in a note to the financial statements, of
the total amount of property, plant and equipment acquired during the period, and the cash
payments that were made to acquire them. These two amounts are different, because some of the
non-current assets might have been acquired under finance lease arrangements.

Illustration 33:
A note to the financial statements is as follows.
During the period, the company acquired property, plant and equipment with an aggregate cost of
Rs. 250,000, of which Rs. 60,000 was acquired by means of leases. Cash payments of Rs. 190,000
were made to purchase property, plant and equipment.
In this illustration, Rs. 190,000 would appear as a cash outflow in the statement of cash flows in
the section for cash flows from investing activities for the period.
 The Rs. 190,000 is the amount of cash actually paid for purchases of property, plant and
equipment in the period.
 The cash payments under the terms of the leases are not included in this part of the
statement of cash flows.

© Emile Woolf International 306 The Institute of Chartered Accountants of Pakistan


Chapter 8: IAS 7: Statement of cash flows

5 CASH FLOWS FROM FINANCING ACTIVITIES


Section overview

 Examples of cash flows from financing activities


 Cash from new share issues
 Cash from new loans/cash used to repay loans
 Dividend payments to equity shareholders
 Financing of a sole proprietor or a partnership

5.1 Examples of cash flows from financing activities


Examples of cash flows from financing activities are listed below:

Cash payments Cash receipts


Cash payments to redeem/buy back shares Cash proceeds from issuing shares
Cash payments to repay a loan or redeem Cash proceeds from a loan or issue of
bonds bonds

As explained earlier, payments of dividends are also usually included within cash flows from
financing activities, in this part of the statement of cash flows. (Some entities may also include
interest payments in this section, instead of including them in the section for cash flows from
operating activities.)

5.2 Cash from new share issues


The cash raised from new share issues can be established by comparing the equity share capital
and the share premium in the statements of financial position at the beginning and the end of the
year.

Illustration 34:
Rs.
Share capital + Share premium at the end of the year X
Share capital + Share premium at the beginning of the year X
Cash obtained from issuing new shares in the year X

Illustration 35: The statements of financial position of Company P at 1 January and 31 December
included the following items:
1 January 31 December
2017 2017
Rs. Rs.
Equity shares of Rs. 1 each 600,000 750,000
Share premium 800,000 1,100,000
The cash obtained from issuing shares during the year is calculated as follows.
Rs.
Share capital + Share premium at the end of 2017 1,850,000
Share capital + Share premium at the beginning of 2017 1,400,000
= Cash obtained from issuing new shares in 2017 450,000

© Emile Woolf International 307 The Institute of Chartered Accountants of Pakistan


Financial accounting and reporting I

5.3 Cash from new loans/cash used to repay loans


Cash from new loans or cash paid to redeem loans in the year can be calculated simply by looking
at the difference between the liabilities for loans and bonds at the beginning and the end of the
year.
 An increase in loans or bonds means there has been an inflow of cash during the year.
 A reduction in loans or bonds means there has been a payment (outflow) of cash.
Remember to add any loans, loan notes or bonds repayable within one year (current liability) to
the loans, loan notes or bonds repayable after more than one year (non-current liability) to get the
total figure for loans, loan notes or bonds.
Illustration 36:
Rs.
Loans at end of year (current and non-current liabilities) X
Loans at beginning of year (current and non-current liabilities) (X)
Cash inflow or outflow X/(X)

Note: The same calculation can be applied to bonds or loan notes that the company might have
issued. Bonds and loan notes are long-term debt.

Illustration 37: The statements of financial position of Company Q at 1 January and 31 December
included the following items:
1 January 31 December
2017 2017
Rs. Rs.
Loans repayable within 12 months 760,000 400,000
Loans repayable after 12 months 1,400,000 1,650,000
The cash flows relating to loans during the year are calculated as follows.
Rs.
Loans outstanding at the end of 2017 2,050,000
Loans outstanding at the beginning of 2017 2,160,000
= Net loan repayments during the year (= cash outflow) 110,000

5.4 Dividend payments to equity shareholders


These should be the final dividend payment from the previous year and the interim dividend
payment for the current year. The dividend payments during the year are shown in the Statement
of Changes in Equity (SOCIE).
You might be expected to calculate dividend payments from figures for retained earnings and the
profit after tax for the year.
The equity dividend payments can be calculated as follows:
Illustration 38:
Rs.
Retained earnings at the beginning of the year X
Profit after tax X
Any other transfer into the account X
Increase in the retained earnings reserve X
Retained earnings at the end of the year (X)
Equity dividend payments X

© Emile Woolf International 308 The Institute of Chartered Accountants of Pakistan


Chapter 8: IAS 7: Statement of cash flows

Example 09:
Question: From the following information, calculate the cash flows from financing activities for
Company X in 2017.
Beginning End
of 2017 of 2017
Rs. Rs.
Share capital (ordinary shares) 400,000 500,000
Share premium 275,000 615,000
Retained earnings 390,000 570,000
1,065,000 1,685,000
Loans repayable after more than 12 months 600,000 520,000
Loans repayable within 12 months or less 80,000 55,000
The company made a profit of Rs. 420,000 for the year after taxation.
Required
Calculate for 2017, for inclusion in the statement of cash flows:
(a) the cash from issuing new shares
(b) the cash flows received or paid for loans
(c) The payment of dividend to ordinary shareholders.
Answer:
Proceeds from new issue of shares Rs.
Share capital and share premium:
At the end of the year (500,000 + 615,000) 1,115,000
At the beginning of the year (400,000 + 275,000) (675,000)
Proceeds from new issue of shares during the year 440,000

Repayment of loans Rs.


Loans repayable:
At the end of the year (520,000 + 55,000) 575,000
At the beginning of the year (600,000 + 80,000) (680,000)
Repayment of loans during the year 105,000

Payment of dividends Rs.


Retained earnings at the beginning of the year 390,000
Profit after taxation for the year 420,000
810,000
Retained earnings at the end of the year (570,000)
Dividends paid during the year 240,000
Cash flows from financing activities can now be presented as follows.
Cash flows from financing activities Rs. Rs.
Proceeds from issue of shares 440,000
Repayment of loans (105,000)
Dividends paid to shareholders (240,000)
Net cash from financing activities 95,000

© Emile Woolf International 309 The Institute of Chartered Accountants of Pakistan


Financial accounting and reporting I

5.5 Financing of a sole proprietor or a partnership


You may face a question asking for the preparation of a statement of cash flows for a sole proprietor
or partnership. Such a question might require the calculation of cash flows between the owners
and the business. These cash flows would be capital introduced and drawings.
It is useful to remember the following relationship:

Illustration 39:

Rs.

Capital at the beginning of the year X

Profit (loss) after tax X/(X)

Capital introduced X

Drawings (X)

Capital at the end of the year X

The drawings and capital introduced figures might be provided in the question in which case you
simply have to slot the figures into the cash flow statement.
Other questions might need you to identify one or other of these as balancing figure.

EXAMPLE 10: Universal Limited (UL)


Question: Following are the extracts from the financial statements of Universal Limited (UL) for the
year ended 30 June 2017:

Statement of financial position as on 30 June 2017

Assets 2017 2016 Equity & liabilities 2017 2016

Rs. in ‘000 Rs. in ‘000

Property, plant and 158,500 120,000 Share capital (Rs. 10 175,000 150,000
equipment each)

Retained earnings 54,434 21,500

Stock in trade 58,000 45,000 Revaluation surplus 10,000 -

Income tax refundable 8,500 - Debentures (Rs. 100 18,000 20,000


each)

Trade receivables 68,000 56,000 Deferred tax liability - 6,000

Cash 39,434 48,000 Interest payable 1,000 2,500

Trade payables 42,000 39,000

Accrued liabilities 20,000 18,000

Unearned maintenance 2,000 4,000

Provision for taxation 10,000 8,000

332,434 269,000 332,434 269,000

© Emile Woolf International 310 The Institute of Chartered Accountants of Pakistan


Chapter 8: IAS 7: Statement of cash flows

Statement of profit or loss for the year ended 30 June 2017


Rs. in '000
Sales 273,000
Cost of sales (187,500)
Gross profit 85,500
Operating expenses (46,766)
Other income 11,200
Profit before interest and tax 49,934
Interest expense (2,000)
Profit before tax 47,934
Tax expense (15,000)
Profit after tax 32,934

Additional information:
i. 60% of sales were made on credit.
ii. UL maintains a provision for doubtful receivables at 6%. During the year, trade receivables of
Rs. 7 million were written off.
iii. Depreciation expense for the year was Rs. 22.5 million. 70% of the depreciation was charged
to cost of sales.
iv. Other income comprises of:
 gain of Rs. 3 million on disposal of vehicles for Rs. 12 million;
 maintenance income of Rs. 8 million; and
 discount of Rs. 10 per debenture which were redeemed during the year.
Required: Based on the above information, prepare UL’s statement of cash flows for the year ended
30 June 2017 using direct method.
Answer:
Statement of Cash Flows
For the year ended 30 June 2017
Cash flows from operating activities Rs. in ‘000

Cash receipts from customers

(Cash sales: 109,200; Credit sales: 144,034) (W-1) 253,234

Cash receipts from customers - maintenance services (W-2) 6,000

Cash paid to suppliers (W-3) (181,750)

Cash paid to other vendors (W-4) (30,250)

Income taxes paid (8,000+6,000+15,000–10,000+8,500) (27,500)

Interest paid (2,500+2,000–1,000) (3,500)

Net cash inflow from operating activities 16,234

© Emile Woolf International 311 The Institute of Chartered Accountants of Pakistan


Financial accounting and reporting I

Rs. in ‘000
Cash flows from investing activities
Purchase of property, plant and equipment [120,000–158,500–9,000
(i.e. 12,000–3,000)–22,500+10,000] (60,000)
Proceeds from disposal of vehicles 12,000
Net cash outflow from investing activities (48,000)

Cash flows from financing activities


Redemption of debentures [(20,000–18,000)–(20×10)] (1,800)
Proceeds from issue of shares (175,000–150,000) 25,000
Net cash inflow from financing activities 23,200
Net decrease in cash and cash equivalents (8,566)
Cash and cash equivalent at the beginning of the year 48,000
Cash and cash equivalent at the end of the year 39,434
Workings:
W-1: Cash receipts from customers - sales
Trade receivables – opening (56,000÷0.94) 59,574
Sales for the year 273,000
Bad debts written off (7,000)
Trade receivables – closing (68,000÷0.94) (72,340)
Cash received from customers 253,234

W-2: Cash receipts from customers - maintenance service


Unearned maintenance – opening (4,000)
Maintenance income for the year 8,000
Unearned maintenance – closing 2,000
6,000

W-3: Cash paid to suppliers --------- Rs. in ‘000 ---------


Trade payables – opening 39,000
Add: Purchases / Manufacturing cost
Stock in trade – closing 58,000
Cost of goods sold less dep. [187,500–(22,500×70%)] 171,750

Stock in trade – opening (45,000) 184,750


Less: Trade payables – closing (42,000)
Cash paid to suppliers 181,750

© Emile Woolf International 312 The Institute of Chartered Accountants of Pakistan


Chapter 8: IAS 7: Statement of cash flows

W-4: Cash paid to other vendors Rs. in ‘000


Accrued liabilities – opening 18,000
Operating expense for the year 46,766
Depreciation (22,500×30%) (6,750)
Bad debt expense (W-4.1) (7,766)
Accrued liabilities – closing (20,000)
30,250
W-4.1: Bad debts expense for the year
Provision for doubtful receivables – opening (56,000÷0.94×0.06) (3,574)
Bad debts written off 7,000
Provision for doubtful receivable – closing (68,000÷0.94×0.06) 4,340
7,766

EXAMPLE 11: TRANGO LIMITED


Question: The following information has been extracted from the financial statements of Trango
Limited for the year ended 31 December 2015.
Statement of comprehensive income for the year ended 31 December 2015

Sales 905,000
Cost of sales (311,000)
Gross profit 594,000
Loss on disposal of non-current asset (9,000)
Wages and salaries (266,000)
Other expenses (including depreciation Rs.46,000) (193,000)
126,000
Interest charges (24,000)
Profit before tax 102,000
Tax on profit (38,000)
Profit after tax 64,000
The asset disposed of had a carrying amount of Rs. 31,000 at the time of the sale.

Extracts from the statement of financial At 1 Jan At 31 Dec


position: 2015 2015
Rs.
Trade receivables 157,000 173,000
Inventory 42,000 38,000
Trade payables 43,600 35,700
Accrued wages and salaries 4,000 4,600
Accrued interest charges 11,200 10,000
Tax payable 45,000 41,000

Required: Present the cash flows from operating activities as they would be presented in a
statement of cash flows using direct method.

© Emile Woolf International 313 The Institute of Chartered Accountants of Pakistan


Financial accounting and reporting I

Answer:

Statement of cash flows Rs.


Cash receipts from customers (W1) 889,000
Cash payments to suppliers (W2) (314,900)
Cash payments to employees (W3) (265,400)
Cash paid for other expense (193,000-46,000) (147,000)
Cash generated from operations 161,700
Taxation paid (W4) (42,000)
Interest charges paid(W4) (25,200)
Net cash flow from operating activities 94,500

Workings:
W1 Cash receipts from customers
Trade receivables at the beginning of the year 157,000
Sales in the year 905,000
Trade receivables at the end of the year (173,000)
Cash from sales during the year 889,000

W2 Cash paid to suppliers


Calculation of Purchases
Closing inventory at the end of the year 38,000
Cost of sales 311,000
Opening inventory at the beginning of the year (42,000)
Purchases in the year 307,000

Trade payables at the beginning of the year 43,600


Purchases in the year (as above) 307,000
350,600
Trade payables at the end of the year (35,700)
Cash paid for materials 314,900

W3 Cash paid to employees


Accrued wages and salaries at the beginning of the year 4,000
Wages and salaries expenses in the year 266,000
Accrued wages and salaries at the end of the year (4,600)
Cash paid for wages and salaries 265,400

W4: Interest and tax payments Tax Interest


Rs. Rs.
Liability at 1 January 2015 45,000 11,200
Taxation charge/interest charge for the year 38,000 24,000
Liability at 31 December 2015 (41,000) (10,000)
Tax paid/interest paid during the year 42,000 25,200

© Emile Woolf International 314 The Institute of Chartered Accountants of Pakistan


Chapter 8: IAS 7: Statement of cash flows

EXAMPLE 12: NARDONE LIMITED


Question: The following information has been extracted from the draft financial information of
Nardone Limited.

Statement of comprehensive income for the year ended 31 December 2015


Rs.000 Rs.000
Sales revenue 490
Administration costs (86)
Distribution costs (78)
(164)
Operating profit 326
Interest expense (23)
Profit before tax 303
Taxation (87)
Profit after tax 216

Statement of financial position


31 December 2015 31 December 2014
Rs.000 Rs.000 Rs.000 Rs.000
Non-current assets (see below) 1,145 957
Current assets:
Inventory 19 16
Receivables 38 29
Bank 19 32
76 77
Total assets 1,221 1,034

Share capital 323 232


Revaluation reserve 170 0
Retained earnings 553 389
1,046 621
Non-current liabilities:
Long-term loans 70 320
Current liabilities:
Trade payables 12 17
Tax payable 93 76
105 93
Total equity and liabilities 1,221 1,034

© Emile Woolf International 315 The Institute of Chartered Accountants of Pakistan


Financial accounting and reporting I

Note on non-current assets

Land Fixtures
and Machinery & Total
buildings fittings

Rs.000 Rs.000 Rs.000 Rs.000

Cost or valuation

At 31 December 2014 830 470 197 1,497

Additions - 43 55 98

Disposals - (18) - (18)

Adjustment on revaluation 70 - - 70

At 31 December 2015 900 495 252 1,647

Depreciation

At 31 December 2014 (90) (270) (180) (540)

Charge for the year (10) (56) (8) (74)

Disposals - 12 - 12

Adjustment on revaluation 100 - - 100

At 31 December 2015 0 (314) (188) (502)

Carrying amount:

At 31 December 2014 740 200 17 957

At 31 December 2015 900 181 64 1,145

You have been informed that included within distribution costs is Rs. 4,000 relating to the loss on
a disposal of a non-current asset. Dividend of Rs. 52,000 was paid during the year.
Required:
Prepare a statement of cash flows for Nardone Limited for the year ended 31 December 2015.

© Emile Woolf International 316 The Institute of Chartered Accountants of Pakistan


Chapter 8: IAS 7: Statement of cash flows

Answer:
Nardone Limited
Statement of cash flows for the year ended 31 December 2015
Rs.000 Rs.000
Cash flows from operating activities
Profit before taxation 303
Adjustments for:
Depreciation 74
Interest charges in the statement of comprehensive income 23
Losses on disposal of non-current assets 4
404
Increase in receivables (38 – 29) (9)
Increase in inventories (19 – 16) (3)
Decrease in trade payables (17 – 12) (5)
Cash generated from operations 387
Taxation paid (W1) (70)
Interest charges paid (23)
Net cash flow from operating activities 294
Cash flows from investing activities
Purchase of non-current assets (98)
Proceeds from sale of non-current assets (W2) 2
Net cash used in (or received from) investing activities (96)
Cash flows from financing activities
Proceeds from issue of shares (323 – 232) 91
Repayment of loans (320 – 70) (250)
Dividends paid to shareholders (52)
Net cash used in (or received from) financing activities (211)
Net increase/(decrease) in cash and cash equivalents (13)
Cash and cash equivalents at beginning of the year 32
Cash and cash equivalents at the end of the year 19

Workings
W1: Taxation paid Rs.000
Taxation payable at the beginning of the year 76
Tax charge for the year (statement of comprehensive income) 87
163
Taxation payable at the end of the year (93)
Therefore tax paid during the year 70

W2: Disposal of machinery Rs.000


Cost of machinery disposed of 18
Accumulated depreciation on machinery disposed of (12)
Net book value at disposal 6
Loss on disposal 4
Therefore cash received from the disposal 2

© Emile Woolf International 317 The Institute of Chartered Accountants of Pakistan


Financial accounting and reporting I

EXAMPLE 13: HOT SAUCE LIMITED


Question: Hot Sauce Limited summarised final accounts are as follows:
Statement of financial position
31 December 2014 31 December 2015
Rs.000 Rs.000 Rs.000 Rs.000
Non-current assets:
Plant and machinery at cost 2,700 3,831
Accumulated depreciation (748) (1,125)
Carrying amount 1,952 2,706
Current assets:
Inventory 203 843
Receivables 147 184
Bank 51 -
401 1,027
Total assets 2,353 3,733
Ordinary share capital (Rs1 shares) 740 940
Share premium account 0 100
Retained earnings 671 1,034
1,411 2,074
Non-current liabilities:
Loans 320 150
Current liabilities:
Bank overdraft 0 766
Trade payables and accruals 152 141
Current taxation 470 602
622 1,509
Total equity and liabilities 2,353 3,733

Statement of comprehensive income for year ended 31 December 2015


R.s 000
Profit before tax 1,195
Taxation (602)
Profit after tax 593

Dividend payments during the year were Rs. 230,000.


The following information is also available:
(1) The only new loan raised during the year was a five-year bank loan amounting to Rs.
65,000.
(2) Interest charged during the year was Rs. 156,000. Interest accrued was Rs. 24,000 last
years and Rs. 54,000 this year.
(3) Depreciation charged during the year amounted to Rs. 401,000. This does not include any
profit or loss on disposal of non-current assets.
(4) During the year plant which originally cost Rs. 69,000 was disposed of for Rs. 41,000.
(5) During the year the company issued 200,000 new shares.
Required: Prepare a statement of cash flows for Hot Sauce Limited for the year ended 31 December
2015.

© Emile Woolf International 318 The Institute of Chartered Accountants of Pakistan


Chapter 8: IAS 7: Statement of cash flows

Answer:
Hot Sauce Limited

Statement of cash flows for the year ended 31 December 2015


Rs.000 Rs.000
Cash flows from operating activities
Profit before taxation 1,195
Adjustments for:
Depreciation 401
Loss on sale of plant (W1) 4
Interest charges in the statement of comprehensive income 156
1,756
Increase in receivables (184 – 147) (37)
Increase in inventories (843 – 203) (640)
Decrease in trade payables (W2) (41)
Cash generated from operations 1,038
Taxation paid (W3) (470)
Interest charges paid (W4) (126)
Net cash flow from operating activities 442

Cash flows from investing activities


Purchase of non-current assets (1,200)
Proceeds from sale of non-current assets (see W1) 41
Net cash used in (or received from) investing activities (1,159)
Cash flows from financing activities
Proceeds from issue of shares (W5) 300
Bank loan raised 65
Repayment of loans (W6) (235)
Dividends paid to shareholders (230)
Net cash used in (or received from) financing activities (100)
Net increase/(decrease) in cash and cash equivalents (817)
Cash and cash equivalents at beginning of the year 51
Cash and cash equivalents at the end of the year (766)
Cost of asset disposed of 69
Accumulated depreciation on asset disposed of (24)
Carrying amount at date of disposal 45
Disposal proceeds (41)
Therefore loss on disposal 4

© Emile Woolf International 319 The Institute of Chartered Accountants of Pakistan


Financial accounting and reporting I

W2: Increase or decrease in trade payables Rs.000 Rs.000


Trade payables and accruals at 31 December 2015 141
Less accrued interest (54)
87
Trade payables and accruals at 31 December 2014 152
Less accrued interest (24)
(128)
Decrease in trade payables and accruals (41)
Tutorial note
The accrued interest is removed from the figures because accrued interest is relevant to the amount
of interest paid in the year. This is a separate item in the statement of cash flows.
W3: Taxation paid
Current taxation liability at 31 December 2014 470
Taxation charge in the year 602
1,072
Current taxation liability at 31 December 2015 (602)
Therefore taxation paid in the year 470

W4: Interest paid


Accrued interest liability at 31 December 2014 24
Interest charge in the year 156
180
Accrued interest liability at 31 December 2015 (54)
Therefore interest paid in the year 126

W5: Proceeds from the issue of shares


Ordinary share capital at 31 December 2015 940
Share premium at 31 December 2015 100
1,040
Ordinary share capital at 31 December 2014 740
Share premium at 31 December 2014 -
(740)
Proceeds from the issue of shares 300

W6: Loans repaid


Loans at 31 December 2014 320
New loan during the year 65
385
Loans at 31 December 2015 (150)
Therefore loans repaid during the year 235

© Emile Woolf International 320 The Institute of Chartered Accountants of Pakistan


Chapter 8: IAS 7: Statement of cash flows

EXAMPLE 14: QUETTA TRACK LIMITED


Question: The statement of financial position of Quetta Track Limited as at 30 June was as follows:
2015 2014
Rs. 000 Rs. 000 Rs. 000 Rs. 000
ASSETS
Non-current assets
Property cost 22,000 12,000
Depreciation (4,000) (1,000)
————— 18,000 ————— 11,000
Plant and equipment
Cost 5,000 5,000
Depreciation (2,250) (2,000)
————— 2,750 ————— 3,000
————— —————
20,750 14,000

Current assets
Inventories 16,000 11,000
Trade receivables 9,950 2,700
Cash and cash equivalents – 1,300
————— 25,950 ————— 15,000
————— —————
Total assets 46,700 29,000
————— —————
EQUITY AND LIABILITIES
Capital and reserves
Equity capital 3,000 3,000
Accumulated profits 16,200 3,800
————— —————
19,200 6,800
Non-current liabilities
Loan 6,000 10,000
Current liabilities
Operating overdraft 11,000 –
Trade payables 8,000 11,000
Income tax payable 1,800 1,000
Accrued interest 700 200
————— 21,500 ————— 12,200
————— —————
Total equity and liabilities 46,700 29,000
————— —————

© Emile Woolf International 321 The Institute of Chartered Accountants of Pakistan


Financial accounting and reporting I

Statement of comprehensive income (extracts)

2015 2014

Rs.000 Rs.000

Operating profit 15,400 5,900

Financing cost (Interest) (1,000) (1,400)

————— —————

Profit before tax 14,400 4,500

Income tax expense (2,000) (1,500)

————— —————

Net profit for the year 12,400 3,000

————— —————
Equipment of carrying amount Rs.250,000 was sold at the beginning of 2015 for Rs.350,000. This
equipment had originally cost Rs.1,000,000.

In recent years, no dividends have been paid.

Required:
Prepare a statement of cash flows, under the indirect method, for the year ended 30 June 2015

Answer:
Quetta Track Limited

Statement of cash flow for the year ended 30 June, 2015.

Rs.000 Rs.000
Cash flows from operating activities
Net profit before tax 14,400
Adjustments for
Depreciation (Rs. 3,000 + 1,000) 4,000
Profit on sale of non-current assets (W3) (100)
Interest expense 1,000
—————
Operating profit before working capital adjustments 19,300
Increase in inventories (5,000)
Increase in trade receivables (7,250)
Decrease in trade payables (3,000)
—————
Cash generated from operations 4,050
Interest paid (W5) (500)
Income taxes paid (W4) (1,200)
—————
Net cash from operating activities 2,350

© Emile Woolf International 322 The Institute of Chartered Accountants of Pakistan


Chapter 8: IAS 7: Statement of cash flows

Cash flows from investing activities

Purchase of property (10,000)

Purchase of plant and equipment (W1) (1,000)

Proceeds from sale of plant and equipment (W3) 350

—————

Net cash used in investing activities (10,650)

Cash flows from financing activities

Part repayment of loan (4,000)


—————
—————
Net cash used in financing activities (4,000)
—————
Net decrease in cash and cash equivalents (12,300)

Cash and cash equivalents at beginning of year 1,300


—————
Cash and cash equivalents at end of period (11,000)
—————

Workings

(1) Plant and machinery – Cost

Rs.000 Rs.000

Bal b/d 5,000 Disposal 1,000


Additions () 1,000 Bal c/d 5,000
——– ——–
6,000 6,000
——– ——–

(2) Plant and machinery – Accumulated depreciation

Rs.000 Rs.000

Disposal 750 Bal b/d 2,000


Bal c/d 2,250 Depreciation charge for year () 1,000
——– ——–
3,000 3,000
——– ——–

© Emile Woolf International 323 The Institute of Chartered Accountants of Pakistan


Financial accounting and reporting I

(3) Plant and machinery – Disposals

Rs.000 Rs.000

Cost 1,000 Accumulated depreciation 750


Profit on sale 100 Proceeds 350
——– ——–
1,100 1,100
——– ——–
(4) Tax payable

Rs.000 Rs.000

Cash paid () 1,200 Bal b/d 1,000


Bal c/d 1,800 Tax charge to P&L 2,000
——– ——–
3,000 3,000
——– ——–
(5) Interest payable

Rs.000 Rs.000

Cash paid () 500 Bal b/d 200


Bal c/d 700 Charge to P&L 1,000
——– ——–
1,200 1,200
——– ——–

Example 15: MARDAN SOFTWARE LIMITED


Question: The following are the summarized accounts of Mardan Software Limited
Statement of financial position at 31 December
2014 2015
Rs.(000) Rs.(000) Rs.(000) Rs.(000)
ASSETS
Non-current assets
Plant and equipment 2,086 2,103
Fixtures and fittings 1,381 1,296
———— ————
3,467 3,399
Current assets
Inventory 1,292 1,952
Trade receivables 713 1,486
Short term investment 1,050 600
Cash 197 512
———— 3,252 ———— 4,550
———— ————
Total assets 6,719 7,949
———— ————

© Emile Woolf International 324 The Institute of Chartered Accountants of Pakistan


Chapter 8: IAS 7: Statement of cash flows

EQUITY AND LIABILITIES


Capital and reserves
Equity capital 4,200 4,500
Share premium reserve 800 900
Accumulated profits (Note 1) 431 1,180
———— ————
5,431 6,580
Current liabilities
Dividend payable 132 154
Income tax payable 257 312
Trade payables 899 903
———— 1,288 ———— 1,369
———— ————
Total equity and liabilities 6,719 7,949
———— ————

Statement of comprehensive income (extracts) for the year ended 31 December 2015

Rs.(000)

Profit before taxation 1,381

Income tax expense (310)

Net profit for the period 1,071

Note 1 Accumulated profits

Rs.(000)

Balance at 1 January 431

Net profit for the year 1,071

Dividend for the year (322)

Balance at 31 December 1,180

Further information:
(1) Plant and equipment with a carrying amount of Rs184,000 was disposed of for Rs.203,000,
whilst a new item of plant was purchased for Rs312,000
(2) Fixtures and fittings with a carrying amount of Rs100,000 were disposed of for Rs95,000;
(3) Depreciation recognised on fixtures and fittings amounted to Rs 351,000.
(4) Dividend for the year was declared during the year. Dividend payable in the statements of
financial position at each year end relate to dividends declared in that year but not paid over
to shareholders by the reporting date.
Required:
Prepare a statement of cash flows for the year ended 31 December 2015 in accordance with IAS
7: Statement of cash flows.

© Emile Woolf International 325 The Institute of Chartered Accountants of Pakistan


Financial accounting and reporting I

Answer:
Mardan Software Limited
Statement of cash flows for the year ended 31 December 2015

Rs.000 Rs.000
Cash flow from operating activities
Net profit before tax 1,381
Adjustments for
Depreciation charges (111 + 351) (W1, W2) 462
Profit on sale of machinery (W1) (19)
Loss on sale of fixtures (W2) 5
—————
Operating profit before working capital adjustments 1,829
Increase in inventories (660)
Increase in trade receivables (773)
Increase in trade payables 4
—————
Cash generated from operations 400
Income tax paid (W3) (255)
—————
Net cash from operating activities 145

Cash flows from investing activities


Purchase of plant and equipment (312 + 366) (W1, W2) (678)
Proceeds from sale of plant and equipment (203 + 95) (W1, W2) 298
—————
Net cash used in investing activities (380)

Cash flows from financing activities


Equity dividends paid (W4) (300)
Proceeds from issuance of ordinary share capital 400
—————
100
—————
Net decrease in cash and cash equivalents (135)

Cash and cash equivalents at beginning of year (1,050 + 197) 1,247


—————

Cash and cash equivalents at end of year (600 + 512) 1,112


—————

© Emile Woolf International 326 The Institute of Chartered Accountants of Pakistan


Chapter 8: IAS 7: Statement of cash flows

Workings
(1) Plant and equipment (carrying amt.)

Rs.000 Rs.000

Balance b/f 2,086 P & E – disposal 184


Bank – purchase 312 Depreciation (al fig) 111
Balance c/f 2,103
——– ——–
2,398 2,398
——– ——–

Plant and equipment– Disposal

Rs.000 Rs.000

P & E – Carrying amt. 184 Cash – proceeds 203


Gain on disposal 19
—— ——
203 203
—— ——

(2) Fixtures and fittings (Carrying amount)

Rs.000 Rs.000

Balance b/f 1,381 F & F – disposal 100


Bank – purchase (Bal. figure) 366 Depreciation 351
Balance c/f 1,296
——– ——–
1,747 1,747
——– ——–

Fixtures and fittings – Disposal

Rs.000 Rs.000

F & F – Carrying amt. 100 Cash – proceeds 95


Loss on disposal 5
—— ——
100 100
—— ——

(3) Tax payable

Rs.000 Rs.000

Bank – tax paid (al fig) 255 Balance b/f 257


Balance c/f 312 P&L a/c 310
—— ——
567 567
—— ——

© Emile Woolf International 327 The Institute of Chartered Accountants of Pakistan


Financial accounting and reporting I

(4) Dividends paid

Rs.000 Rs.000

Bank – dividends paid (al fig) 300 Balance b/f 132


Balance c/f 154 2015 dividend 322
—— ——
454 454
—— ——

Example 16: TARBELA TRADERS LIMITED


Question: The statement of financial position of Tarbela Traders Limited is presented below:
Statement of financial position

31 December 2015 31 December 2014


Rs.000 Rs.000 Rs.000 Rs.000
Non-current assets (at WDV)
Premises 37,000 38,000
Equipment 45,800 17,600
Motor vehicles 18,930 4,080
101,730 59,680
Investments 25,000 17,000
126,730 76,680
Current assets
Inventories 19,670 27,500
Trade receivables and prepayments 11,960 14,410
Cash and bank balances 5,500 5,400
37,130 47,310
Total assets 163,860 123,990

Capital and reserves


Share capital 67,940 67,940
Retained earnings 10,670 7,100
78,610 75,040
Non-current liabilities
Interest-bearing borrowings 25,000 28,000
Current liabilities
Trade payables and accrued expenses 32,050 20,950
Bank overdraft 28,200 –
60,250 20,950
163,860 123,990

© Emile Woolf International 328 The Institute of Chartered Accountants of Pakistan


Chapter 8: IAS 7: Statement of cash flows

Profit for the year ended 31 December 2015 is Rs. 3,570,000 (after accounting for);

Rs.000

Depreciation

Premises 1,000

Equipment 3,000

Motor vehicles 3,000

Profit on disposal of equipment 430

Loss on disposal of motor vehicle 740

Interest expense 3,000

The written down value of the assets at date of disposal was:

Equipment 5,200

Motor vehicles 2,010

Interest accrued at 31 December 2015 is Rs.400,000.


Required:
Prepare a statement of cash flows for the year ended 31 December 2015.

Answer:
Tarbela Trader Limited
Statement of cash flows for the year ended 31 December 2015

Rs.000 Rs.000
Cash flows from operating activities
Net profit 3,570
Adjustments for
Depreciation 7,000
Net loss on disposals 310
Interest expense 3,000

Operating profit before working capital changes 13,880


Decrease in trade receivables (11,960 – 14,410) 2,450
Decrease in inventories (19,670 – 27,500) 7,830
Increase in trade payables ((32,050 – 400) – 20,950) 10,700
Cash generated from operations 34,860
Interest paid (3,000 – 400) (2,600)
Net cash from operating activities 32,260

© Emile Woolf International 329 The Institute of Chartered Accountants of Pakistan


Financial accounting and reporting I

Rs.000 Rs.000
Cash flows from investing activities
Purchase of long-term investments (25,000 – 17,000) (8,000)
Purchase of equipment and cars
(36,400 (W1)+ 19,860 (W2)) (56,260)
Proceeds from sale of equipment and cars (W3) 6,900
Net cash used in investing activities (57,360)

Cash flows from financing activity


Borrowing repayment (3,000)

Net decrease in cash and cash equivalents (28,100)

Cash and cash equivalents at beginning of period (3,600 + 1,800) 5,400


Cash and cash equivalents at end of period
(4,800 + 700 – 28,200) (22,700)

Workings
(1) Equipment (WDV)

Rs.000 Rs.000
Bal b/d 17,600 Disposal 5,200
Depreciation 3,000
Additions () 36,400 Bal c/d 45,800
——– ——–
54,000 54,000
——– ——–

(2) Motor vehicles (WDV)

Rs.000 Rs.000
Bal b/d 4,080 Disposal 2,010
Depreciation 3,000
Additions () 19,860 Bal c/d 18,930
——– ——–
23,940 23,940
——– ——–

(3) Disposals

Rs.000 Rs.000
Equipment 5,200
Motor vehicle 2,010 Loss on disposal (vehicles) 740
Profit on disposal (equipment) 430 Proceeds () 6,900
——– ——–
7,640 7,640
——– ——–

© Emile Woolf International 330 The Institute of Chartered Accountants of Pakistan


Chapter 8: IAS 7: Statement of cash flows

Example 17: THE SINDH ROBOTICS COMPANY


Question: The statements of financial position and statement of comprehensive incomes of The
Sindh Robotics Company for two consecutive financial years are shown below.
Statements of financial position
31 December 2013 31 December 2014
Cost Dep’n Net Cost Dep’n Net
Rs.000 Rs.000 Rs.000 Rs.000 Rs.000 Rs.000
Non-current assets
Land 43,000 – 43,000 63,000 – 63,000
Buildings 50,000 10,000 40,000 90,000 11,000 79,000
Plant 10,000 4,000 6,000 11,000 5,000 6,000
—————— —————— —————— —————— —————— ——————
103,000 14,000 89,000 164,000 16,000 148,000
—————— —————— —————— ——————
Investments 50,000 80,000

31 December 2013 31 December 2014


Cost Dep’n Net Cost Dep’n Net
Rs.000 Rs.000 Rs.000 Rs.000 Rs.000 Rs.000
Current assets
Inventories 55,000 65,000
Trade receivables 40,000 50,000
Bank 3,000 –
—————— 98,000 —————— 115,000
—————— ——————
237,000 343,000
—————— ——————
Capital
Issued shares of
Rs. 1 each 40,000 50,000
Share premium 12,000 14,000
Revaluation – 20,000
surplus
Accumulated profit 25,000 25,000
—————— ——————
77,000 109,000
Non-current
liabilities
10% loan 100,000 150,000
borrowings

Current liabilities
Trade payables 40,000 60,000
Dividend payable 20,000 20,000
Bank overdraft – 4,000
—————— ——————
60,000 84,000
—————— ——————
237,000 343,000
—————— ——————

© Emile Woolf International 331 The Institute of Chartered Accountants of Pakistan


Financial accounting and reporting I

Statements of comprehensive incomes 2013 2014


Rs.000 Rs.000
Revenue 200,000 200,000
Cost of sales (100,000) (120,000)
———— ————
Gross profit 100,000 80,000
Distribution and administration expenses (50,000) (47,000)
———— ————
50,000 33,000
Interest (10,000) (13,000)
———— ————
Net profit for year 40,000 20,000
———— ————
Only one dividend is declared each year which is paid in the following year. No sales of non-
current assets have occurred during the relevant period. Ignore taxation.
Required:
Prepare a statement of cash flows for the year ended 31 December 2014 using the direct method.

Answer:
The Sindh Robotics Company
Statement of cash flows for the year ended 31 December 2014

Rs.000 Rs.000
Cash flows from operating activities
Cash receipts from customers (W1) 190,000
Cash paid to suppliers and employees (W2) (155,000)
Cash generated from operations 35,000
Interest paid (13,000)
Dividends paid* (20,000)
Net cash from operating activities 2,000
Cash flows from investing activities
Purchase of property and plant (40,000 + 1,000) (41,000)
Purchase of investments (30,000)
Net cash used in investing activities (71,000)
Cash flows from financing activities
Proceeds from issued shares (10,000 + 2,000) 12,000
Proceeds from long-term borrowings 50,000
Net cash from financing activities 62,000
Net decrease in cash and cash equivalents (7,000)
Cash and cash equivalents at 1 January 2014 3,000
Cash and cash equivalents at 31 December 2014 (4,000)

* Could be shown as a financing cash flow.

© Emile Woolf International 332 The Institute of Chartered Accountants of Pakistan


Chapter 8: IAS 7: Statement of cash flows

Workings
(1) Receipts from sales
Receivables control

Rs.000 Rs.000
Balance b/d 40,000 Cash receipts (al fig) 190,000
Sales 200,000 Balance c/d 50,000
_______ ________

240,000 240,000
—————— ——————

(2) Payments
Payables and wage control

Rs.000 Rs.000
Cash paid (al fig) 155,000 Balance b/d 40,000
Depreciation * 2,000 Purchases (W3) 130,000
Balance c/d 60,000 Expenses 47,000

________ ________

217,000 217,000
—————— ——————

(3) Cost of sales

Rs.000 Rs.000
Opening inventory 55,000 Cost of sales 120,000
Purchases and wages 130,000 Closing inventory 65,000
________ ________

185,000 185,000
—————— ——————

* Alternatively, depreciation could be adjusted against cost of sales.

EXAMPLE 18: ABIDA LIMITED


Question: Abida Ltd. made a net profit of Rs. 256,800 for the year ended June 30, 2015 after
charging depreciation of Rs. 17,500 and loss on disposal of furniture of Rs. 6,800. The sale proceeds
of the furniture were Rs. 12,000.
During the year, the net book value of non-current assets decreased by Rs. 7,400; receivables
increased by Rs. 11,700; inventories decreased by Rs. 21,600 and creditors increased by Rs. 8,900.
A long-term loan of Rs. 75,000 was repaid during the year and Abida withdrew Rs. 120,000 for her
own use.
Required:
Prepare the statement of cash flows for the year ended June 30, 2015.

© Emile Woolf International 333 The Institute of Chartered Accountants of Pakistan


Financial accounting and reporting I

Answer:

Abida Limited

Cash flow for year ended June 30, 2015

Profit for the year 256,800

Depreciation 17,500

Loss on sale of furniture 6,800

Increase/decrease in working capital

Decrease in inventories 21,600

Increase in payables 8,900

Increase in receivables (11,700)

18,800

299,900

Add: Proceeds from sale of non-current assets 12,000

311,900

Less: Purchase of non-current assets (W) 28,900

Payment of long term loan 75,000

Drawings 120,000

223,900

Net increase in bank balance 88,000

W Non-current assets

Decrease in assets 7,400 Depreciation 17,500

Purchase of assets – balancing figure 28,900 Sale of furniture 12,000

Loss on above sale 6,800

36,300 36,300

© Emile Woolf International 334 The Institute of Chartered Accountants of Pakistan


Chapter 8: IAS 7: Statement of cash flows

EXAMPLE 19: MOOSANI LIMITED


Question: The comparative statements of financial position of Moosani Ltd. show the following
information:

December 31

2015 2014

Rs. Rs.

Cash 5,200 41,400

Accounts receivable 31,700 21,500

Inventory 25,000 19,400

Investments - 16,900

Furniture 80,000 64,000

Equipment 86,000 43,000

Total 227,900 206,200

Allowance for doubtful accounts 6,500 9,700

Accumulated depreciation on equipment 24,000 18,000

Accumulated depreciation on furniture 8,000 15,000

Trade creditors 10,800 6,500

Accrued expenses 4,300 10,800

Bills payable 6,500 8,600

Long-term loans 31,800 53,800

Capital 136,000 83,800

Total 227,900 206,200

Additional data related to 2015 is as follows:


(i) Equipment that had cost Rs. 23,000 and was 40% depreciated at the time of
disposal was sold for Rs. 6,500.
(ii) Payments against long-term loans amounted to Rs. 22,000 of which Rs. 12,000
was paid by Mr. Moosani out of his personal account.
(iii) On January 1, 2015, the furniture was completely destroyed by a fire.
Proceeds received from the insurance company amounted to Rs. 60,000.
(iv) Investments were sold at Rs. 7,500 above their cost.
(v) Mr. Moosani withdraws Rs. 15,000 each month for his personal use.

Required:
Prepare a statement of cash flows for the year ended 31 December 2015.

© Emile Woolf International 335 The Institute of Chartered Accountants of Pakistan


Financial accounting and reporting I

Answer:
Moosani Limited
Statement of cash flows for the year ended 31 December 2015
Rs.000 Rs.000
Cash flows from operating activities
Net profit for the year (W1) 220,200
Adjustments for
Depreciation – equipment (24,000 + 9,200 – 18,000) 15,200
– furniture 8,000
Loss on sale of equipment (23,000 – 9,200 – 6,500) 7,300
Gain on sale of investments (7,500)
Insurance claim over book value (60,000 – [64,000 – 15,000]) (11,000)

Operating profit before working capital adjustments 232,200


Increase in payables 4,300
Decrease in bills payable (2,100)
Decrease in accrued expenses (6,500)
Increases in receivables (13,400)
Increase in inventory (5,600)
Cash generated from operations 208,900

Net cash from operating activities 208,900

Cash flows from investing activities


Insurance claim against furniture 60,000
Sale of investments (16,900 + 7,500) 24,400
Sale of equipment 6,500
Capital Expenditure – purchase of equipment
(86,000 + 23,000 – 43,000) (66,000)
Capital Expenditure – purchase of furniture
(80,000 + 64,000 – 64,000) (80,000)
Net cash used in investing activities (55,100)

Cash flows from financing activities


Capital invested (payment of long-term loan) 12,000
Payment of long-term loan (22,000)
Drawings / withdrawals of capital (15,000 x 12) (180,000)

Net cash used in financing activities (190,000)


Net decrease in cash and cash equivalents (36,200)
Cash and cash equivalents at beginning of year 41,400
Cash and cash equivalents at end of period 5,200

© Emile Woolf International 336 The Institute of Chartered Accountants of Pakistan


Chapter 8: IAS 7: Statement of cash flows

Working
W1: Profit for the year Rs.
Capital b/f 83,800
Capital introduced (loan repayment) 12,000
Less: drawings (180,000)
Profit for the year (balancing figure) 220,200
Capital c/f 136,000

EXAMPLE 20: SAKHAWAT HUSSAIN LIMITED


Question: The statements of financial position of Sakhawat Hussain Ltd. as at December 31, 2015
and 2014 are as follows:

2015 2014
Rs. Rs.
Current assets 4,750,000 2,850,000
Investments 2,600,000 2,500,000
Non-current assets 9,750,000 9,600,000
Accumulated depreciation (2,950,000) (2,450,000)
14,150,000 12,500,000
Non-current liability (loan) 2,000,000 2,000,000
Current liabilities 1,850,000 1,450,000
Interest liability 200,000 150,000
Capital 9,000,000 8,000,000
Profit and loss account 1,100,000 900,000
14,150,000 12,500,000

Other information for the year 2015 is as follows:


(i) Investments costing Rs. 250,000 were sold for Rs. 320,000.
(ii) Fully depreciated furniture costing Rs. 200,000 was written-off.
(iii) Non-current assets costing Rs. 960,000 with a net book value of Rs. 160,000 were sold for
Rs. 250,000.
(iv) Interest amounting to Rs. 180,000 was paid during the year.
(v) Sakhawat Hussain withdrew Rs. 1,200,000 from the profits of 2014 and 2015.
(vi) 20% of the opening and closing balances of current assets are represented by cash.

Required:
Prepare a statement of cash flows for the year ended December 31, 2015.

© Emile Woolf International 337 The Institute of Chartered Accountants of Pakistan


Financial accounting and reporting I

Answer:
Sakhawat Hussain Limited
Statement of cash flows for the year ended December 31, 2015
Rs.000 Rs.000
Cash flows from operating activities
Net profit before tax 1,400,000
Adjustments for
Depreciation on non-current assets
(2,950,000 – 2,450,000)+200,000+(960,000 – 160,000) 1,500,000
Profit on sale of investment (70,000)
Profit on sale of non-current assets (90,000)
Interest expense (180 + 200 – 150) 230,000

Operating profit before working capital adjustments 2,970,000


Increase in payables 400,000
Increase in current assets 80% of (4,750,000 – 2,850,000) (1,520,000)

Cash generated from operations 1,850,000


Interest paid (180,000)
Net cash from operating activities 1,670,000

Cash flows from investing activities


Purchase of non-current assets
(9,750,000 + 200,000 + 960,000 – 9,600,000) (1,310,000)
Purchase of investment
(2,600,000+250,000 – 2,500,000) (350,000)
Proceeds from sale of investment 320,000
Proceeds from sale of non-current assets 250,000 (1,090,000)
Net cash used in investing activities

Cash flows from financing activities


Capital introduced 1,000,000
Withdrawal by owner against profits (1,200,000)
Net cash used in financing activities (200,000)
Net increase in cash and cash equivalents 380,000
Cash and cash equivalents at beginning of year 570,000
Cash and cash equivalents at end of period 950,000
Working: Profit for the year

Closing balance 1,100,000


Drawings 1,200,000
2,300,000
Less: opening balance 900,000
Net profit for the year 1,400,000

© Emile Woolf International 338 The Institute of Chartered Accountants of Pakistan


Chapter 8: IAS 7: Statement of cash flows

EXAMPLE 21: JUNAID JANJUA LIMITED


Question: Junaid Janjua Ltd. has provided you the following statements of financial position and
statement of comprehensive income.
Statements of financial position as on December 31, 2015

2015 2014

Rupees

Cash 145,000 32,000

Accounts receivable 280,000 104,000

Long-term investments 220,000 170,000

Inventory 424,000 200,000

Prepaid insurance 24,000 36,000

Office supplies 14,000 7,000

Land 1,810,000 2,500,000

Building 2,800,000 2,300,000

Accumulated depreciation (890,000) (720,000)

Equipment 1,200,000 1,150,000

Accumulated depreciation (380,000) (350,000)

Total assets 5,647,000 5,429,000

Accounts payable 158,000 263,000

Wages payable 40,000 24,000

Short-term loans 580,000 580,000

Long-term loans 985,000 1,160,000

Capital 3,884,000 3,402,000

Total liabilities and equity 5,647,000 5,429,000

Statement of comprehensive income for the year ended December 31, 2015

2015
Rupees
Sales revenue 9,280,000
Cost of goods sold (6,199,000)
Gross profit 3,081,000

© Emile Woolf International 339 The Institute of Chartered Accountants of Pakistan


Financial accounting and reporting I

2015
Operating expenses Rupees
Selling expenses 634,000
Administrative expenses 1,348,000
Depreciation expenses 230,000
(2,212,000)
Income from operations 869,000
Other revenues/expenses
Gain on sale of land 64,000
Gain on sale of long term investment 32,000
Loss on sale of equipment (15,000)
81,000
Net income 950,000
Drawings (568,000)
Retained earnings 382,000
Notes:
(a) Part of the long term loan amounting to Rs. 100,000 was paid by Mr. Junaid from his personal
account.
(b) Long term investments costing Rs. 100,000 were sold during the year.
(c) Depreciation charged during the year on equipment amounted to Rs. 60,000. Equipment
having a book value of Rs. 75,000 was sold during the year.
Required: Prepare a statement of cash flows for the year ended December 31, 2015.

Answer:
Junaid Janjua Limited
Statement of cash flows for the year ended 31 December 2015
1

Cash flow from operating activities


Net income 950,000
Adjustments for:
Depreciation (170,000 + 60,000) 230,000
Gain on sale of land (64,000)
Gain on sale of long term investment (32,000)
Loss on sale of equipment 15,000
Operating profit before working capital adjustment 1,099,000

Increase in Accounts receivable (176,000)


Increase in Inventory (224,000)
Decrease in Prepaid insurance 12,000
Increase in Office supplies (7,000)
Decrease in accounts payable (105,000)
Increase in wages payable 16,000
Net cash from operating activities 615,000

© Emile Woolf International 340 The Institute of Chartered Accountants of Pakistan


Chapter 8: IAS 7: Statement of cash flows

Cash flows from investing activities


Proceeds from investment 100,000
Proceeds from sale of:
- Land (2,500,000 – 1,810,000 + 64,000) 754,000
- Equipment (75,000 – 15,000) 60,000
- Long term investments (100,000 + 32,000) 132,000 946,000
Fixed capital expenditure
– building (2,800,000 – 2,300,000) (500,000)
*
– equipment (1,200,000+105,000 –1,150,000) (155,000)
Long term investments (220,000 +100,000-170,000) (150,000)
Cash generated from investing activities 241,000

Cash flows from financing activities


Payment of long term loan (1,160,000– 985,000) (175,000)
Drawings (568,000)
Cash generated from financing activities (743,000)

Increase in cash and cash equivalent 113,000


Cash - opening 32,000
Cash - closing 145,000
*Book value 75,000 + accumulated depreciation 30,000 = Cost Rs. 105,000

EXAMPLE 22: AMIN INDUSTRIES LIMITED


Question: The statements of financial position of Amin Industries Ltd. as at 31 August 2014 and
2015 are as follows:

2015 2014 2015 2014


Rs. Rs. Rs. Rs.
Capital 33,433,000 27,942,000 Non-current assets –
book value 15,172,000 12,346,000

Current liabilities Current assets


Short term Investments
finance 2,545,000 1,616,000 4,911,000 -
Creditors 3,457,000 2,850,000 Inventory 12,178,000 14,950,000
6,002,000 4,466,000 Trade debts – net of
provision for bad debts 6,732,000 4,887,000
Bank 442,000 225,000
24,263,000 20,062,000
39,435,000 32,408,000 39,435,000 32,408,000

© Emile Woolf International 341 The Institute of Chartered Accountants of Pakistan


Financial accounting and reporting I

The following information is also available:


Rupees
Profit during the year ended 31 August 2015 3,161,000
Mr. Amin’s withdrawals during the year 3,120,000
Accumulated depreciation on non-current assets – 31 August 2014, 5,605,000
Accumulated depreciation on non-current assets – 31 August 2015 7,470,000
Provision for bad debts – 31 August 2014 385,000
Provision for bad debts – 31 August 2015 484,000
During the year non-current assets costing Rs. 1,500,000 with a carrying amount of Rs. 867,000
were sold for Rs. 1,284,000.
Required: Prepare a statement of cash flows for the year ended 31 August 2015. Show necessary
workings.
Answer:
Amin Industries Limited
Statement of cash flows for the year ended 31 August 2015
Cash flows from operating activities Rs. Rs.
Profit for the year 3,161,000
Adjustments for:
Depreciation charge 2,498,000
Profit on sale of non-current assets
(1,284,000 – 867,000) (417,000)
Provision for doubtful debts (484,000 – 385,000) 99,000
Operating profit before working capital adjustments 5,341,000

Decrease in inventory 2,772,000


(Increase) in trade debts (1,944,000)
Increase in payables 607,000
Increase in short term finance 929,000
Net cash from operating activities 7,705,000

Cash flows from investing activities


Purchase of non-current assets (6,191,000)
Sale proceeds of non-current assets 1,284,000
Purchase of investment (4,911,000)
Net cash from investing activities (9,818,000)

Cash flows from financing activities


Capital input 5,450,000
Withdrawals by Mr. Amin (3,120,000)
Net cash from financing activities 2,330,000
Increase in cash and cash equivalents 217,000
Opening bank balance 225,000
Closing bank balance 442,000

© Emile Woolf International 342 The Institute of Chartered Accountants of Pakistan


Chapter 8: IAS 7: Statement of cash flows

Workings Non-current assets – cost

Opening (12,346+5605) 17,951,000 Sale 1,500,000

Additions – balancing figure 6,191,000 Closing (15,172+7,470) 22,642,000

24,142,000 24,142,000

Accumulated depreciation

On assets sold (1,500-867) 633,000 Opening 5,605,000

Closing balance 7,470,000 Charge for the year 2,498,000

8,103,000 8,103,000

Trade debts

Opening (4,887+385) 5,272,000

Increase in balance 1,944,000 Closing (6,732+484) 7,216,000

7,216,000 7,216,000

Capital account – Mr. Amin

Withdrawals 3,120,000 Opening 27,942,000

Closing balance 33,433,000 Profit for the year 3,161,000

Capital introduced 5,450,000

36,553,000 36,553,000

EXAMPLE 23: NADIR LIMITED


Question: Following information pertains to Nadir Limited:
Extract from statement of profit or loss for the year ended 31 December 2017

Rs. in ‘000
Profit before taxation 8,955
Taxation (2,945)
Profit after taxation 6,010

© Emile Woolf International 343 The Institute of Chartered Accountants of Pakistan


Financial accounting and reporting I

Extract from statement of financial position as on 31 December 2017

2017 2016 2017 2016


Equity and liabilities Assets
---- Rs. in ‘000 ---- ---- Rs. in ‘000 ----
Share capital 12,400 10,000 Property plant &
Share premium 1,400 - equipment – net 21,400 15,800
Retained earnings 13,450 12,440 Current assets:
Surplus on revaluation 4,000 - Stock-in-trade 5,600 5,750
Non-current liabilities: Trade receivables – net 6,840 4,446
Long-term loans 4,100 5,000 Other receivables 2,385 800
Current liabilities: Cash & bank 2,355 3,204
Trade payables 1,900 1,400
Accruals & other payables 680 660
Tax liability 650 500
38,580 30,000 38,580 30,000
Other information:
(i) Shares issued during the year were as follows:
a. 10% bonus shares in March 2017.
b. Right shares in July 2017.
(ii) During the year, a plant costing Rs. 9,500,000 and having a book value of Rs. 5,200,000
was disposed of for Rs. 4,800,000 of which Rs. 1,800,000 are still outstanding.
(iii) Depreciation for the year amounted to Rs. 7,350,000.
(iv) Financial charges for the year amounted to Rs. 1,100,000. Accrued financial charges as on
31 December 2017 amounted to Rs. 112,000 (2016: Rs. 48,000).
(v) Provision for doubtful trade receivables is maintained at 5%.
Required:
Prepare statement of cash flows for the year ended 31 December 2017, in accordance with IAS 7
‘Statement of Cash Flows’ using indirect method.
Answer:
Nadir Limited
Statement of cash flows for the year ended 31 December 2017

Cash flows from operating activities:


Profit before tax 8,955
Adjustment for:
Depreciation 7,350
Loss on disposal 5,200–4,800 400
Financial charges 1,100
Increase in provision for doubtful receivables (6,840×5÷95) – (4,446×5÷95) 126
17,931
Working capital changes:

© Emile Woolf International 344 The Institute of Chartered Accountants of Pakistan


Chapter 8: IAS 7: Statement of cash flows

31 December 31 December
2017 2016

Stock-in-trade 5,600 5,750 150

Other receivables 585 800 215


2,385–1,800

Trade receivables-gross 7,200 4,680 (2,520)


(6,840÷0.95);(4,446÷0.95)

Trade payables (1,900) (1,400) 500

Accrued exp./Other payables (568) (612) (44)


(680–112);(660–48)

Increase in working capital (1,699)

Cash generated from operations 16,232

Payment of interest 1,100 –112+48 (1,036)

Payment of taxes 2,945–650+500 (2,795)

12,401

Cash flows from investing activities

Additions to PP&E 15,800–7,350–5,200+4,000–21,400 (14,150)

Disposals of PP&E 4,800–1,800 3,000

(11,150)

Cash flows from financing activities

Issue of right shares (12,400–10,000)–(10,000×10%)+1,400 2,800

Loan repaid 5,000–4,100 (900)

Cash dividend paid [(12,440+6,010)–(10,000×10%)]–13,450 (4,000)

(2,100)

Net cash outflows (849)

Cash and cash equivalent at beginning of the year 3,204

Cash and cash equivalent at year-end 2,355

© Emile Woolf International 345 The Institute of Chartered Accountants of Pakistan


Financial accounting and reporting I

EXAMPLE 24: LIAQUAT INDUSTRIES


Question: The statement of financial position of Liaquat Industries as at 31 December 2016 is as
follows:

Equity and 2016 2015 2016 2015


Assets
liabilities -------- Rupees -------- -------- Rupees --------
Owner’s capital 13,938,060 13,665,280 Freehold land 4,778,400 6,600,000
Long-term loan 1,000,000 1,000,000 Building – WDV 5,057,600 4,171,200
Short term loan 1,331,200 1,531,200 Vehicle – WDV 600,000 800,000
Accounts payable 417,120 694,320 Equipment – WDV 1,643,100 2,112,000
Accrued interest 105,600 63,360 Capital work in 1,478,400 1,821,600
progress
Long-term deposits 580,800 448,800
Inventory 685,608 320,628
Accounts receivable 1,273,272 595,452
Cash 694,800 84,480
16,791,980 16,954,160 16,791,980 16,954,160
The following information has been extracted from income statement:

Rupees
Depreciation expenses 932,500
Finance cost 141,872
Gain on sale of fixed assets (net) 98,960
Net profit before tax 1,525,948
Additional information:
a Details of gain on sale of fixed assets are as follows:

Rupees
Gain on sale of freehold land 168,960
Loss on disposal of equipment due to fire (70,000)
98,960
The loss on disposal of equipment represents the WDV of the equipment. The amount of
insurance claim received, amounting to Rs. 30,000 was erroneously credited to accumulated
depreciation.
b Repairs to building amounting to Rs. 50,000 were erroneously debited to building account
on 31 December 2016.
c Transfers from capital work in progress to building amounted to Rs. 1,200,000.
d The owner withdrew Rs. 150,000 per month.
Required:
Prepare statement of cash flows for the year ended 31 December 2016, in accordance with IAS – 7
using indirect method.

© Emile Woolf International 346 The Institute of Chartered Accountants of Pakistan


Chapter 8: IAS 7: Statement of cash flows

Answer:
Liaquat Industries Limited
Statement of cash flows for the year ended 31 December 2016

Cash flows from operating activities Rupees


Net profit before tax (1,525,948 – 50,000+ 30,000) 1,505,948

Adjustments for:
Depreciation expenses 932,500
Gain on disposal (70,000 – 30,000 – 168,960) (128,960)
Finance cost 141,872
Adjusted profit before working capital changes 2,451,360

Working capital changes:


Accounts receivable (595,452 – 1,273,272) (677,820)
Inventory (320,628 – 685,608) (364,980)
Accounts payable (417,120 – 694,320) (277,200)
Net cash from operating activities 1,131,360

Cash flows from investing activities


Proceed from sale of fixed assets (W-2) 2,020,560
Capital expenditure (1,821,600 – 1,200,000 – 1,478,400) (856,800)
Long term deposits (448,800 – 580,800) (132,000)
Net cash from investing activities 1,031,760

Rupees
Cash flow from financing activities
Interest paid (105,600 – 63,360 – 141,872) (99,632)
Drawing made by the owner (150,000×12) (1,800,000)
Amount injected by the owner (W-1) 546,832
Repayment of short term loan (1,331,200 – 1,531,200) (200,000)
Net cash used in financing activities (1,552,500)
Net increase in cash and cash equivalents 610,320
Cash at the beginning of year 84,480
Cash at the end of year 694,800

W-1: Movement in capital account


Opening capital 13,665,280
Less: Drawings (150,000×12) (1,800,000)
Add: Profit (1,525,948–50,000+30,000) 1,505,948
13,371,228
Less: Closing capital (13,938,060+30,000–50,000) 13,918,060
Capital injected 546,832

© Emile Woolf International 347 The Institute of Chartered Accountants of Pakistan


Financial accounting and reporting I

W-2: Disposal proceeds from sale of fixed assets


Freehold land – Opening 6,600,000
Lees: Freehold land – Closing (4,778,400)
Disposal cost 1,821,600
Add: Gain on disposal of freehold land 168,960
Sale proceeds from disposal of freehold land 1,990,560
Insurance claim received against fixed assets 30,000
2,020,560

EXAMPLE 25: QUALITY ENTERPRISES


Question: Following are the extracts from income statement of Quality Enterprises (QE) for the year
ended 31 December 2015 and its statement of financial position as at that date, together with
some additional information:
Income statement for the year ended 31 December 2015
Rs. in ‘000
Profit from operations 6,402
Other income 1,357
Interest expense (100)
Profit before tax 7,659
Income tax expense (1,376)
Profit for the year 6,283

Statement of financial position as at 31 December 2015


2015 2014 2015 2014
Equity and liabilities Assets
--- Rs. in ‘000 --- --- Rs. in ‘000 ---
Non-current assets
Owner’s capital 14,219 10,703 Property, plant and 19,628 11,845
equipment
Unappropriated profit 10,652 6,697 Investments 7,645 6,498
27,273 18,343
Revaluation surplus 2,676 1,911
10% bank loan 6,000 -
Current liabilities Current assets
Trade and other 3,337 4,953 Inventories 4,642 3,073
payables
Income tax payable 1,300 994 Trade and other receivables 2,273 3,865
Bank overdraft - 27 Cash and bank 3,996 4
4,637 5,974 10,911 6,942
38,184 25,285 38,184 25,285
Additional information:
(i) During the year, movements in property, plant and equipment include:
 Depreciation amounting to Rs. 5,280,000.
 Machinery having a carrying amount of Rs. 2,481,000 was sold for Rs. 3,440,000.
 Factory building was revalued from a carrying amount of Rs. 5,963,000 to Rs. 8,000,000.
 An office building which had previously been revalued was sold at its carrying Amount of
Rs. 2,599,000.

© Emile Woolf International 348 The Institute of Chartered Accountants of Pakistan


Chapter 8: IAS 7: Statement of cash flows

(ii) The owner of QE withdrew Rs. 300,000 per month. The amounts were debited to
unappropriated profit.
(iii) Trade debts written off during the year amounted to Rs. 200,000. The provision for bad debts
as at 31 December 2015 was Rs. 400,000 (2014: Rs. 550,000)
(iv) The interest on bank loan is payable on 30th June every year. The bank loan was received on
1 November 2015. Interest for two months has been accrued and included in trade and other
payables.
(v) Other income includes investment income of Rs. 398,000. As at 31 December 2015, trade
and other receivables included investment income receivable amounting to Rs. 96,000 (2014:
Rs. 80,000).
Required:
Prepare a statement of cash flows for Quality Enterprises for the year ended 31 December 2015,
using the indirect method.

Answer:
Quality Enterprises Statement of cash flows for the year ended 31 December 2015

Rupees

Cash flow from operating activities

Profit before tax 7,659,000

Non-cash adjustments

Investment income (398,000)

Interest expense 100,000

Depreciation charge 5,280,000

Bad debt expense [(200,000 +(400,000 – 550,000)] 50,000

Profit on disposal of property, plant and equipment (959,000)


(3,440,000–2,481,000)

Changes in working capital

Increase in inventories (3,073,000–4,642,000) (1,569,000)

Decrease in trade and other receivables 1,558,000

[(3,865,000–80,000–2,273,000+96,000
+(550,000– 400,000– 200,000))]

Decrease in trade and other payables (1,716,000)


[(4,953,000–(3,337,000–100,000))]

Net changes in working capital (cash generated from operations) 10,005,000

Income tax paid (W-2) (1,070,000)

Net cash from operating activities 8,935,000

© Emile Woolf International 349 The Institute of Chartered Accountants of Pakistan


Financial accounting and reporting I

Rupees
Cash flow from investing activities

Purchase of property, plant and equipment (W-3) (16,106,000)


Proceeds from sale of property, plant and equipment 6,039,000
(3,440,000+2,599,000)
Investment income received (W-1) 382,000
Purchase of investments (7,645,000 – 6,498,000) (1,147,000)

Net cash used in investing activities (10,832,000)

Cash flow from financing activities

Obtain bank loan 6,000,000


Additional capital (14,219,000 – 10,703,000) 3,516,000
Drawings (3,600,000)

Net cash from financing activities 5,916,000

Net increase in cash and cash equivalents 4,019,000


Cash and cash equivalents at beginning of period (4,000–27,000) (23,000)

Cash and cash equivalents at end of period 3,996,000

W-1: Investment income received Amount in Rs.

Balance b/d 80,000 Cash (balancing) 382,000


Income Statement 398,000 Balance c/d 96,000

478,000 478,000

W-2: Tax paid Amount in Rs.

Taxes paid (balancing) 1,070,000 Balance b/d 994,000


Balance c/d 1,300,000 Income Statement 1,376,000

2,370,000 2,370,000

W-3: Property, plant and Equipment Amount in Rs.

Balance b/d 11,845,000 Disposals (2,481,000+2,599,000) 5,080,000


Revaluation surplus
(8,000,0005,963,000) 2,037,000 Depreciation 5,280,000
Additions (balancing) 16,106,000 Balance c/d 19,628,000
29,988,000 29,988,000

© Emile Woolf International 350 The Institute of Chartered Accountants of Pakistan


Chapter 8: IAS 7: Statement of cash flows

EXAMPLE 26: KLEA


Question: The statement of financial position and statement of profit or loss for Klea for the year to
31st March 2015 are provided below.

Statement of financial position as at 31 st March 2015


2015 2014
Rs. in ‘000
Assets
Non-current assets
Intangible assets 300 200
Property, plant and equipment 3,450 1,600
Long term investments 400 200
4,150 2,000
Current assets
Inventory 3,200 2,000
Trade receivables 2,400 2,000
Cash and cash equivalents 32 580
5,632 4,580
Total assets 9,782 6,580

Equity and liabilities


Equity
Issued share capital 3,000 2,000
Share premium account 838 560
Retained earnings 910 354

Total equity 4,748 2,914


Revaluation surplus 1,000 -

Non-current liabilities
Interest-bearing loans and liabilities 1,600 2,000
Current liabilities
Bank overdraft 414 -
Trade payables 1,600 1,266
Taxation 420 400
2,434 1,666

Total liabilities 4,034 3,666


Total equity and liabilities 9,782 6,580

© Emile Woolf International 351 The Institute of Chartered Accountants of Pakistan


Financial accounting and reporting I

Statement of profit or loss for the year ended 31st March 2015
Revenue 10,000
Other income 100
Change in inventory of finished goods and WIP 1,300
Raw materials and consumables used 4,000
Employee benefits costs 3,000
Depreciation and amortisation expense 800
Other expenses 1,724

Total expenses (9,524)


1,876
Finance costs (320)
Finance income 50

Profit before tax 1,606


Income tax expense (650)
Profit for the year 956

Additional information
(i) Non-current assets Rs. in ‘000

2015 2014

Cost Deprec’n Cost Deprec’n

Intangible assets 700 400 400 200

Property, plant and equipment 5,000 1,550 3,000 1,400

(ii) At 1 April 2014 land was revalued from Rs. 1million to Rs. 2 million.
(iii) During the year, plant and machinery costing Rs. 600,000 and depreciated by Rs. 500,000
was sold for Rs. 150,000.
(iv) The interest bearing loans relate to debentures which were issued at their nominal value. Rs.
400,000 of these debentures was redeemed at par during the year.
(v) Ordinary shares were issued for cash during the year.
(vi) Rs. 100,000 of current asset investments held as cash equivalents were sold during the year
for Rs. 94,000.
Dividends paid in the year were Rs. 200,000 relating to the 2014 proposed dividend and Rs.
200,000 interim dividends for 2015.
Required:
Prepare a statement of cash flows for Klea for the year ended 31 March 2015 in accordance with
IAS 7 using the indirect method.

© Emile Woolf International 352 The Institute of Chartered Accountants of Pakistan


Chapter 8: IAS 7: Statement of cash flows

Answer:

KLEA’s Statement of cash flows for the year ended 31 st March 2015
Rs. in ‘000
Cash flows from operating activities
Profit before taxation 1,606
Adjustments for:
Depreciation (W4) 800
Finance income (50)
Interest expense 320
2,676

Increase in trade receivables (400)


Increase in inventories (1,200)
Increase in trade payables 334

Cash generated from operations 1,410


Interest paid (320)
Income taxes paid (W1) (630)

Net cash from operating activities 460


Cash flows from investing activities
Purchase of intangible assets (W2) (300)
Purchase of property, plant and equipment (W3) (1,600)
Proceeds from sale of equipment 150
Purchase of long-term investments (200)
Finance income received 50

Net cash used in investing activities (1,900)

Cash flows from financing activities


Proceeds from issue of share capital (1,000 + 278) 1,278
Payments to redeem debentures (400)
Dividends paid (400)

Net cash used in financing activities 478

Net decrease in cash and cash equivalents (962)


Cash and cash equivalents at 1 April 2014 580
Cash and cash equivalents at 31 March 2015 (32 - 414) (382)
(Note: Alternative classifications of the cash flows in accordance with IAS 7 should receive full credit
i.e. interest and dividends received as investing activities or operating cash flows, interest and
dividends paid as financing or operating cash flows.)

© Emile Woolf International 353 The Institute of Chartered Accountants of Pakistan


Financial accounting and reporting I

Notes
(1) Analysis of cash and cash equivalents
Rs. in ‘000
2015 2014
Cash on hand and balances with bank 32 580
Bank overdraft (414) -
-------------- --------------
Cash and cash equivalents (382) 580
-------------- --------------
(2) Material non-cash transactions
During the year land was re-valued upwards by Rs.1million

Workings
(W1) Taxation paid
Rs. in ‘000
Taxation creditor brought forward 400
Taxation expense for period 650
--------------
1,050
Taxation creditor carried forward (420)
--------------
Taxation paid in the year 630
--------------

(W2) Intangible assets


Net book value brought forward 200
Capitalised in the year (from (i)) 300
--------------
500
Amortisation charged in year (from (i)) (200)
--------------
Intangibles acquired in the year 300
--------------

(W3) Property, plant and equipment


Cost brought forward 3,000
Revaluation in year (from (ii)) 1,000
Disposals (from (iii)) (600)
Additions (balancing figure) 1,600
--------------
Cost carried forward 5,000
--------------

(W4) Depreciation and amortisation


Depreciation (150 movement + 500 on disposal) 650
Amortisation 200
Profit on disposal (W5) (50)
--------------
Charge shown in statement of profit or loss 800
--------------
Hence add back of depreciation and amortisation also takes account of the profit on disposal
of the plant and machinery.

© Emile Woolf International 354 The Institute of Chartered Accountants of Pakistan


Chapter 8: IAS 7: Statement of cash flows

(W5) Disposal
Rs. in ‘000
Cost of disposal 600
Accumulated depreciation (500)
Net book value 100
Proceeds of sale 150
--------------
Profit on sale 50
--------------

Example 27: Sunday Traders Limited


Question: Following are the extracts from the financial statements of Sunday Traders Limited (STL)
for the year ended 30 June 2019:
Statement of financial position as on 30 June 2019

2019 2018 2019 2018


Assets Equity & liabilities
Rs. in million Rs. in million
Property, plant & equipment 8,555 7,240 Share capital 4,650 3,450
(Rs. 100 each)
Investment property 1,800 1,120 Share premium 1,600 1,240
Stock in trade 4,800 4,500 Retained earnings 1,652 (655)
Prepayments 184 268 Long term loans 6,024 6,523
Trade receivables 3,800 3,600 Trade payables 3,422 5,390
Cash 194 480 Advance from
250 40
customers
Accrued liabilities 310 180
Interest payable 135 110
Current maturity:
long-term loans 850 700
Provision for taxation 440 230
19,333 17,208 19,333 17,208

Statement of profit or loss for the year ended 30 June 2019


Rs. in million
Sales 29,700
Cost of sales (15,750)
Gross profit 13,950
Distribution cost (6,185)
Administrative cost (2,302)
Other income 404
Profit before interest and tax 5,867
Interest expense (1,210)
Profit before tax 4,657
Tax expense (1,150)
Profit after tax 3,507

© Emile Woolf International 355 The Institute of Chartered Accountants of Pakistan


Financial accounting and reporting I

Additional information:
(i) 72% of sales were made on credit.
(ii) Depreciation expense for the year amounted to Rs. 750 million which was charged to
distribution and administrative cost in the ratio of 3:1.
(iii) Distribution cost includes:
 Rs. 40 million in respect of loss on disposal of equipment. The written down value at
the time of disposal was Rs. 152 million.
 impairment loss on vehicles amounting to Rs. 24 million.
(iv) Loan instalments (including interest) of Rs. 1,984 million were paid during the year.
(v) Other income comprises of:
 increase in fair value of investment property amounting to Rs. 220 million.
 rent received from investment property amounting to Rs. 184 million.
(vi) During the year, STL issued right shares at premium.
Required:
Prepare STL’s statement of cash flows for the year ended 30 June 2019 using direct method.

Answer:
Sunday Traders Limited
Statement of Cash Flows
For the year ended 30 June 2019
Cash flows from operating activities Rs. in million
Cash receipts from customers (Cash sales: 8,316 ; Credit sales: 21,394)
(W-1) 29,710
Cash receipts from tenants 184

Cash paid to suppliers (W-2) (18,018)


Cash paid to other vendors (W-3) (7,459)

Cash generated from operations 4,417


Interest paid 110+1,210–135 (1,185)
Income taxes paid 230+1,150–440 (940)

Net cash inflow from operating activities 2,292

Cash flows from investing activities

Purchase of property, plant and equipment (8,555–7,240)+24+152+750 (2,241)


Proceeds from disposal of property, plant and equipment 152–40 112

Purchase of investment property (1,800–1,120)–220 (460)

Net cash outflow from investing activities (2,589)

© Emile Woolf International 356 The Institute of Chartered Accountants of Pakistan


Chapter 8: IAS 7: Statement of cash flows

Cash flows from financing activities Rs. in million


Proceeds from issue of shares (4,650–3,450)+(1,600–1,240) 1,560
Dividend paid 1,652+655–3,507 (1,200)
Repayment of loans 1,984–1,185 (799)
New loans acquired (6,024–6,523)+799+(850–700) 450
Net cash inflow from financing activities 11
Net decrease in cash and cash equivalents (286)
Cash and cash equivalent at the beginning of the year 480
Cash and cash equivalent at the end of the year 194

Workings:
W-1: Cash receipts from customers – sales
Sales for the year 29,700
Increase in trade receivables balances 3,600–3,800 (200)
Increase in advance from customers 250–40 210
Cash received from customers 29,710
W-2: Cash paid to suppliers
Cost of sales 15,750
Increase in stock balances 4,800–4,500 300
Decrease in trade payable balances 5,390–3,422 1,968
Cash paid to suppliers 18,018
W-3: Cash paid to other vendors
Distribution cost 6,185
Administrative cost 2,302
Depreciation (750)
Loss on disposal (40)
Impairment (24)
Increase in accrued liabilities balances 180–310 (130)
Decrease in prepayment balances 184–268 (84)
7,459
.

Example 28: Broad Peak Limited


Question: You are working as Finance Manager in Broad Peak Limited (BPL). Faraz has recently
joined BPL as an internee for three months. You have asked him to develop an understanding
of the statement of cash flows. After going through few statements, he has raised the following
queries:
(i) Depreciation is not a cash flow but was still appearing as an addition in the statement
of cash flows.
(ii) In the statement of cash flows of a competitor, interest paid was shown as a financing
activity but BPL showed it in operating activities.
(iii) BPL purchased inventories throughout the year but total purchases of inventory were
not shown in the statement. However, only decrease in inventory was added.
(iv) Cash and bank balance in the statement of financial position was not in agreement with
the opening and closing balances at the end of statement of cash flows.
Required:
Briefly answer the queries raised by Faraz.

© Emile Woolf International 357 The Institute of Chartered Accountants of Pakistan


Financial accounting and reporting I

Answer:
(i) A statement of cash flows begins with net profit which is arrived after deducting depreciation
expense. So to convert the net profit into net cash flow the deduction of depreciation is
reversed (i.e. added).
(ii) As per IAS 7, interest paid can be shown as either cash flow from financing activities or cash
flow from operating activities. Both classifications are correct as long as they are
consistently applied by an entity.
(iii) A statement of cash flows begins with net profit which is arrived after deducting cost of
sales. So to convert the effect of cost of goods sold into outflow for purchases of inventory,
change in inventory is adjusted i.e. increase is deducted and decrease is added.
(iv) Statement of financial position shows cash and bank balances while the statement of cash
flows ends with cash and cash equivalents which may differ from cash and bank balances
due to existence of bank overdraft and short term investments.

Example 29: Taxila Limited (TL)


Question: Statement of financial position of Taxila Limited (TL) as on 30 June 2020 is as follows:
Assets 2020 2019 Equity & liabilities 2020 2019
Rs. in million Rs. in million
Property, plant and equipment Share capital (Rs. 100
1,619 1,200 each) 1,200 800
Investment property 290 120 Share premium 290 150
Inventories 205 180 Retained earnings 260 90
Trade receivables 342 291 Revaluation surplus 215 200
Prepayments and other Long-term loans
receivables 14 20 367 445
Short-term investments Trade and other
60 48 payables 144 120
Cash and bank balances 24 6 Current portion of
long-term loans
78 60
2,554 1,865 2,554 1,865
Additional information:
(i) Equipment having fair value of Rs. 240 million was acquired by issuing 2 million shares.
(ii) As a result of revaluation carried out on 30 June 2020, property, plant and equipment was
increased by Rs. 80 million out of which Rs. 35 million was credited to profit and loss
account.
(iii) During the year, fully depreciated items of property, plant and equipment costing Rs. 36
million were sold for Rs. 8 million out of which Rs. 3 million is still outstanding.
(iv) Depreciation on property, plant and equipment for the year amounted to Rs. 290
million.
(v) An investment property was acquired for Rs. 180 million. TL applies cost model for
subsequent measurement of its investment property.
(vi) Financial charges for the year amounted to Rs. 45 million. Trade and other payables include
accrued financial charges of Rs. 12 million (2019: Rs. 17 million).
(vii) Short-term investments amounting to Rs. 35 million are readily convertible to cash (2019:
Rs. 20 million). Investment income for the year amounted to Rs. 6 million.
Required:
Prepare TL’s statement of cash flows for the year ended 30 June 2020 in accordance with the
requirements of IFRSs.

© Emile Woolf International 358 The Institute of Chartered Accountants of Pakistan


Chapter 8: IAS 7: Statement of cash flows

Answer:
Taxila Limited
Statement of cash flows for the year ended 30 June 2020
Rs. in million
Cash flows from operating activities
Profit (W-1) 140
Adjustments for:
Depreciation on property, plant and equipment 290
Depreciation on investment property 120+180–290 10
Gain on disposal of property, plant and equipment (8)
Revaluation gain (35)
Interest expense 45
Operating profit before working capital changes 442
Changes in working capital:
Increase in inventory 205–180 (25)
Decrease in prepayments and other receivables (14–3)–20 9
Increase in trade receivables 342–291 (51)
Decrease in short-term investments (60–35)–(48–20) 3
Increase in trade and other payables (144–12)–(120–
17) 29
(35)
Cash generated from operations 407
Interest paid 17+45–12 (50)
Net cash flows from operating activities 357

Cash flows from investing activities


Purchase of property, plant and equipment (W-2) (389)
Purchase of investment property (180)
Proceeds from disposal of property, plant & equipment 8–3 5
Net cash flows used in investing activities (564)

Cash flows from financing activities


Proceeds from issue of shares at premium (W-3) 300
Repayment of long term loan (367+78)– (445+60) (60)
Net cash flows from financing activities 240
Net increase in cash and cash equivalents 33
Cash and cash equivalents at beginning of the year 6+20 26
Cash and cash equivalents at the end of the year 24+35 59

© Emile Woolf International 359 The Institute of Chartered Accountants of Pakistan


Financial accounting and reporting I

W-1: Profit for the year Rs. in million


Retained earnings – closing 260
Transfer from revaluation surplus 200+(80–35)–215 (30)
Retained earnings – opening (90)
140

W-2: Purchase of property, plant and equipment Rs. in million


Opening balance 1,200
Equipment acquired against issuance of shares 240
Revaluation surplus 80
Depreciation for the year (290)
Closing balance (1,619)
389

W-3: Issuance of shares at premium Rs. in million


Share capital and share premium – closing 1,200+290 1,490
Issuance of shares for property, plant and equipment (240)
Share capital and share premium – opening800+150 (950)
300

© Emile Woolf International 360 The Institute of Chartered Accountants of Pakistan


Chapter 8: IAS 7: Statement of cash flows

6 OBJECTIVE BASED QUESTIONS


01. Faria Limited is involved in the business of furniture. At 1 January 2018 the company’s issued
share capital consists of 50,000 Rs. 1 shares. During the year 2018 company has made a bonus
issue of 1 for 5 shares.
What is impact of bonus issue on cash flows of the business?
(a) Decrease in cash flows from operating activities
(b) Increase in cash flows from financing activities
(c) No impact
(d) Increase in cash generated from operations

02. A company has incurred a loss of Rs. 40,000 during the year 2018; however, the balance in the
bank account at end of the year is more than the balance at start of the year.
What does this mean?
(a) Company has allowed a longer credit period to the credit customers
(b) Company has purchased more stock
(c) Company has made a right issue during the year
(d) Company has purchased fixed assets during the year

03. A company has provided the following information:


2018 2017
Rs. Rs.
Share capital 110,000 100,000
Share premium 30,000 40,000
A bonus issue of 1 for every 10 shares held has been made during the year.
What is the amount to be reported in cash flow from financing activities for the year 2018?
(a) Rs. 10,000 Inflow
(b) 0
(c) Rs. 10,000 outflow
(d) Cannot be determined

04. A company has provided following balances


Rs.
Non–current asset – 31 December 2018 125,000
Accumulated depreciation 1 January 2018 25,000
Accumulated depreciation 31 December 2018 38,000
During the year an asset having cost Rs. 10,000 was sold for Rs. 6,000 and gain on disposal
was Rs. 3,000.
What is the charge for depreciation for the year to be adjusted in statement of cash flows?
(a) Rs. 13,000
(b) Rs. 19,000
(c) Rs. 20,000
(d) Rs. 38,000

© Emile Woolf International 361 The Institute of Chartered Accountants of Pakistan


Financial accounting and reporting I

05. A company has provided following information as at 31 March 2019:


2019 2018
Rs. Rs.
Retained earnings 50,000 38,000
Following adjustments were made during the year 2019:
Dividends paid Rs. 5,000
Transfer to general reserves Rs. 12,000
Tax charge Rs. 4,000
What is the amount of profit before tax for the year 2019 for the purposes of preparing statement
of cash flows?
(a) Rs. 29,000
(b) Rs. 33,000
(c) Rs. 24,000
(d) Rs. 25,000

06. A company has provided the following data:


Rs.
Receivables at 1 April 2018 12,000
Receivables at 31 March 2019 25,000
Credit sales during the year 75,000
Discount allowed during the year 3,000
What is the amount to be shown as cash received from customers in statement of cash flows
using direct method?
(a) Rs. 62,000
(b) Rs. 75,000
(c) Rs. 59,000
(d) Rs. 65,000

07. Which TWO of the following are considered as inflows in a company’s statement of cash flows?
(a) Bonus shares issued
(b) Decrease in accounts receivables
(c) Increase in inventory
(d) Increase in accounts payables

08. Which of the following item will appear in cash flows from financing activities section of
statement of cash flows?
(a) Cash paid to acquire non-current assets
(b) Dividends paid
(c) Bonus shares issued
(d) Depreciation for the year

© Emile Woolf International 362 The Institute of Chartered Accountants of Pakistan


Chapter 8: IAS 7: Statement of cash flows

09. Following data is available for a company for the year ended 31 December 2018:
Rs.
Operating profit before working capital changes 30,000
Increase in accounts receivables 5,000
Increase in inventory 3,000
Increase in accounts payable 2,000
Interest paid 500
What is the net cash generated from cash flows from operating activities for the year ended 31
December 2018?
(a) Rs. 23,500
(b) Rs. 24,500
(c) Rs. 29,500
(d) Rs. 19,500

10. Which of the following is an advantage of statement of cash flows?


(a) It determines the profitability of a business
(b) It helps users to estimate the future expected cash flows of the business
(c) It determines the ratio of business debts and equity
(d) It helps in determining the net assets of a business

11. A company has made following investments during the year:


Rs.
6 months Advance rent paid to landlord 30,000
Short term investments bond (highly liquid) 25,000
Debentures purchased- redeemable after 7 years 50,000
Non–current assets purchased 45,000
What is the amount to be shown in investing activities for the year?
(a) Rs. 45,000
(b) Rs. 150,000
(c) Rs. 95,000
(d) Rs. 100,000

12. A company has provided following data at the end of year 2017:
2017
Rs.
Share capital Rs. 1 each 100,000
Share premium 3,000
The company has made a right issue of 1 for 5 shares during the year 2018 at Rs. 1.2 per share.
What is the amount to be shown in the cash flows from financing activities?
(a) Rs. 24,000 outflow
(b) Rs. 24,000 inflow
(c) Rs. 20,000 inflow
(d) Rs. 4000 inflow

© Emile Woolf International 363 The Institute of Chartered Accountants of Pakistan


Financial accounting and reporting I

13. How should gain on sale of used equipment be reported in a cash flow statement, using indirect
approach?
(a) In operating activities as deduction from Profit before tax
(b) In investing activities as a reduction in cash inflow
(c) In investing activities as an increase in cash inflows
(d) In operating activities as addition to profit before tax

14. Which TWO of the following are added as non-cash adjustments to the profit before tax in the
cash flow from operating activities section of statement of cash flows?
(a) Interest expense
(b) Interest income
(c) Loss on sale of non–current assets
(d) Tax charge for the year

15. Where, in a company are financial statements complying with international accounting
standards, should you find the proceeds of non-current assets sold during the period?
(a) Statement of cash flows and statement of financial position
(b) Statement of changes in equity and statement of financial position
(c) Statement of profit or loss and statement of cash flows
(d) Statement of cash flows only

16. Zahid & Co. reported a profit Rs. 40,000 for the year, after charging the following:
Rs.

Depreciation 4,000

Loss on sale of assets 3,000


During the year there was a decrease in accounts receivables of Rs. 1,000.
What was the net cash flow generated from operations based on above data?
Rs. ___________

17. Asmat Limited made a profit for the year of Rs. 320,500, after accounting for depreciation Rs.
32,500. During the year following transactions took place:
Rs.

Purchase of machinery 125,000

Increase in accounts receivables 45,000


Increase in inventory 28,000

Increase in accounts payable 12,600


What is the net increase in cash and bank balance during the year?
Rs. ___________

© Emile Woolf International 364 The Institute of Chartered Accountants of Pakistan


Chapter 8: IAS 7: Statement of cash flows

18. A company has provided following information:


Rs.
4% Loan notes 1,000,000
Interest payable 1 January 2018 10,000
Interest payable 31 December 2018 20,000
What is the amount to be reported as interest paid during the year 2018 in the Statement of
Cash Flows?
Rs. ___________

19. Furqan Limited has provided following information about non–current assets:
Rs.
Cost as at 1 January 2018 350,000
Cost as at 31 December 2018 450,000
During the year an asset costing Rs. 100,000 and having net book value of Rs. 40,000 was sold
at a profit of Rs. 30,000.
What is the net to be shown as outflow in the “Cash flow from investing activities” section in
Statement of Cash Flows?
Rs. ___________

20. The following amounts have been calculated for inclusion in the statement of cash flow of House
Limited:
Rs.
Net cash inflow from financing activities 145,000
Net cash outflow from investing activities 160,000
Increase in cash and cash equivalents 24,000
Income taxes paid 65,000
Interest paid 12,000
How much cash has been generated from operations?
Rs. ___________

21. A cash flow statement provides information that enables users to evaluate the changes in:
(a) Solvency
(b) Net assets
(c) Its financial structure
(d) Its liquidity

22. Daily sales and purchases and employee costs comprise:


(a) Operating activities
(b) Investing activities
(c) Financing activity
(d) Component of cash and cash equivalent

© Emile Woolf International 365 The Institute of Chartered Accountants of Pakistan


Financial accounting and reporting I

23. Which of the following involves a movement of cash?


(a) A rights issue
(b) Depreciation of fixed assets
(c) Creation of a provision for doubtful debts
(d) A bonus issue

24. Activities that result in changes in the size and composition of the equity capital and borrowings
of an entity are called:
(a) Operating activities
(b) Investing activities
(c) Financing activity
(d) None of these

25. Which of the following are not the operating activities?


(a) Interest paid
(b) Cash payments of income taxes
(c) Collections from customers
(d) Payment of dividends

26. Amplifier Limited had sales of Rs.120 million during the year. Trade and other receivables
increased from Rs.12 million to Rs.16 million, an increase of Rs. 4 million. What amount of cash
was received from customers during the year?
(a) Rs.124 million
(b) Rs.116 million
(c) Rs.120 million
(d) None of these

27. Cost of sales for Shah Textile Limited during the year was Rs.100 million. Opening inventory
was Rs.20 million and closing inventory was Rs. 28 million. Opening trade payables were Rs.5
million and closing trade payables were Rs.9 million. What amount of cash was paid to
suppliers?
(a) Rs.102 million
(b) Rs.104 million
(c) Rs.108 million
(d) Rs.110 million

28. Zaman Limited extracted general ledger from which it shows salaries and wages expense of
Rs.50 million during the year. Its cash flow statement reported cash paid to employees of Rs.42
million. The opening balance of accrued salaries and wages was Rs.3.6 million. What was the
closing balance for accrued salaries and wages?
(a) Rs.11.6 million
(b) Rs.11.8 million
(c) Rs.4.4 million
(d) Rs.3.8 million

© Emile Woolf International 366 The Institute of Chartered Accountants of Pakistan


Chapter 8: IAS 7: Statement of cash flows

29. Sale proceeds from disposal of property, plant and equipment are classified as:
(a) Financing activities
(b) Operating activities
(c) Investing activities
(d) Either financing or operating activities, depending on which method (direct or indirect)
is used to determine cash flows from operating activities

30. Which one of the following events will increase the cash balances of a business?
(a) Loan repayment to banks
(b) Bank granting it an overdraft facility
(c) Debtors paying amounts owed
(d) Sale of stock on credit

31. A company with healthy profits is facing a cash shortage. Which of the following events could
account for this?
(a) Delaying payments to creditors
(b) The shortening of the credit period granted to debtors
(c) The recent acquisition of machinery
(d) An increase in dividend proposed by the directors

32. Which one of the following companies is most likely to run into cash flow problems?
(a) A loss making company making components of vital strategic importance to the
government
(b) A profitable new retailer about to embark on ambitious expansion plans
(c) A company which has recently sold part of its operations so as to concentrate on its
core areas
(d) Reasonably profitable, long established company with no expansion plans

33. What is the immediate effect of making a capital repayment on a loan on cash flow and profits?
(a) On profit - None; On cash – Decrease
(b) On profit - Increase; On cash – Decrease
(c) On profit - Decrease; On cash – Decrease
(d) On profit - Decrease; On cash – None

34. A company has a negative cash flow from operating activities. What could explain this negative
cash flow?
(a) High levels of dividend payments
(b) A substantial investment in new fixed assets
(c) A sudden increase in credit sales
(d) The repayment of a loan

© Emile Woolf International 367 The Institute of Chartered Accountants of Pakistan


Financial accounting and reporting I

35. Which of the following is NOT a cash outflow for the firm?
(a) Dividends
(b) Interest payments.
(c) Taxes
(d) Bad debts

36. Which of the following would cause negative net cash flow from operating activities?
(a) Decrease in depreciation expense
(b) A substantial investment in fixed assets
(c) A significant increase in credit sales
(d) Repayment of a long-term loan

© Emile Woolf International 368 The Institute of Chartered Accountants of Pakistan


Chapter 8: IAS 7: Statement of cash flows

6 OBJECTIVE BASED ANSWERS


01. (c) Bonus issue of shares involves transfer from Reserves to share capital of the
company. There is no cash flow involved.
02. (c) The statement indicates that the company had net cash receipts (inflows)
despite the losses, which is indicative of receipts of cash by issuing right
shares.
03. (b) No cash is paid or received for bonus issue of share capital.
Entry to record bonus issue
Dr CR
Rs. Rs.
Share premium 10,000
Share capital 10,000

Share Capital + Share premium


b/d 100,000+40,000 140,000
c/d
110,000 + 30,000 140,000
140,0000 140,000

04. (c)
Accumulated depreciation
Particulars Rs. Particulars Rs.
Disposal (see below) 7,000 b/f 25,000
c/f 38,000 Depreciation 20,000
45,000 45,000

Disposal
Particulars Rs. Particulars Rs.
Asset 10,000 Provision for dep. 7,000
(balancing)
Gain on disposal 3,000 Cash 6,000
13,000 13,000

05. (b)
Retained earnings
Particulars Rs. Particulars Rs.
Dividends paid 5,000 b/f 38,000
Transfer to reserves 12,000 Profit for the year 29,000
c/f 50,000
67,000 67,000
Profit after tax 29,000 + Tax 4,000 = Rs. 33,000 profit before tax

© Emile Woolf International 369 The Institute of Chartered Accountants of Pakistan


Financial accounting and reporting I

06. (c)
Accounts receivables
Particulars Rs. Particulars Rs.
b/f 12,000 Cash (bal.) 59,000
Sales 75,000 Discount allowed 3,000
c/f 25,000
87,000 87,000

07. (b) & (d) Decrease in accounts receivables indicates that they have paid the debt,
hence, inflow for us.
Increase in accounts payable indicates that we have not paid them, thus
reducing outflows (or increasing cash flows)
Bonus shares issued do not affect cash flows.
Increase in inventory is cash outflows.
08. (b) Dividend is paid to shareholders who provide finance to the business;
therefore, it is treated as financing activity.
Cash paid to acquire non-current assets is shown in investing activities.
Bonus issues have no impact on cash flows of the business.
Depreciation is non-cash item and is adjusted in operating activities.
09. (a)
Rs.
Operating profit before working capital changes 30,000
Increase in accounts receivables (5,000)
Increase in inventory (3,000)
Increase in accounts payable 2,000
Interest paid (500)
23,500

10. (b) Users of financial statements may predict future cash flows from past data of
how the entity generates and uses its cash.
Profitability is reflected in statement of comprehensive income.
Debt/Equity and net assets are reflected in statement of financial position.
11. (c) Only debentures and non – current assets purchased are included in investing
activities; Rs. 50,000+45,000= Rs. 95,000
Investment in short term bonds will be considered cash equivalent and
advance rent would affect operating activities cash flows.
12. (b) Shares issued = 100,000/5 = 20,000
Cash received = 20,000xRs.1.2= Rs. 24,000
13. (a) The gain on disposal in included in profit before tax as other income. This is
deducted back in order to determine the cash figure.
14. (a & c) Interest expense is added back as interest paid is separately reported.
Loss on disposal is added back as this is included in profit before tax as an
expense.
Interest income is deducted back.
Tax charge need not be added back as already the amount taken is profit
before tax.

© Emile Woolf International 370 The Institute of Chartered Accountants of Pakistan


Chapter 8: IAS 7: Statement of cash flows

15. (d)
16. Rs. 48,000
Rs.
Profit before tax 40,000
Adjustments for non-cash items
Depreciation 4,000
Loss on sale of fixed assets 3,000
Operating profit before working capital changes 47,000
Decrease in accounts receivables 1,000
Cash generated from operations 48,000

17. Rs. 167,600


Cash flows from operating activities Rs.
Profit before tax 320,500
Depreciation 32,500
Operating profit before working capital changes 353,000
Increase in accounts receivables (45,000)
Increase in inventory (28,000)
Increase in accounts payable 12,600
292,600

Cash flow from investing activities


Purchase of machinery (125,000)
167,600

18. Rs. 30,000


Interest payable
Particulars Rs. Particulars Rs.
Cash 30,000 b/f 10,000
c/f 20,000 Interest expense 40,000
50,000 50,000
Interest expense = Rs. 1,000,000x4%= Rs. 40,000
19. Rs. 130,000
Amounts to be shown in Cash flows from investing activities are;
Cash flows from investing activities Rs.
Cash paid to acquire assets (200,000)
Cash received on disposal 70,000
130,000

© Emile Woolf International 371 The Institute of Chartered Accountants of Pakistan


Financial accounting and reporting I

Non-current assets
Particulars Rs. Particulars Rs.
b/f 350,000 Disposal 100,000
Cash 200,000 c/d 450,000
550,000 550,000

Disposal
Particulars Rs. Particulars Rs.
Asset 100,000 Acc. Dep [100,000 – 60,000
40,000]
Gain on disposal 30,000 Cash 70,000
130,000 130,000

20. Rs. 116,000


Rs.
Cash generated from operations (β) 116,000
Interest paid (12,000)
Income taxes paid (65,000)
Net cash from operating activities (β) 39,000
Net cash outflow from investing activities (160,000)
Net cash inflow from financing activities 145,000
Increase in cash and cash equivalents 24,000

21. (d)
22. (a)
23. (a)
24. (c)
25. (d)
26. (b)
27. (b)
28. (a)
29. (c)
30. (c)
31. (c)
32. (b)
33. (a)
34. (c)
35. (d)
36. (c) A significant increase in credit sales

© Emile Woolf International 372 The Institute of Chartered Accountants of Pakistan


Certificate in Accounting and Finance

9
Financial accounting and reporting I

CHAPTER
Conceptual framework for financial
reporting

Contents
1 Introduction

2 Qualitative characteristics of useful financial information

3 Recognition

4 Measurement

5 Concept of capital and capital maintenance

6 Objective based questions and answers

* The student must refer original handbook of IFRS.

© Emile Woolf International 373 The Institute of Chartered Accountants of Pakistan


Financial accounting and reporting I

1 INTRODUCTION

Section overview

 Purpose of conceptual framework


 Contents of conceptual framework
 Status of conceptual framework
 Elements of financial statements

1.1 Purpose of conceptual framework


The Conceptual Framework for Financial Reporting (Conceptual Framework) describes the
objective of, and the concepts for, general purpose financial reporting.
The purpose of the Conceptual Framework is to assist:
 IASB to develop IFRSs that are based on consistent concepts;
 preparers to develop consistent accounting policies when no Standard applies to a particular
transaction or other event, or when a Standard allows a choice of accounting policy; and
 all parties to understand and interpret the Standards.
The Conceptual Framework contributes to the stated mission of the IFRS Foundation and IASB
i.e. to develop Standards that bring transparency, accountability and efficiency to financial markets
around the world.
The Conceptual Framework provides the foundation for Standards (IASs and IFRSs) that:

 contribute to transparency by enhancing the international comparability and quality of


financial information, enabling investors and other market participants to make informed
economic decisions.
 strengthen accountability by reducing the information gap between the providers of capital
and management. IFRSs and Conceptual Framework are also source of information for
regulators.
 contribute to economic efficiency i.e. the use of a single, trusted accounting language
derived from Standards based on the Conceptual Framework lowers the cost of capital and
reduces international reporting costs.

1.2 Contents of conceptual framework


The Conceptual framework is divided into eight chapters, namely:
Chapter 1 — The Objective Of General Purpose Financial Reporting
Chapter 2 — Qualitative Characteristics Of Useful Financial Information
Chapter 3 — Financial Statements And The Reporting Entity
Chapter 4 — The Elements Of Financial Statements
Chapter 5 — Recognition And Derecognition
Chapter 6 — Measurement
Chapter 7 — Presentation And Disclosure
Chapter 8 — Concepts Of Capital And Capital Maintenance

© Emile Woolf International 374 The Institute of Chartered Accountants of Pakistan


Chapter 9: Conceptual Framework for financial reporting

1.3 Status of conceptual framework


The Conceptual Framework is not a Standard and nothing in the Conceptual Framework overrides
any Standard (IASs or IFRSs) or any requirement in a Standard.

1.4 Elements of financial statements


The elements of financial statements defined in the conceptual framework are:
 assets, liabilities and equity, which relate to a reporting entity’s financial position; and
 income and expenses, which relate to a reporting entity’s financial performance
These elements are linked to the economic resources, claims and changes in economic resources
and claims and are explained as under:

Item discussed Elements Definition or description


Economic Asset A present economic resource controlled by the entity as a
resource result of past events.
An economic resource is a right that has the potential to
produce economic benefits.
Claim Liability A present obligation of the entity to transfer an economic
resource as a result of past events.
Equity The residual interest in the assets of the entity after
deducting all its liabilities.
Changes in Income Increases in assets, or decreases in liabilities, that result in
economic increase in equity, other than those relating to contributions
resources and from holders of equity claims.
claims, reflecting
Expenses Decreases in assets, or increases in liabilities, that result in
financial
decreases in equity, other than those relating to distributions
performance
to holder of equity claims.
Other changes in - Contributions from holders of equity claims, and distributions
economic to them.
resources and
- Exchanges of assets or liabilities that do not result in
claims
increases or decreases in equity.

© Emile Woolf International 375 The Institute of Chartered Accountants of Pakistan


Financial accounting and reporting I

2 QUALITATIVE CHARACTERISTICS OF USEFUL FINANCIAL INFORMATION

Section overview

 Information to be useful for decision making


 Fundamental qualitative characteristics
 Enhancing qualitative characteristics
 Cost constraint on useful information

2.1 Information to be useful for decision making


If financial information is to be useful, it must be relevant and faithfully represent what it purports
to represent. The usefulness of financial information is enhanced if it is comparable, verifiable,
timely and understandable.
It means information must have certain characteristics in order for it to be useful for decision
making. The IASB Conceptual Framework describes:

 fundamental qualitative characteristics; and


 enhancing qualitative characteristics

2.2 Fundamental qualitative characteristics


The fundamental qualitative characteristics are

 relevance; and
 faithful representation
Relevance
Relevant financial information is capable of making a difference in the decisions made by users.
Information may be capable of making a difference in a decision even if some users choose not to
take advantage of it or are already aware of it from other sources.
The relevance of information is affected by its materiality. Information is material if omitting it or
misstating it could influence decisions that users make on the basis of financial information about
a specific reporting entity.
Faithful representation (True and fair view)
Financial reports represent economic phenomena in words and numbers. To be useful, financial
information must not only represent relevant phenomena, but it must also faithfully represent the
substance of the phenomena that it purports to represent. Although, in many circumstances, the
substance of an economic phenomenon and its legal form are the same, an accountant should be
careful to identify when this might not be the case.
To be a perfectly faithful representation, a depiction would have three characteristics. It would be
complete, neutral and free from error. Of course, perfection is seldom, if ever, achievable. The
objective is to maximise those qualities to the extent possible.

2.3 Enhancing qualitative characteristics


The qualitative characteristics that enhance the usefulness of information that is relevant and a
faithful representation are:

 comparability;
 verifiability
 timeliness; and
 understandability

© Emile Woolf International 376 The Institute of Chartered Accountants of Pakistan


Chapter 9: Conceptual Framework for financial reporting

Comparability
Comparability enables users to identify and understand similarities in, and differences among,
items. Information about a reporting entity is more useful if it can be compared with similar
information about other entities and with similar information about the same entity for another
period or another date.
Consistency is related to comparability but is not the same. Consistency refers to the use of the
same methods for the same items, either from period to period within a reporting entity or in a
single period across entities. Consistency helps to achieve the goal of comparability.
Verifiability
This quality helps to assure users that information faithfully represents the economic phenomena
it purports to represent. Verifiability means that different knowledgeable and independent
observers could reach consensus that a particular depiction is a faithful representation. Quantified
information need not be a single point estimate to be verifiable. A range of possible amounts and
the related probabilities can also be verified.
Verification can be direct or indirect.

 Direct verification means verification through direct observation, e.g. by counting cash or
inventory.
 Indirect verification means checking the inputs to a model, formula or other technique and
recalculating the outputs using the same methodology. For example, the carrying amount of
inventory might be verified by checking the inputs (e.g. costs) and recalculating the closing
inventory using the same assumption (e.g. FIFO).
Timeliness
This means having information available to decision-makers in time to be capable of influencing
their decisions. Generally, the older the information is the less useful it is.
Understandability
Information is made understandable by classifying, characterising and presenting it in a clear and
concise manner. Some phenomena are inherently complex and cannot be made easy to
understand, however, excluding the relevant information is not justified in such circumstances.
Financial reports are prepared for users who have a reasonable knowledge of business and
economic activities and who review and analyse the information diligently.

2.4 Cost constraint on useful information


Cost is a pervasive constraint on the information that can be provided by financial reporting.
Reporting financial information imposes costs, and it is important that those costs are justified by
the benefits of reporting that information.
The benefits obtained from financial information should exceed the cost of obtaining and providing
it. Information should not be provided if the cost is not worth the benefit.

© Emile Woolf International 377 The Institute of Chartered Accountants of Pakistan


Financial accounting and reporting I

3 RECOGNITION

Section overview

 Recognition criteria
 Recognition links elements of financial statements

3.1 Recognition criteria


Only items that meet the definition of an asset, a liability or equity are recognised in the statement
of financial position. Similarly, only items that meet the definition of income or expenses are
recognised in the statement(s) of financial performance.
However, not all items that meet the definition of one of those elements are recognised. Not
recognising an item that meets the definition of one of the elements makes the statement of
financial position and the statement(s) of financial performance less complete and can exclude
useful information from financial statements. On the other hand, in some circumstances,
recognising some items that meet the definition of one of the elements would not provide useful
information.
An asset or liability is recognised only if recognition of that asset or liability and of any resulting
income, expenses or changes in equity provides users of financial statements with information that
is useful, i.e. with:

 relevant information about the asset or liability and about any resulting income, expenses or
changes in equity; and
 a faithful representation of the asset or liability and of any resulting income, expenses or
changes in equity.
Items that fail to meet the criteria for recognition should not be included in the financial statements.
However, some of these items may have to be disclosed as additional details in a note to the
financial statements.

3.2 Recognition links elements of financial statements

© Emile Woolf International 378 The Institute of Chartered Accountants of Pakistan


Chapter 9: Conceptual Framework for financial reporting

Example 01: CRITERIA FOR RECOGNITION


 A manufacturing unit valuing Rs. 5 million, owned and controlled by the Company
 A fleet of trucks valuing Rs 100 million, controlled by another company
 A highly skilled workforce, getting an annual compensation of Rs. 12.5 million
Required:
Which of the above assets will be recognized in the financial statements of a company in
accordance with the recognition criteria?
Answer:
 It will be recognised as an asset. It meets the definition of an asset being present economic
resource controlled by the entity.
 The fleet of truck will not be recognized because it is not controlled by the entity.
 Workforce will not be recognized by the entity because there is no control on as workers can
quit the entity at any time.
However, in case advance salaries have been paid, the entity has present right to future
services from the workforce.

Example 02:
Question: ABC received Rs. 160,000 in cash on 20 December 2004 from RM in return for having
provided financial advice during the 2004 financial year.
Required:
(a) Explain, with reference to the relevant definitions, which elements should possibly be
recognized in the 2004 financial year.
(b) Briefly identify whether and/ or how your answer would change if the cash received had
been received for financial advice to be provided in the 2005 financial year.
Answer:
Part (a)
The cash received meets the definition of an asset i.e. present resource now controlled by the entity
and entity may spend it as it may wish. Services have already been provided, therefore, there is no
obligation (no change in liability). Increase in equity shall be recognised as an income.
An asset and an income shall be recognised in year 2004.
Part (b)
The cash received meets the definition of an asset i.e. present resource now controlled by the entity
and entity may spend it as it may wish. Services have not been provided and there is present
obligation to provide services, resulting in increase in liability. No income can be recognised as
there is no equity increase.
An asset and a liability shall be recognised in year 2004.

Example 03:
Question: Read the following scenarios:
1. An amount paid to landlord totalling Rs.120,000 on 1st January 2012 against the rent for the
year ended 31st December 2012. Year end of the entity is 30 June 2012.
2. An expenditure incurred on repairs and maintenance of plant amounting Rs.300,000.
3. There has been legal dispute between the entity and its customer and company expects the
outflow of Rs. 200,000 in order to settle the dispute.
4. Entity purchased goods costing Rs. 20,000 for trading purposes and the same was sold for
Rs. 25,000.

© Emile Woolf International 379 The Institute of Chartered Accountants of Pakistan


Financial accounting and reporting I

Required:
Which of the above, would be recognized as expense &/or asset in the financial statements of a
company in accordance with the criteria given in conceptual framework.
Answer:
1. Increase in asset (advance rent: Future benefits) Rs. 60,000 and decrease in asset (Cash) Rs.
120,000 resulting in net decrease in equity is Rent expense (Rs. 60,000).
2. Decrease in asset (Cash) Rs. 300,000 and no increase in other assets (unless increase in
present resources) resulting in net decrease in equity is Repair expense (Rs. 300,000).
3. Increase in liability (obligation to settle) Rs. 200,000 and no increase in any assets resulting
in net decrease in equity is Expense (Rs. 200,000).
4. When purchased inventory, it was a present economic resource and recognised as an asset.
When sold, it becomes expense (cost of sales) due to decrease in assets resulting in decrease
in equity.

Example 04:
Question: Read the following scenarios
1. Advance received from customer amounting Rs. 50,000 against the goods to be delivered
after 6 months
2. Services provided to ABC and Co. on credit amounting Rs.30,000.
3. Account Receivables already written off in previous years amounting Rs. 30,000 were received
during the year.

Required:
Which of the above, would be recognized as income &/or liability in the financial statements of a
company in accordance with the criteria given in conceptual framework.

Answer:
1. Increase in asset (Cash) Rs. 50,000 and also an increase in liability (obligation to deliver) Rs.
50,000 and there is no income as no increase in equity.

2. Increase in asset (Right to receive) Rs. 30,000 and no increase in liability (services already
provided) and resulting net increase in equity Rs. 30,000 recognised as income.

3. Increase in asset (cash) Rs. 30,000 but no decrease in asset (RA was already written off)
resulting in Net increase in equity is Income.

© Emile Woolf International 380 The Institute of Chartered Accountants of Pakistan


Chapter 9: Conceptual Framework for financial reporting

4 MEASUREMENT

Section overview

 Measurement bases
 Historical cost
 Current value

4.1 Measurement bases


Elements recognised in financial statements are quantified in monetary terms. This requires the
selection of a measurement basis. A measurement basis is an identified feature, for example,
historical cost, fair value or fulfilment value, of an item being measured.
Applying a measurement basis to an asset or liability creates a measure for that asset or liability
and for related income and expenses. Consideration of the qualitative characteristics of useful
financial information and of the cost constraint is likely to result in the selection of different
measurement bases for different assets, liabilities, income and expenses.

4.2 Historical cost


Historical cost measures provide monetary information about assets, liabilities and related income
and expenses, using information derived, at least in part, from the price of the transaction or other
event that gave rise to them.
The historical cost of an asset when it is acquired or created is the value of the costs incurred in
acquiring or creating the asset, comprising the consideration paid to acquire or create the asset
plus transaction costs. The historical cost of a liability when it is incurred or taken on is the value
of the consideration received to incur or take on the liability minus transaction costs.
In some cases, a current value of the asset or liability is used as a deemed cost on initial recognition
and that deemed cost is then used as a starting point for subsequent measurement at historical
cost. Unlike current value, historical cost does not reflect changes in values, except to the extent
that those changes relate to impairment of an asset or a liability becoming onerous.
The historical cost of an asset is updated over time to depict, if applicable:
 the consumption of asset (depreciation or amortisation);
 payments received that extinguish part or all of the asset;
 the effect of events that cause asset to be no longer recoverable (impairment); and
 accrual of interest to reflect any financing component of the asset.
The historical cost of a liability is updated over time to depict, if applicable:

 fulfilment of part or all of the liability (payment);


 the effect of events that increase the value of the obligation (estimate change); and
 accrual of interest to reflect any financing component of the liability.
One way to apply a historical cost measurement basis to financial assets and financial liabilities is
to measure them at amortised cost. The amortised cost of a financial asset or financial liability
reflects estimates of future cash flows, discounted at a rate determined at initial recognition. For
variable rate instruments, the discount rate is updated to reflect changes in the variable rate. The
amortised cost of a financial asset or financial liability is updated over time to depict subsequent
changes, such as the accrual of interest, the impairment of a financial asset and receipts or
payments.

© Emile Woolf International 381 The Institute of Chartered Accountants of Pakistan


Financial accounting and reporting I

4.3 Current value


Current value measures provide monetary information about assets, liabilities and related income
and expenses, using information updated to reflect conditions at the measurement date. Because
of the updating, current values of assets and liabilities reflect changes, since the previous
measurement date, in estimates of cash flows and other factors reflected in those current values.
Unlike historical cost, the current value of an asset or liability is not derived, even in part, from the
price of the transaction or other event that gave rise to the asset or liability.
Current value measurement bases include:

 fair value
 value in use for assets and fulfilment value for liabilities
 current cost
Fair value
Fair value is the price that would be received to sell an asset, or paid to transfer a liability, in an
orderly transaction between market participants at the measurement date.
Fair value reflects the perspective of market participants—participants in a market to which the
entity has access. The same assumptions they would use when pricing the asset or liability while
acting in their economic best interest.
In some cases, fair value can be determined directly by observing prices in an active market. In
other cases, it is determined indirectly using measurement techniques, for example, cash-flow-
based measurement techniques, reflecting all the following factors:
 estimates of future cash flows.
 possible variations caused by the uncertainty inherent in the cash flows.
 the time value of money.
 the price for bearing the uncertainty inherent in the cash flows (a risk premium or risk
discount).
 other factors, for example, liquidity, if market participants would take those factors into
account in the circumstances.
The fair value is not increased or decreased by the transaction costs incurred when acquiring the
asset and when the liability is incurred or taken on. In addition, fair value does not reflect the
transaction costs that would be incurred on the ultimate disposal of the asset or on transferring or
settling the liability.
Value in use for assets and fulfilment value for liabilities
Value in use is the present value of the cash flows, or other economic benefits, that an entity
expects to derive from the use of an asset and from its ultimate disposal. Fulfilment value is the
present value of the cash, or other economic resources, that an entity expects to be obliged to
transfer as it fulfils a liability.
Those amounts of cash or other economic resources include not only the amounts to be transferred
to the liability counterparty, but also the amounts that the entity expects to be obliged to transfer to
other parties to enable it to fulfil the liability.
Value in use and fulfilment value do not include transaction costs incurred on acquiring an asset
or taking on a liability. However, value in use and fulfilment value include the present value of any
transaction costs an entity expects to incur on the ultimate disposal /fulfilment.
Value in use and fulfilment value reflect entity-specific assumptions rather than assumptions by
market participants. In practice, there may sometimes be little differences. Value in use and
fulfilment value cannot be observed directly and are determined using cash-flow-based
measurement techniques. Value in use and fulfilment value reflect the same factors described for
fair value, but from an entity-specific perspective rather than from a market-participant perspective.

© Emile Woolf International 382 The Institute of Chartered Accountants of Pakistan


Chapter 9: Conceptual Framework for financial reporting

Current cost
The current cost of an asset is the cost of an equivalent asset at the measurement date, comprising
the consideration that would be paid at the measurement date plus the transaction costs that would
be incurred at that date. The current cost of a liability is the consideration that would be received
for an equivalent liability at the measurement date minus the transaction costs that would be
incurred at that date.
Current cost, like historical cost, is an entry value: it reflects prices in the market in which the entity
would acquire the asset or would incur the liability. Hence, it is different from fair value, value in
use and fulfilment value, which are exit values. However, unlike historical cost, current cost reflects
conditions at the measurement date.
In some cases, current cost cannot be determined directly by observing prices in an active market
and must be determined indirectly by other means. For example, if prices are available only for
new assets, the current cost of a used asset might need to be estimated by adjusting the current
price of a new asset to reflect the current age and condition of the asset held by the entity.

Example 05: Measurement Bases


Question: Adeel Limited (AL) owns a machine which it purchased two years ago for Rs. 200,000.
The accumulated depreciation on the machine to date is Rs. 80,000 based on 5 years life using
straight line method.
The machine could be sold in the market for Rs. 100,000 but there would be dismantling costs of
Rs. 10,000.
The cash flows from the existing machine are estimated to be Rs. 50,000 for the next two years
followed by Rs. 40,000 in the last year. Relevant discount rate is 10%.
To replace the machine with a new version would cost Rs. 220,000.
Required:
Measure the machine using different measurement bases for AL using the above information.
Answer:
Historical cost Rs.
Cost 200,000
Less: Accumulated depreciation (80,000)
120,000

Fair value
The fair value is market value (exit price) of Rs. 100,000 without deducting cost to sell of Rs.
10,000.

Value in use Rs.


Year 1 Rs. 50,000 x 1.1-1 45,455
Year 2 Rs. 50,000 x 1.1-2 41,322
Year 3 Rs. 40,000 x 1.1-3 30,053
116,830

Current cost Rs.


Cost of new asset 220,000
Less: Accumulated depreciation* Rs. 220,000 / 5 x 2 years (88,000)
132,000
*The replacement cost is of new machine and needs to be adjusted for two years usage.

© Emile Woolf International 383 The Institute of Chartered Accountants of Pakistan


Financial accounting and reporting I

Example 06: Measurement Bases


Question: Briefly describe the measurement bases that may be used to measure the value of
assets in the financial statements.
Answer:
(i) Historical cost
The historical cost of an asset, when it is acquired or created is the value of the cost incurred
in acquiring or creating the asset, comprising the consideration paid to acquire or create the
asset plus transaction cost.

(ii) Current value


 Current value measures provide monetary information about assets using information
updated to reflect conditions at the measurement date.
 Current value measurement bases include:
– Fair value
– Value in use for assets
– Current cost

Fair value

Fair value is the price that would be received to sell an asset in an orderly transaction
between market participants at the measurement date. Fair value reflects the perspective
of market participants.

Value in use

Value in use is the present value of the cash flows or other economic benefit that an entity
expects to derive from the use of an asset and from its ultimate disposal. Value in use reflect
entity specific assumptions rather than assumptions by market participants.

Current cost

The current cost of an asset is the cost of an equivalent asset at the measurement date
comprising the consideration that would be paid at the measurement date plus the
transaction cost that would be incurred at that date.

Current cost, like historical cost is an entry value; while fair value is an exit value. However,
unlike historical cost, current cost reflects conditions at the measurement date.

© Emile Woolf International 384 The Institute of Chartered Accountants of Pakistan


Chapter 9: Conceptual Framework for financial reporting

5 CONCEPT OF CAPITAL AND CAPITAL MAINTENANCE

Section overview

 Concepts of capital
 Capital maintenance concepts and determination of profit
 Capital maintenance adjustments

5.1 Concept of capital


Financial concept of capital
Under a financial concept of capital, such as invested money or invested purchasing power, capital
is synonymous with the net assets or equity of the entity.
Physical concept of capital
Under a physical concept of capital, such as operating capability, capital is regarded as the
productive capacity of the entity based on, for example, units of output per day.
Consider the basic accounting equation.

Formula: Accounting equation


Assets = Liabilities + Equity Or Assets  Liabilities = Equity
A = L + E A  L = E
Net assets

Like any other equation, changes on one side of the accounting equation are matched by changes
in the other side. Therefore, Profit or loss for a period can be calculated from the difference between
the opening and closing net assets after adjusting for any distributions during the period.

Formula: Profit

Change in equity = Closing equity  Opening equity

Increase in equity = Profit + capital introduced  distributions

Profit = Increase in equity  capital introduced + distributions

This shows that the value ascribed to opening equity is crucial in the measurement of profit.

5.2 Capital maintenance concepts and determination of profit


Only inflows of assets in excess of amounts needed to maintain capital may be regarded as profit
and therefore as a return on capital. Hence, profit is the residual amount that remains after
expenses (including capital maintenance adjustments, where appropriate) have been deducted
from income.
Financial capital maintenance
Under this concept a profit is earned only if the financial (money) amount of the net assets at the
end of the period exceeds the net assets at the beginning of the period excluding any distributions
to, and contributions from, owners during the period.
Financial capital maintenance can be measured in either:
 nominal monetary units (also called Historical Cost Accounting); or
 units of constant purchasing power (also called Constant Purchasing Power Accounting).

© Emile Woolf International 385 The Institute of Chartered Accountants of Pakistan


Financial accounting and reporting I

Physical capital maintenance


Under this concept a profit is earned only if the physical productive capacity (or operating capability
of the entity or the resources or funds needed to achieve that capacity) at the end of the period
exceeds the net assets at the beginning of the period excluding any distributions to, and
contributions from, owners during the period.
This is also called Current Cost Accounting as the physical capital maintenance concept requires
the adoption of the current cost basis of measurement.

5.3 Capital maintenance adjustments


Financial capital maintenance (money terms)
Profit represents the increase in nominal money capital over the period. Thus, increases in the
prices of assets held over the period, i.e. holding gains, are, conceptually, profits. They may not
be recognised as such until disposal.
As mostly historical costs is used, no adjustments to profit is required.
Financial capital maintenance (real terms)
Profit represents the increase in invested purchasing power over the period. Thus, only that part
of the increase in the prices of assets that exceeds the increase in the general level of prices is
regarded as profit. The rest of the increase is treated as a capital maintenance adjustment and,
hence, as part of equity.

Illustration 01: Adjustment to maintain opening equity

Debit Credit

Statement of profit or loss X

Inflation reserve X

Physical capital maintenance


Profit represents the increase in physical capital over the period. All (specific) price changes
affecting the assets and liabilities of the entity are viewed as changes in the measurement of the
physical productive capacity of the entity; hence, they are treated as capital maintenance
adjustments that are part of equity and not as profit.

Illustration 02: Adjustment to maintain opening equity

Debit Credit

Statement of profit or loss X

Current cost reserve X

Illustration 03: Capital maintenance concepts


X Limited commenced business on 1 January with a single item of inventory which costs Rs.
10,000.
During the year it sold the item for Rs. 14,000 (cash).
During the year general inflation was 5% but the inflation specific to the item was 10%.
Profit is calculated under each concept in the following ways.

© Emile Woolf International 386 The Institute of Chartered Accountants of Pakistan


Chapter 9: Conceptual Framework for financial reporting

Capital maintenance concept


Financial Financial (real
(money terms) terms) Physical
Statement of profit or loss Rs. Rs. Rs.
Revenue 14,000 14,000 14,000
Cost of sale (10,000) (10,000) (10,000)
Inflation adjustment (inflation rate applied to opening equity):
5% Rs.10,000 (500)
10% Rs.10,000 (1,000)
4,000 3,500 3,000

Statement of financial position Rs. Rs. Rs.


Net assets 14,000 14,000 14,000
Equity:
Opening equity
Before adjustment 10,000 10,000 10,000
Inflation reserve (see above) 500 1,000
After adjustment 10,000 10,500 11,000
Retained profit (profit for the year) 4,000 3,500 3,000
14,000 14,000 14,000

Commentary on the above illustration


Under historical cost accounting, the profit is Rs. 4,000. If the business paid this out as a dividend
it would have Rs. 10,000 left.
Rs. 10,000 is the opening equity expressed as a number of units of currency. This means that the
company would have maintained its equity expressed as a number of units of currency. However,
inflation in the period has caused the purchasing power of the currency to decline. This means that
Rs. 10,000 no longer has the same purchasing power that it had a year ago. The company has not
maintained its capital in real terms.
To maintain its opening equity in real terms the company would have to ensure that it had the same
purchasing power at the year-end as it had at the start. Inflation was 5% so the company would
need Rs. 10,500 at the year-end in order to have the same purchasing power as it had at the start
of the year. The company can achieve this by transferring Rs.500 from profit and loss into an
inflation reserve. Profit would then be reported as Rs. 3,500.
If the business paid out Rs. 3,500 as a dividend it would have Rs. 10,500 left. This is not enough
to buy the same asset that it had at the start of the year. The asset has been subject to specific
inflation of 10% therefore the company would need Rs. 11,000 at the year-end in order to buy the
same asset.
This means that the company would not have the same capacity to operate as it had a year ago.
To maintain its opening equity in physical terms the company would have to ensure that it had the
same ability to operate at the year-end as it had at the start. In other words, it would need to have
Rs. 11,000. The company can achieve this by transferring Rs. 1,000 from profit and loss into an
inflation reserve. Profit would then be reported as Rs. 3,000.

© Emile Woolf International 387 The Institute of Chartered Accountants of Pakistan


Financial accounting and reporting I

Comparing the two concepts


Neither the IASB Conceptual Framework nor accounting standards require the use of a specific
capital maintenance concept. In practice, almost all entities use money financial capital
maintenance, but both concepts can provide useful information.
Financial capital maintenance is likely to be the most relevant to investors as they are interested
in maximizing the return on their investment and therefore its purchasing power.
Physical capital maintenance is likely to be most relevant to management and employees as they
are interested in assessing an entity’s ability to maintain its operating capacity. This is particularly
true for manufacturing businesses, where management may need information about the ability of
the business to continue to produce the same or a greater volume of goods.

Example 07: CARRIE


Question: Carrie starts in business on 1 January Year 1. Carrie’s sole shareholder contributed
capital of Rs. 1,000. Carrie purchased one item of inventory for Rs. 1,000 and sold that inventory
for cash of Rs. 1,400. At the end of Year 1 the replacement cost of the same item of inventory is
Rs. 1,100. General inflation during the year was 7%.
Required
Calculate the profit for the year and set out a summary statement of financial position as of 31
December Year 1 under the following capital maintenance concepts.
(a) Physical capital maintenance
(b) Financial capital maintenance
i. Historical cost accounting
ii. Constant purchasing power accounting
Answer:

(a) Physical (b) Financial Capital


Capital Maintenance
Maintenance
(ii) Constant
(i) Historical
purchasing
cost
power
accounting
Profit for the year accounting
Rs. Rs. Rs.
Sales 1,400 1,400 1,400
Cost of sales (1,000) (1,000) (1,000)
Inflation adjustment
- Specific (1,100 – 1,000) (100) - -
- General (1,000  7%) - - (70)
------------- ------------- -------------
Profit 300 400 330
------------- ------------- -------------
Balance sheet as at 31 December Year 1
Cash at bank 1,400 1,400 1,400
------------- ------------- -------------
Share capital (1,000 + 100) (1,000 + 70) 1,100* 1,000 1,070*
Reserves 300 400 330
------------- ------------- -------------
1,400 1,400 1,400
------------- ------------- -------------

© Emile Woolf International 388 The Institute of Chartered Accountants of Pakistan


Chapter 9: Conceptual Framework for financial reporting

Tutorial note
Share capital at the year end is restated under the physical capital maintenance concept for an
increase in specific price changes and under Constant Purchasing Power accounting for general
price changes. This is the other side of the entry to the inflation adjustments in the statement of
profit or loss

Example 08:
Question: Read the following statements:
A. In case of conflict between requirements of conceptual framework and IFRS, the requirements
of conceptual framework shall prevail.
B. Conceptual framework is not an International financial reporting standard (IFRS)
C. HR related cost is recognized as an asset in the financial statements since economic benefit
is probable from human resource
D. Internally generated goodwill is recognized as asset and measured at fair value in the financial
statements
E. When economic benefits arise over several accounting periods, and the association with
income can only be decided in broad terms, expenses should be recognized in profit and loss
of each accounting period on the basis of systematic and rational allocation procedure
F. When an item of expenditure is not expected to provide any future economic benefit, it is
recognized as an asset in the financial statements
G. In fair value method, assets are measured at the amount that would be paid to purchase the
same or a similar asset currently.

Required:
Analyse the above statements as true or false along with reasons for the selected answer.

Answer:
A. False. Nothing in the Conceptual Framework overrides any Standard or any requirement in a
Standard.
B. True. The Conceptual Framework is not a Standard. However, it provides foundation for
consistent development for IFRSs.
C. False.HR related cost can never be capitalized as it does not meet the definition criteria of
asset “controlled by the entity”
D. False. Internally generated goodwill is not recognised because its cost or value cannot be
measured reliably. IAS 38 specifically prohibits recognition of internally generated goodwill.
E. True, because of matching principle
F. False. Instead, an expense shall be recognised in that case.
G. False. This describes “current cost” which is entry value. “Fair value” is an exit value.

© Emile Woolf International 389 The Institute of Chartered Accountants of Pakistan


Financial accounting and reporting I

2 OBJECTIVE BASED QUESTIONS


01. Which of the following measurement basis is an ‘entry value’ and ‘reflects the conditions at the
measurement date’?
(a) Historical cost
(b) Fair value
(c) Value in use / fulfilment value
(d) Current cost

02. Financial capital maintenance (money terms) is also referred to as:


(a) Historical cost accounting
(b) Current cost accounting
(c) Constant purchasing power accounting
(d) Fair value accounting

03. Which of the following concepts measures profit in terms of an increase in the productive
capacity of an entity?
(a) Physical capital maintenance
(b) Historical cost accounting
(c) Financial capital maintenance
(d) Going concern concept

04. Which of the following statements is true about historical cost accounts in times of rising
prices?
(a) Profits will be overstated, and assets will be understated
(b) Asset values will be overstated
(c) Unrecognized gains will be recorded incorrectly
(d) Depreciation will be overstated

05. Which of the following measurement basis fulfils following two conditions when measuring an
asset or liability:
(i) Transactions costs at acquisition are ignored in valuation
(ii) Transaction costs at disposal or ultimate disposal are considered in valuation
(a) Historical cost
(b) Fair value
(c) Value in use / fulfilment value
(d) Current cost

© Emile Woolf International 390 The Institute of Chartered Accountants of Pakistan


Chapter 9: Conceptual Framework for financial reporting

06. Which of the following is NOT a purpose of the International Accounting Standards Board’s
Conceptual Framework?
(a) To assist the Board in the preparation and review of IFRS Standards.
(b) To assist auditors in forming an opinion on whether financial statements comply with
IFRS Standards.
(c) To assist in determining the treatment of items not covered by an existing IFRS
Standards.
(d) To be authoritative where a specific IFRS Standard conflicts with the Conceptual
Framework.

07. Which of the following items should be recognized as an asset in the statement of financial
position of an entity?
(a) A skilled and efficient workforce which has been very expensive to train. Some of
these staff is still employed by the entity.
(b) A highly lucrative contract signed during the year which is due to commence shortly
after the year-end.
(c) A government grant relating to the purchase of an item of plant several years ago
which has a remaining life of four years.
(d) A receivable from a customer which has been sold (factored) to a finance company.
The finance company has full recourse to the entity for any losses.

08. Which of the following criticisms does NOT apply to historical cost financial statements during
a period of rising prices?
(a) They contain mixed values, some items are at current values, some at out-of-date
values
(b) They are difficult to verify as transactions could have happened many years ago
(c) They understate assets and overstate profit
(d) They overstate gearing in the statement of financial position

09. Financial capital maintenance (real terms) is also referred to as:


(a) Historical cost accounting
(b) Current cost accounting
(c) Constant purchasing power accounting
(d) Fair value accounting

10. Physical capital maintenance is also referred to as:


(a) Historical cost accounting
(b) Current cost accounting
(c) Constant purchasing power accounting
(d) Fair value accounting

© Emile Woolf International 391 The Institute of Chartered Accountants of Pakistan


Financial accounting and reporting I

11. In which of the following, no adjustment for inflation is considered?


(a) Financial capital maintenance (money terms)
(b) Financial capital maintenance (real terms)
(c) Physical capital maintenance
(d) Fair value accounting

12. In which of the following, inflation adjustment is made on general rate of inflation?
(a) Financial capital maintenance (money terms)
(b) Financial capital maintenance (real terms)
(c) Physical capital maintenance
(d) Fair value accounting

13. In which of the following, inflation adjustment is made on specific rate of inflation?
(a) Financial capital maintenance (money terms)
(b) Financial capital maintenance (real terms)
(c) Physical capital maintenance
(d) Fair value accounting

14. Financial capital maintenance is likely to be most relevant to:


(a) Investors
(b) Management and employees
(c) Neither (a) nor (b)
(d) Capital maintenance is always irrelevant to decision making

15. Physical capital maintenance is likely to be most relevant to:


(a) Investors
(b) Management and employees
(c) Neither (a) nor (b)
(d) Capital maintenance is always irrelevant to decision making

16. An entity made a profit of Rs. 350,000 for the year 2019 based on historical cost accounting
principles. It had opening capital of Rs. 1,000,000.
Specific price indices increase during the year by 20% and general price indices by 5%.
How much profit should be recorded for 2019 under money financial capital maintenance
concept?
Rs. ___________

© Emile Woolf International 392 The Institute of Chartered Accountants of Pakistan


Chapter 9: Conceptual Framework for financial reporting

17. An entity made a profit of Rs. 350,000 for the year 2019 based on historical cost accounting
principles. It had opening capital of Rs. 1,000,000.
Specific price indices increase during the year by 20% and general price indices by 5%.
How much profit should be recorded for 2019 under real financial capital maintenance concept?
Rs. ___________

18. An entity made a profit of Rs. 350,000 for the year 2019 based on historical cost accounting
principles. It had opening capital of Rs. 1,000,000.
Specific price indices increase during the year by 20% and general price indices by 5%.
How much profit should be recorded for 2019 under physical capital maintenance concept?
Rs. ___________

19. An entity acquired an item of plant on 1 October 2012 at a cost of Rs. 500,000. It is being
depreciated over five years, using straight-line depreciation and an estimated residual value of
10% of its historical cost or current cost as appropriate. As at 30 September 2014, the
manufacturer of the plant still makes the same item of plant and its current price is Rs. 600,000.
What is the correct carrying amount to be shown in the statement of financial position as at 30
September 2014 under historical cost accounting?
Rs. ___________

20. An entity acquired an item of plant on 1 October 2012 at a cost of Rs. 500,000. It is being
depreciated over five years, using straight-line depreciation and an estimated residual value of
10% of its historical cost or current cost as appropriate. As at 30 September 2014, the
manufacturer of the plant still makes the same item of plant and its current price is Rs. 600,000.
What is the correct carrying amount to be shown in the statement of financial position as at 30
September 2014 under current cost accounting?
Rs. ___________

21. An entity made a profit of Rs. 480,000 for the year 2018 based on historical cost accounting
principles. It had opening capital of Rs. 1,100,000. During 2018, specific price indices increased
by 15% while general price indices increased by 12%. How much profit should be recorded for
2018 under real financial capital maintenance concept?
(a) Rs. 480,000
(b) Rs. 315,000
(c) Rs. 348,000
(d) Rs. 645,000

22. Which of the following statements is correct about financial statements based on historical cost
in times of rising prices?
(a) Profits will be overstated and assets will be understated
(b) Assets will be overstated
(c) Profits as well as assets will be understated
(d) Depreciation will be overstated

© Emile Woolf International 393 The Institute of Chartered Accountants of Pakistan


Financial accounting and reporting I

23. The IASB's Framework identifies qualitative characteristics.


(i) Relevance
(ii) Comparability
(iii) Verifiability
(iv) Understandability
(v) Faithful representation.
Which of the above are not listed as enhancing characteristics?
(a) (i), (iv) and (v)
(b) (ii), (iii) and (iv)
(c) (ii) and (iii)
(d) (i) and (v)

24. The IASB’s Conceptual Framework for Financial Reporting identifies qualitative characteristics
of financial statements.
Which TWO of the following characteristics are NOT fundamental qualitative characteristics
according to the IASB’s The Conceptual Framework for Financial Reporting?
(a) Relevance
(b) Reliability
(c) Faithful representation
(d) Comparability

© Emile Woolf International 394 The Institute of Chartered Accountants of Pakistan


Chapter 9: Conceptual Framework for financial reporting

2 OBJECTIVE BASED ANSWERS


01. (d) Historical cost and current cost both are entry values (unlike fair value and value
in use), however, historical costs reflects conditions on acquisition date and
current cost reflects conditions at measurement date.

02. (a)

03. (a) Physical capital maintenance looks at profit in terms of the physical productive
capacity of the business, taking into account specific price changes relevant to
the entity.

04. (a) In times of rising prices, asset values will be understated, as historical cost will
not be a true representation of the asset values. Additionally, the real purchase
cost of replacement items will not be incorporated, meaning that profits are
overstated.

05. (c) Value in use and fulfilment value do not include transaction costs incurred on
acquiring an asset or taking on a liability. However, value in use and fulfilment
value include the present value of any transaction costs an entity expects to
incur on the ultimate disposal /fulfilment.

06. (d) Where there is conflict between the conceptual framework and an IFRS
Standard, the IFRS Standard will prevail. An example of this is IAS 20
Government grants, where deferred grant income is held as a liability, despite
not satisfying the definition of a liability.

07. (d) As the receivable is ‘sold’ with recourse it must remain as an asset on the
statement of financial position and is not derecognized.

08. (b) Historical cost is the easiest to verify as the cost can be proved back to the
original transaction. Fair value is often more difficult to verify as it may involve
elements of estimation.

09. (c)

10. (b)

11. (a)

12. (b)

13. (c)

14. (a)

15. (b)

16. Rs. 350,000 Money financial capital maintenance looks at the actual physical cash. No
inflation adjustment is required.

17. Rs. 300,000 Rs. 350,000 – (1,000,000 x 5%) = Rs. 300,000

18. Rs. 150,000 Rs. 350,000 – (1,000,000 x 20%) = Rs. 150,000

© Emile Woolf International 395 The Institute of Chartered Accountants of Pakistan


Financial accounting and reporting I

19. Rs. 320,000 Historical cost annual depreciation = Rs. 90,000 ((500,000 × 90%)/5 years).
After two years carrying amount would be Rs. 320,000 = (500,000 -
(2×90,000)).

20. Rs. 384,000 Current cost annual depreciation = Rs. 108,000 ((600,000 × 90%)/5 years).
After two years carrying amount would be Rs. 384,000 = (600,000 -
(2×108,000)).

21. (c) Rs. 348,000

22. (a) Profits will be overstated and assets will be understated.

23. (d) Relevance and faithful representation are fundamental characteristics. Without
these characteristics, information cannot be useful.

24. (b) & (d) It is important to learn that the two fundamental characteristics are relevance
and faithful representation.

© Emile Woolf International 396 The Institute of Chartered Accountants of Pakistan


Certificate in Accounting and Finance

10

CHAPTER
Financial accounting and reporting I

Interpretation
of financial statements

Contents
1 Users of Financial statements and their information needs
2 Interpretation of financial statements by ratio analysis
3 Profitability ratios
4 Working capital efficiency ratios
5 Liquidity ratios
6 Debt ratios or long term solvency ratio
7 Financial Statement Analysis
8 Limitations of financial statements and ratio analysis
9 Objective based questions and answers

* The student must refer original handbook of IFRS.

© Emile Woolf International 397 The Institute of Chartered Accountants of Pakistan


Financial accounting and reporting I

1 USERS OF FINANCIAL STATEMENTS AND THEIR INFORMATION NEEDS


Section overview

 Users of the financial statements and their information needs

1.1 Users of the financial statements and their information needs


The IASB Conceptual Framework identifies several groups of people who may use financial
statements:
 Investors and potential investors
 Lenders
 Employees
 Suppliers
 Customers
 Government and government agencies
 The general public and
 The management
All these groups are interested in financial performance, financial position and cash flows. Some
users are mainly interested in performance and profitability, while others may be more interested
in liquidity and gearing or other matters.
For example:
 A private investor needs to know whether to continue to hold shares or to sell them. He or
she will tend to be most interested in profitability ratios (such as gross and net profit margin
and return on capital employed) and investor ratios (such as earnings per share, dividend
cover and price earnings ratio).
 A potential acquirer needs information about an entity’s profitability and probably also
information about whether or not the entity is managed efficiently. The acquirer’s
management is likely to focus on profit margins, return on capital employed, asset turnover
and working capital ratios.
 Employees are interested in fair wages, adequate fringe benefits and bonus linked with
productivity/profitability. Ratio analysis provides them adequate information regarding
efficiency and profitability of the entity.
 A bank that has been approached to lend money to an entity needs to know whether it will
receive interest payments when these are due and whether the money that it lends will
eventually be repaid. A bank manager will normally be most interested in cash flows and
liquidity ratios, (current ratio, acid test ratio) gearing and interest cover. A potential lender
will also be interested in predicting future performance as without sales there will be no cash.
 The management do financial forecasting, planning (of a future based on the past and
present) and controlling (budgets) as a decision maker.
An examination question might ask you to interpret an entity’s financial statements for the benefit
of specific people or groups of people. Therefore, your analysis should focus on the needs of the
users.
 What do they need to know?
 What are they interested in?
 What decision do they need to make?

© Emile Woolf International 398 The Institute of Chartered Accountants of Pakistan


Chapter 10: Interpretation of financial statements

2 INTERPRETATION OF FINANCIAL STATEMENTS WITH FINANCIAL RATIOS


Section overview

 Purpose and Importance of Financial Ratio Analysis


 Categories of Financial Ratios

Financial statements are used to make decisions. They are used by shareholders and investors,
and also by lenders, as well as by management. The financial statements contain a large number
of figures, but the figures themselves do not necessarily have much meaning to a user of the
financial statements. However, the figures can be analysed and interpreted by calculating financial
ratios.
Financial ratios can help the user of the financial statements to assess:
 the financial position of the entity, and
 its financial performance

2.1 Purpose and Importance of Financial Ratio Analysis


Financial ratio analysis helps a business in a number of ways. The importance and advantages of
financial ratios are given below:
 Ratios help in analysing the performance trends over a long period of time.
 They also help a business to compare the financial results to those of competitors.
 Ratios assist the management in decision making.
 They also point out problem and weak areas along with the strength areas.
 Ratios help to develop relationships between different financial statement items.
 Ratios have the advantage of controlling for differences in size. For example, two
businesses may be quite different in size but can be compared in terms of profitability,
liquidity, etc., by the use of ratios.

2.2 Categories of Financial Ratios


The main financial ratios can be classified as:
 Profitability Ratios Working capital ‘turnover’ ratios
 Liquidity ratios
 Debt ratios
 Investor ratios
The details of above ratios are given in next section

© Emile Woolf International 399 The Institute of Chartered Accountants of Pakistan


Financial accounting and reporting I

3 PROFITABILITY RATIO
Section overview

 Return on capital employed


 Return on shareholder capital (equity) or investment
 Return on assets
 Analysing return: profitability and asset utilisation
 Profit/sales ratio
 Cost / sales ratios
 Sales/capital employed ratio or assets turnover ratio
 Percentage annual growth in sales

3.1 Return on capital employed


Profit-making companies should try to make a profit that is large enough in relation to the amount
of money or capital invested in the business. The most important profitability ratio is probably
return on capital employed (ROCE) or return on investment (ROI).
For a single company:

Formula:

Profit before interest and taxation

ROCE = X 100%
(Share capital and reserves + long-term debt capital + preference
share capital)

The capital employed is the share capital and reserves, plus long-term debt capital such as bank
loans, bonds and loan stock.
Where possible, use the average capital employed during the year. This is usually the average of
the capital employed at the beginning of the year and end of the year.
A higher ROCE indicates more efficient use of capital. ROCE should be higher than the company’s
capital cost; otherwise it indicates that the company is not employing its capital effectively and is
not generating shareholder value.

Illustration 01:
Sting Company achieved the following results in Year 1.

1 January 31 December
Year 1 Year 1

Rs. Rs.

Share capital 200,000 200,000

Share premium 100,000 100,000

Retained earnings 500,000 600,000

Bank loans 200,000 500,000

© Emile Woolf International 400 The Institute of Chartered Accountants of Pakistan


Chapter 10: Interpretation of financial statements

Rs.

Profit before tax 210,000

Income tax expense 65,000

Profit after tax 145,000

Interest charges on bank loans were Rs.30, 000. Dividend payments to shareholders were Rs.45,
000. Sales during the year were Rs.5, 800,000.
Based on above Information the analysis and calculation are as under:
Capital employed at the beginning of the year = Rs.1, 000,000.
Capital employed at the end of the year = Rs.1, 400,000.
Average capital employed = [Rs.1, 000,000 + Rs.1, 400,000]/2 = Rs.1, 200,000.
Profit before interest and taxation = Rs.210, 000 + Rs.30, 000 = Rs.240, 000.
240,000
ROCE =  100% = 20%
1,200,000
Comment:
The 20% return on the capital employed indicates a return of Rs.20 on every 100 rupees invested
into the business. The ratio shows how efficiently the entity’s long term funds are being employed.
The investors are interested to invest in the company that has a higher ROCE than the other
available option(s). Besides the company’s return should always be higher than the cost at which
the funds were acquired. For example if a company borrows at 15% and achieves a return of only
10%, that says they are actually losing money.

ROCE is a useful measure of comparing profitability across competing entities based on the
amount of capital they use. It becomes more useful when the comparison is between capital-
intensive entities. Moreover, for a single company, the ROCE trend over the years is a significant
performance indicator. Generally speaking, the investors are more inclined to invest in the
companies that have stable and rising ROCE figures as compared to those where the ROCE is
volatile and inconsistent.

Illustration 02:
The two companies A and B operating in the similar lines of business for the year ender 31 st
December 2016. The following data is available.

Company A Company B

Rs. Rs.

Profit before Interest and Taxation 539,900 2,616,100

Capital employed 3,659,000 12,193,400

Based on above Information the analysis and calculation are as under:


Company A

539,900
ROCE =  100% = 14.8%
3,659,000
Company B

2,616,100
ROCE =  100% = 21.5%
12,193,400

© Emile Woolf International 401 The Institute of Chartered Accountants of Pakistan


Financial accounting and reporting I

Comment:
Based on the given figures, Company B appears to be utilising its capital better than Company A.
Company B can reinvest a greater portion of its profits back into the business operations to the
benefit of the shareholders. While we see that Company B’s ROCE is higher than that of Company
A yet there is not a lot to be attained from using data from one angle and at a single point of time.
Greater insight can be achieved if trends over time are analysed. As Company B though is
performing better than A yet it might have been facing a constant decline in the ROCE that may
point to a loss of competitive advantage.

3.2 Return on shareholder capital (equity) OR Investment


Return on shareholder capital (ROSC), or return on equity, measures the return on investment
that the shareholders of the company have made. This ratio normally uses the values of the
shareholders’ investment as shown in the statement of financial position (rather than market
values of the shares).

Formula:
Profit after taxation and preference dividend
ROSC = X 100%
Share capital and reserves

The average value of shareholder capital should be used if possible. This is the average of the
shareholder capital at the beginning and the end of the year.
Profit after tax is used as the most suitable measure of return for the shareholders, since this is a
measure of earnings (available for payment as dividends or for reinvestment in the business).

Illustration 03:
Using the figures in the previous illustration:
Shareholders’ capital at the beginning of the year = Rs. 200,000 + Rs. 100,000 + Rs. 500,000
= Rs. 800,000.
Shareholders’ capital at the end of the year= Rs. 200,000 + Rs. 100,000 + Rs. 600,000
= Rs. 900,000.
Average shareholders’ capital employed = [Rs. 800,000 + Rs. 900,000]/2 = Rs. 850,000.

145,000
ROSC =  100% = 17.06%
850,000
Comment:
The ROSC measures the ability of the entity to generate profits from the investments made by its
shareholders. The figure above shows a percentage of 17.06% meaning that the entity generates
a return of Rs.17 for every 100 rupees invested by the shareholder into the business. ROSC is the
indicator of effective management of equity financing.

3.3 Return on assets


Formula:
Profit before interest and taxation
ROA = X 100%
Assets

The normal convention is to use ‘total assets’ which includes both current and non-current
assets. However, other variations are sometimes used such as non-current assets only.

© Emile Woolf International 402 The Institute of Chartered Accountants of Pakistan


Chapter 10: Interpretation of financial statements

The return on assets ratio is a profitability ratio and measures the return produced by the total
assets. It helps both, the management and the investors, to know how well the entity can convert
its investment in assets into profits. The figures of ROA depend highly on the industry and hence
can vary substantially. This suggests that when ROA has to be used as a comparative measure
then the best practice is to compare it against a company’s previous ROA figures or the ROA of a
company in the similar business line.

Illustration 04:
A company’s chartered accountant is calculating ROA for the year 2016. The year-end figures of
total non- current assets and total current assets are Rs. 882,900 and Rs. 360,000 respectively.
The net profit for the year was Rs.685,000
Based on above Information the analysis and calculation are as under:
(a) Total assets

Calculating ROA using total assets

685,000
ROA =  100%= 55%
1,242,900

(b) Non-current assets

Calculating ROA using non-current assets

685,000
ROA =  100%= 77.6%
882,900

Comment:

This means that on average every single rupee invested in business’s assets generated 55 paisa in
profit.

If taken from the perspective of non-current assets only, they contribute to the
extent of 77.6%. That says for every 100 rupees invested in the non-current assets around 78
rupees are generated in profit.

The analysis would be more useful and meaningful when compared with the entity’s own
performance over the years and against the figures of the firm(s) competing in the similar industry.

3.4 Analysing return: profitability and asset utilisation


The size of the return on capital employed, or the size of the return on shareholders’ capital,
depends on two factors:
 The profitability of the goods or services that the entity has sold
 The volume of sales that the entity has achieved with the capital and assets it has
employed: this is known as asset utilisation or asset turnover.

3.5 Profit/sales ratio


The profit/sales ratio is the ratio of the profit that has been achieved for every Rs.1 of sales.

Formula:

Profit
Profit/sales ratio = X 100%
Sales

© Emile Woolf International 403 The Institute of Chartered Accountants of Pakistan


Financial accounting and reporting I

Profit/sales ratios are commonly used by management to assess financial performance, and a
variety of different figures for profit might be used. The definition of profit can be any of the
following:
 Profit before interest and tax
 Gross profit (= Sales minus the Cost of sales)
 Net profit (= Profit after tax)
It is important to be consistent in the definition of profit, when comparing performance from one
year to the next.
So there are 3 types of profit to sales ratio:
a) Profitability/Operating profit ratio
b) Gross profit margin ratio
c) Net profit ratio
The gross profit ratio is often useful for comparisons between companies in the same industry, or
for comparison with an industry average. It is also useful to compare the net profit ratio with the
gross profit ratio. A high gross profit ratio and a low net profit ratio indicate high overhead costs for
administrative expenses and selling and distribution costs.

Illustration 05:
Using the figures in the previous illustration, profit/sales ratios can be calculated as follows:
 If profit is defined as profit before interest and tax, the profit/sales ratio =
Rs.240,000/Rs.5,800,000 = 0.0414 = 4.14%
 If profit is defined as profit after interest and tax, the profit/sales ratio =
Rs.145,000/Rs.5,800,000 = 0.025 = 2.5%
Comment:
The figure suggests that Rs.4.14 is earned on every 100 rupees of sales before interest and tax
are deducted. After the deduction this figure becomes Rs.2.5 in the given scenario.
The profit to sales ratios show the percentage of sales that is left over after the business has paid
all its expenses. The ratio helps to determine how effectively a company’s sales are converted into
net income. Again the figures have to be compared with the industry averages and over the years
for the same company to arrive at a more meaningful conclusion.

Illustration 06:
Using the given figures the gross profit ratio of a company may be computed as:
Gross Profit Rs.235,000
Net Sales Rs.910,000
235,000
GP Margin=  100% = 26%
910,000
Comment:
The rounded off figure of GP margin is 26% that implies the company may reduce the selling price
of its products up to around 26% without incurring any loss. The GP ratio is an important ratio as it
evaluates the operational performance of the entity. Gross profit is an important figure for the
business, it should be sufficient enough to cover all the expenses and provide for the profit to the
investors.
In general, a higher ratio is a better ratio. The profitability of the business can be measured by
comparing it with the competing entities in the similar industry and with the past trend for the same
company. A consistent growth over the years indicates a sustainable continuous improvement in
the business’s processes and practices.

© Emile Woolf International 404 The Institute of Chartered Accountants of Pakistan


Chapter 10: Interpretation of financial statements

3.6 Cost/ sales ratios


It is also useful to monitor the ratio of different types of cost to sales. The following ratios can be
useful to highlight an unexpected change in a period or to indicate a difference between the
company and another in a similar industry:
 Ratio of (Cost of sales/Sales) × 100%
 Ratio of (Administration costs/Sales) × 100%
 Ratio of (Selling and distribution costs/Sales) × 100%

Illustration 07:
Following figures have been extracted from the Income Statement of Alpha ltd.

Rs.

Net Sales 650,000

Cost of sales 422,500

Administration costs 26,000

Selling and distribution costs 39,000

The cost of goods sales ratio, administration costs ratio and Selling and distribution costs ratio are
calculated as:
Cost of sales ratio
422,500
=  100% = 65%
650,000
Administration costs ratio
26,000
=  100% = 4%
650,000
Selling and distribution costs ratio
39,000
=  100% = 6%
650,000
Comment:

Costs ratios represent what extent of sales is an individual expense or a group of expenses. The
lower the ratio the better is the profitability status of the organisation. Care must be taken in dealing
with the variable expenses as they vary with the change in the sales volume. This ratio doesn’t
normally change significantly with the rise or decline in the sales volume. Whereas the ratios for
fixed expenses change significantly with the increase or decrease in the sales volume.

In the given scenario the cost of sales/sales ratio states that every 65 rupees out of 100 rupees of
sales represent cost of sales. These are the direct costs that vary with the level of sales.

Looking at the other two ratios we find that in this particular period every 4 rupees out of every 100
rupees of sales were spent on the administration costs and 6 rupees were expensed on selling and
distribution costs.

These ratios help the management in controlling and estimating future expenses.

© Emile Woolf International 405 The Institute of Chartered Accountants of Pakistan


Financial accounting and reporting I

3.7 Sales/capital employed ratio or Assets turnover ratio


The sales/capital employed ratio is also called the ‘asset turnover ratio’. It measures the amount
of sales achieved during the period for each Rs.1 of investment in assets.

Formula:

Sales
Asset turnover ratio = (Share capital and reserves + long-term debt capital + preference
shares)

It is measured as ‘x times a year’.


The sales/capital employed ratio is also a ratio of sales to (assets – current liabilities). This is
because capital employed = total assets minus liabilities excluding long-term debt.

Illustration 08:
Using the figures in the previous illustration, the asset turnover ratio = Rs.5,800,000/Rs.1,200,000
= 4.83 times.
Note that:
ROCE = Profit/sales ratio × Asset turnover ratio
(Where profit is defined as profit before interest and taxation).
Using the figures in the previous illustration:

ROCE = Profit/sales × Sales/capital employed


240,000 240,000 5,800,000
= ×
1,200,000 5,800,000 1,200,000
20% = 4.14% × 4.83 times

Comment:
The Sales/Capital employed ratio measures how efficiently an organisation’s assets generate
revenues. The figure in the solution says that every single rupee of the capital employed in the
business is generating revenue of Rs.4.83. It must also be taken into account that the age of a
company’s assets can heavily impact hence result in different asset turnover ratios for similar
companies. For example a company having older assets with lower book values might have a higher
asset turnover ratio than the one with the similar revenues but newer, higher net book value assets.
A constantly declining assets turnover ratio or a lower ratio as compared to the industry averages
might indicate towards the issues related to the excess production capacity, poor inventory
management, and sloppy collection methods etc. The higher the ratio the better it is considered
yet capital investment for purchasing assets in anticipation of future growth or sale of existing
unnecessary assets for an anticipated decline in future can suddenly and may be artificially change
the company’s assets turnover ratio. Besides companies in the capital-intensive industries tend to
have a lower assets turnover ratio than the ones operating with fewer assets. Therefore for a more
meaningful analysis, the companies should be compared within the same industry.

3.8 Percentage annual growth in sales


It can be useful to measure the annual growth (or decline) in sales, measured as a percentage of
sales in the previous year.
For example, if sales in the year just ended were Rs.5,800,000 and sales in the previous year
were Rs.5,500,000, the annual growth in sales has been (Rs.300,000/Rs.5,500,000) × 100% =
5.45%.

© Emile Woolf International 406 The Institute of Chartered Accountants of Pakistan


Chapter 10: Interpretation of financial statements

4 WORKING CAPITAL EFFICIENCY RATIOS

Section overview

 Purpose of working capital efficiency ratios


 Average time to collect (receivables days or days sales outstanding)
 Average time for holding inventory
 Average time to pay suppliers
 Cash operating cycle/working capital cycle
 Turnover (multiples) ratios

4.1 Purpose of working capital efficiency ratios


Working capital efficiency ratios measure the efficiency with which the entity has managed its
receivables, inventory and trade payables. The ratios are usually measured in terms of an average
number of days.
The working capital ratios are a useful measure of whether the entity has too much or too little
invested in working capital.
Excessive investment in working capital is indicated by a long cash cycle (a long working capital
cycle) that appears to be getting even longer. When too much is invested in working capital, the
return on capital employed and ROSC will be lower than they should be.
Under-investment in working capital is an indication of possible liquidity difficulties. When
working capital is low in comparison with the industry average, this might indicate that current
assets are being financed to an excessive extent by current liabilities, particularly trade payables
and a bank overdraft.
(The cash cycle, also called the operating cycle and the working capital cycle, is explained later).

4.2 Average time to collect (receivables days or days sales outstanding)


This ratio estimates the time that it takes on average to collect the payment from customers after
the sale has been made. It could be described as the average credit period allowed to customers
or the ‘average collection period’.

Formula:
Trade receivables
Average days to collect = X 365 days
Sales

Trade receivables should be the average value of receivables during the year. This is the average
of the receivables at the beginning of the year and the receivables at the end of the year. However,
the value for receivables at the end of the year is also commonly used.
Sales are usually taken as total sales for the year. However, if sales are analysed into credit sales
and cash sales, it is probably more appropriate to use the figure for credit sales only.
The average time to collect money from credit customers should not be too long. A long average
time to collect suggests inefficient collection of amounts due from receivables.

4.3 Average time for holding inventory


This ratio is an estimate of the average time that inventory is held before it is used or sold.

Formula:
Inventory
Inventory holding period = X 365 days
Cost of sales

© Emile Woolf International 407 The Institute of Chartered Accountants of Pakistan


Financial accounting and reporting I

In theory, inventory should be the average value of inventory during the year. This is the average
of the inventory at the beginning of the year and the inventory at the end of the year.
However, the value for inventory at the end of the year is also commonly used, particularly in
examinations.

4.4 Average time to pay suppliers


The average time to pay suppliers may be calculated as follows:

Formula:
Trade payables
Average time to pay = X 365 days
Cost of purchases

Trade payables should be the average value of trade payables during the year. This is the average
of the trade payables at the beginning of the year and the trade payables at the end of the year.
However, the value for trade payables at the end of the year is also commonly used. When the
cost of purchases is not available, the cost of sales should be used instead. This figure is obtained
from the profit and loss information in the statement of profit or loss and other comprehensive
income.

Illustration 09:
The following information is available for The Brush Company for Year 1.

1 January Year 1 31 December Year 1


Rs. Rs.
Inventory 300,000 360,000
Trade receivables 400,000 470,000
Trade payables 150,000 180,000

Sales in Year 1 totalled Rs.3, 000,000 and the cost of sales was Rs.1, 800,000
Based on above the analysis and calculation are as under:

Average inventory = [Rs.300,000 + Rs.360,000]/2 = Rs.330,000


Average trade receivables = [Rs.400,000 + Rs.470,000]/2 = Rs.435,000
Average trade payables = [Rs.150, 000 + Rs.180, 000]/2 = Rs.165, 000.
Turnover ratios
Average days to collect = [435,000/3,000,000] × 365 days = 52.9 days
Inventory turnover period = [330,000/1,800,000] × 365 days = 66.9 days
Average time to pay = [165,000/1,800,000] × 365 days = 33.5 days.
Comment:
The relationship among the three ratios given above is quite critical. The sale of inventory generates
receivables, that when collected are used to settle creditors and the cycle goes on. The firm’s ability
to get the cycle repeated continuously depends heavily on the entity’s short-term liquidity and the
cash generating ability. In the given scenario the firm is responsible to settle its payables at least
around 19 days before it receives payments from its customers. The working capital need then
would be financed through some bank operating line and that would bring a cost which in turn
would reduce the profitability of the organisation. The credit policy therefore needs to get revised
and improved and sloppy collections to be gotten rid of. Also the agreement with the supplier might
be refreshed for the purpose of extending the credit period.

© Emile Woolf International 408 The Institute of Chartered Accountants of Pakistan


Chapter 10: Interpretation of financial statements

4.5 Cash operating cycle/working capital cycle


The cash operating cycle or working capital cycle is the average time of one cycle of business
operations:
 From the time that suppliers are paid for the resources they supply
 To the time that cash is received from customers for the goods (or services) that the entity
makes (or provides) with those resources and then sells.
A cash cycle or operating cycle is measured as follows. Figures are included for the purpose of
illustration:

Days Days
Inventory turnover A 40.2
Average days to collect B 88.2
–––––
128.4
Average time to pay (C) (33.5)
––––––––– –––––
Cash cycle/operating cycle A+B–C 94.9
––––––––– –––––
The working capital ratios and the length of the cash cycle should be monitored over time. The
cycle should not be allowed to become unreasonable in length, with a risk of over-investment or
under-investment in working capital.
A positive working capital cycle balances incoming and outgoing payments to minimise net working
capital and maximise free cash flow. For example, a company that pays its suppliers in 30 days
but takes 60 days to collect its receivables has a working capital cycle of 30 days. This 30 day
cycle usually needs to be funded through a bank operating line, and the interest on this financing
is a carrying cost that reduces the company's profitability.
Growing businesses require cash, and being able to free up cash by shortening the working capital
cycle is the most inexpensive way to grow. Sophisticated buyers review closely a target's working
capital cycle because it provides them with an idea of the management's effectiveness at managing
their balance sheet and generating free cash flow.
In a manufacturing organization there are 3 types of inventory that are:
a) Raw material inventory
b) Work in process inventory
c) Finished goods inventory
So inventory turnover ratio is analysed as:
Raw material inventory holding period

Formula:

Raw material Inventory


Raw material Inventory holding period = X 365 days
Material consumed

Work in process inventory holding period

Formula:
Work in process Inventory
Work in Porcess Inventory holding period = X 365 days
Cost of goods manufactured

© Emile Woolf International 409 The Institute of Chartered Accountants of Pakistan


Financial accounting and reporting I

Finished goods inventory holding period

Formula:
Finished goods Inventory
Finished goods Inventory X 365 days
holding period = Cost of goods sold

4.6 Turnover (multiples) ratios


Inventory turnover

Formula:
Cost of sales
Inventory turnover = times
Average inventory

Receivables (debtor) turnover

Formula:
Credit sales
Receivables turnover = times
Average trade receivables

Payables (creditor) turnover

Formula:
Credit purchases times
Payables turnover =
Average trade payables

© Emile Woolf International 410 The Institute of Chartered Accountants of Pakistan


Chapter 10: Interpretation of financial statements

5 LIQUIDITY RATIOS
Section overview

 The meaning of liquidity


 Current ratio
 Quick ratio or acid test ratio

5.1 The meaning of liquidity


Liquidity means having cash or access to cash readily available to meet obligations to make
payments.
For the purpose of ratio analysis, liquidity is measured on the assumption that the only sources of
cash available are:
 Cash in hand or in the bank, plus
 Current assets that will soon be converted into cash during the normal cycle of trade.
It is also assumed that the only immediate payment obligations faced by the entity are its current
liabilities.
There are two ratios for measuring liquidity:
 Current ratio
 Quick ratio, also called the acid test ratio.
The more suitable ratio for use depends on whether inventory is considered a liquid asset that will
soon be used or sold, and converted into cash from sales.

5.2 Current ratio


The current ratio is the ratio of current assets to current liabilities.
Formula:
Current assets
Current ratio =
Current liabilities

The amounts of current assets and current liabilities in the statement of financial position at the
end of the year may be used. It is not necessary to use average values for the year.
It is sometimes suggested that there is an ‘ideal’ current ratio of 2.0 times (2:1).
However, this is not necessarily true and in some industries, much lower current ratios are
normal. It is important to assess the liquidity ratios by considering:
 Changes in the ratio over time
 The liquidity ratios of other companies in the same period
 The industry average ratios.
Liquidity should be monitored by looking at changes in the ratio over time.

Illustration 10:
The following information is available for X ltd. for Year 1. The current ratio can be calculated and
interpret as:
Current Assets Rs.1,100,000
Current Liabilities Rs.400,000
1,100,000
Current Ratio = = 2.75 times
400,000

© Emile Woolf International 411 The Institute of Chartered Accountants of Pakistan


Financial accounting and reporting I

Comment
A current ratio of 2:1 or higher is considered satisfactory for most of the entities yet the analysts
should be very careful at interpreting it. A simple calculation of current ratio might not disclose the
exact liquidity position of the company. A deeper analysis into the individual items of current assets
and liabilities would add value to the results. A higher current ratio might not indicate the ability to
pay off the entity’s current obligations efficiently as a huge portion of current assets may comprise
of needless, obsolete and/or slow moving inventory items.

5.3 Quick ratio or acid test ratio


The quick ratio or acid test ratio is the ratio of current assets excluding inventory to current
liabilities. Inventory is excluded from current assets on the assumption that it is not a very liquid
item.

Formula:
Current assets excluding inventory
Quick ratio =
Current liabilities

The amounts of current assets and current liabilities in the statement of financial position at the
end of the year may be used. It is not necessary to use average values for the year.
This ratio is a better measurement of liquidity than the current ratio when inventory turnover times
are very slow, and inventory is not a liquid asset.
It is sometimes suggested that there is an ‘ideal’ quick ratio of 1.0 times (1:1).
However, this is not necessarily true and in some industries, much lower quick ratios are normal.
As indicated earlier, it is important to assess liquidity by looking at changes in the ratio over time
and comparisons with other companies and the industry norm.

Illustration 11:
Kashif’s Clothing Store has applied for a loan to remodel the shop front. The bank has asked him
for a detailed balance sheet, so it can compute the quick ratio. Kashif's balance sheet includes the
following figures:
Cash: Rs.20,000
Accounts Receivable: Rs.10,000
Inventory: Rs.5,000
Stock Investments: Rs.2,000
Prepaid taxes: Rs.500
Current Liabilities: Rs.30,000
The bank can compute Kashif's quick ratio like this.
20,000  10,000  2,000
Quick Ratio = = 1.07 times
30,000

Comment
The Quick ratio of Kashif’s Clothing store turns out to be 1.07 times that says Kashif can pay off all
his current liabilities with liquid assets and can still have some quick assets left over. The Acid-test
ratio gives a more rigorous assessment of the company’s ability to pay off its current liabilities as
it considers only highly liquid assets. Had it been below 1 it would have represented the company
as an overly leveraged company that is struggling to; maintain or increase sales, settling its
creditors quickly, or/and collecting receivables on time.

© Emile Woolf International 412 The Institute of Chartered Accountants of Pakistan


Chapter 10: Interpretation of financial statements

6 DEBT RATIOS OR LONG TERM SOLVENCY RATIO


Section overview

 Gearing (debt to equity) ratio


 Interest cover ratio

Debt ratios are used to assess whether the total debts of the entity are within control and are not
excessive.
6.1 Gearing (debt to equity) ratio
Gearing, also called leverage, measures the total long-term debt of a company as a percentage
of either:
 The equity capital in the company, or
 The total capital of the company

Formula:
Long-term debt
Gearing = X 100%
Share capital and reserves

Alternatively:
Formula:
Long-term debt
Gearing = X 100%
Share capital and reserves + Long-term debt

It is usually appropriate to use the figures from the statement of financial position at the end of the
year. However, a gearing ratio can also be calculated from average values for the year.
When there are redeemable preference shares it is usual to include them within debt capital. This
is because redeemable preference shares behave more like a long-term loan or bond with fixed
annual interest followed by future redemption.
Irredeemable preference shares behave more like Equity (as they are never redeemed) and should
therefore be treated as equity.
A company is said to be high-geared or highly-leveraged when its debt capital exceeds its share
capital and reserves. This means that a company is high-geared when the gearing ratio is above
either 50% or 100%, depending on which method is used to calculate the ratio.
A company is said to be low-geared when the amount of its debt capital is less than its share
capital and reserves. This means that a company is low-geared when the gearing ratio is less than
either 50% or 100%, depending on which method is used to calculate the ratio.
A high level of gearing may indicate the following:
 The entity has a high level of debt, which means that it might be difficult for the entity to
borrow more when it needs to raise new capital.
 High gearing can indicate a risk that the entity will be unable to meet its payment
obligations to lenders, when these obligations are due for payment.
The gearing ratio can be used to monitor changes in the amount of debt of a company over time.
It can also be used to make comparisons with the gearing levels of other, similar companies, to
judge whether the company has too much debt, or perhaps too little, in its capital structure.
6.2 Interest Cover Ratio
Formula:
Interest cover Profit before interest and tax
= Times
ratio = Interest payable

© Emile Woolf International 413 The Institute of Chartered Accountants of Pakistan


Financial accounting and reporting I

7 FINANCIAL STATEMENTS ANALYSIS


Section overview

 Overview
 Horizontal analysis
 Vertical analysis
 Impact of specific transactions on ratios

7.1 Overview
Financial statement analysis is the process of analysing a company's past, current and projected
performance for decision-making purposes
Financial statement analysis allows analysts to identify trends by comparing ratios across multiple
periods and statement types to allow analysts to measure liquidity, profitability, company-wide
efficiency, and cash flow.
Financial statement analysis is of the following types:
 Horizontal analysis
 Vertical analysis
 Ratio analysis (already explained in above sections)

7.2 Horizontal analysis


Horizontal analysis is used to compare historical data, such as ratios, or line items, over a number
of accounting periods.
Financial analysts and investors need to identify trends and growth patterns in the company’s
performance over a number of years, a year-end balance sheet or income statement is not enough
to evaluate whether the company is operating efficiently and profitably.
Horizontal analysis also makes it easier to compare growth rates and profitability among different
companies.
The following is the formula for horizontal analysis:
Amount in comparison year – Amount in base year x 100
Base year
The following figure is an example of how to prepare a horizontal analysis for two years.
Carnations Ltd
Profit & Loss Account
For the year ended December 31, 2018

%age change
2018 2017
from 2017 to 2018
Rs. in millions
Sales 86,320 75,200 14.79
Cost of Sales (44,618) (40,900) 9.09
Gross Profit 41,702 34,300 21.58
Distribution costs (19,597) (15,380) 27.42
Administrative expenses (2,339) (2,053) 13.93

© Emile Woolf International 414 The Institute of Chartered Accountants of Pakistan


Chapter 10: Interpretation of financial statements

%age change
2018 2017
from 2017 to 2018
Rs. in millions
Other operating expenses (1,322) (1,052) 25.67
Other income 1,488 1,000 48.80
Profit before interest 19,932 16,815 18.54
Finance cost (343) (300) 14.33
Profit before taxation 19,589 16,515 18.61

7.3 Vertical analysis


In vertical analysis each category of accounts on the balance sheet is shown as a percentage of
the total account.
Line items on an income statement can be stated as a percentage of gross sales, while line items
on a balance sheet can be stated as a percentage of total assets or liabilities. This analysis of
income statements gives the company a heads up if cost of goods sold or any other expense
appears to be too high when compared to sales and allows the management to identify the reasons
and take action to fix the problem(s).
Carnations Ltd
Statement of Financial Position
For the year ended December 31, 2018

2018 2017
------------------ Rs. in millions ------------------
Assets
Non-Current Assets
Property, plant & equipment 15,000 42.33% 12,000 32.71%
Intangibles 500 1.41% 600 1.64%
Long term investments 120 0.34% 100 0.27%
Long term loans 200 0.56% 150 0.41%
Long term deposits and
prepayments 70 0.20% 180 0.49%
15,890 44.84% 13,030 35.51%
Current Assets
Stores and spares 650 1.83% 585 1.59%
Stock in trade 6,000 16.93% 5,500 14.99%
Trade debts 2500 7.05% 1200 3.27%
Loans and advances 800 2.26% 300 0.82%
Short term deposits and 750 900
2.12%
prepayments 2.45%
Other receivables 350 0.99% 175 0.48%
Cash and bank balances 8,500 23.98% 15,000 40.88%
19,550 55.16% 23,660 64.49%
Total assets 35,440 36,690

© Emile Woolf International 415 The Institute of Chartered Accountants of Pakistan


Financial accounting and reporting I

2018 2017

------------------ Rs. in millions ------------------

Equity and liabilities

Share capital and reserves

Share capital 1,000 2.82% 1,000 2.73%


Reserves 2,950 8.32% 7,095 19.34%

Liabilities

Non-current liabilities

Staff retirement benefits 290 0.82% 295 0.80%

Current liabilities

Trade and other payables 30,000 84.65% 27,500 74.95%

Provisions 1200 3.39% 800 2.18%


Total current liabilities 31,200 88.04% 28,300 77.13%

Total liabilities 31,490 88.85% 28,595 77.94%

Total equity and liabilities 35,440 36,690

Carnations Ltd
Profit & Loss Account
For the year ended December 31, 2018

2018 2017

------------------ Rs. in millions ------------------

Sales 86,320 100% 75,200 100%

Cost of Sales (44,618) 51.69% (40,900) 54.39%

Gross Profit 41,702 48.31% 34,300 45.61%

Distribution costs (19,597) 22.70% (15,380) 20.45%

Administrative expenses (2,339) 2.71% (2,053) 2.73%

Other operating expenses (1,322) 1.53% (1,052) 1.40%

Other income 1,488 1.72% 1,000 1.33%

Profit before interest 19,932 23.09% 16,815 22.36%

Finance cost (343) 0.40% (300) 0.40%

Profit before taxation 19,589 22.69% 16,515 21.96%

© Emile Woolf International 416 The Institute of Chartered Accountants of Pakistan


Chapter 10: Interpretation of financial statements

7.4 Impact of specific transactions on ratios


It is important to understand how certain future transactions or events might affect already
calculated ratios, for example, an entity may be required to certain current ratio of at least 1.75 as
part of its running finance agreement with the bank.
The following mathematical rules are useful in most circumstances:
 Increase in numerator will increase the measure of ratio
 Decrease in numerator will decrease the measure of ratio
 Increase in denominator will decrease the measure of ratio
 Decrease in denominator will increase the measure of ratio
 Increase in numerator and denominator by equal amount will increase the lower side of
fraction bar, more proportionately.
 Decrease in numerator and denominator by equal amount will decrease the lower side of
fraction bar, more proportionately.

Example 01: Epivac Limited


Question: Epivac Limited is considering to take some of the following measures during the last week
of the year ending 31 March 2021 in order to show better financial performance:
(i) Pay balance of a major supplier from bank overdraft facility and avail 5% discount.
(ii) Sell slow moving stock items at a price equal to cost.
(iii) Recover debtors’ balances by offering cash discounts of 10%.
(iv) Offer extended credit terms of 90 days which would increase sales at existing margins.
(v) Dispose-off some non-current assets at gain.
Required: State the effect (increase, decrease, no effect) of each of the above measure on the
following financial ratios:
(a) Gross profit %
(b) Net profit %
(c) Current ratio
(d) Stock turnover (times)
(e) Return on non-current assets
(f) Quick ratio
Answer:

S. Measures
Ratios
No.
(i) (ii) (iii) (iv) (v)
Gross profit %
(a) Increase* Decrease Decrease* No effect No effect
Gross Profit / Sales
Net profit %
(b) Increase Decrease Decrease Increase Increase
PAT / Sales
Current ratio
(c) Increase No effect Decrease Increase Increase
Current assets / Current liab.
Stock turnover (times)
(d) Decrease* Increase No effect Increase No effect
Cost of Sales / Inventory
Return on non-current assets
(e) Increase No effect Decrease Increase Increase
PBIT / Non-current assets
Quick ratio
(f) Increase Increase Decrease Increase Increase
(RA + Cash) / Current liab.
*The settlement discount is deducted from revenue or cost of purchases.

© Emile Woolf International 417 The Institute of Chartered Accountants of Pakistan


Financial accounting and reporting I

8 LIMITATIONS OF FINANCIAL STATEMENTS AND RATIO ANALYSIS


Section overview

 Limitations of financial statements and ratio analysis


 Window dressing

8.1 Limitations of financial statements and ratio analysis


Financial statements are time and cost producing. Ratio analysis can be used to compare
information taken from the financial statements to gain an analytical understanding of the results,
financial position and cash flows of a business. This analysis is a useful tool, especially for an
outsider such as a supplier, lender or an investor. However, there are a number of limitations of
ratio analysis which are given below:
Historical
All of the information used in ratio analysis is derived from actual historical results. This does not
mean that the same results will carry forward into the future. However, you can use ratio analysis
on pro forma information and compare it to historical results for consistency.
Historical versus current cost
The information on the income statement is stated in current costs (or close to it), whereas many
elements of the balance sheet are stated at historical cost (which could vary substantially from
current costs). This disparity can result in unusual ratio results.
Inflationary effect
If the rate of inflation has changed in any of the periods under review, this can mean that the
numbers are not comparable across periods. For example, if the inflation rate was 100% in one
year, sales would appear to have doubled over the preceding year, when in fact sales did not
change at all.
Aggregation
The information in a financial statement line item that you are using for a ratio analysis may have
been aggregated differently in the past, so that running the ratio analysis on a trend line does not
compare the same information through the entire trend period.
Accounting policies and estimates
Different companies in a similar industry may have different policies for recording the
same accounting transaction. This means that comparing the ratio results of different companies
may be like comparing apples and oranges. For example, one company might use reducing
balance method while another company uses straight-line depreciation.
Business conditions
You need to place ratio analysis in the context of the general business environment. For example,
60 days of sales outstanding for receivables might be considered poor in a period of rapidly
growing sales, but might be excellent during an economic contraction when customers are in
severe financial condition and unable to pay their bills.
Interpretation
It can be quite difficult to ascertain the reason for the results of a ratio. For example, an acid test
ratio of 2:1 might appear to be excellent, until you realize that the company just sold a large amount
of its stock to bolster its cash position. A more detailed analysis might reveal that the acid test ratio
will only temporarily be at that level, and will probably decline in the near future.
Company strategy
It can be difficult to interpret a ratio analysis comparison between two companies that are pursuing
different strategies. For example, one company may be following a low-cost strategy, and so is
willing to accept a lower gross margin in exchange for more market share. Conversely, a company
in the same industry is focusing on a high customer service strategy where its prices are higher
and gross margins are higher, but it will never attain the revenue levels of the first company.

© Emile Woolf International 418 The Institute of Chartered Accountants of Pakistan


Chapter 10: Interpretation of financial statements

8.2 Window dressing


One should also be careful of unethical practices like window dressing while interpreting the
financial statements.

Window dressing is the adaptation of the rules and practices to present financial statements in a
way that business situation appears better than it actually is. This manipulates the financial
information and misleads the users of financial statements.
Examples
Some of the ways in financial statements may be manipulated include:
 Delay in paying suppliers, so that the period-end cash balance appears higher.
 Using lower estimate for allowance for doubtful debts.
 Capitalize smaller expenditures that would normally be charged to expense, to increase
reported profits.
 Offer customers an early shipment discount, thereby accelerating revenues from a future
period into the current period.
 Lower depreciation expense by using higher useful lives or residual values, etc.

Example 02: Wasim Pvt. Ltd


Question: Wasim Pvt. Ltd. is an importer and retailer of vegetable oils. Extracts from the financial
statements for this year and last are set out below:
Income statements for the years ended 30 September
Year 7 Year 6
Rs.000 Rs.000
Revenue 2,160 1,806
Cost of sales (1,755) (1,444)
----------------- -----------------
Gross profit 405 362
Distribution costs (130) (108)
Administrative expenses (260) (198)
----------------- -----------------
Profit before tax 15 56
Income tax expense (6) (3)
----------------- -----------------
Profit for the period 9 53
----------------- -----------------

Statements of financial position as of 30 September


Year 7 Year 6
Rs.‘000 Rs.‘000
Assets
Non-current assets
Property, plant and equipment 78 72

Current assets
Inventories 106 61
Trade receivables 316 198
Cash - 6
----------------- -----------------
422 265
----------------- -----------------
Total assets 500 337
----------------- -----------------

© Emile Woolf International 419 The Institute of Chartered Accountants of Pakistan


Financial accounting and reporting I

Equity and liabilities


Equity
Ordinaryshares 110 85
Preference shares 23 11
Share premium 15 -
Revaluation reserve 20 20
Retained earnings 78 74
----------------- -----------------
246 190
Current liabilities
Bank overdraft 49 -
Trade payables 198 142
Current tax payable 7 5
----------------- -----------------
254 147
----------------- -----------------
Total equity and liabilities 500 337
----------------- -----------------
Required: Calculate profitability ratios, liquidity ratios and working capital ratios for Wasim (private)
Limited for two years.
Answer:
Profitability ratios
Year 7 Year 6
Gross profit % =
Gross profit 405 362
x 100 x 100 = 19% x 100 = 20%
Sales 2,160 1,806

Net profit % =
Net profit 9 53
x 100  x 100 
= 0.4% x 100 = 2.9%
Sales 2,160 1,806

Profit before interest and tax


Return on capital employed =

Share capital 
and reserves + Long-term debt capital
15 56
x 100 = 6% x 100 = 29%
246 190

Sales
Asset turnover = x 100
Share capital and reserves
 + Long- term debt capital
2,160 1,806
= 8.8 times = 9.5 times
246 190

Liquidity Ratios
Current ratio =  
Current assets 422 265
Current liabilities = 1.7 times = 1.8 times
254 147
Quick ratio =
Current assets excluding inventory 422 - 106 265 - 61
Current liabilities   1.2 times
  1.4 times
254 147

 
© Emile Woolf International 420 The Institute of Chartered Accountants of Pakistan
Chapter 10: Interpretation of financial statements

Working capital ratios

Average time to collect =


Trade receivables 316 x 365 198 x 365
x 365  53 day s  40 day s
Sales 2,160 1,806

Average time to pay =


Trade payables 198 x 365 142 x 365
x 365  =
41 day s = 36 day s
Cost of purchases 1,755 1, 444

Inventory turnover =
Inventory 106 x 365 61 x 365
x 365   day s
 22 = 15 day s
Cost of sales 1,755 1, 444

 
Example 03: AMIR & MO Limited
Question: The income statements and statements of financial position of two manufacturing
companies in the same sector are set out below.

Amir Mo
Rs. Rs.
Revenue 150,000 700,000
Cost of sales (60,000) (210,000)
------------------- -------------------
Gross profit 90,000 490,000
Interest payable (500) (12,000)
Distribution costs (13,000) (72,000)
Administrative expenses (15,000) (35,000)
------------------- -------------------
Profit before tax 61,500 371,000
Income tax expense (16,605) (100,170)
------------------- -------------------
Profit for the period 44,895 270,830
------------------- -------------------
Assets
Non-current assets
Property - 500,000
Plant and equipment 190,000 280,000
------------------- -------------------
190,000 780,000

© Emile Woolf International 421 The Institute of Chartered Accountants of Pakistan


Financial accounting and reporting I

Current assets
Inventories 12,000 26,250
Trade receivables 37,500 105,000
Cash at bank 500 22,000
------------------- -------------------
50,000 153,250
------------------- -------------------
Total assets 240,000 933,250
------------------- -------------------
Required:
Calculate profitability ratios, liquidity ratios and working capital ratios for Amir and Mo to make
comparison.
Answer:
Profitability ratios

Amir Mo
Gross profit % =
Gross profit 90,000 490,000
x 100 x 100 = 60% x 100 = 70%
Sales 150,000 700,000

Net profit % =
Net profit 44,895 270,830
x 100  x 100
= 30% x 100 = 39%
Sales 150,000 700,000

Profit before interest and tax


Return on capital employed =
Share capital and reserves + Long - term debt capital
 
Amir 61,500 + 500
x 100 = 28.5%
207, 395 +10,000

Mo 371,000 +12,000
x 100 = 47%
565,580 + 250,000
 Sales
Asset turnover = x 100
Share capital and reserves + Long - term debt capital
Amir  150,000
= 0.7 times
207,395 + 10,000

Mo 700,000
= 0.85 times
565,580 + 250,000

Liquidity Ratios

 Amir Mo
Current ratio =
Current assets 50,000 153,250
Current liabilities = 2.2 times = 1.3 times
22,605 117,670

Quick ratio =
Current assets excluding inventory 50,000 - 12,000 153,250 - 26,250
Current liabilities  22,605
 times
= 1.7
117,670
= 1.1 times

 

© Emile Woolf International 422 The Institute of Chartered Accountants of Pakistan


Chapter 10: Interpretation of financial statements

Working capital ratios

Average time to collect =


Trade receivables 37,500 105,000
x 365 x 365 = 91 day s x 365 = 55 day s
Sales 150,000 700,000

Average time to pay =


Trade payables 22,605 117,670
x 365  day s
x 365 = 137 x 365 = 204 day s
Cost of purchases 60,000 210,000

Inventory turnover =
Inventory 12,000 26,250
x 365   s
x 365 = 73 day x 365 = 46 day s
Cost of sales 60,000 210,000

 
Example 04: Alpha Limited and Omega Limited
Question: Alpha Limited and Omega Limited are in the same trade, but operate in different areas.
Their accounts for the year ended 31 December, 2016 are as follows:

Profit and loss account Alpha Limited Omega Limited


Rs.’000 Rs.’000 Rs.’000 Rs.’000
Sales 1,440 1,720
Less: Cost of sales 1,120 1,342
Gross profit 320 378
Less: Overheads 220 300
Profit before tax 100 78
Taxation 40 30
Dividends 20 24
60 54
Retained earnings 40 24
Statement of financial position
Share capital of Rs. 1 each 600 200
Reserves 240 104
840 304
8% Debentures - 120
840 424
Represented by:
Non-current assets at cost 660 520
Less: Depreciation 200 160
460 360
Current assets:
Inventory 280 172
Receivables 310 300
Cash 30 32
620 504

© Emile Woolf International 423 The Institute of Chartered Accountants of Pakistan


Financial accounting and reporting I

Current liabilities:
Taxation 40 30
Creditors 180 344
Bank overdraft - 42
Dividends 20 24
240 440
Net Current assets 380 64
840 424
Required:
Compute the current ratio, acid test ratio, creditors ratio and collection period for each of the
companies and carry out the comparative analysis of the companies based on the computed ratios.

Answer:
Liquidity Ratios

Alpha Limited Omega Limited


Current ratio=
Current assets 620,000 504,000
Current liabilities 240,000 440,000
=2.58 times =1.15 times
Quick ratios=
Current assets – inventory 620,000-280,000 504,000-172,000
Current liabilities 240,000 440,000
=1.42 times =.75 times

Working capital ratios

Average time to collect =


Trade receivables x 365 310,000 x365 300,000 x 365
Sale 1440,000 1720,000
x365= 79 days = 64 days
Average time to pay=
Trade payables x 365 180,000 x 365 344,000 x 365
Credit purchases 1120,000 1342,000
= 59 days =94days

The comments on comparative analysis of both companies based on the ratios computed above
are as under:
(i) In terms of working capital and liquidity, Alpha Limited is in a better position to honor its
obligations as they fall due because its current ratio and acid test ratio are higher than those
of Omega Limited.
(ii) Omega Limited’s payment period is better than that of Alpha Limited’s because Omega
Limited uses supplier’s funds to finance its operation.

© Emile Woolf International 424 The Institute of Chartered Accountants of Pakistan


Chapter 10: Interpretation of financial statements

(iii) Omega Limited’s collection period is also better than that of Alpha Limited. It extends shorter
credit period to its customers than Alpha Limited.
(iv) Omega Limited’s credit policy is better than that of Alpha Limited. This is because there is 30
days difference between its payments period and collection periods compared with Alpha
Limited that had a longer collection period than its payment period.

Example 05: Boom Limited (BL)


Question: Boom Limited (BL) is a manufacturer of sports goods. Following financial statements for
the year ended 31 December 2017 have been submitted to the Chief Executive Officer (CEO)
Statement of profit or loss
Rs. in ‘000
Revenues 21,000
Cost of sales (17,500)
Gross profit 3,500
Operating expenses (1,900)
Finance cost (450)
Profit before tax 1,150
Taxation (345)
Profit after tax 805

Statement of financial position

Rs. in ‘000
Property, plant and equipment 7,500
Current assets 1,500
9,000
Share capital 4,000
Reserves 1,000
Non-current liabilities 3,000
Current liabilities 1,000
9,000

Although performance of BL has improved from the last year, CEO wants to compare the results
with other companies operating in sports manufacturing industry. In this respect, following industry
data has been gathered:

Gross profit margin 23.5%


Net profit margin 7.7%
Current ratio 2.75
Gearing ratio 50:50
Return on non-current asset 32.9%
Return on capital employed 27.4%
Return on equity 31.3%

Required:
(a) Compute BL’s ratios for comparison with the industry.
(b) For each ratio, give one possible reason for variation from the industry.

© Emile Woolf International 425 The Institute of Chartered Accountants of Pakistan


Financial accounting and reporting I

Answer:
Ratios (a) BL's ratios Industry's (b) Reasons for variation from industry
ratios
Gross profit 16.67% 23.50% Lower than industry
margin  Purchase of raw material at higher prices
as compared to its competitors
 Inability to obtain economies of scale in
production as compared to its competitors
 Higher production costs due to
inefficiencies
 Deliberately keeping selling prices
lower to gain the market share
Net profit 3.83% 7.70% Lower than industry
margin 805  BL’s gross profit margin is 6.8% lower than
x 100
21,000 industry (16.6% Vs 23.5%) whereas net
profit margin is only 3.9% lower which
indicates that BL’s operating expenses as
a percentage of sales are approximately
2.9% lower than the industry

Current ratio 1.50 2.75 Lower than industry


1,500 Since gearing ratio is lower than the
1,000 industry so BL might have:
 obtained running finances as compared to
long-term financing by the industry availed
extended credit terms from suppliers
 Low inventory levels are maintained by BL
 Shorter credit terms are given to debtors

Gearing ratio 37.5: 62.5 50 : 50 Lower than industry


3,000  Difficulty in raising long-term
4,000 + 1,000 finance from banks due to low
+ 3,000 profits
 Running finance or extended
credit terms from suppliers are
available for BL

Return on 21.33% 32.90% Lower than industry


non- current 1,150 +  Lower profit margins
assets 450 x
100  Relatively newer non-current
7,500 assets have higher carrying
value
Return 20.00% 27.40% Lower than industry
on 1,150 + 450  Lower profit margins
capital 4,000 + x  High shareholder’s equity
employed 1,000 + 100
3,000

Return 16.10% 31.30% Lower than industry


on 805  Lower profit margins
equity x
4,000 + 100  Higher shareholder’s equity/low
1,000 gearing ratio

© Emile Woolf International 426 The Institute of Chartered Accountants of Pakistan


Chapter 10: Interpretation of financial statements

Example 06: Progressive Steel Limited


Question: Progressive Steel Limited (PSL) commenced business in 2015. The following
comparative data pertains to the year ended 30 June 2017:

PSL Industry
Description
2017 2016 2017
Gross profit margin 13% 13% 16%
Net profit margin 8% 7% 10%
Return on shareholders’ equity 22% 18% 25%
Current ratio 1.2 1.6 1.5
Debt to equity ratio 40:60 30:70 50:50
Cash operating cycle in days 119 135 118

Required:
For each ratio/data give possible reasons for variation from comparative and industry data.

Answer:
Reasons for fluctuation with
Ratios Reason for fluctuation with Industry
previous year
Gross profit In line with previous year. Lower than industry
margin No variation.  The company is in initial phase and may
have kept the selling prices lower than
the industry to gain the market share.
 The company may not have been able
to purchase raw material at prices
which is available to its competitors.
 The company may not have been able
to obtain economies of scale in its
production which may have been
obtained by its competitors.
 Possibility of higher production costs.
Net profit Higher than previous year: Lower than industry however, the difference
margin  Tight control over is mainly attributed to lower gross profit
operating costs. margin.
 Increase in other income.
Decrease in fixed cost per
unit due to increase in sale.
Return on Higher than previous year: Lower than industry
shareholder's  Reduction in tax rates.  Lower gross profit and net profit
equity margins.
 Reduction in interest
rates.  Lower leverage.
 Decrease in equity  Higher net assets resulting in higher
might be due to equity.
buyback of shares.
 Distribution of profits
from previous year
which resulted in
decrease in equity.

© Emile Woolf International 427 The Institute of Chartered Accountants of Pakistan


Financial accounting and reporting I

Current ratio Lower than previous year: Lower than industry


 The company might
have obtained running
finance facility to fund
its operations in the
current year.
 Long term loan
payments might have
become due in the next
12 month, which
decreases the current
ratio.
 Decrease in current
assets due to better
inventory management/
reduction in credit period
of debtors.
Debt to equity Higher than previous year Lower than industry
ratio  Decrease in reserves  Being a new entrant the company may
due to dividend pay- out. be in the phase of expansion thereby
 Further debt obtained raising debt accordingly.
during the period.
 Decrease in equity might
be due to buyback of
shares.
Cash operating Lower than previous year In line with industry.
cycle  Increase in current
liabilities might be due
to increase in credit
period.
 Decrease in current
assets which might be
due to greater stock
turnover or better
inventory management.
 By giving lower credit
days to debtors.
.

Example 07: Dairy Foods Limited


Question: The following information has been gathered by an analyst, in respect of Dairy Foods
Limited (DFL) which specializes in various dairy products.

Industry
Ratio 2016 2015 2014
average
Profit margin % 11% 10% 8% 10.45%
Quick ratio 1.38 1.40 1.42 1.52
Current ratio 1.84 1.67 1.59 1.73
Days purchases in payables 80 91 89 82

In the latest annual report to the shareholders, Directors of DFL have claimed that liquidity position
of the Company has improved significantly.
Required: Critically analyse and discuss whether you agree with the claim.

© Emile Woolf International 428 The Institute of Chartered Accountants of Pakistan


Chapter 10: Interpretation of financial statements

Answer:
While analyzing liquidity positions of DFL, it is noted that current ratio has steadily increased over
the years and is better than industry average. However, the quick ratio has steadily declined and is
even lower than industry average. This is a clear evidence that the increase in liquidity is caused by
an increase in inventory.

Further, by considering the nature of highly perishable inventories kept by a dairy food company, it
is a possibility that DFL may bear high inventory losses due to short expiry. Based on the above, I
do not agree with the claim of DFL’s directors.

Example 08: Comparison


Question: Extracts from latest financial statements of two companies are as follows;
Extracts from statements of financial position
A B Assets A B
Equity & Liabilities
Rs. in millions Rs. in millions
51,690 72,114 Fixed assets 34,460 48,076
Equity & reserves
- 36,057 Stock in trade 21,700 20,000
Long term loan
35,790 45,135 Trade debtors 24,470 44,030
Trade creditors
12,000 8,500 Cash and bank 18,850 49,700
Other payables
99,480 161,806 99,480 161,806

Extract from Statement of Comprehensive Income

A B

Rs. in million

Revenue 161,600 220,150

Cost of sales (135,160) (180,520)

Gross profit 26,440 39,630

Operating expenses (9,840) (13,870)

Interest expense (720) (2,313)

Profit before tax 15,880 23,447

Income tax (333) (409)

Profit after tax 15,547 23,038

Required:
Analyze the profitability, liquidity and working capital ratios of both the companies.

© Emile Woolf International 429 The Institute of Chartered Accountants of Pakistan


Financial accounting and reporting I

Answer:

Profitability ratios A B

Gross profit ratio (GP ÷ sales) 16.36% 18.00%

Profit to sales (Profit after tax ÷ sales) 9.62% 10.46%

Return on capital employed (Profit before interest and tax ÷


capital employed)
32.11% 23.81%

Return on asset employed (Profit before interest and tax ÷ 16.69% 15.92%
assets)

Company B's gross profit and net profit ratio is slightly higher as compared to Company A. The
difference is not significant and may be on account of higher level of sales resulting in lesser fixed
costs per unit.
Company A’s return on capital employed ratio and return on asset employed ratio are better than
Company B, because Company B has accumulated large balances of cash despite of availing long
term loan. Had Company B had used its cash balances to pay off the long term loan; it would have
both of these ratio better than Company A.

Liquidity Ratios A B

Current ratio (current assets ÷ current liabilities) 1.36 2.12

Quick ratio (current asset-inventory ÷ liabilities) 0.91 1.75

Company B has better current and quick ratio. However, it appears that these ratios are better than
Company A due to substantially high amount of trade debts in term of percentage of sales as sales
days. It also represents a risk that these trade debts may prove irrecoverable. Moreover, they may
be indicative of inefficient in debt collection as well.

Working capital turnover ratios A B

Stock turnover days (Stock ÷ Cost of goods sold × 365) [A] 58.60 40.44

Debtor turnover days (Debtor ÷ Revenue × 365) [B] 55.27 73.00

Creditor turnover days (Creditor ÷ Cost of goods sold × 365) [C] 96.65 91.26

Cash operating cycle [A+B–C] (days) 17.22 22.18

Stock turnover of Company B is better than that of Company A. Company B is turning over its stock
9 times whereas company A is doing it 6 times a year.
Company A is more effectively collecting its debtors than Company B. This could also be due to the
fact that Company B is following a lenient credit policy to attract more revenue. This fact is also
supported from higher stock turnover ratio of Company B.
Company A have availed better credit facility from its creditors but it may have forgone some
settlement discounts which might have resulted in lower gross profit ratio than that of Company B.
Overall cash operating cycle of Company A is better than Company B. Furthermore, Company B has
accumulated large balances of cash despite the fact that it has also availed long term loan. Excess
cash balance should have been used to pay off the long term loan to reduce the finance cost.

© Emile Woolf International 430 The Institute of Chartered Accountants of Pakistan


Chapter 10: Interpretation of financial statements

Example 09: Shispare Limited and Trivor Limited


Question: Following are the summarised financial statements of Shispare Limited (SL) and its
competitor Trivor Limited (TL) for the year ended 31 December 2019:
Statement of financial position

Assets SL TL Equity & SL TL


liabilities
Rs. in million Rs. in million

Fixed assets 5,400 7,800 Capital and 8,400 9,450


reserves
Current Long-term loan 1,900 4,600
assets:

Inventory 4,800 7,100 Current liabilities:

Debtors 2,700 3,200 Creditors 2,900 4,500

Cash 1,200 800 Accrued 900 350


expenses

8,700 11,100 3,800 4,850

14,100 18,900 14,100 18,900

Statement of profit or loss

SL TL

--- Rs. in million ---

Sales 16,700 35,400

Cost of goods sold (11,400) (27,800)

Gross profit 5,300 7,600

Operating expenses (3,500) (4,900)

Finance cost (250) (600)

Net profit 1,550 2,100

Required:
Compute relevant ratios for SL and TL to assess which company seems to:
(i) give more incentives to its customers to pay on time
(ii) avail extended credit terms from its suppliers
(iii) be more efficient in the use of capital
(iv) keep lower selling prices to gain the market share
(v) have better liquidity position
(vi) have higher ability to convert its assets into profit
(vii) control operating expenses more efficiently
(viii) have higher ability to raise bank loan in future

© Emile Woolf International 431 The Institute of Chartered Accountants of Pakistan


Financial accounting and reporting I

Answer:

Relevant ratios SL TL
Debtors collection period (i) 59.01 days 32.99 days
Debtors 2,700 3,200
= ×365 = ×365 = ×365
Sales 16,700 35,400
TL is giving more incentives to its customers to pay on time.

Creditors payment period (ii) 92.85 days 59.08 days


Creditors 2,900 4,500
= ×365 = ×365 = ×365
Purchases 11,400 27,800
SL avail extended credits terms

Return on capital (iii) 17.48% 19.22%


Profit before interest 1,550+250 2,100+600
= ×100 = ×100 = ×100
Capital employed 8,400+1,900 9,450+4,600
TL is more efficient in the use of capital

Gross profit margin (iv) 31.74% 21.47%


Gross profit 5,300 7,600
= ×100 = ×100 = ×100
Sales 16,700 35,400
TL is deliberately keeping selling prices lower to gain the market share.

Relevant ratios SL TL
Current ratio (v) 2.29 2.29
Current assets 8,700 11,100
= = =
Current liabilities 3,800 4,850
Quick ratio 1.03 0.82
Current assets-inventory 8,700 − 4,800 11,100 − 7,100
= = =
Current liabilities 3,800 4,850
SL has better liquidity position

Return on assets (vi) 12.77% 14.29%


Profit before interest 1,550+250 2,100+600
= ×100 = ×100 = ×100
Total assets 14,100 18,900
TL has higher ability to convert its assets into profit

Operating expenses %age (vii) 20.95% 13.84%


Operating expenses 3,500 4,900
= ×100 = ×100 = ×100
Sales 16,700 35,400
TL is efficiently controlling the operating expenses.

Gearing ratio (viii) 0.18 0.33


Debt 1,900 4,600
= = =
Debt + Equity 8,400+1,900 9,450+4,600
SL is going to raise a bank loan relatively easily in future

© Emile Woolf International 432 The Institute of Chartered Accountants of Pakistan


Chapter 10: Interpretation of financial statements

Example 10: Limitations of ratio analysis


Question: Ratios are computed by using numerical values from financial statements to gain
meaningful information about an entity. However, due to inherent limitations of ratio analysis,
it may not reflect the correct financial situation.
Required:
Briefly explain any four limitations of ratio analysis.
Answer:
(i) Historical
All information used in ratio analysis is derived from actual historical results. This does not
mean that the same results will carry forward into the future. However, ratio analysis can be
used on pro forma information and compare it to historical results for consistency.
(ii) Historical versus current cost
The information on the income statement is stated in current costs (or close to it), whereas
many elements of the balance sheet are stated at historical cost (which could vary
substantially from current costs). This disparity can result in unusual ratio results.
(iii) Inflationary effect
If the rate of inflation has changed in any of the periods under review, this can mean that the
numbers are not comparable across periods. For example, if the inflation rate was 100% in
one year, sales would appear to have doubled over the preceding year, when in fact sales did
not change at all.
(iv) Aggregation
The information in a financial statement line item that is used for a ratio analysis may have
been aggregated differently in the past, so that running the ratio analysis on a trend line does
not compare the same information through the entire trend period.

© Emile Woolf International 433 The Institute of Chartered Accountants of Pakistan


Financial accounting and reporting I

9 OBJECTIVE BASED QUESTIONS


01. The acid test also known as quick ratio should include which of the following?
(1) Inventory
(2) Accounts receivables
(3) Bank overdraft
(4) Accruals
(a) (1), (2), (3) and (4)
(b) (1), (2) and (3) only
(c) (1), (2) and (4) only
(d) (2), (3) and (4) only

02. Salik has net current liabilities in its statement of financial position. He has decided to pay off
its accounts payables using surplus cash.
What will be the effect of the above transaction on the current ratio?
(a) Decrease
(b) Increase
(c) No effect
(d) The ratio could either increase or decrease

03. Extracts from the statement of Comprehensive Income and statement of financial position of
the Huda Limited are shown below:
Rs.
Revenue from sales (all on credit) 900,000
Cost of goods sold 756,000
Purchases (all on credit) 504,000
Receivables 112,500
Trade payables 75,600
Inventory 333,000
What is the length of the working capital cycle (also known as the operating or cash cycle) to
the nearest day?
(a) 170 days
(b) 152 days
(c) 261days
(d) 60 days

04. Which of the following should be included in acid test or quick ratio?
(a) Finished goods inventory
(b) raw materials and consumables
(c) long-term loans
(d) Accounts payable

© Emile Woolf International 434 The Institute of Chartered Accountants of Pakistan


Chapter 10: Interpretation of financial statements

05. Given below are included in the financial statements of Haris Limited for the year ending 30
June:
2015 2016
Rs. 000 Rs. 000
Receivables for disposal of PPE 36 54
Accounts receivables 108 72
144 126
Sales for the year amounted to Rs. 630,000, of which Rs. 90,000 were cash sales.
The average receivables turnover (in times) during the year ended 30 June 2016 was?
(a) 7
(b) 6
(c) 5
(d) 4

06. Dawn Limited has provided following information:


31st December 31st December
2014 2015
Rs. 000 Rs. 000
Non -current assets 36 61.2
Inventory 18 23.4
Accounts receivable 25.2 19.8
Bank balance 5.4 3.6
Accounts payable 19.8 16.2
Sales on credit 252 216
Purchases on credit 144 108
Which TWO of the following statements are correct?
(a) Net current assets have increased between the two years
(b) The quick (acid test) ratio has improved between the two years
(c) Customers are paying more quickly in 2015 than in 2014
(d) Dawn Limited is paying its suppliers more quickly in 2015 than in 2014

07. Amir sells fish and Abid sells books. Both operate on a 50% mark-up on cost.
However, their gross profit ratios are as follows.
Amir 25%
Abid 33%
The highest gross profit ratio of the bookseller may be because?
(a) There is more wastage with fish stocks than with book stocks
(b) Amir has a substantial bank loan whereas the Abid’s business is entirely financed by
her family
(c) Amir has expensive high street premises whereas Abid has cheaper back street
premises
(d) Amir’s turnover is declining whereas that of the Abid is increasing

© Emile Woolf International 435 The Institute of Chartered Accountants of Pakistan


Financial accounting and reporting I

08. Maira Limited has a current ratio of 2:1.


This ratio will decrease if Maira Limited
(a) will receive cash in respect of a long-term loan
(b) will receive cash in respect of a short-term loan
(c) pays an existing trade payable
(d) writes off an existing receivable against the provision for doubtful debts

09. Haris Limited has positive working capital.


What effect will the payment of a declared (payable) dividend using cash balances have upon
the current ratio and working capital?
(a) Increase in current ratio and increase in working capital
(b) Increase in current ratio and no effect on working capital
(c) No effect on current ratio and working capital
(d) Decrease in current ratio and decrease in working capital

10. After declaring a final dividend, Kashan Limited has a current ratio of 2.0 and a quick asset
ratio of 0.8.
If the company now uses its positive cash balance to pay that final dividend, what will be the
effect upon the two ratios?
(a) Increase in current ratio and increase in quick asset ratio
(b) Increase in current ratio and decrease in quick asset ratio
(c) Decrease in current ratio and increase in quick asset ratio
(d) Decrease in current ratio and decrease in quick asset ratio

11. The draft accounts of Super Star Limited for the year ended 31 December 2018 include the
following:

Revenue Rs. 360 million

Gross profit Rs. 90 million

It was subsequently discovered that the revenue was overstated by Rs. 45 million and the
closing inventory understated by Rs. 15 million.
After correction of these errors the gross profit percentage will be?
(a) 9.5%
(b) 19.0%
(c) 23.8%
(d) 33.3%

© Emile Woolf International 436 The Institute of Chartered Accountants of Pakistan


Chapter 10: Interpretation of financial statements

12. The following has been extracted from the financial statements of a business.
SOCI Rs. SOFP Rs.
Profit from operations 86,400 7% debenture 117,000
Debenture interest (8,190) Ordinary share capital 171,000
Profit for the year 72,360 Share premium 13,500
Retained earnings 63,000
What was the return on capital employed (ROCE)?
(a) 19.9%
(b) 23.7%
(c) 29.2%
(d) 34.9%

13. Waris Limited buys and sells a single product. The following is an extract from its statement
of financial position at 31 December 2018.
2018 2017
Rs. 000 Rs. 000
Inventory 75 60
Receivables 24 36
Sales and purchases during 2018 were Rs. 300,000 and Rs. 180,000 respectively. 20% of
sales were for cash.
Which TWO of the following are correct?
(a) Average receivables collection period is 37 days
(b) Average receivables collection period is 46 days
(c) Gross profit % is 35%
(d) Gross profit % is 45%

14. A company’s gearing ratio (Debt/Equity) is 60%. This would rise if


(a) The decrease in shareholders’ equity is equal in amount to decrease in long term loans
(b) The increase in shareholders’ equity is equal in amount to decrease in long term loans
(c) The part of long term loans are repaid
(d) Right issue of shares of 1 for 5 is made

15. A business has the following trading account for the year ending 31 May 2018:
Rs. Rs.
Sales 450,000
Opening inventory 40,000
Purchases 260,500
300,500
Less: closing inventory (60,000) (240,500)
Gross profit 209,500

© Emile Woolf International 437 The Institute of Chartered Accountants of Pakistan


Financial accounting and reporting I

Its rate of inventory turnover for the year is?


(a) 4.9 times
(b) 5.3 times
(c) 7.5 times
(d) 9 times

16. Tara Ltd produces a single product with a margin on sales of 25%.
Total sales for the year Rs. 400,000
Receivables collection period 64 days
Average receivables Rs. 32,000
The value of inventory held during the year was constant.
The cost of credit sales was?
Rs. ___________

17. The following are extracts from the financial statements of Laiba Ltd for the year ended 31
December 2018.
Statement of financial position Statement of Comprehensive Income
Rs. 000 Rs. 000
Issued share capital 3,600 Operating profit 1,431
Reserves 1,800 Debenture interest (216)
5,400 1,215
12% debenture 2018 1,800
7,200
What is the return (%) on long-term funds?

___________%

18. The opening inventory for a business was Rs. 108,000. The closing inventory was Rs.
144,000.
Inventory turnover for the year was 10 times.
The gross margin was 30%.
What were the sales for the year?
Rs. ___________

19. Adeel Limited has trade payables (creditors) of Rs. 12,000 and a bank overdraft of Rs. 3,000.
Its current ratio is 2.5: 1 and its quick (acid test) ratio is 1.5:1.
What is the value of its inventory (stock)?
Rs. ___________

© Emile Woolf International 438 The Institute of Chartered Accountants of Pakistan


Chapter 10: Interpretation of financial statements

20. Extracts from statement of financial position of Turab Limited at 31 March 2019 are presented
below:
Rs. 000
Loans due in more than one year 32
5% loan notes 24
Ordinary shares - Rs. 1 each fully paid 80
6% redeemable preferred shares, Rs. 1 each fully paid 16
Retained profits 104
Revaluation reserve 40
The gearing ratio is (to one decimal place)?

___________%

21. Which ratio is considered as safe margin of solvency?


(a) Liquid ratio
(b) Quick ratio
(c) Current ratio
(d) Net profit ratio

22. Liquid ratio is also known as


i. Quick ratio
ii. Acid test ratio
iii. Working capital ratio
iv. Stock turnover ratio
(a) i. and ii.
(b) i. and iii.
(c) iii. and iii.
(d) iii. and iv.

23. Care pharmacy Ltd. has a current ratio equal to 1.6 and a quick ratio equal to 1.2. The
company has Rs.200 million in sales and its current liabilities are Rs.10 million. what is the
value of company's current assets?
(a) Rs.14 million
(b) Rs.16 million
(c) Rs.18 million
(d) Rs.20 million

© Emile Woolf International 439 The Institute of Chartered Accountants of Pakistan


Financial accounting and reporting I

24. Determine working capital turnover ratio if, current assets is Rs.150 million, current liabilities
is Rs.100 million and Sales during the year are Rs.500 million.
(a) 5 times
(b) 10 times
(c) 15 times
(d) 20 times

25. A debtor turnover ratio of 12 times means that:


(a) one-twelfth of debtors will turn out to be bad debts
(b) debtors are about twelve times as big as creditors
(c) in any given month, twelve debtors are expected to pay in full
(d) the average debtor takes about one month to pay.

26. What is the impact of collection of debtors on the current ratio;


(a) Increase
(b) Decrease
(c) No impact
(d) None of the above

27. Which of the following is not included in the computation of acid test ratio?
(a) Debtors
(b) Cash at bank
(c) Short-term investments
(d) Stock

28. Determine inventory turnover ratio if, opening inventory is Rs.31 million, closing inventory is
Rs.29 million, sales are Rs.320 million and gross profit margin is 25%.
(a) 6 times
(b) 8 times
(c) 11 times
(d) 16 times

29. Debt-equity ratio is a sub-part of


(a) Short-term solvency ratios
(b) Long-term solvency ratios
(c) Debtors turnover ratios
(d) None of the above

© Emile Woolf International 440 The Institute of Chartered Accountants of Pakistan


Chapter 10: Interpretation of financial statements

30. Night Limited has a current ratio of 1.8. This ratio will increase if Night Limited:
(a) receives cash in respect of a short term loan
(b) receives cash from an existing receivable
(c) pays an existing trade payable
(d) purchases inventory on credit

31. Night Limited has a current ratio of 1.8. This ratio will increase if Night Limited:
(a) receives cash in respect of a short term loan
(b) receives cash from an existing receivable
(c) pays an existing trade payable
(d) purchases inventory on credit

32. Which of the following companies is most likely to face cash flow problems?
(a) A loss making government organisation
(b) A company which has recently sold part of its operations so as to concentrate on its
core areas
(c) A reasonably profitable and long established company with no expansion plans
(d) A profitable retailer about to embark on ambitious expansion plans

33. A company has current ratio and quick ratio of 2.0 and 0.8 respectively. If the company uses
its positive cash balance to pay a creditor, it will:
(a) increase current ratio as well as quick ratio
(b) increase current ratio and decrease quick ratio
(c) have no effect on current ratio as well as quick ratio
(d) decrease current ratio as well as quick ratio

34. Which of the following would increase gearing ratio?


(a) Issuance of shares at premium
(b) Issuance of shares at discount
(c) Issuance of bonus shares
(d) Declaration and payment of cash dividend

© Emile Woolf International 441 The Institute of Chartered Accountants of Pakistan


Financial accounting and reporting I

9 OBJECTIVE BASED ANSWERS


01. (d) Acid test ratio is calculated as (current assets - inventory) / current
liabilities

02. (a) Assume initially that Salik has assets of Rs. 100,000 and liabilities of Rs.
150,000
Its current ratio prior to the transaction will be: Rs. 100,000/Rs. 150 ,000=
0.67
If it then pays its trade payables by Rs. 50,000 the current assets will be Rs.
50,000 and the liabilities Rs. 100,000.
Hence the new current ratio is 50,000/100,000 = 0.50, i.e. a decrease.

03. (b)
days
Inventory turnover: 333/756× 365 160.8
Average collection period: 112.5/900 × 365 45.6
Average time to pay: 75.6/504× 365 (54.8)
151.6

04. (d) The acid test ratio excludes all inventory balances, and is based on short-
term creditors only.

05. (b) Credit sales = 630,000 – 9,000 = Rs. 540,000


Average receivables = (72000 + 108000)/2 =90,000
Receivable turnover =540,000/ 90,000 = 6

06. (a & c)
2014 2015
Net current (18 + 25.2 + 5.4 28.8 (23.4+ 19.8+ 30.6
assets – 19.8) 3.6 – 16.2)
Quick ratio (25.2 + 5.4)/19.8 1.55 (19.8 + 1.44
3.6)/16.2
Collection (25.2 /252) × 36.5 (19.8/216) × 33.5 days
period 365 days 365
Payment (19.8/144) × 365 50.2 (16.2/108) × 54.8 days
period days 365

07. (a) Cost plus 50% is equivalent to a gross profit ratio of 33%. Amir’s gross
profit margin may be low because of wastage.
The loan interest and rental would not affect gross profit (only affects net
profit) and declining turnover would not directly affect the gross profit
percentage.

08. (b) Receiving cash for a long-term loan increases current assets with no
change in current liabilities, hence improves the ratio. Payment on an
existing creditor improves the ratio.
Writing off a receivable against a provision has no effect on current assets.

© Emile Woolf International 442 The Institute of Chartered Accountants of Pakistan


Chapter 10: Interpretation of financial statements

Therefore receiving cash in respect of a short-term loan must be the correct


choice.
Thus, suppose current ratio is 2:1, that means assets 100,000 and current
liabilities 50,000
Now say Maira Limited receives loan = Rs. 50,000
Current assets will be 100,000 + 50,000=150,000
Current liabilities 50,000 + 50,000 = 100,000
New Current ratio 1.5:1

09. (b) Before payment of proposed dividend


Rs.
Current assets 150,000
Current liabilities including proposed dividend Rs. 50,000
30,000
Current ratio 3:1
Working capital 100,000
After payment of proposed dividend
Rs.
Current assets 120,000
Current liabilities 20,000
Current ratio 6:1
Working capital 100,000

10. (b) Suppose that inventories are Rs. 120,000, cash plus receivables are Rs.
80,000 and creditors (including a Rs. 10,000 dividend) are Rs. 100,000.
Payment of the dividend will cause cash plus receivables to fall to Rs.
70,000 and creditors to fall to Rs. 90,000.
The current ratio will increase to 2.11 (190 ÷ 90).
The quick ratio will decrease to 0.77 (70 ÷ 90).

11. (b) Rs. m Rs. m


Turnover 360-45 315
Cost of sales 270-15 (255)
Gross profit 60
Gross profit % =60/315= 19.0%

12. (b) ROCE = 86,400/364,500= 23.7%

13. (b) & (d) Credit sales = Rs. 300,000 x 80%= Rs. 240,000
Average receivables =Rs. (24,000+ 36,000)/2 = Rs. 30,000
Receivables’ turnover =Rs. 240,000/Rs. 30,000 = 8
Collection period =365/8 = 46 days
Rs. 000 %
Sales 300 100
Cost of sales (60+180-75) (165) 55
Gross profit 135 45

© Emile Woolf International 443 The Institute of Chartered Accountants of Pakistan


Financial accounting and reporting I

14. (b) Since long term loans being numerator are lower amount (i.e. 60% of equity),
the equal amount increase would result in higher ratio than before. Equal
amount decrease would result in lower ratio and repayment of long term loan
or issue of shares would also result in lower gearing ratio.

15. (a) Inventory turnover = COS/ average inventory


= 240,500 /[(40,000 + 60 ,000)/2 = 4.9 times

16. Rs. 136,875 Receivables of Rs. 32,000 represent 64 days’ credit sales.
Therefore, receivables of Rs. 182,500 would represent 365 days’ credit
sales. [32,000 x 365/64]
Cost of credit sales = Rs. 182,500 x 75% = Rs. 136,875

17. 19.88% Return on long-term funds = Operating profit (before debenture interest)
/(Share capital + Reserves + Debentures)
=1,431/7,200
= 19.88%

18. Rs. Inventory turnover =CGS/ average inventory


1,800,000
CGS = average inventory x inventory turnover
= ((108,000 + 144,0000)/2) x 10
= 126,000x10 = Rs. 1,260,000
Sales = Rs. 1,260,000 / 0.7 = Rs. 1,800,000

19. Rs. 15,000 Current ratio = Current assets/ current liabilities


Current assets = Current ratio x Current liabilities
= 2.5 x (Rs. 12,000 + Rs. 3000) = Rs. 37,500
Quick ratio = (C.A – inventory)/ C.L
C.A – inventory = Quick ratio x C.L
Rs. 37,500 – inventory = 1.5 x 15,000
Rs. 37,500 – inventory = Rs. 22,500
Inventory = Rs. 15,000

20. 24.3 % Debt = 32 + 24 + 16 = 72


Equity = 80 + 104 + 40 = 224
Gearing = 72/(72 + 224) = 24.3 %

21. (c)

22. (a)

23. (b)

24. (b)

25. (d)
26. (c)

27. (d)

28. (b)

© Emile Woolf International 444 The Institute of Chartered Accountants of Pakistan


Chapter 10: Interpretation of financial statements

29. (b)

30. (c) Pays an existing trade payable


31. (c) Average debtors are collected in 2 months
32. (d) A profitable retailer about to embark on ambitious expansion plans
33. (b) increase current ratio and decrease quick ratio
34. (d) Declaration and payment of cash dividend

© Emile Woolf International 445 The Institute of Chartered Accountants of Pakistan


Financial accounting and reporting I

© Emile Woolf International 446 The Institute of Chartered Accountants of Pakistan


Certificate in Accounting and Finance

11

CHAPTER
Financial accounting and reporting I

Accounting for Not-for-Profit


Organisations

Contents
1 Not for profit organisations (NPOs)
2 Income and expenditure account
3 Statement of financial position
4 Accounting Standard for NPOs (ASNPO)
5 ASNPO: Contribution revenue and receivable
6 ASNPO: Inventories and non-current assets
7 ASNPO: Preparation of financial statements
8 Objective based questions and answers

* The student must refer original handbook of IFRS.


© Emile Woolf International 447 The Institute of Chartered Accountants of Pakistan
Financial accounting and reporting I

1 NOT FOR PROFIT ORGANISATIONS (NPOS)

Section overview

 Introduction
 Different terminology
 Receipt and payment account

1.1 Introduction
Not-for-Profit Organisations (NPOs) are organisations, normally without transferable ownership
interests, organized and operated exclusively for social, educational, professional, religious, health,
charitable or any other not-for-profit purpose. An NPO's members, contributors and other resource
providers do not, in such capacity, receive any financial return directly from the NPO.
NPOs may be:
 companies formed under Section 42 of Companies Act, 2017;
 trusts formed under Trust Act, 1882;
 societies formed under the Societies Registration Act, 1860; or
 any other recognisable form of organisation giving value to the groups of people they
administer to.
The financial objective of a profit-oriented entity is to make profit and maximise shareholders’
wealth while financial objective of NPO is to provide its services effectively by achieving value for
money. NPO applies or intends to apply its profits, if any, or other income in promoting its objects,
and prohibits the distribution of surplus to its members, sponsors, promoters, etc.
NPOs have income which they raise and costs which must be paid just like other organisations
and although profit is not their objective but they have to account for their income and costs. NPOs
are accountable for their effectiveness, economy and efficiency in utilising the funds.
Revenues of NPOs normally arise from donations, government grants and other contributions as
well as from membership fees, the sale of goods, the rendering of services or the use by others of
NPO resources yielding rent, interest, royalties or dividends.
Some accounting rules are as relevant to NPOs as to profit-oriented entities, for example,
requirements relating to inventory, non-current assets and recognition of revenue. However, some
areas might be completely irrelevant, for example, earnings per share.

1.2 Different terminology


NPOs use different accounting terminology from profit-oriented entities.
Profit-oriented entities Not-for-Profit Organisations
Statement of comprehensive income Statement of income and expenditure
Net profit Excess of income over expenditure / Surplus
Net loss Excess of expenditure over income / Deficit
Equity / Share capital and equity reserves Net assets / Accumulated fund / Accumulated
surplus / Accumulated deficit / Fund balance
Statement of changes in equity Statement of changes in net assets

© Emile Woolf International 448 The Institute of Chartered Accountants of Pakistan


Chapter 11: Accounting for Not-for-Profit Organisations

1.3 Receipts and payments account


When accounts are prepared on cash or disbursement basis rather than accrual basis of
accounting, a receipt and payment account is prepared and presented. This is simply a summary
of cash receipts and payments during the accounting period. It includes capital and revenue items
both.
A receipt and payment account gives far less information than a set of financial statements based
on the accruals concept.

Illustration 01:

Receipts and payments account


Balance b/d X Donation to Dam Fund X
Subscriptions X Repairs X
Functions – ticket revenue X Telephone X
Sale of land X Extension of building X
Bank interest X Furniture X
Bequest X Electricity expenses X
Sundry income X Salary and wages X
Sundry expenses X
Balance c/d X
X X
Balance b/d X

© Emile Woolf International 449 The Institute of Chartered Accountants of Pakistan


Financial accounting and reporting I

2 INCOME AND EXPENDITURE ACCOUNT


Section overview

 Introduction
 Categories of income
 Subscriptions account
 Life membership fee
 Donations
 Surplus from running an operation
 Surplus from running an event
 Type of expenditures
 Format

2.1 Introduction
An income and expenditure account is an accruals based statement listing the different types of
income of a club followed by the different categories of expenditure of the club.

2.2 Categories of income


A club may have several categories of income including:
 Membership fees and subscriptions;
 Life membership fees;
 Donations to the club;
 Investment income;
 Surplus from running a coffee bar or a shop;
 Surplus from running an event;
Note that if a club has a coffee bar or shop or runs an event the “profit” from these is generally
calculated separately (in an account known as a trading account) and presented as a line in the
income and expenditure account.

2.3 Subscriptions account


At each year end there will usually be some members who have paid their subscriptions in advance
and some who are in arrears. These are both included as balances brought down and carried down
on a single subscription account. Cash received is credited to this account and the balance on the
account is transferred to the income and expenditure account (as income for the year).
Illustration 02: Subscription account
Subscription account
Rs. Rs.
Balance b/d Balance b/d
(members in arrears) X (members who have prepaid) X
Income and expenditure X Cash X
Balance c/d Balance c/d
(members who have prepaid) X (members in arrears) X
X
Balance b/d Balance b/d (members who
(members in arrears) X have prepaid) X

© Emile Woolf International 450 The Institute of Chartered Accountants of Pakistan


Chapter 11: Accounting for Not-for-Profit Organisations

Example 01: Subscription account


Question: At 31 March 2016 a cricket club had membership subscriptions in arrears amounting to
Rs. 48,000 and had received Rs. 12,000 subscriptions in advance.
During the year to 31 March 2017 the club received Rs. 624,000 including 26 memberships for
the year to 31 March 2018 at Rs. 1,200 per annum.
At 31 March 2017 16 members owed subscriptions of Rs. 1,200 each.
Required: How the above transactions would be recorded in the subscription’s ledger account for
the year to 31 March 2017?
Answer:

Subscriptions

Rs. Rs.

Balance b/d: Balance b/d:


Members in arrears 48,000 Advance payments 12,000

Cash 624,000

Membership fees for the


year (to I&E) 576,000

Balance c/d: Balance c/d:


Advance payments Members in arrears
(26 × 1,200) 31,200 (16 × 1,200) 19,200

655,200 655,200

Balance b/d: 19,200 Balance b/d: 31,200

Write off of subscriptions


Questions often include the write off of subscriptions from members who have stopped attending
the club.

Illustration 03: Write off of subscriptions

Debit Credit
Income and expenditure account X
Subscription account X

Example 02: Subscription account


Question: At 31 March 2016 a cricket club had membership subscriptions in arrears amounting to
Rs. 48,000 and had received Rs. 12,000 subscriptions in advance.
During the year to 31 March 2017 the club received Rs. 624,000 including 26 memberships for
the year to 31 March 2018 at Rs. 1,200 per annum.
At 31 March 2017 16 members owed subscriptions of Rs. 1,200 each.
Half of the members who were in arrears at the end of the previous period still had not paid by 31
March 2017. It was decided to write these amounts off.
Required: How the above transactions would be recorded in the subscription’s ledger account for
the year to 31 March 2017?

© Emile Woolf International 451 The Institute of Chartered Accountants of Pakistan


Financial accounting and reporting I

Answer:
Subscriptions
Rs. Rs.
Balance b/d: Balance b/d:
Members in arrears 48,000 Advance payments 12,000
Cash 624,000
Membership fees for the
year (to I&E) 600,000 Bad debts (1/2  48,000) 24,000
Balance c/d: Advance Balance c/d: Members in
payments (26 × 1,200) 31,200 arrears (16 × 1,200) 19,200
679,200 679,200
Balance b/d: 19,200 Balance b/d: 31,200

2.4 Life membership fee


A club should have an accounting policy for these. Possible policies include:
 Recognition as income when received.
 Recognition as income over a specified period.
 Recognition in an equity reserve (an accumulated fund).
Recognition as income when received

Illustration 04:

Debit Credit

Bank (cash received) X

Income and expenditure account X

Recognition as income over a specified period

Illustration 05:

On receipt: Debit Credit

Bank (cash received) X

Deferred income ( accredit account on the face of the


statement of financial position) X

Each year over the specified future period:

Deferred income X

Income and expenditure account X

This treatment recognises the amount received as income over several years.

© Emile Woolf International 452 The Institute of Chartered Accountants of Pakistan


Chapter 11: Accounting for Not-for-Profit Organisations

Recognition in an equity reserve (an accumulated fund)

Illustration 06:
Debit Credit
Bank (cash received) X
Life membership fund (an accumulated fund account in
equity) X

This might then be transferred to the accumulated surplus of income over expenditure over a pre-
defined period or on the death of the member.

2.5 Donations
A club might receive a donation or bequest. If the donation has not been made for a specific
purpose the club might recognise the donation as income in the period in which it is received.
A club might receive a donation for a particular purpose. For example, a member might donate
money for a new cricket square. In this case the money is credited to a fund account set up for the
purpose.

Illustration 07:
Debit Credit
Bank (cash received) X
Cricket square fund (an accumulated fund account in
equity) X

2.6 Surplus from running an operation


If a club has a coffee bar or shop the “profit” from these is generally calculated separately (in an
account known as a trading account).
Any expenses directly related to the operation of a coffee bar or shop would be deducted from the
gross profit of the operation and the net profit would be presented on a separate line in the income
and expenditure account.

Illustration 08: Coffee bar trading account

Rs. Rs.
Income
Sales X
Opening inventory X
Purchases X
X
Closing inventory (X)
Cost of sales (X)
Gross profit X
Coffee shop worker’s salary (X)
Net profit (this figure to the face of the income and
expenditure account) X

© Emile Woolf International 453 The Institute of Chartered Accountants of Pakistan


Financial accounting and reporting I

2.7 Surplus from running an event


If a club runs an event any surplus (or loss) generally calculated separately and presented as a
separate line in the income and expenditure account.

Illustration 09: Event surplus

Rs.
Sports day entry fees X
Cost of prizes (X)
Surplus/deficit (this figure to the face of the income
and expenditure account) X

2.8 Type of expenditures


The expenditures related to non -profit organization are quite similar to profit organizations like:
 Salaries
 Rent
 Electricity
 Repair and maintenance
 Depreciation of fixed assets etc.

2.9 Formats
There are no mandatory formats for income & expenditure account. It can either be presented in
a statement form or an account form, both illustrated below:

Illustration 10: Income and expenditure account (Statement Form) for the year ended XX/XX/XX

Rs. Rs.
Income
Subscription income X
Donations X
Interest on bank deposit X
Coffee bar/shop profit X
Tournament income X
Less: Prizes (X)
X
X
Expenditure
Club expenses X
Rent X
Electricity X
Depreciation X
Repairs X
X
Surplus (deficit) of income over expenditure X

© Emile Woolf International 454 The Institute of Chartered Accountants of Pakistan


Chapter 11: Accounting for Not-for-Profit Organisations

Illustration 11: Income & expenditure account (Account Form) for the year ended June 30, XXXX

Amount Amount
Expenditure Income
(Rs.) (Rs.)
Salaries X Subscriptions X
Rent X Interest X
Travelling expenses X Gain on sale of furniture X
Printing and stationary X
General charges X
Periodicals X
Depreciation on furniture X
Excess of income over expenditure X
X X

© Emile Woolf International 455 The Institute of Chartered Accountants of Pakistan


Financial accounting and reporting I

3 STATEMENT OF FINANCIAL POSITION


Section overview

 Format
 Special funds

3.1 Format
A not for profit organisation may or may not prepare a statement of financial position but if it does
so the statement of financial position would be similar to that of a business. The main difference is
in the equity section. The equivalent of the capital section of a business is called the accumulated
fund. The below illustrations shows two formats, statement form and account form:

Illustration 12: Statement of financial position (Statement Form) of a club

Rs.
Assets
Non-current assets
Club house X

Current assets
Subscriptions in arrears X
Investments X
Shop inventory X
Prepayments X
Cash X
Total assets X

Equity and liabilities


Accumulated fund
At start of year X
Surplus / (deficit) for the year X
At end of year X

Current liabilities
Subscriptions in advance X
Accruals X
Total accumulated fund and liabilities X

© Emile Woolf International 456 The Institute of Chartered Accountants of Pakistan


Chapter 11: Accounting for Not-for-Profit Organisations

Illustration 13: Statement of financial position (Account Form) of a club


As at 31 December XXXX

General Fund & Liabilities Rs. in ‘000 Assets Rs. in ‘000

General fund X Non-Current Assets

Excess of income over expenditure X Land/Capital advance X

X Furniture & fixtures X

Long term advance X Van X

Long term deposits X

Long term prepayment X

Current Liabilities

Creditors X Current Assets

Accrued expenses X Stock X

Advance subscription X Accounts receivable X

X Advance & prepayments X

Bank X

Total General Fund & Liabilities X Total Assets X

3.2 Special funds


An organisation might also have other funds in addition to the accumulated fund.
These “special” funds arise in a number of circumstances including:
 when an organisation receives cash for a designated purpose; or
 when an organisation sets aside resources for a designated purpose.
The organisation might also set aside assets (say cash) to match to the fund so that they can be
used for the specified purpose.

Illustration 14: Receipt of cash for a specified purpose

Debit Credit
Cash X
Special fund X

Allocation of assets to the fund


Special fund cash X
Cash X

The following journals reflect cash being spent on the specified purpose.

© Emile Woolf International 457 The Institute of Chartered Accountants of Pakistan


Financial accounting and reporting I

Illustration 15: Receipt of cash for a specified purpose


Debit Credit
Special fund X
Cash (or “Special fund cash” if so allocated) X

Illustration 16: Special fund


Ali has been very successful in business.
When he was a young man he very much enjoyed playing cricket and has very fond memories of
his days at the village cricket club.
He has donated Rs. 1,000,000 to the club to fund the building of a new club house.
This would be accounted for as follows:
Debit Credit
Cash 1,000,000
Special fund (clubhouse) 1,000,000
Allocation of assets to the fund
Special fund cash 1,000,000
Cash 1,000,000

An organisation itself might set aside funds for a particular purpose.

Illustration 17: Set up fund for a specified purpose


Debit Credit
Accumulated fund X
Special fund X
Allocation of assets to the fund
Special fund cash X
Cash X

Illustration 18: Special fund


A social club in a small town has managed to accumulate a significant balance on its accumulated
fund over the years.
Its members have decided that the club should establish a fund to contribute to the school fees of
children of high promise from the town. Parents of such children would apply to the club for a grant
of Rs. 50,000.
Rs. 1,500,000 is to be set aside for this purpose.
This would be accounted for as follows:
Setting up the fund Debit Credit
Accumulated fund 1,500,000
Special fund (Education fund) 1,500,000
Allocation of assets to the fund
Special fund cash 1,000,000
Cash 1,000,000

© Emile Woolf International 458 The Institute of Chartered Accountants of Pakistan


Chapter 11: Accounting for Not-for-Profit Organisations

On the award of a grant. Debit Credit


Special fund (Education fund) 50,000
Special fund cash 50,000

Example 03: Nawabshar Youth Movement


Question: The following were the assets and liabilities of the Nawabshar Youth Movement at 30
April 2017.

Rs. 000

Fixtures and fittings (net) 16,340

Inventory of refreshment (coffee bar) 4,460

Land 51,600

Subscription received in advance 4,900

Payables for drinks supplied (coffee bar) 6,780

Cash at bank 7,466

The accountant’s receipts and payments account for the year to 30 April 2018 shows the following:

Receipts Rs. 000

Donations received 500

Rent of hall 5,600

Members’ subscription 24,000

Sale of brochure 1,740

Sale of dance tickets 3,400

Sale of refreshments (coffee bar) 10,200

Payments

Repairs and maintenance 3,218

Salaries and wages 6,309

Gifts and donations 600

Dance expenses 950

Refreshment supplies (coffee bar) 19,415

Sundry expenses 10,000

Further information:

(i) Wages of Rs. 556,000 were due but unpaid at the year-end.

(ii) Inventories of drinks at 30 April 2018 were Rs. 14,210,000

(iii) Provide for depreciation on fixtures and fittings at Rs. 1,900,000

(iv) Subscription due but not paid at 30 April 2018 was Rs. 1,900,000
Required: Based on the above information, prepare the club’s income and expenditure account
for the year ended 30 April 2018 and the statement of financial position as at that date.

© Emile Woolf International 459 The Institute of Chartered Accountants of Pakistan


Financial accounting and reporting I

Answer:
Income and expenditure account Rs.000
Income:
Subscriptions (W1) 30,800
Donations 500
Rent of hall 5,600
Sales of brochure 1,740
Sales of dance tickets 3,400
Net income from coffee bar (W4) 7,315
49,355
Less expenses
Repairs and Maintenance 3,218
Salaries and Wages (W2) 6,865
Gifts and Donations 600
Dance expenses 950
Sundry expenses 10,000
Depreciation of fixtures and fittings 1,900
(23,533)
Net surplus 25,822

Statement of financial position as at 30 April 2018 Rs.000


Non-current assets
Land 51,600
Fixtures and Fittings 16,340
Depreciation (1,900)
14,440
66,040
Current Assets:
Inventory of drinks 14,210
Subscriptions unpaid 1,900
Cash and Bank Balance 12,414
94,564
Financed By:
Accumulated Fund (W5) 68,186
Surplus of income over expenditure 25,822
94,008
Current Liabilities : Wages accrued 556
94,564

© Emile Woolf International 460 The Institute of Chartered Accountants of Pakistan


Chapter 11: Accounting for Not-for-Profit Organisations

W1 Subscriptions account
Rs. 000 Rs. 000
Balance b/d 4,900
Subscriptions for the period 30,800 Bank 24,000
Balance c/d 1,900
30,800 30,800

W2 Salaries and wages


Rs. 000 Rs. 000
Bank 6,309 Expenditure 6,865
Balance c/d 556
6,865 6,865

W3 Payables
Rs. 000 Rs. 000
Bank 19,415 Balance b/d 6,780
Expenditure 12,635
19,415 19,415

W4 Coffee bar
Sales 10,200
Opening inventory 4,460
Purchases (W3) 12,635
Closing inventory (14,210)
(2,885)
Profit (gross) 7,315

W5 Accumulated fund at start of the year


Assets:
Fixtures and Fittings 16,340
Inventory of refreshments 4,460
Land 51,600
Cash and Bank Balances 7,466
79,866
Liabilities:
Subscription received in Advance 4,900
Payables for drinks supplied 6,780
(11,680)
Accumulated fund 68,186

© Emile Woolf International 461 The Institute of Chartered Accountants of Pakistan


Financial accounting and reporting I

Example 04: Peshawar Business Club


Question: The statement of financial position of Peshawar Business Club as at 31 December 2017
is shown as follows:

Accumulated Carrying
Cost
depreciation amount
Rs.000 Rs.000 Rs.000
Furniture and Fittings 40,000 10,000 30,000
Games Equipment 20,000 7,200 12,800
Motor van 30,000 10,000 20,000
90,000 27,200 62,800

Current Assets: Cash at bank and at hand 9,200


72,000

Financed by: Accumulated funds 72,000

The following transactions took place during the year 1 January 2018 to 31 December 2018:
Receipts Rs. 000
Subscriptions (10,000 members @ 1,600 each) 16,000
Donations 1,600
Sale of tickets for annual dinner 10,800
Payments
Electricity 4,000
Expenses for annual dinner 6,200
New games equipment 3,200
Cleaners’ wages 2,080
Repairs and renewal 1,660
Motor van repairs 2,520
Further information:
(i) An electricity bill of Rs. 900,000 was owed at 31 December 2018.
(ii) Depreciation should be calculated at 10% of cost of the assets.
Required: Based on the above information, prepare the receipt and payment account and income
and expenditure account of Peshawar Business Club for the year ended 31 December 2018 and
statement of financial position as at that date.
Answer:

Receipts and payments


Rs. 000 Rs. 000
Balance b/d 9,200 Electricity 4,000
Subscription 16,000 Expenses for annual dinner 6,200
Donations 1,600 New games equipment 3,200
Sale of Tickets for annual dinner 10,800 Cleaner’s wages 2,080
Repairs and renewals 1,660
Motor van repairs 2,520
Balance c/d 17,940
37,600 37,600
.

© Emile Woolf International 462 The Institute of Chartered Accountants of Pakistan


Chapter 11: Accounting for Not-for-Profit Organisations

Income and expenditure account


Income: Rs.000
Subscriptions 16,000
Donations 1,600
Sales of dance tickets 10,800
28,400
Less expenses
Electricity (4,000,000 + 900,000) 4,900
Annual expenses 6,200
Cleaner’s wages 2,080
Repairs and renewals 1,660
Motor van repairs 2,520
Depreciation (W) 9,320
(26,680)
Net surplus 1,720

Working: Depreciation (W)


Rs.000
Furniture and Fittings 0.1 @ 40,000,000 4,000
Game Equipment (20,000,000 + 3,200,000) x 0.1) 2,320
Motor Van 30,000,000 @ 0.1 3,000
9,320

Statement of financial position as at 30 April 2018


Non-current assets
Cost Accumulated depreciation Carrying amount
Rs.000 Rs.000 Rs.000
Furniture and Fittings 40,000 14,000 26,000
Games and Equipment 23,200 9,520 13,680
Motor Van 30,000 13,000 17,000
93,200 36,520 56,680

Current Assets: Rs. 000


Cash and Bank Balance 17,940
74,620
Financed By:
Accumulated Fund 72,000
Surplus of income over expenditure 1,720
73,720
Current Liabilities : Electricity accrual 900
74,620

© Emile Woolf International 463 The Institute of Chartered Accountants of Pakistan


Financial accounting and reporting I

Example 05: GILTAN GOLF CLUB


Question: The treasurer of the Giltan Golf Club has prepared the following receipts and payments
account for the year ended 31 March 2016.
Rs.(000) Rs.(000)
Balance at 1 April 2015 682 Functions 305
Subscriptions 2,930 Repairs 146
Functions 367 Telephone 67
Sale of land 1,600 Extension of club house 600
Bank interest 60 Furniture 135
Bequest (legacy) 255 Heat and light 115
Sundry income 46 Salary and wages 2,066
Sundry expenses 104
Balance at 31 March 2016 2,402
5,940 5,940

(a) Subscriptions received included Rs. 65,000 which had been in arrears at 31 March 2015 and
Rs. 35,000 which had been paid for the year commencing 1 April 2016.
(b) Land sold had been valued in the club's books at cost Rs. 500,000.
(c) Accrued expenses
31 March 2015 31 March 2016
Rs. (000) Rs. (000)
Heat and light 32 40
Wages 12 14
Telephone 14 10
58 64
(d) Depreciation is to be charged on the original cost of assets appearing in the books at 31
March 2016 as follows:
Buildings 5%
Fixtures and fittings 10%
Furniture 20%

(e) The following balances are from the club's books at 31 March 2015:
Rs.(000)
Land at cost 4,000
Buildings at cost 3,200
Buildings allowance for depreciation 860
Fixtures and fittings at cost 470
Fixtures allowance for depreciation 82
Furniture at cost 380
Furniture allowance for depreciation 164
Subscriptions in arrears 80
(including Rs. 15,000 irrecoverable - member had emigrated)
Subscriptions in advance 30

Required: Prepare an income and expenditure account for the year ended 31 March 2016 and a
Statement of financial position as at that date.

© Emile Woolf International 464 The Institute of Chartered Accountants of Pakistan


Chapter 11: Accounting for Not-for-Profit Organisations

Answer:
Income and expenditure account for Giltan Golf Club for year ending 31 March 2016
Rs.(000) Rs.(000)
Income
Functions surplus (367  305) 62
Sale of land (1,600  500) 1,100
Bank interest 60
Bequest 255
Sundry income 46
Subscriptions(W1) 2,860
4,383
Expenditure
Bad debts 15
Repairs 146
Telephone (67  14 + 10) 63
Heat and light (115  32 + 40) 123
Salaries and wages (2,066  12 + 14) 2,068
Sundry expenses 104
Depreciation - building 190
Depreciation - furniture 103
Depreciation - fixtures and fittings 47
(2,859)
Surplus for the year 1,524

Giltan golf club: Statement of financial position as at 31 March 2016


Non-current assets Cost Accumulated Carrying
depreciation amount
Rs.(000) Rs.(000) Rs.(000)
Land (4,000 – 500) 3,500 - 3,500
Buildings(W3) 3,800 (1,050) 2,750
Fixtures and fittings(W4) 470 (129) 341
Furniture(W5) 515 (267) 248
8,285 (1,446) 6,839
Current assets
Bank 2,402
9,241

Accumulated Fund(W2) 7,618


Surplus for the year 1,524
9,142
Current liabilities
Accruals 64
Subscriptions in advance 35
99
9,241

© Emile Woolf International 465 The Institute of Chartered Accountants of Pakistan


Financial accounting and reporting I

Workings
W1 Subscriptions account

Rs.(000) Rs.(000)

Subs in arrears b/d 80 Subs in advance b/d 30


Income and expenditure 2,860 Bank 2,930
Subs in advance c/d 35 Bad debts 15
_____ _____
2,975 2,975
_____ _____

W2 Opening statement of affairs 2015


Assets Rs.(000)
Bank 682
Subscriptions in arrears 80
Land 4,000
Buildings (3,200  860) 2,340
Fixtures (470  82) 388
Furniture (380  164) 216
Liabilities 7,706
Accruals (58 + 30) (88)
7,618

W3 Buildings

Cost Acc. Depreciation


Rs.(000) Rs.(000)
Balance b/d 3,200 860
Extension to clubhouse 600
Depreciation (5%  3,800) 190
3,800 1,050

W4 Fixtures and fittings

Cost Acc. Depreciation


Balance b/d 470 82
Depreciation (10%  470) 47
470 129

W5 Furniture

Cost Acc. Depreciation


Balance b/d 380 164
Additions 135
Depreciation (20%  515) 103
515 267

© Emile Woolf International 466 The Institute of Chartered Accountants of Pakistan


Chapter 11: Accounting for Not-for-Profit Organisations

Example 06: GULSHAN CRICKET CLUB


Question: The following balances have been obtained from the books of Gulshan Cricket Club:

June 30, 2014 June 30, 2015


Building 6,024,000 6,438,150
Furniture 3,012,000 2,710,800
Books 1,129,500 1,246,950
Sports equipment 1,807,200 1,595,200
Investments - 436,000
Advance subscription 86,000 92,000
Prepaid expenses 122,000 176,000
Expenses payable 186,900 207,600
Subscriptions receivable 326,000 357,000
Cash 1,204,800 1,586,500
The following information is also available in respect of the year ended June 30, 2015:
(i) Depreciation for the year has been credited directly to the asset accounts. The rates of
depreciation are as follows:

Building 5%
Furniture and books 10%
Sports equipment 20%

(ii) The club had 600 members on June 30, 2015. No fresh members were admitted during the
year but 10 members left the club on January 1, 2015. Subscription per member is Rs. 500
per month.
Required:
(a) Summary of receipts and payments made during the year ended June 30, 2015.
(b) Income and Expenditure Account for the year ended June 30, 2015.

Answer:
Receipt & payment account for the year ended June 30, 2015
Receipts Rupees Payments Rupees
Balance b/d 1,204,800 Additions: to:
Building 753,000
Subscriptions received 3,605,000 Sports Equipment 186,800
Books 256,000
Investments made 436,000
Expenses (payments)
Balancing 1,591,500
Balance c/d 1,586,500
4,809,800 4,809,800

© Emile Woolf International 467 The Institute of Chartered Accountants of Pakistan


Financial accounting and reporting I

Income & expenditure account for the year ended June 30, 2015
Expenditures Rupees Incomes Rupees
Expenses A/c 1,558,200 Subscription
(600 x 6000 + 10 x 3000) 3,630,000
Dep. Exp. -Building 338,850
-Furniture 301,200
-Sports
Equipment 398,800
-Books 138,550

Surplus of Income over Exp. 894,400


3,630,000 3,630,000

Workings
Building Account
Rupees Rupees
Balance b/d 6,024,000 Depreciation
(6,438,150×5/95) 338,850
Addition 753,000
Balance c/d 6,438,150
6,777,000 6,777,000

Sports Equipment Account


Rupees Rupees
Depreciation
Balance b/d 1,807,200 (1,595,200 20/80) 398,800
Addition 186,800
Balance c/d 1,595,200
1,994,000 1,994,000

Furniture Account
Rupees Rupees
Depreciation
Balance b/d 3,012,000 (2,710,800 10/90) 301,200
Balance c/d 2,710,800
3,012,000 3,012,000

Books Account
Rupees Rupees
Depreciation
Balance b/d 1,129,500 (1,246,950 10/90) 138,550
Addition 256,000
Balance c/d 1,246,950
1,385,500 1,385,500

© Emile Woolf International 468 The Institute of Chartered Accountants of Pakistan


Chapter 11: Accounting for Not-for-Profit Organisations

Subscription Account
Rupees Rupees
Sub. Receivables - Balance
b/d 326,000 Adv. Subscription - b/d 86,000
Income & Exp. Account 3,630,000 Cash Received 3,605,000
Adv. Subscription - Balance Sub. Receivables - Balance
c/d 92,000 c/d 357,000
4,048,000 4,048,000

Expenses Account
Rupees Rupees
Balance b/d 122,000 Balance b/d 186,900
Payment made (Rcpt. & Pay. Income & Exp A/c (Bal.
A/c) 1,591,500 Amount) 1,558,200
Balance c/d 207,600 Balance c/d 176,000
1,921,100 1,921,100

Example 07: SEHAT CLUB


Question: Following is the Receipts and Payments Account of Sehat Club for the year ended 30
June 2015:
Receipts and payments account for the year ended 30 June 2015

Receipts Rupees Payments Rupees


Opening balance 15,000 Salaries 63,500
Subscriptions 201,000 Rent 34,000
Entrance fees 63,000 Travelling expenses 1,500
Donations 38,000 Printing and stationery 1,000
Interest 16,000 General charges 2,500
Receipt on disposal of furniture 500 Periodicals 500
Investments 200,000
Closing balance 30,500
333,500 333,500

The club’s statement of financial position as on 30 June 2014 was as follows:


Statement of financial position as on 30 June 2014

Liabilities Rupees Assets Rupees


General Fund 172,500 Furniture – net 40,000
Liabilities: Rent 11,000 Sports equipment – net 20,000
Salaries 17,500 Investments 100,000
Subscription receivable 15,000
Interest receivables 11,000

© Emile Woolf International 469 The Institute of Chartered Accountants of Pakistan


Financial accounting and reporting I

Bank balance 15,000


201,000 201,000
Other details for the year ended 30 June 2015 are as follows:
(i) Furniture purchased on 1 July 2013 costing Rs. 4,000 was disposed of on 1 January 2015
at a scrap value of Rs. 500.
(ii) On 1 July 2014, furniture having written down value of Rs. 6,000 was traded-in with new
furniture having fair value of Rs. 6,700.
(iii) Depreciation is charged on diminishing balance basis at 20% on furniture and 15% on
sports equipment.
(iv) Sports equipment worth Rs. 12,000 were received at year end as donation.
(v) Following amounts are receivable /outstanding as at 30 June 2015:

Subscription receivable 8,000


Entrance fee receivable 3,000
Salaries outstanding 4,000
Rent outstanding 2,000

Required:
Prepare an income and expenditure account of Sehat Club for the year ended 30 June 2015 and
its statements of financial position on that date.

Answer:
Sehat Club: Income and Expenditure Account for the year ended 30 June 2015
Amount Amount
Expenditure Income
(Rs.) (Rs.)
Salaries (63.5+4-17.5) 50,000 Subscriptions (201+8-15) 194,000
Rent (34+2-11) 25,000 Entrance fees (63+3) 66,000
Travelling expenses 1,500 Donation (38+12) 50,000
Printing and stationary 1,000 Interest (16-11) 5,000
General charges 2,500 Gain on trade-in of furniture 700
Periodicals 500
Depreciation on furniture *7,820
Depreciation on sports 3,000
equipment
Loss on furniture disposed of 2,380
(2880-500)
Excess of income over 222,000
expenditure
315,700 315,700

Sehat Club: Statement of financial position as at 30 June 2015

Assets Rupees

Furniture (see account below) 30,000

Sports equipment (20-3+12) 29,000

Investments (100+200) 300,000

© Emile Woolf International 470 The Institute of Chartered Accountants of Pakistan


Chapter 11: Accounting for Not-for-Profit Organisations

Subscription receivable 8,000

Entrance fee receivable 3,000

Bank balance 30,500

400,500

Equity and liabilities Rupees

General fund

Opening balance 172,500

Add: Excess of income over expenditure 222,000 394,500

Liabilities:

Salaries payable 4,000

Rent payable 2,000

400,500

Furniture Account

Rupees Rupees

Balance b/d 40,000 Asset disposed of 2,880


(4,000 – 800 – 320)

New furniture 6,700 Asset exchanged 6,000

Depreciation expense *7,820

Balance c/d 30,000

46,700 46,700

* Depreciation on furniture:
20% of (40,000+6,700–3,200–6,000) = 7,500+320 (i.e. 10% of Rs. 3,200).

Example 08: AB SPORTS AND SOCIAL CLUB


Question: You have agreed to take over the role of bookkeeper for the AB sports and social club.
The summarised statement of financial position on 31 December 2014 as prepared by the previous
bookkeeper contained the following items.
Assets Rs.

Heating oil for clubhouse 1,000


Shop and cafe inventories 7,000
New sportswear, for sale, at cost 3,000
Used sportswear, for hire, at valuation 750

© Emile Woolf International 471 The Institute of Chartered Accountants of Pakistan


Financial accounting and reporting I

Equipment for grounds man


Cost 5,000
Depreciation 3,500 1,500
Subscriptions due 200

Bank

Current account 1,000

Deposit account 10,000


Capital and liabilities
Accumulated fund 23,150
Payables:
Shop and cafe inventories 1,000
Sportswear 300
The bank account summary for the year to 31 December 2015 contained the following items.

Receipts Rs.
Subscriptions 11,000
Bank
Shop and café 20,000
Sale of sportswear 5,000
Hire of sportswear 3,000
Interest on deposit account 800
39,800

Payments Rs.
Rent and repairs of clubhouse 6,000
Heating oil 4,000
Sportswear 4,500
Grounds person 10,000
Shop and cafe purchases 9,000
Transfer to deposit account 6,000
39,500

You discover that the subscriptions due figure as at 31 December 2014 was arrived at as follows.
Subscriptions unpaid for 2013 10
Subscriptions unpaid for 2014 230
Subscriptions paid for 2015 40
Corresponding figures at 31 December 2015 are:
Subscriptions unpaid for 2013 10
Subscriptions unpaid for 2014 20
Subscriptions unpaid for 2015 90
Subscriptions paid for 2016 200

© Emile Woolf International 472 The Institute of Chartered Accountants of Pakistan


Chapter 11: Accounting for Not-for-Profit Organisations

Subscriptions due for more than 12 months should be written off with effect from 1 January 2015.
Asset balances at 31 December 2015 include:

Heating oil for club house 700


Shop and cafe inventories 5,000

New sportswear, for sale, at cost 4,000

Used sportswear, for hire, at valuation 1,000

Closing payables at 31 December 2015 are for:

shop and cafe inventories 800

Sportswear 450

heating oil for clubhouse 200

Two thirds of the sportswear purchases made in 2015 had been added to inventory of new
sportswear in the figures given in the list of assets above, and one third had been added directly to
the inventory of used sportswear for hire.
Half of the resulting new sportswear for sale at cost at 31 December 2015 is actually over two
years old. You decide, with effect from 31 December 2015, to transfer these older items into the
inventory of used sportswear, at a valuation of 25% of their original cost.
No cash balances are held at 31 December 2014 or 31 December 2015. The equipment for the
grounds person is to be depreciated at 10% per annum, on cost.
Required:
Prepare the income and expenditure account and statement of financial position for the AB
sports club for 2015.
Answer:
AB Sports and social club: Income and expenditure account
Rs. Rs.
Subscriptions (W1) 10,720
Shop and cafe profit (W2) 9,200
Sale of sportswear (W3) 1,400
Hire of sportswear (W4) 1,700
Interest on deposit account 800

23,820
Rent of clubhouse 6,000

Heating oil (1,000  4,000  200  700) 4,500

Grounds person 10,000


Bad debts (unpaid subscriptions = 10 +20) 30
Depreciation 500
21,030

Net surplus 2,790

© Emile Woolf International 473 The Institute of Chartered Accountants of Pakistan


Financial accounting and reporting I

AB Sports and Social Club statement of financial position as at 31 December 2014


Non-current assets
Equipment for grounds person
Cost 5,000
Depreciation 4,000

1,000
Current assets
Heating oil 700
Shop and cafe inventories 5,000
New sportswear 2,000
Hire sportswear 1,500
Subscriptions due 90
Bank
Current account 1,300
Deposit account 16,000
26,590
27,590

Rs. Rs.
Capital and liabilities
Accumulated fund b/f 23,150
Surplus for year 2,790
25,940
Current liabilities
Shop and cafe 800
Sportswear 450
Heating oil 200
Subscriptions prepaid 200
1,650
27,590
Workings
(W1) Subscriptions
Summary subscriptions account
Rs. Rs.
Opening balance (10 + 230) 240 Opening balance 40
Income for period 10,720 Bank 11,000
Bad debts (10 + 20) 30
Closing balance 200 Closing balance 90
11,160 11,160

© Emile Woolf International 474 The Institute of Chartered Accountants of Pakistan


Chapter 11: Accounting for Not-for-Profit Organisations

(W2) Shop and cafe results


Rs. Rs.
Sales 20,000
Opening inventory 7,000
Purchases (9,000  800  1,000) 8,800
15,800
Closing inventory 5,000
10,800
Profit (gross) 9,200

(W3) Sale of sportswear

Sales 5,000
Opening inventory 3,000
Purchases (4,500  450  300)  2 3 3,100
6,100
Closing inventory 4,000
2,100
Profit (gross) 2,900
Loss on sportswear transferred 1,500
Profit 1,400

(W4) Hire of sportswear Rs. Rs.


Rentals 3,000
Opening balance 750
Additions of cost (4,500  450  300)  1 3 1,550
2,300
Closing inventory at valuation 1,000
1,300
Surplus 1,700

Example 09: MONARCH SPORTS CLUB


Question: The Monarch Sports Club has the following summary of its cash book for the year ended
30 June 2015:

Rs. Rs.
Opening bank balance 12,500
Receipts:
Subscriptions 18,000
Life membership fees 3,000
Competition receipts 7,500
Entrance fees 2,500
Equipment sold
1,000 32,000
44,500

© Emile Woolf International 475 The Institute of Chartered Accountants of Pakistan


Financial accounting and reporting I

Payments:
Transport to matches 3,700
Competition prizes 4,300
Coaching fees 2,100
Repairs to equipment 800
Purchase of new equipment 4,000
Purchase of sports pavilion 35,000
(49,900)
Closing balance (overdrawn) (5,400)

The following information is available regarding the position at the beginning and end of the
accounting year:

1 July 2014 30 June 2015


Rs. Rs.
Subscriptions in advance 1,100 900
Subscriptions in arrears 200 300
Coaching fees outstanding 150 450

Of the subscriptions outstanding at the beginning of the year, only half were eventually received.
The equipment sold during the year had a net book value of Rs. 1,200 at 1 July 2014. Equipment
is to be depreciated at 20% per annum straight line. Life membership fees are taken to cover 10
years.
The treasurer insists that no depreciation needs to be charged on the sports pavilion, as buildings
do not decrease in value. He says that the last club of which he was treasurer did charge
depreciation on its buildings but that when the club came to replace them, there was still
insufficient money in the bank to pay for the new building.

Required:
Prepare an income and expenditure account for the Monarch Sports Club for the year ended 30
June 2015.

Answer:
Monarch Sports Club: Income and expenditure account year ended 30 June 2015
Rs. Rs.
Income
Annual subscriptions (W1) 18,400
Life membership (3,000  10%) 300
Entrance fees 2,500
Surplus from competitions (W2) 3,200
24,400
Expenditure
Transport 3,700
Coaching fees (2,100  150 + 450) 2,400
Repairs 800
Bad debts 100
Loss on disposal of equipment (W3) 200

© Emile Woolf International 476 The Institute of Chartered Accountants of Pakistan


Chapter 11: Accounting for Not-for-Profit Organisations

Depreciation (W4) 800


(8,000)
Surplus for the year 16,400

Workings
(W1)

Subscriptions account
Rs. Rs.
Balance b/d (in arrears) 200 Balance b/d (in advance) 1,100
I + E a/c 18,400 Cash
Balance c/d (in advance) 900 Bad debts 100
Balance c/d (in arrears) 300
19,500 19,500

(W2) Competitions

Rs.
Receipts 7,500
Prizes (4,300)
Surplus 3,200

(W3) Sale of equipment


Disposals account
Rs. Rs.
NBV 1,200 Cash 1,000
Loss to I & E a/c 200
1,200 1,200
(W4) Depreciation
20%  4,000 = 800.

Example 10: LH SPORTS CLUB


Question: The LH Sports Club opened on 1 May 2014 having purchased premises for Rs. 80,000
and furniture for Rs. 18,000, through a contribution from a member. The club secretary has
produced the following income and expenditure account for the year to 30 April 2015.

Income Rs. Rs.


Joining fees (89 members Rs.200 each) 17,800
Annual subscriptions 12,000
Cafe profits 8,450
Dinner Dance surplus 830
Equipment hire receipts 1,750
40,830
Expenditure
Premises costs 10,990
Equipment costs 5,590

© Emile Woolf International 477 The Institute of Chartered Accountants of Pakistan


Financial accounting and reporting I

Secretary’s expenses 470


Bank charges 125 (17,175)
Surplus for the year 23,655

The income and expenditure account has been prepared after taking into account the following
items at 30 April 2015:
 cafe inventories Rs. 1,400
 payables for cafe supplies Rs.1,320
 rates and insurances prepaid Rs. 2,280

The following items have not been taken into account:


 the equipment costs figure includes Rs. 4,000 for the purchase of equipment
 depreciation is to be provided as follows:
 at 2% on premises
 at 10% on furniture
 at 20% on equipment
 joining fees are to be spread over a five-year period
 the annual subscriptions figure includes Rs.960 paid in advance
 subscriptions outstanding at the end of the year, and expected to be collected, amount to
Rs.300.
The bank balance at 30 April 2015 was Rs. 21,295.

Required:
(a) Calculate the correct surplus for the year.
(b) Prepare the statement of financial position at 30 April 2015.

Answer:
Surplus for the year Rs. Rs.
Surplus per draft income and expenditure account 23,655
Add capital expenditure 4,000
Deduct depreciation
Premises 1,600
Furniture 1,800
Equipment 800
(4,200)
Less 80% joining fee (14,240)
Less net subscriptions in advance (960  300) (660)
New surplus for year 8,555

LH Sports Club: Statement of financial position as at 30 April 2015


Assets Rs. Rs.
Non-current assets
Premises 78,400
Furniture 16,200
Equipment 3,200 97,800

© Emile Woolf International 478 The Institute of Chartered Accountants of Pakistan


Chapter 11: Accounting for Not-for-Profit Organisations

Current assets
Inventory 1,400
Subscriptions in arrears 300
Prepaid rates and insurance 2,280
Bank 21,295 25,275
123,075

Capital and liabilities Rs. Rs.


Accumulated fund at 1 May 2014 98,000
Surplus for year 8,555
Accumulated fund at 30 April 2015 106,555
Joining fees c/f 14,240

Current liabilities
Payables 1,320
Subscriptions in advance 960 2,280
123,075
Working
Non-current assets Cost Depreciation Net
Rs. Rs. Rs.
Premises 80,000 (1,600) 78,400
Furniture 18,000 (1,800) 16,200
Equipment 4,000 (800) 3,200
102,000 4,200 97,800

Example 11: LEISURE CLUB


Question: The accountant of Leisure Club was terminated on account of charges of fraud on 31
December 2016 and Mr. Emad has been appointed in his place. Emad has gathered the following
information in respect of the year ended 31 December 2016:
(i) The club has 3,300 members and the membership fee is Rs. 10,000 per annum. The fee
payable by each member becomes due on the first day of the quarter in which he became a
member. The fee received in each quarter was as follows:

Quarter First Second Third Fourth


Subscription received (Rs.) 9,900,000 8,250,000 5,500,000 9,350,000
Last year the fee was Rs. 9,000 per annum. However, the number of members was the same.
(ii) A summary of the bank account for the year is shown below:
Deposits Rupees Withdrawals Rupees
Balance as at 1 Jan. 2016 3,700,500 Insurance 175,000
Cash deposited into bank 37,848,500 Rent and rates 4,200,000
Written off amount recovered 1,860,000 Utilities 4,365,000
Disposal of fixed assets 750,000 Freehold land purchased 17,000,000
Members subscription received Cash withdrawals from bank 6,120,000
directly in bank account 19,800,000 Payment to creditors 18,155,000
Repairs and maintenance 700,000

© Emile Woolf International 479 The Institute of Chartered Accountants of Pakistan


Financial accounting and reporting I

Exercise equipment 7,350,000


Balance as at 31 Dec. 2016 5,894,000
63,959,000 63,959,000
(iii) Amounts paid from petty cash were as follows:
Rupees
Salaries 2,300,000
Sundry expenses 640,000
(iv) The club has a tuck shop which earns a profit margin of 20% of sales. All sales of tuck shop
are made on cash. During the year, stock costing Rs. 500,000 was destroyed by fire.
(v) The opening WDV of fixed assets was Rs. 28,000,000. Exercise equipment was purchased
on 1 October 2016. Fixed assets having opening WDV of Rs. 800,000 were disposed of on
31 March 2016. Fixed assets are depreciated @ 20% under the reducing balance method.
(vi) The opening and closing balances of cash in hand were Rs. 300,000 and Rs.
25,000 respectively.
(vii) The following balances have been extracted through a scrutiny of the available records:
2016 2015
------- Rupees -------
Creditors 3,330,000 2,500,000
Prepaid rent 175,000 168,000
Stock- tuck shop 2,500,000 2,300,000
Required:
(a) Determine the amount of loss incurred by the club due to fraud committed by the
previous accountant.
(b) An income and expenditure account for the year ended 31 December 2016.
(c) Statement of financial position as at 31 December 2016.

Answer:
Determination of amount of loss incurred due to fraud Rupees
Opening cash balance 300,000

Cash receipts
Collection from members [(3,300 x 10,000) – 19,800,000] 13,200,000
Bank withdrawals 6,120,000
Tuck shop sales (W-2) 22,856,250
42,176,250
Cash payments
Salaries (2,300,000)
Sundry expenses (640,000)
Cash deposited into bank (37,848,500)
(40,788,500)
Closing cash should have been 1,687,750
Closing cash-actual (25,000)
Loss due to fraud 1,662,750

© Emile Woolf International 480 The Institute of Chartered Accountants of Pakistan


Chapter 11: Accounting for Not-for-Profit Organisations

Income and expenditure account

Income Rupees
Subscription income (W-1) 31,817,500
Income from tuck shop (22,856,250(W-2) – 18,285,000 (W-2)) 4,571,250
Other income – Bad debts recovered 1,860,000
38,248,750
Expenditures
Salaries 2,300,000
Insurance 175,000
Rent expense (168,000 + 4,200,000 – 175,000) 4,193,000
Utilities 4,365,000
Repair and maintenance 700,000
Depreciation (W-3.1) 5,847,500
Sundry expenses 640,000
Loss on disposal [750,000 – (800,000 – 40,000)] 10,000
Loss of inventory due to fire 500,000
Loss due to fraud 1,662,750
20,393,250
Excess of income over expenditures 17,855,500

Statement of Financial Position

2016 2016
Fund and Liabilities Assets
Rupees Rupees

Accumulated fund 39,181,500 Fixed asset - WDV (W-3) 45,742,500


(Balancing)

Creditors 3,330,000 Stock 2,500,000

Unearned subscription (W-1) 11,825,000 Prepaid rent 175,000

Cash and bank 5,919,000


(5,894,000+25,000 )

Total fund and liabilities 54,336,500 Total assets 54,336,500

2015
W-1: Determination of subscription income Rupees

Opening unearned subscription income (11,825,000(W-1.1)×9/10) 10,642,500

Add: Receipts for the year (3,300×10,000) 33,000,000

Less: Closing unearned subscription income (W-1.1) (11,825,000)

© Emile Woolf International 481 The Institute of Chartered Accountants of Pakistan


Financial accounting and reporting I

Subscription income for the year 31,817,500

W-1.1: Closing unearned subscription income

Quarter - 1 -

Quarter – 2 2,062,500

Quarter – 3 2,750,000

Quarter – 4 7,012,500

11,825,000

W-2: Tuck shop sales & cost of sales

Opening stock 2,300,000

Add: Purchases of stock (W-2.1) 18,985,000

Less: Loss due to fire, charged to expenditures (500,000)

Less: Closing stock (2,500,000)

Cost of goods sold 18,285,000

Sales (18,285,000/0.80) 22,856,250

2015
W-2.1: Purchases of tuck shop Rupees
Closing creditors 3,330,000
Add: Payments to creditors 18,155,000
Less: Opening creditors (2,500,000)
Purchases 18,985,000

W-3: Fixed assets and depreciation


Opening WDV before disposal 28,000,000
Add: Addition (7,350,000 + 17,000,000) 24,350,000
Less: WDV of assets disposed off (800,000 – 40,000) (760,000)
Less: Depreciation for the year (W-3.1) (5,847,500)
Closing WDV 45,742,500

W-3.1: Depreciation

Depreciation on opening WDV [(28,000,000–800,000)×20%] 5,440,000


Depreciation on disposed asset (800,000×20%×3/12) 40,000
Depreciation on addition (7,350,000×20%×3/12) 367,500

Depreciation for the year 5,847,500

© Emile Woolf International 482 The Institute of Chartered Accountants of Pakistan


Chapter 11: Accounting for Not-for-Profit Organisations

Example 12: SEAVIEW CLUB


Question: Seaview Club started its operations on 1 February 2015. Sponsor of the club contributed
Rs. 50 million towards general fund for the start of operations and placed the amount in the bank.
Following is the receipts and payments summary for the period from 1 February 2015 to 31
December 2015:

Receipts Rs. in ‘000 Payments Rs. in ‘000


Sponsor's contribution 50,000 Furniture & fixtures 1,200
Joining fees 20,800 Van 1,500
Subscription from members 29,952 Salaries 1,000
Sale of beverages 1,500 Rent 3,600
Utilities 570
Insurance 120
Repairs and maintenance 275
Purchase of beverages 1,367
Advance for plot of land 65,000
Balance 27,620
102,252 102,252

Additional information:
(i) The joining fee for award of membership is Rs. 50,000. Annual subscription is Rs. 24,000.
All new members pay three years’ subscription in advance.
The memberships were awarded as follows:

Month March June September December

No. of members 112 98 101 105

(ii) The club sells beverages at a gross profit margin of 20%. All sales are billed in the first week
of the next month and the payment is received in the same month. Sale of beverages during
December 2015 amounted to Rs. 150,000.
(iii) 25% of total purchases of beverages made during the year remained unsold at year-end.
(iv) Salaries are paid on the first day of next month. The amount of salaries includes an advance
amounting to Rs. 10,000 paid to an employee on 1 December 2015. The advance is
repayable on 1 February 2016.
(v) Rent for three years was paid in advance on 1 February 2015.
(vi) Presently the club is operating on rental premises. However, a plot of land has been
purchased on which construction would commence shortly. Title of land would be transferred
after completion of legal formalities.
(vii) Payments for utilities include security deposit paid to utility companies amounting to Rs.
20,000. Utility bills are paid on the 7th day of the next month.
(viii) Insurance premium was paid on 1 February 2015 covering a period of 12 months.
(ix) Repairs and maintenance include an advance of Rs. 100,000 paid to a contractor for
construction of a parking shed. Repair bills amounting to Rs. 7,000 were outstanding at year-
end.

© Emile Woolf International 483 The Institute of Chartered Accountants of Pakistan


Financial accounting and reporting I

(x) Furniture & fixtures and van were purchased on 1 February 2015. Depreciation on these
assets is to be charged at 10% and 20% respectively.

Required: Prepare statement of financial position as at 31 December 2015 and income &
expenditure account of Seaview Club for the period ended31 December 2015.
Answer:
Seaview Club
Income & Expenditure Account for the period ended 31 December 2015
Expenditure Rs.in ‘000 Income Rs.in ‘000
Salaries and wages (1000-10+99) 1,089 Joining fee 20,800
Rent (3,600/3x11/12) 1,100 Subscription income 4,630
(W-1)
Utilities (570-20+55) 605 Profit on sale of 330
beverages (W-2)
Insurance (120/12x11) 110
Repairs and maintenance (275–100+7) 182
Depreciation expense 385
(1,200x10%x11/12+1,500x20%x11/12)
Excess of income over expenditure 22,289
25,760 25,760

Statement of Financial Position as at 31 December 2015


General Fund & Liabilities Rs. in ‘000 Assets Rs. in ‘000

General fund 50,000 Non-Current Assets

Excess of income over 22,289 Land/Capital advance 65,000


expenditure

72,289 Furniture & fixtures (1,200110) 1,090

Van (1,500275) 1,225

Deferred income/long term Advance for parking shed 100


advance (W-1)
15,338 Long term deposits 20

Long term prepayment 1,300

Current Liabilities 68,735

Creditors (1,760 – 1,367) 393 Current Asset

Accrued expenses (7+55+99) 161 Stock (W-2) 440

Advance subscription (W-1) 9,984 Debtors for beverages (credit 150


sale)

10,538 Advance & prepayments (W-3) 1,220

Bank 27,620

29,430

Total General Fund & Liabilities 98,165 Total Assets 98,165

© Emile Woolf International 484 The Institute of Chartered Accountants of Pakistan


Chapter 11: Accounting for Not-for-Profit Organisations

W-1: Subscription income


Subscription for 3 years is Rs. 72,000 so subscription for 1 year is Rs. 24,000 or Rs. 2,000 per
month

No. of No. of Subscription income Deferred subscription


members months for the year income
Month
A×B×2,000 A×(36B)×2,000
A B
--------------- Rupees ---------------

March 112 10 2,240,000 5,824,000

June 98 7 1,372,000 5,684,000

September 101 4 808,000 6,464,000

December 105 1 210,000 7,350,000

4,630,000 25,322,000

Less: Short term (9,984,000)


[(112+98+101+105)×24,000]

Long term 15,338,000

W-2: Beverage sale results Rs. in ‘000

Sales (1,500 + 150) 1,650


Less: Cost of sales 1,760

Purchases (1,320/0.75) (440) 1,320


Closing stock (1,760×25%) 330

W-3: Advance & prepayments Rs. in ‘000

Rent [(3,600–1,100)–1,300(long term)] 1,200


Insurance (120 – 110) 10

Advance salary 10
1,220

© Emile Woolf International 485 The Institute of Chartered Accountants of Pakistan


Financial accounting and reporting I

4 ACCOUNTING STANDARD FOR NOT-FOR-PROFIT ORGANISATIONS (ASNPO)

Section overview

 Introduction
 Primary sources: basis of accounting
 Recognition and measurement
 Financial statements
 Fund accounting and inter-fund transfers and balances

4.1 Introduction
So far in this chapter, we have discussed and applied accounting concepts that are normally used
by many small NPOs including those which do not maintain proper double entry records. However,
some NPOs are required to comply with Accounting Standards for Not-for-Profit Organisations.
The Institute of Chartered Accountants of Pakistan (ICAP) issued the ‘Accounting Standard for
Not-for-Profit Organisations’ and as per Securities and Exchange Commission of Pakistan’s
(SECP) directives, ASNPO is applicable to associations not-for-profit registered under the
Companies Act, 2017.
ASNPO is applicable to NPOs registered under the Companies Act, 2017, however, for other NPOs
it is recommended to prepare financial statements in accordance with ASNPO and stating that its
financial statements have been prepared in accordance with ‘Accounting Standard for Not-for-
Profit Organisations’.
An NPO that is registered under the Companies Act, 2017 is also required to comply with the
presentation and disclosure requirements of the Fifth Schedule of the Companies Act, 2017.
ASNPO is applicable so far as not in conflict with the provisions of the Companies Act, 2017.
An NPO applying ASNPO will also apply the primary source of how to account and report
transactions and events.

4.2 Primary sources: basis of accounting


An NPO applying ASNPO will also apply the primary source of how to account and report
transactions and events. The primary source of how to account and report transactions and events
in financial statements will vary according to the class of the NPO.
When the concepts contained in ASNPO, conflict with a primary source, the requirements of the
primary source shall prevail.
It is necessary to refer to other sources when the primary sources do not deal with the accounting
and reporting in financial statements of transactions or events encountered by the NPO or when
additional guidance is needed to apply a primary source to specific circumstances.
Applicable financial reporting framework (primary source)
The primary source for public interest entities and large sized NPOs is IFRS issued by International
Accounting Standards Board (IASB) and as notified by SECP.
The primary source for medium and small sized NPOs is International Financial Reporting
Standards for Small and Medium-sized Entities (IFRS for SMEs) issued by IASB and as notified
by SECP.
Micro NPOs (defined in ASNPO as organisations not registered under Companies Act, 2017 with
annual gross revenue less than Rs. 25 million) may opt to prepare their accounts on cash and
disbursement basis.
However, if a micro NPO opt to prepare and present its financial statements on accrual basis, it
will prepare financial statements in accordance with ASNPO and Accounting and Financial
Reporting Standards for Small Sized Entities (AFRS for SSE) as applicable in Pakistan.

© Emile Woolf International 486 The Institute of Chartered Accountants of Pakistan


Chapter 11: Accounting for Not-for-Profit Organisations

Accounting policies
An NPO selects and applies its accounting policies for a period consistently for similar transactions,
other events and circumstances.
Accounting treatments that are not in accordance with ASNPO are not rectified either by disclosure
of the accounting policies used or by information provided in notes or supporting schedules.

4.3 Recognition and measurement


Recognition criteria
The recognition criteria under ASNPO are as follows:
 the item has an appropriate basis of measurement and a reasonable estimate can be made
of the amount involved; and
 for items involving obtaining or giving up economic benefits, it is probable that such benefits
will be obtained or given up.
The recognition criteria provide general guidance on when an item is recognized in the financial
statements. Professional judgment is required to consider the specific circumstances to identify
whether any particular item is recognized or not.

Revenue Revenues are generally recognized when performance is achieved and


reasonable assurance regarding measurement and collectability of the
consideration exists.
Unrestricted Unrestricted contributions to NPO do not normally arise from the sale of goods
contributions or the rendering of services and, consequently, performance achievement is
generally not relevant to the recognition of unrestricted contributions; such
revenues are generally recognized when received or receivable.
Restricted Restricted contributions are recognized based on the nature of the related
contributions restriction.
Gains Gains are generally recognized when realized.
Expenses Expenses and losses are generally recognized when an expenditure or
and losses previously recognized asset does not have future economic benefit. Expenses
are related to a period on the basis of transactions or events occurring in that
period or by allocation applying the matching concept.

Measurement
Financial statements of NPOs are prepared primarily using the historical cost basis of
measurement whereby transactions and events are recognized in financial statements at the
amount of cash or cash equivalents paid or received or the fair value ascribed to them when they
took place.
Financial statements are prepared with capital maintenance measured in financial terms and with
no adjustment being made for the effect on capital of a change in the general purchasing power of
the currency during the period.

4.4 Financial statements


Financial statements of NPO shall normally include:
 statement of financial position (or balance sheet)
 statement of income and expenditure
 statement of changes in net assets
 statement of cash flows.
Notes to financial statements and supporting schedules to which the financial statements are
cross-referenced are an integral part of such statements; the same does not apply to information
set out in other material attached to or submitted with financial statements.

© Emile Woolf International 487 The Institute of Chartered Accountants of Pakistan


Financial accounting and reporting I

Alternative titles of financial statements may be used in a manner that results in fair presentation.
Comparative information
Financial statements shall be prepared on a comparative basis, unless the comparative information
is not meaningful. Comparative information is normally meaningful. However, this may not be the
case in some rare circumstances, such as when the financial structure of the NPO has significantly
changed.

4.5 Fund accounting

Definition: Fund accounting


Fund accounting comprises the collective accounting procedures resulting in a self-balancing set
of accounts for each fund established by legal, contractual or voluntary actions of an NPO.
Elements of a fund can include assets, liabilities, net assets, revenues and expenses (and gains
and losses, where appropriate). Fund accounting involves an accounting segregation, although not
necessarily a physical segregation, of resources.

Net assets or fund balances may be internally or externally restricted. Internally restricted net
assets or fund balances are often referred to as reserves or appropriations.

Definition: Restrictions
Restrictions are stipulations imposed that specify how resources must be used. External
restrictions are imposed from outside the NPO, usually by the contributor of the resources. Internal
restrictions are imposed in a formal manner by the NPO itself, usually by resolution of the board
of directors/council/board of trustees.

An NPO that uses fund accounting in its financial statements should provide a brief description of
the purpose of each fund reported.
Financial statements that are reported using fund accounting may follow the multi-column format
whereby resources or similar groups of resources are each assigned a separate column. Other
formats may be used to report using fund accounting provided that financial information about the
NPO as a whole is presented in the NPO's financial statements.
An NPO may present its financial statements using different formats for the individual statements.
For example, a statement of income and expenditure and changes in net assets presented in the
multi-column format may be accompanied by a statement of financial position that presents assets,
liabilities and net assets in a single column without presenting each financial statement item by
individual fund.
There are two methods of fund accounting, restricted fund method and deferral method, both will
be discussed later in this chapter.
The funds can be classified into following three categories:
Endowment fund
An endowment fund is a self-balancing set of accounts which reports the accumulation of
endowment contributions. Only endowment contributions and investment income subject to
restrictions stipulating that it be added to the principal amount of the endowment fund would be
reported as revenue of the endowment fund.
Allocations of resources to the endowment fund that result from the imposition of internal
restrictions are recorded as inter-fund transfers.
Restricted fund
A restricted fund is a self-balancing set of accounts the elements of which are restricted or relate
to the use of restricted resources. Only restricted contributions, other than endowment
contributions, and other externally restricted revenue would be reported as revenue in a restricted
fund.

© Emile Woolf International 488 The Institute of Chartered Accountants of Pakistan


Chapter 11: Accounting for Not-for-Profit Organisations

Allocations of resources that result from the imposition of internal restrictions are recorded as inter-
fund transfers to the restricted fund.
General fund / unrestricted fund
A general fund is a self-balancing set of accounts which reports all unrestricted revenue and
restricted contributions for which no corresponding restricted fund is presented. The fund balance
represents net assets that are not subject to externally imposed restrictions.

Example 13: Types of funds


Question: Consider the following independent circumstances:
(a) A professional body of accountants (the NPO) sets-up a fund for financial support of deserving
students. For this purpose, Rs. 100 million have been allocated that will be invested and 80%
of the investment income shall be used for student support and 20% of investment income
shall be added to fund investments. The fund investments shall not be available for use by the
NPO for its operations and the NPO shall preserve the principal amount of fund.
(b) A healthcare NPO has raised money through special marketing drive in which overseas
contributors deposited $100 each in its ‘Save a life fund’ account. The contributions shall be
used for the NPO’s routine operations which focuses on providing life-saving drugs to patients
who cannot afford the cost.
(c) A educational NPO has set-up a fund for development of new school in nearby rural area. The
fund-raising drive has been successful as many people have contributed for the cause. The
fund-raising clearly stated that the funds so raised shall only be used for construction and
operations of school at that specific location.
Required:
Identify the type of above funds.
Answer:
(a) Endowment fund
(b) General fund
(c) Restricted fund

© Emile Woolf International 489 The Institute of Chartered Accountants of Pakistan


Financial accounting and reporting I

5 ASNPO: CONTRIBUTION REVENUE AND RECEIVABLE

Section overview

 Contribution
 Contribution receivable
 Membership fee
 Government funding
 Contributed materials and services
 Revenue recognition: restricted fund method
 Revenue recognition: deferral method

5.1 Contribution
Contributions can come from many sources, including individuals, corporations, governments and
other NPOs. Contributions include contributions receivable that meet the criteria for recognition in
the financial statements.

Definition: Contribution
A contribution is a non-reciprocal transfer to an NPO of cash or other assets or a non-reciprocal
settlement or cancellation of its liabilities. Government funding provided to an NPO is considered
to be a contribution.

Restrictions
Restrictions (explicit or implicit) on contributions may only be externally imposed.
Types of contribution

Restricted A restricted contribution is a contribution subject to externally imposed


contribution stipulations that specify the purpose for which the contributed asset is to be
used.

Endowment An endowment contribution is a type of restricted contribution subject to


contribution externally imposed stipulations specifying that the resources contributed be
maintained permanently, although the constituent assets may change from time
to time.

Unrestricted An unrestricted contribution is a contribution that is neither a restricted


contribution contribution nor an endowment contribution.

Example 14: Types of contributions


Question: Consider the following independent circumstances:
(a) A healthcare NPO received Rs. 10 million from wealthy individuals subject to the condition that
this amount shall only be used for acquisition of land for construction of a hospital in a specific
village.
(b) A healthcare NPO received Rs. 25 million contribution from a wealthy individual in the year
20X2. The sole purpose of the amount is to support the NPO’s general operations in the year
20X4 and 20X5.
(c) A educational NPO received a plot of land from Mr. Salman subject to the condition that this
land shall only be used for construction of a primary education school to be run by that NPO.
The fair value of this plot of land is Rs. 12 million.

© Emile Woolf International 490 The Institute of Chartered Accountants of Pakistan


Chapter 11: Accounting for Not-for-Profit Organisations

(d) A educational NPO received a plot of land from Mr. Jamal subject to the condition that this land
or sale proceeds from its disposal shall only be used to achieve general objectives of that NPO.
The fair value of this plot of land is Rs. 15 million.
(e) An educational NPO received Rs. 50 million from alumni donors subject to the condition that
the principal balance shall be invested as per specified investment policy and NPO cannot use
the principal balance to fund operations. However, the NPO can utilise the investment earnings
to pay for things such as academic programs or building new school facilities.
Required:
Identify the type of contributions in above circumstances.
Answer:
(a) Restricted contribution
(b) Restricted contribution
(c) Restricted contribution
(d) Unrestricted contribution
(e) Endowment contribution

Revenue recognition
Revenue from contributions is recognised by following either restricted fund method or deferral
method. An NPO is required to select one method and apply it consistently over the periods and
any change from one method to the other shall be treated as change in accounting policy.

5.2 Contribution receivable


Recognition
A contribution receivable should be recognized as an asset when it meets the following criteria:
 the amount to be received can be reasonably estimated; and
 ultimate collection is reasonably assured.
Pledge
A pledge is a promise to contribute cash or other assets to an NPO. Similar to any other contribution
receivable, an uncollected pledge would only be recognized:
 if it meets the above recognition criteria;
 there is reasonable assurance that the NPO will comply with conditions, if any, attached to
the contribution; and
 contribution is not dependant on any contingent event outside NPO’s control.
Bequest
Bequests are often subject to considerable uncertainty surrounding both the timing of the receipt
and the amount that will actually be received. In many cases, the recognition criteria will not be
satisfied and the bequest will not be recognized until it is received.

5.3 Membership fee


Many NPOs receive membership fees. Such fees are considered fees for services when members
receive services having a value commensurate with fees paid. In other cases, membership fees
may be in substance contributions.
An NPO would decide whether its membership fees are contributions or fees for services and
account for them accordingly on a consistent basis. Some membership fees have characteristics
of both fees for services and contributions. Such fees would be divided into the portion that relates
to fees for services and the portion that is in substance a contribution.

© Emile Woolf International 491 The Institute of Chartered Accountants of Pakistan


Financial accounting and reporting I

Example 15: Membership fee


Question: ABC Golf Club is members only club providing its members with sports facilities in the
grounds owned and maintained by it against annual subscription fee.
At 30 June 20X2, the club had membership subscriptions in arrears amounting to Rs. 48,000,000
and had received Rs. 12,000,000 in advance.
During the year to 30 June 20X3, the club received Rs. 650,000,000 from its members. This
amount includes:
 Rs. 26,000,000 received as donation from members (no conditions attached).
 Rs. 31,200,000 received for membership fee for the year to 30 June 20X4.
At 30 June 20X3, members owed Rs. 19,200,000 of subscriptions.
Half of the members who were in arrears at the end of the previous period still had not paid by 30
June 20X3. It was decided to write these amounts off.
Required:
How the revenue from above should be reported in financial statements of ABC Golf Club for the
year ended on 30 June 20X3?
Answer:
The donation of Rs. 26 million received shall be recognised as contribution revenue separately from
fee for services to members.
The subscription income (fee for services) may be calculated as follows:

Subscription income

Rs. Rs.

b/d 48,000,000 b/d 12,000,000

I&E 600,000,000 Cash (Rs. 650m – 26m) 624,000,000

Bad debts (Rs. 48m x 50%) 24,000,000

c/d 31,200,000 c/d 19,200,000

679,200,000 679,200,000

The subscription income may be reported in statement of income and expenditure at Rs. 600
million (gross basis) or at Rs. 576 million (net of bad debts expense).

5.4 Government funding


Certain types of government funding are calculated and paid as if they were fees for services.
However, because the services being funded are provided to the NPO's community of service, and
not directly to the government funder, government funding is considered to be a contribution.
Government funding is a significant component of many NPOs’ total contributions. Restrictions on
government funding may be indicated by following factors:
 the fact that the funding is provided based on the NPO's approved operating budget.
 the requirement to report to the government funder as to how the resources were actually
used.
 the funding left over at the end of the period must be returned to the government funder.
 the funding received relates to expenses of the future period being funded.

© Emile Woolf International 492 The Institute of Chartered Accountants of Pakistan


Chapter 11: Accounting for Not-for-Profit Organisations

Example 16: Government funding


Question: Mujahid Healthcare (MH) is a registered NPO. It has received government funding of Rs.
20 million for which it has to provide vaccine (dosage and administration) for a viral disease to
general public (8,000 dosages x Rs. 2,500 each) without taking any fee from them.
Required:
Discuss the accounting treatment of above from perspective of MH.
Answer:
The amount of Rs. 20 million is being calculated on dosage basis (i.e. 8,000 dosages x Rs. 2,500)
which might indicate that Rs. 20 million should be recognised as fee-for-services in statement of
income and expenditure.
However, since the service is not being provided to government but rather to MH’s community of
service (i.e. general public to whom they provide healthcare services), the government funding of
Rs. 20 million shall be considered as contribution.
Further, since the purpose of government funding is specified, it shall be considered as restricted
contribution.

5.5 Contributed materials and services


A contribution of assets other than cash would be measured at fair value estimated using market
or appraisal values. For contributed materials and services that are normally purchased, fair value
would be determined in relation to the purchase of similar materials and services.
Contributed materials and services that are part of a constructed or developed capital asset would
be recognized at fair value.
The NPO may choose to recognize contributions of materials and services, but should do so only
when:
 a fair value can be reasonably estimated; and
 when the materials and services are used in the normal course of the NPO's operations and
would otherwise have been purchased.
Often these contributions are not recorded because of record-keeping and valuation difficulties.
For example, it may be impractical to record the receipt of contributed services where the NPO
depends heavily on the use of volunteers to provide services. Where contributed materials and
services meet the criteria of fair value measurement, recording their value would provide useful
information.

5.6 Revenue recognition: restricted fund method


Restricted fund method
The restricted fund method of accounting for contributions is a specialized type of fund accounting
that involves the reporting of details of financial statement elements by fund in such a way that the
NPO reports:
 total general funds;
 one or more restricted funds; and
 an endowment fund, if applicable.
Reporting of financial statement elements segregated on a basis other than that of use restrictions
(for example, by program or geographic location) does not constitute the restricted fund method.

© Emile Woolf International 493 The Institute of Chartered Accountants of Pakistan


Financial accounting and reporting I

The contributions and related income are recognised as follows:

Contribution / income Recognition

Endowment contributions Recognise as revenue of the endowment fund in the current


period.

Restricted contributions Restricted contributions for which a corresponding restricted fund


reported in restricted fund is presented should be recognized as revenue of that fund in the
current period.

Restricted contributions Restricted contributions for which no corresponding restricted


reported in general fund fund is presented should be recognized in the general fund in
accordance with the deferral method.
In such case, deferred contributions balances should be
presented in the statement of financial position outside net assets
as liability.

Unrestricted contributions Recognise as revenue of the general fund in the current period.

Allocation of unrestricted This allocation shall be reported as an inter-fund transfer in the


resources to restricted fund statement of changes in net assets.

Net investment income Externally restricted investment income that must be added to
(includes revenue, gains or principal resources held for endowment, recognise as direct
losses on investments) increase or decrease in net assets.
Other externally restricted investment income, as per the type of
restrictions discussed above.
In case there is no external restriction, recognise in the statement
of income and expenditure.

5.7 Revenue recognition: deferral method


When an NPO uses fund accounting in their financial statements without following the restricted
fund method, contributions would be accounted for using the deferral method. The contributions
and related income are recognised as follows:

Contribution / income Recognition

Endowment contributions Recognise as direct increases in net assets in the current period
and excluded from revenue.

Restricted contributions for Recognise as revenue in current period.


expenses of current period

Restricted contributions for Defer and recognise as revenue in the same period(s) as the
expenses of future periods related expenses are recognised.
When the only restriction on a contribution is that it cannot be used
until a particular future period, the total amount of the contribution
would be recognized as revenue in that future period, whether or
not it has been spent.

Restricted contributions for In case of depreciable assets, defer and recognise as revenue on
the purchase of capital the same basis as the depreciation/amortisation expense related
assets to the acquired capital assets.
In case of non-depreciable assets, recognise as direct increase in
net assets.

© Emile Woolf International 494 The Institute of Chartered Accountants of Pakistan


Chapter 11: Accounting for Not-for-Profit Organisations

Contribution / income Recognition


In order for a contribution to be accounted for as a contribution
restricted for the purchase of a capital asset, the contributor must
specify the portion of the contribution that is to be used to
purchase capital assets. If the contributor does not so specify,
then the contribution would be recognized as revenue when spent
for the particular purpose covered by the restriction, regardless of
the fact that some of the expenditures may relate to the purchase
of capital assets.

Restricted contributions for In case debt was incurred to fund expenses of future periods,
the repayment of debt defer and recognise as revenue in same period(s) as the related
expenses are recognised.
In case debt was incurred to fund the purchase of capital asset
(depreciable), defer and recognise as revenue on the same basis
as the depreciation/amortisation expense related to the acquired
capital assets.
In case debt was incurred to fund the purchase of capital asset
(non-depreciable), recognise as direct increase in net assets.
Otherwise, recognise as revenue in the current period.

Unrestricted contributions Recognise as revenue in the current period.

Net investment income Externally restricted investment income that must be added to
(includes revenue, gains or principal resources held for endowment are recognised as direct
losses on investments) increase or decrease in net assets.
Other externally restricted investment income are recognised as
per the type of restrictions discussed above.
In case there is no external restriction, recognise in the statement
of income and expenditure.

Deferred contributions balances should be presented in the statement of financial position


outside net assets as liability.

© Emile Woolf International 495 The Institute of Chartered Accountants of Pakistan


Financial accounting and reporting I

6 ASNPO: INVENTORIES AND NON-CURRENT ASSETS

Section overview

 Inventories
 Collections
 Property, plant and equipment
 Intangible assets

6.1 Inventories
Contribution of materials
When an NPO recognizes contributions of materials and goods, the cost of inventories shall reflect
the fair value at the date of contribution.
To be distributed at no charge or for a nominal charge
An NPO shall measure inventories at the lower of cost and current replacement cost when they
are held for:
 distribution at no charge or for a nominal charge; or
 consumption in the production process of goods to be distributed at no charge or for a
nominal charge.

Example 17: Inventories


Question: Medicine-for-All is an NPO which provides medicine to communities living in
underdeveloped areas at nominal charge. It has following inventories:
Replacement
Cost NRV* Fair value
Item Type cost
Rupees
Panadol Received in kind Nil 6,000 26,000 28,000
Neubrol 24,000 4,000 24,500 25,000
Imodium 12,000 3,000 12,000 12,500
Purchased for cash
Motilium 15,000 2,500 14,700 15,200
Rijix 18,000 3,500 18,300 17,900

*provided at nominal charge


Required:
Calculate the amount of inventory that should be presented in the statement of financial position
of Medicine-for-All from above data.
Answer:
Item Basis Rupees
Panadol Cost equal to fair value but replacement cost is lower 26,000
Neubrol Cost (lower) 24,000
Imodium Cost / replacement cost (equal) 12,000
Motilium Replacement cost (lower) 14,700
Rijix Cost (lower) 18,000
Total 94,700
.

© Emile Woolf International 496 The Institute of Chartered Accountants of Pakistan


Chapter 11: Accounting for Not-for-Profit Organisations

6.2 Collections
Definition: Collections
Collections are works of art, historical treasures or similar assets that are:
 held for public exhibition, education or research;
 protected, cared for and preserved; and
 subject to an organisational policy that requires any proceeds from their sale to be used
to acquire other items to be added to the collection or for the direct care of the existing
collection.

Although items meeting the definition of a collection exhibit the characteristics of ‘assets’ they are
excluded from the definition of property, plant & equipment, and intangible assets. Collections are
made up of items that are often rare and unique. They have cultural and historical significance.

Although collections are usually held by museums or galleries, other NPOs may also have items
that meet the definition of a collection. For example, an NPO's library may include rare books which
might be considered to be a collection. The regular library materials, however, would not usually
meet the definition of a collection.
NPOs holding collections act as custodians for the public interest. They undertake to protect and
preserve the collection for public exhibition, education or research. The existence of a policy
requiring that any proceeds on the sale of collection items be used to acquire additional items or
for the direct care of the collection provides evidence of the NPO's commitment to act as custodian
of the collection.
Capitalisation
The cost of capitalizing collections often would exceed the incremental benefit of the information
gained, especially for NPOs that have been in existence for several decades. Accordingly, although
the capitalization of collections is not precluded, it is not required.
Certain works of art and historical treasure not to be depreciated
Certain works of art and historical treasures may have lives that are so long as to be virtually
unlimited. Works of art and historical treasures in this category are those that have cultural,
aesthetic, or historical value that is worth preserving perpetually. In addition, the NPO must have
the technological and financial ability to continue to protect and preserve them. Works of art and
historical treasures of this type would not be depreciated.

6.3 Property, plant and equipment


Definition: Tangible capital assets
Tangible capital assets are identifiable tangible assets that meet all of the following criteria:
 are held for use in the provision of services, for administrative purposes, for production of
goods or for the maintenance, repair, development or construction of other tangible
capital assets;
 have been acquired, constructed or developed with the intention of being used on a
continuing basis;
 are not intended for sale in the ordinary course of operations; and
 are not held as part of a collection.

Recognition
Property, Plant and Equipment (PPE) shall be recognized as an asset, if and only if:
 it is probable that future economic benefits associated with the item will flow to the NPO;
and
 the cost of the item can be measured reliably.

© Emile Woolf International 497 The Institute of Chartered Accountants of Pakistan


Financial accounting and reporting I

Measurement for contributed assets


A contributed asset would be recognized at its fair value at the date of contribution. Fair value of a
contributed asset may be estimated using market or appraisal values.
When an estimate of fair value cannot reasonably be made, both the asset and the related
contribution would be recognized at nominal value.
A tangible capital asset purchased by an NPO at a value substantially below fair value would also
be recognized at its fair value with the difference between the consideration paid for the tangible
capital asset and fair value reported as a contribution.
A tangible moveable capital asset procured from a grant may be recognised at carrying amount
deducting the grant. The grant is recognised in profit or loss over the life of the depreciable asset
as a reduced depreciation expense.
If it is a grant for a specified period and the asset has to be returned at the end of the grant period,
asset shall be valued at fair value less present value of the estimated residual amount at the time
of grant / contribution.
Construction or development over time
The cost of PPE includes direct construction or development costs (such as materials and labour)
and overhead costs directly attributable to the construction or development activity. PPE which is
developed or constructed by an NPO might include contributed materials or labour, which would
be recognized at fair value at the date of contribution.
Land
Land normally has an unlimited life and would not be depreciated.
Depreciation /Amortisation (under fund accounting)
When a fund accounting basis of reporting is used, the choice of the fund or funds to which
depreciation expense would be charged would be based on providing the most meaningful
presentation.
Some NPOs may wish to show depreciation as an expense of the operating fund. This presentation
emphasizes that depreciation is part of the cost of service delivery.
Other NPOs may prefer to show depreciation as an expense of the PPE fund. This presentation
shows all revenues and expenses associated with tangible capital assets in a single fund.
Unamortised deferred contributions
When PPE no longer contribute to the NPO's ability to provide services, its carrying amount would
be written down to residual value, if any. A write-down would be necessary, for example, when the
NPO no longer plans to use the asset because it has been damaged or rendered obsolete.
When an asset's carrying amount is written down, a corresponding amount of any unamortized
deferred contributions related to the asset would be recognized as revenue, provided that all
restrictions have been complied with.

6.4 Intangible assets


Definition: Intangible asset
Intangible asset is defined as an identifiable non-monetary asset without physical substance held
for use in the production or supply of goods or services, for rental to others, or for administrative
purposes.

Recognition
The NPO shall recognize an intangible asset as an asset if, and only if:
 it is probable that the expected future economic benefits that are attributable to the asset will
flow to the NPO; and
 the cost or value of the asset can be measured reliably.

© Emile Woolf International 498 The Institute of Chartered Accountants of Pakistan


Chapter 11: Accounting for Not-for-Profit Organisations

Internally generated assets: Expense


Internally Generated research costs, goodwill, brands, training costs are always expensed out.
Expenditure on intangible item that was initially recognized as an expense shall not subsequently
be capitalized as part of the cost of an intangible asset.
Internally generated assets: Capitalised
Development costs, which is the next step after research phase, can be capitalized if all of the
following conditions are fulfilled:
(a) the technical feasibility of completing the asset;
(b) the intention to complete the asset exists;
(c) the ability to use or sell the asset;
(d) how the asset will generate the future economic benefit and ability to demonstrate the
existence of market;
(e) the availability of adequate resources to complete; and
(f) the NPO's ability to reliably measure the cost of development of the asset.
Website costs
Website costs are categorized into five basic stages that are:
 Stage 1: planning stage
 Stage 2: application and infrastructure development
 Stage 3: the graphical design development
 Stage 4: content development
 Stage 5: operations.
Costs incurred in stage 1 and stage 5 are always expensed. However, costs incurred from stage
2 to 4 can be capitalized if it fulfils the criteria of capitalisation of development asset discussed
earlier, particularly criteria (d).
Unamortised deferred contributions
Same requirements as for PPE.
Contributed intangible assets
Same requirements as for PPE.

© Emile Woolf International 499 The Institute of Chartered Accountants of Pakistan


Financial accounting and reporting I

7 ASNPO: PREPARATION OF FINANCIAL STATEMENTS

Section overview

 General
 Statement of financial position
 Statement of income and expenditure
 Statement of changes in net assets
 Statement of cash flows

7.1 General
The accounting and approach for preparation of financial statements of an NPO is similar to other
entities except for the issues specifically addressed in ASNPO.
This section will discuss general presentation requirements and formats of following:
 statement of financial position (or balance sheet)
 statement of income and expenditure
 statement of changes in net assets
 statement of cash flows.

7.2 Statement of financial position


The preparation of financial statements of an NPO is same
The statement of financial position should present the following:
 net assets subject to restrictions to be maintained permanently as endowments;
 designated net assets;
 unrestricted net assets; and
 total net assets.
Information about the NPO's liquidity is presented by classifying current assets separately from
noncurrent assets and current liabilities separately from non-current liabilities. Cash and other
assets subject to external restrictions limiting their use to beyond one year from the date of the
statement of financial position would be classified as non-current assets.
Under the deferral method of accounting for contributions, endowment contributions are
accumulated in the net assets balance. Internally restricted balances are reflected as
appropriations of unrestricted net assets in the net assets balance. Externally restricted
contributions are accumulated in the statement of financial position as deferred contributions.
Under the restricted fund method of accounting for contributions, endowment contributions are
accumulated in the endowment fund balance. Other internally and externally restricted
contributions are accumulated in the statement of financial position as part of the appropriate
restricted fund balance. If there is no appropriate restricted fund, externally restricted contributions
are accumulated as deferred contributions in the general fund.

© Emile Woolf International 500 The Institute of Chartered Accountants of Pakistan


Chapter 11: Accounting for Not-for-Profit Organisations

Illustration 19:
Statement of financial position (Format)
Not-for-Profit Organisation
Statement of financial position
As at 31 December 20X2

20X2 20X1
Non-current assets Rs. 000 Rs. 000
Capital assets /property and equipment) 1,987 XX
Intangible assets 50 XX
Collections 80 XX
Investments 4,157 XX
6,274 XX
Current assets
Office supplies stock 55 XX
Prepaid expenses 58 XX
Grants/contribution receivable 17 XX
Cash and cash equivalents 183 XX
313 XX
6,587 XXX

Fund balances / Net assets


Net assets: restricted for endowments 208 XX
Net assets: Externally restricted for specific projects 241 XX
Net assets: internally restricted for special projects 340 XX
General fund / Unrestricted net assets 2,698 XX
3,487 XX
Non-current liabilities
Deferred grants/contributions 1,800 XX
Loans 300 XX
2,100 XX
Current liabilities
Deferred grants/contributions 600 XX
Accrued expenses 400 XX
1,000 XX
6,587 XX

© Emile Woolf International 501 The Institute of Chartered Accountants of Pakistan


Financial accounting and reporting I

Illustration 20:
Statement of financial position (Multi-Columnar Format)
Not-for-Profit Organisation
Statement of financial position
As at 31 December 20X2
20X2 20X1
Rs. 000 Rs. 000
General Special
Endowment Total
Non-current assets operations projects
Capital assets 1,580 407 1,987 XX
Intangible assets 50 50 XX
Collections 80 80 XX
Investments 3,052 897 208 4,157 XX
4,762 1,304 208 6,274 XXX
Current assets
Office supplies stock 52 3 55 XX
Prepaid expenses 51 7 58 XX
Grants/contribution receivable 17 17 XX
Cash and cash equivalents 166 17 183 XX
286 27 0 313 XX
5,048 1,331 208 6,587 XXX

Fund balances / Net assets


Externally restricted 241 208 449 XX
Internally restricted 340 340 XX
Unrestricted 2,698 2,698 XX
2,698 581 208 3,487 XX
Non-current liabilities
Deferred grants/contributions 1,300 500 1,800 XX
Loans 300 300 XX
1,600 500 0 2,100 XX
Current liabilities
Deferred grants/contributions 400 200 0 600 XX
Accrued expenses 350 50 400 XX
750 250 0 1,000 XX
5,048 1,331 208 6,587 XX

7.3 Statement of income and expenditure


Classification of expenses
NPO may classify expenses in the statement of income and expenditure:
 by object (for example, salaries, rent, utilities);
 by function (for example, administrative, research, ancillary operations); or
 by program.

© Emile Woolf International 502 The Institute of Chartered Accountants of Pakistan


Chapter 11: Accounting for Not-for-Profit Organisations

An NPO would classify its expenses in the manner that results in the most meaningful presentation
in the circumstances. Whether the NPO prepares its budgets by function or object would be a factor
to consider in deciding which method of expense classification would be most appropriate for the
NPO's financial statements.
Attribution of expenses (under classification by function)
When attributing an expense among various operating functions, an NPO considers an approach
such as the following:
 an expense that contributes directly to the output of one function is applied directly to that
function, for example, the cost of a staff member exclusively devoted to that function.
 an expense that contributes directly to the output of more than one function is attributed on
a reasonable and consistent basis to each function to which it applies (for example, the rent
applicable to the space used for more than one separately reported function, and the
remuneration expense of an executive director of a small health care NPO who, in addition
to managing the NPO, provides direct health care services to clients of that NPO).
Statement of income and expenditure — deferral method
The statement of income and expenditure should present:
 for each financial statement item, a total that includes all funds reported; and
 total excess or deficiency of revenues and gains over expenses and losses for the period.
Under the deferral method of accounting for contributions, total excess of revenues over expenses
for all funds reports the change in the NPO's unrestricted resources in the period.

Illustration 21:
Statement of income and expenditure (Format)
Not-for-Profit Organisation
Statement of income and expenditure (deferral method)
For the year ended 31 December 20X2
20X2 20X1
Income Rs. 000 Rs. 000
Fee-for-services 5,300 XX
Government grants 1,200 XX
Contributions 170 XX
Fundraising events 350 XX
Investment income 31 XX
Other income 2 X
7,053 XXX

Expenditures
Salaries 3,070 XX
Rent 1,320 XX
Office supplies used 610 XX
Utilities 880 XX
Marketing and communications 422 XX
Amortisation of capital assets 153 XX
(6,455) (XXX)
Excess of income over expenditure 598 XX

© Emile Woolf International 503 The Institute of Chartered Accountants of Pakistan


Financial accounting and reporting I

Statement of income and expenditure — restricted fund method


The statement of income and expenditure should present the following for the period:
 the total for each financial statement item recognized in the general fund;
 the total for each financial statement item recognized in the restricted funds, other than the
endowment fund;
 the total for each financial statement item recognized in the endowment fund; and
 excess or deficiency of revenues and gains over expenses and losses for each of the general
fund, restricted funds other than the endowment fund and the endowment fund.
Under the restricted fund method of accounting for contributions, the general fund presents all
revenues and expenses related to unrestricted resources.

Illustration 22:
Statement of income and expenditure (Format)
Not-for-Profit Organisation
Statement of income and expenditure (restricted fund method)
For the year ended 31 December 20X2
20X2 20X1
Rs. 000 Rs. 000
General Restricted Endowment
Total
Income fund fund fund
Fee-for-services 5,300 5,300 XX
Government grants 1,200 500 1,700 XX
Contributions 170 20 20 210 XX
Fundraising events 350 350 XX
Investment income 31 8 18 57 XX
Other income 2 2 X
7,053 528 38 7,619 XXX
Expenditures
Salaries 3,070 320 3,390 XX
Rent 1,320 1,320 XX
Office supplies used 610 20 630 XX
Utilities 880 57 937 XX
Marketing & communications 422 422 XX
Amortisation of capital assets 153 30 183 XX
(6,455) (427) 0 (6,882) (XX)
Excess of income over expenditure 598 101 38 737 XX

Presentation of revenues and expenses — gross versus net


Revenues and expenses should be recognized and presented at their gross amounts and this
information may be presented in the notes to the financial statements. The determination of
whether to report the revenues and expenses on a gross or net basis depends on the relative facts
and circumstances and requires significant judgment.

© Emile Woolf International 504 The Institute of Chartered Accountants of Pakistan


Chapter 11: Accounting for Not-for-Profit Organisations

Example 18: Funded development


Question: An NPO receives funding to undertake a specific research project. The NPO contracts at
its own discretion with a scientist to perform the research. The NPO would not have undertaken the
research project had the funds not been made available.
Required:
Whether the funding revenue and cost of scientist’s services be presented on gross basis or net
basis?
Answer:
Although the NPO would not have undertaken the research project without the availability of the
funding, the NPO acts as the principal in contracting with the scientist. It specifies the details of the
research to be carried out by the scientist, and has discretion in selecting the scientist and in
establishing the price to be paid. Thus, the expenses incurred are obligations of the NPO. The
funding revenue and cost of scientist services should be presented on gross basis in statement of
income and expenditure.

Example 19: Funded development


Question: An NPO receives funding (reimbursement) to undertake a specific research project from
a textile company. The NPO allocates an employee to textile company for the conduct of research.
The NPO would be reimbursed for all the costs related to that employee.
Required:
Whether the reimbursement and employee-related costs be presented on gross basis or net basis?
Answer:
The NPO has an employee who is seconded to a textile company to work under their direction and
the NPO is reimbursed for all of the costs related to that employee. As the NPO is the employer,
they would report their employee-related costs as expenses and would report the reimbursement
of their costs as revenues on gross basis in statement of income and expenditure.

Example 20: Direct fundraising activities


Question: An NPO engages in a number of fundraising activities, including a fundraising telethon,
a telephone campaign, a direct mail campaign, special events and a lottery. The NPO uses an
outside fundraising consultant to conduct the telethon and uses the NPO's own staff and volunteers
in the telethon and the other activities. Funds solicited in each of the activities are raised in the
name of the NPO.
Required:
Whether the fundraised and related costs be presented on gross basis or net basis?
Answer:
Even though the NPO uses an outside fundraising consultant to conduct the telethon, the NPO is
the principal in the relationship with the donors as the funds are raised in its name. The NPO has
discretion in selecting the outside fundraiser, in establishing the fees to be paid and in determining
the specifications of the telethon. The NPO also has the credit risk if donors to the telethon do not
pay according to their pledge. Thus, the NPO should recognize the gross amounts fundraised in
each of the activities as revenue of the NPO, and the total expenses of each activity, including the
fees charged by any outside party, as expenses of the NPO, separately.

© Emile Woolf International 505 The Institute of Chartered Accountants of Pakistan


Financial accounting and reporting I

Example 21: Fund raising conducted by others


Question: An NPO is actively engaged in helping communities in flood affected area. A group of
students organised a music concert, announcing that the net proceeds of the event shall be given
to the NPO.
Required:
Whether to report the revenue and costs of the event on gross basis or net basis?

Answer:
The NPO is not the principal in the fundraising event as it was not involved in organizing the event
and did not bear any risks in connection with it. The amount received by the NPO is a donation from
the organizers of the event. Neither the gross revenues nor the gross expenses of the event are
recognized in the NPO's financial statements. The net proceeds received are recognized as a
contribution. Disclosure of gross revenues and expenses is not required.

7.4 Statement of changes in net assets


The statement of changes in net assets should present changes in the following for the period:
 restricted net assets (to be maintained permanently as endowments);
 internally restricted net assets;
 externally restricted net assets (other than endowment assets);
 unrestricted net assets; and
 total net assets.
The statement of changes in net assets may be referred to as ‘the statement of changes in fund
balances’ when the NPO uses fund accounting in its financial statements.
Inter-fund transfers should be presented in the statement of changes in net assets. Allocations of
revenues and expenses between funds that are made when the NPO first recognizes the revenue
or expense are not considered to be transfers.

Illustration 23:
Statement of changes in net assets (Format)
Not-for-Profit Organisation
Statement of changes in net assets (deferral method)
For the year ended 31 December 20X2
Internally Externally
Externally
Unrestricted restricted restricted
restricted Total
General fund special fund endowment
fund
fund
Rs. 000
Balance 1 Jan 2,145 315 140 150 2,750
Surplus 598 598
Endowment
20 20
Contributions
Restricted grants &
520 520
contributions
Investment income 8 18 26
Fund utilisation (427) (427)
Internally imposed
(25) 25 0
restrictions
Transfers (20) 20 0
Balance 31 Dec 2,698 340 241 208 3,487

© Emile Woolf International 506 The Institute of Chartered Accountants of Pakistan


Chapter 11: Accounting for Not-for-Profit Organisations

Illustration 24:
Statement of changes in net assets (Format)
Not-for-Profit Organisation
Statement of changes in net assets (restricted fund method)
For the year ended 31 December 20X2

Internally Externally
Externally
Unrestricted restricted restricted
restricted Total
General fund special fund endowment
fund
fund

Rs. 000

Balance 1 Jan 2,145 315 140 150 2,750

Surplus 598 101 38 737

Internally imposed
(25) 25 0
restrictions

Transfers (20) 20 0

Balance 31 Dec 2,698 340 241 208 3,487

4.5 Statement of cash flows


Operating activities
Cash flows from operations include all cash receipts and disbursements resulting from the main,
ongoing service delivery activities of an NPO and exclude cash flows from financing and investing
activities.
Cash receipts from operations include unrestricted contributions, restricted contributions that are
to be used for operations and other revenues arising from the NPO's ordinary activities, such as
fees for services, proceeds on the sale of goods and unrestricted investment income.
Cash disbursements for operations would comprise expenditures made by the NPO in carrying out
its service delivery activities.
Investing activities
Components of cash flows from investing activities would include the acquisition of capital assets,
the purchase of investments, and the proceeds on disposal of major categories of assets, such as
capital assets and investments.
Financing activities
Components of cash flows from financing activities would include cash contributed that is restricted
for the purpose of acquiring capital assets and cash contributed for endowment. Cash receipts and
disbursements related to the assumption and repayment of debt would also be presented as
components of cash flows from financing activities.

© Emile Woolf International 507 The Institute of Chartered Accountants of Pakistan


Financial accounting and reporting I

Illustration 25:
Statement of cash flows (Format)
Not-for-Profit Organisation
Statement of cash flows
As at 31 December 20X2

20X2 20X1
Cash flows from operating activities Rs. 000 Rs. 000
Surplus (deficit) of income over expenditure 197 (14)
Adjustments:
Amortisation of capital assets 24 30
Amortisation of deferred grant / contributions (84) (80)
Finance cost 16 12
153 (52)
Changes in non-cash working capital balances
Office supplies (18) (8)
Prepaid expenses (3) 4
Accrued expenses 8 (11)
Cash generated from operations 140 (67)
Interest paid (20) (15)
Grants and contributions received 280 250
Net cash from operating activities 400 168

Cash flows from investing activities


Purchase of capital assets (800) (300)
Purchase of investments (1,400) (1,100)
Sale proceeds from investments 81 75
Investment income received 11 18
Net cash used in investing activities (2,108) (1,307)

Cash flows from financing activities


Loan obtained (repaid) 100 (500)
Endowment contributions 1,250 950
Restricted contributions for capital assets 300 800
Cash flows from financing activities 1,650 1,250

Net increase (decrease) in cash and cash equivalent (58) 111


Cash and cash equivalents at beginning of year 211 100
Cash and cash equivalents at end of year 153 211

© Emile Woolf International 508 The Institute of Chartered Accountants of Pakistan


Chapter 11: Accounting for Not-for-Profit Organisations

Example 22: Professional Sports Club (I)


Question: The following information relates to Professional Sports Club (PSC), a Not-for-Profit
Organisation.
Trial balance as at 30 June 20X3
Dr. Cr.
Rs. m Rs. m
Total Funds as on 01 July 20X3 1,715
Long term assets (net) 428
Investments 1,204
Short term bank loan 17
Prepaid and accrued expenses 8 11
Cash at bank 43
Fee-for-services 340
Fundraising in various tournaments (net proceeds) 15
Contributions 94
Government funding 150
Investment income 144
Salaries 403
Rent and utilities 354
Other expenses 46
2,486 2,486

Additional information:
1. The composition of fund balances is as follows:
Rs. m
Fund for Supporting the Young-Talent (Externally imposed stipulation
50
that resources contributed be maintained permanently)
Fund for gymnasium and training centre (Externally imposed
115
stipulation for specific use of resources)
Fund for acquiring a franchise in a popular league (Internally imposed
3
stipulation for specific use of resources)
Fund for general operations: no restrictions 1,547
1,715

2. The details of contributions (same restrictions apply as are applicable to related fund) are as
follows:
Rs. m
Contribution for ‘Supporting the Young-Talent’ 15
Contribution for gymnasium and training centre 2
Contribution to acquire freehold land (external restriction). However,
12
the land has not been acquired yet.
Contribution to repay loan that was taken to fund current year
8
expenses
Contributions (unrestricted but PSC itself imposed restriction that Rs.
57
3 million will be allocated for acquiring franchise in a popular league)
94

© Emile Woolf International 509 The Institute of Chartered Accountants of Pakistan


Financial accounting and reporting I

3. The government funding was received to support PSC general operations for five years starting
from 1st January 20X4.
4. The investment income of Rs. 6 million is externally restricted to be added to principal amount
of resources for Young-Talent fund to be maintained permanently. There is no other restrictions
on investment income.
5. Long term assets in the trial balance include freehold land of Rs. 20 million and collections of
Rs. 8 million. These collections represent items of such historic value that is worth preserving
perpetually and PSC is committed to protect and preserve them as part of its organisation
policy.
6. Long term assets are depreciated at 20% reducing balance method. All the amortisation is
allocated to general operations.
7. As part of agreement with contributors of ‘Supporting the Young-Talent’, PSC is required to
allocate Rs. 5 million from unrestricted fund to the restricted fund, annually.
8. The allocation of expenses is as follows:
Gymnasium
General
and training
operations
centre
Rs. m Rs. m
Salaries 370 33
Rent and utilities 325 29
Other expenses 40 6
735 68
Required:
Prepare the following (under deferral method) for PSD:
 Statement of income and expenditure for the year ended 30 June 20X4.
 Statement of changes in net assets for the year ended 30 June 20X4.
 Statement of financial position as at 30 June 20X4 (single column).
Answer:
Professional Sports Club
Statement of income and expenditure
For the year ended 30 June 20X4
Income Rs. m
Fee-for-services 340
Fundraising proceeds 15
Contributions [8 + 57] 65
Government funding [150 / 5 years x 6/12] 15
Investment income [144 – 6] 138
573
Expenditures
Salaries 370
Rent and utilities 325
Other expenses 40
Amortisation of capital assets [(428 – 8 – 20) x 20%] 80
(815)
Surplus / (Deficit) (242)

© Emile Woolf International 510 The Institute of Chartered Accountants of Pakistan


Chapter 11: Accounting for Not-for-Profit Organisations

Professional Sports Club


Statement of changes in net assets
For the year ended 30 June 20X4
Internally
Unrestricted Externally restricted
restricted
Franchise Gymnasium Young-Talent Total
General fund acquisition & Training Endowment
Centre Fund fund
Rs. m
Balance 1 July 1,547 3 115 50 1,715
Surplus / (deficit) (242) (242)
Endowment
15 15
Contributions
Restricted grants &
2 2
contributions
Investment income 6 6
Fund utilisation (68) (68)
Internally imposed
(3) 3 0
restrictions
Transfers (5) 5 0
Balance 30 June 1,297 6 49 76 1,428

Professional Sports Club


Statement of financial position
As at 30 June 20X4
Non-current assets Rs. m
Capital assets [428 – 8 – 80] 340
Collections 8
Investments 1,204
1,552
Current assets
Prepaid expenses 8
Cash at bank 43
51
1,603
Fund balances / Net assets
Net assets: restricted for endowments 76
Net assets: Externally restricted gymnasium and training centre 49
Net assets: internally restricted for franchise rights 6
General fund / Unrestricted net assets 1,297
1,428

Non-current liabilities
Deferred grants/contributions [150 – 15 – 30] 105

© Emile Woolf International 511 The Institute of Chartered Accountants of Pakistan


Financial accounting and reporting I

Current liabilities
Deferred grants/contributions [12 for land + 150 / 5] 42
Short term bank loan 17
Accrued expenses 11
70
1,603
.

Example 23: Professional Sports Club (II)


Question: The following information relates to Professional Sports Club (PSC), a Not-for-Profit
Organisation.
Trial balance as at 30 June 20X3

Dr. Cr.
Rs. m Rs. m
Total Funds as on 01 July 20X3 1,715
Long term assets (net) 428
Investments 1,204
Short term bank loan 17
Prepaid and accrued expenses 8 11
Cash at bank 43
Fee-for-services 340
Fundraising in various tournaments (net proceeds) 15
Contributions 94
Government funding 150
Investment income 144
Salaries 403
Rent and utilities 354
Other expenses 46
2,486 2,486

Additional information:
1. The composition of fund balances is as follows:

Rs. m
Fund for Supporting the Young-Talent (Externally imposed stipulation
50
that resources contributed be maintained permanently)
Fund for gymnasium and training centre (Externally imposed
115
stipulation for specific use of resources)
Fund for acquiring a franchise in a popular league (Internally imposed
3
stipulation for specific use of resources)
Fund for general operations: no restrictions 1,547
1,715

© Emile Woolf International 512 The Institute of Chartered Accountants of Pakistan


Chapter 11: Accounting for Not-for-Profit Organisations

2. The details of contributions (same restrictions apply as are applicable to related fund) are as
follows:

Rs. m
Contribution for ‘Supporting the Young-Talent’ 15
Contribution for gymnasium and training centre 2
Contribution to acquire freehold land (external restriction). However,
12
the land has not been acquired yet.
Contribution to repay loan that was taken to fund current year
8
expenses
Contributions (unrestricted but PSC itself imposed restriction that Rs.
57
3 million will be allocated for acquiring franchise in a popular league)
94

3. The government funding was received to support PSC general operations for five years starting
from 1st January 20X4.
4. The investment income of Rs. 6 million is externally restricted to be added to principal amount
of resources for Young-Talent fund to be maintained permanently. There is no other restrictions
on investment income.
5. Long term assets in the trial balance include freehold land of Rs. 20 million and collections of
Rs. 8 million. These collections represent items of such historic value that is worth preserving
perpetually and PSC is committed to protect and preserve them as part of its organisation
policy.
6. Long term assets are depreciated at 20% reducing balance method. All the amortisation is
allocated to general operations.
7. As part of agreement with contributors of ‘Supporting the Young-Talent’, PSC is required to
allocate Rs. 5 million from unrestricted fund to the restricted fund, annually.
8. The allocation of expenses is as follows:

Gymnasium
General
and training
operations
centre

Rs. m Rs. m

Salaries 370 33

Rent and utilities 325 29

Other expenses 40 6

735 68

Required:
Prepare the following (under restricted fund method) for PSD:
 Statement of income and expenditure for the year ended 30 June 20X4.
 Statement of changes in net assets for the year ended 30 June 20X4.
 Statement of financial position as at 30 June 20X4 (single column).

© Emile Woolf International 513 The Institute of Chartered Accountants of Pakistan


Financial accounting and reporting I

Answer:
Professional Sports Club
Statement of income and expenditure
For the year ended 30 June 20X4

Income Unrestricted Restricted Endowment Total


Rs. m
Fee-for-services 340 340
Fundraising proceeds 15 15
Contributions [8 + 57] 65 2 15 82
Government funding 15 15
[150 / 5 years x 6/12]
Investment income 138 6 144
573 2 21 596
Expenditures
Salaries 370 33 403
Rent and utilities 325 29 354
Other expenses 40 6 46
Amortisation [(428 – 8 – 20) x 20%] 80 80
(815) (68) 0 (883)
Surplus / (Deficit) (242) (66) 21 (287)

Professional Sports Club


Statement of changes in net assets
For the year ended 30 June 20X4

Internally
Unrestricted Externally restricted
restricted
Total
Franchise Gymnasium Young-Talent
General fund acquisition & Training Endowment
Centre Fund fund

Rs. m

Balance 1 July 1,547 3 115 50 1,715

Surplus / (deficit) (242) (66) 21 (287)

Internally imposed
(3) 3 0
restrictions

Transfers (5) 5 0

Balance 30 June 1,297 6 49 76 1,428

© Emile Woolf International 514 The Institute of Chartered Accountants of Pakistan


Chapter 11: Accounting for Not-for-Profit Organisations

Professional Sports Club


Statement of financial position
As at 30 June 20X4

Non-current assets Rs. m


Capital assets [428 – 8 – 80] 340
Collections 8
Investments 1,204
1,552
Current assets
Prepaid expenses 8
Cash at bank 43
51
1,603

Fund balances / Net assets


Net assets: restricted for endowments 76
Net assets: Externally restricted gymnasium and training centre 49
Net assets: internally restricted for franchise rights 6
General fund / Unrestricted net assets 1,297
1,428
Non-current liabilities
Deferred grants/contributions [150 – 15 – 30] 105

Current liabilities
Deferred grants/contributions [12 for land + 150 / 5] 42
Short term bank loan 17
Accrued expenses 11
70
1,603
.

© Emile Woolf International 515 The Institute of Chartered Accountants of Pakistan


Financial accounting and reporting I

Example 24: Chongtar International Hospital


Question: Following is the trial balance of Chongtar International Hospital as on 31 December
2019:
Debit Credit
---- Rs. in million ----
Burns ward - capital work in progress 55.3
Cafeteria sales 24.4
Cash and bank balances 8.4
Donations for burns ward (externally restricted) 75.1
Expenses and gifts for ‘walk on diabetes day’ 2.6
Fees from patients 125.0
General donations 82.6
General fund 195.6
Inventory – cafeteria 4.7
Inventory – medicines 19.4
Inventory – hospital supplies 8.5
Medical equipment 185.4 64.2
Miscellaneous expenses 8.5
Other fixed assets 110.7 54.7
Payables 38.9
Purchases – cafeteria 16.4
Purchases – medicines 60.5
Purchases – hospital supplies 18.7
Receivables – panel corporates 31.4
Rent 19.6
Sponsorship for ‘walk on diabetes day’ 2.2
Salaries – administrative staff 24.0
Salaries – doctors and nursing staff 38.2
Short term investments 38.0
Utilities 12.4
662.7 662.7
Additional information:
(i) Cost of closing physical inventory of medicines and hospital supplies was Rs. 25.8
million and Rs. 13.8 million respectively. Medicines costing Rs. 3.1 million were found
expired. Medicines are only used to treat the admitted patients and are not sold
separately.
(ii) Year-end physical count of cafeteria inventory could not take place. Goods are sold in
cafeteria at a gross margin of 25% on sales.
(iii) Rent outstanding at year-end was Rs. 1.4 million.
(iv) 15% of salaries and 10% of rent are related to cafeteria.
(v) Hospital facilities of Rs. 48.6 million were provided free of charge to the patients.
(vi) ‘Walk on diabetes day’ was organised in December 2019. Expenses relating to the event
amounting to Rs. 1.2 million were outstanding and unrecorded at year end.
(vii) Medical equipment having fair value of Rs. 36.8 million were received as donation on
1st January 2019. These have been brought into use but have not been recorded in the
books.
(viii) Depreciation is charged on reducing balance method at 15% per annum.

© Emile Woolf International 516 The Institute of Chartered Accountants of Pakistan


Chapter 11: Accounting for Not-for-Profit Organisations

Required:
(a) Prepare income and expenditure account for the year ended 31 December 2019
(b) Prepare statement of financial position as on 31 December 2019

Answer:
Chongtar International Hospital
Part (a)
Income and expenditure account for the year ended 31 December 2019
Rs. in million
Income:
Fees from patients 125.0
General donations 82.6
Amortisation of contribution of capital asset 36.8 x 15% 5.5
Fundraising – Walk on diabetes day 2.2
Profit from cafeteria 0.4
215.7
Expenditures:
Salaries - Administrative staff 24×0.85 20.4
Salaries - Doctors and nursing staff 38.2
Medicines used (19.4+60.5) – (25.8–3.1) 57.2
Hospital supplies used (8.5+18.7)–13.8 13.4
Rent (19.6 +1.4)×0.9 18.9
Walk on diabetes day (2.6+1.2) 3.8
Depreciation - Medical equipment [(185.4–64.2)+36.8]×15% 23.7
Depreciation - Other fixed assets (110.7–54.7)×15% 8.4
Utilities 12.4
Miscellaneous expenses 8.5
(204.9)
Excess of income over expenditure 10.8
Cafeteria trading account for the year ended 31 December 2018
Rs. in million
Sales 24.4
Cost of goods sold:
Opening stock 4.7
Purchases 16.4
Closing stock Balancing (2.8)
(18.3)
Gross profit 24.4×0.25 6.1
Expenses:
Salaries 24×0.15 (3.6)
Rent (19.6+1.4)×0.1 (2.1)
Profit from cafeteria 0.4

© Emile Woolf International 517 The Institute of Chartered Accountants of Pakistan


Financial accounting and reporting I

Part (b)
Statement of financial position as on 31 December 2019
Rs. in million
Non-current assets:
Medical Equipment (185.4–64.2)+36.8–23.7 134.3
Other fixed assets (110.7–54.7–8.4) 47.6
Burns ward - CWIP 55.3
237.2
Current assets:
Cafeteria stock 2.8
Medicines stock (25.8 -3.1) 22.7
Hospital supplies 13.8
Receivables 31.4
Short term investments 38.0
Cash in hand 8.4
117.1
354.3
Net assets:
General fund: Unrestricted (195.6 + 10.8) 206.4
Burns ward fund: restricted 75.1
281.5
Non-current liabilities:
Deferred contributions (36.8 x 85%) 31.3
31.3
Current liabilities:
Creditors 38.9
Accrued expenses 1.4+1.2 2.6
41.5
354.3
.

© Emile Woolf International 518 The Institute of Chartered Accountants of Pakistan


Chapter 11: Accounting for Not-for-Profit Organisations

8 OBJECTIVE BASED QUESTIONS


01. Which of the following is generally considered as a non-profit oriented organization?
(a) Charitable organization
(b) Corporation
(c) Audit firms
(d) Insurance companies

02. Expenditures greater than incomes of a non-profit organization give rise to a:


(a) Loss
(b) Profit
(c) Surplus
(d) Deficit

03. An advance receipt of subscription from a member of the non-profit organization is considered
as a/an:
(a) Expense
(b) Liability
(c) Equity
(d) Asset

04. Income and expenditure account is based on;


(a) Cash accounting
(b) Accrual accounting
(c) Government accounting
(d) Management accounting

05. Life membership fees of not for profit concerns is?


(a) Capital Receipts
(b) Revenue Receipts
(c) Both (a) & (b)
(d) None (a) & (b)

06. When cash is received for life membership, which one of the following double entries is passed?
(a) Cash Debit and capital Credit
(b) Life membership Debit and cash Credit
(c) Investment Debit and cash Credit
(d) Cash Debit and life membership fund Credit

© Emile Woolf International 519 The Institute of Chartered Accountants of Pakistan


Financial accounting and reporting I

07. XYZ club has a bar that maintains a separate trading account for its trading activities. Which of
the following is the treatment of profit or loss on bar trading activities?
(a) Profit or loss is directly shown in the statement of financial position
(b) Profit or loss is to be presented in income and expenditure account
(c) Profit or loss is credited in income statement
(d) Profit or loss is added to accumulated fund

08. Which of the following is the accounting equation for a non-profit organization?
(a) Asset = Capital + Liabilities
(b) Capital + Liabilities = Assets
(c) Accumulated fund + Liabilities = Assets
(d) Liabilities = Asset + Accumulated fund

09. Subscription earned but not yet received is considered as a/an:


(a) Asset
(b) Liability
(c) Income
(d) Expenditure

10. A non-profit organization received Rs. 100,000 as the entrance fee of a new member. If 20% of
the fee has to be capitalized, what is the amount of fee needs to be shown in the income and
expenditure account?
(a) Rs. 20,000
(b) Rs. 80,000
(c) Rs. 90,000
(d) Rs. 10,000

11. Rs. 1,000,000 received as the annual membership subscription. Out of this, Rs. 200,000 is
pertaining to the previous accounting period whereas Rs. 100,000 is receivable at the end of the
current accounting period.
Calculate the amount of subscription that will be shown in the income and expenditure account.
(a) Rs. 100,000
(b) Rs. 900,000
(c) Rs. 1,200,000
(d) Rs. 800,000

12. Income and expenditure accounts show:


(a) Cash available to an organization
(b) Closing capital of an organization
(c) Cash available in the bank account
(d) Surplus or deficit for the current accounting period

© Emile Woolf International 520 The Institute of Chartered Accountants of Pakistan


Chapter 11: Accounting for Not-for-Profit Organisations

13. On what basis the ‘receipts and payments account’ is prepared?


(a) Cash basis
(b) Accrual basis
(c) Both accrual and cash basis
(d) None of the two

14. Payment of Honorarium to secretary is treated as?

(a) Capital Expenditure

(b) Revenue Expenditure

(c) Cash Expenses

(d) None of these

15. Income and Expenditure Account records:


(a) Capital items
(b) Revenue items
(c) A and B both
(d) None of these

16. A club has 500 members. Annual membership fees are Rs. 1,000. Therefore, membership fees
for the year should be Rs. 500,000.
The club’s subscription records for the year ended 31 December 2013 show the following:
At 31 December At 31 December
2012 2013
Subscriptions in advance 10,000 6,000
Subscriptions in arrears 18,000 22,000
Calculate the amount of cash received during the year.
Rs. ___________

17. At 31 March 2012 a cricket club had membership subscriptions in arrears amounting to Rs.
48,000 and had received Rs. 12,000 subscriptions in advance.
During the year to 31 March 2013 the club received Rs. 624,000 including 26 memberships for
the year to 31 March 2014 at Rs. 1,200 per annum in advance.
At 31 March 2013 16 members owed subscriptions of Rs. 1,200 each.
Calculate the amount of subscription income during the year.
Rs. ___________

© Emile Woolf International 521 The Institute of Chartered Accountants of Pakistan


Financial accounting and reporting I

18. At 31 March 2012 a cricket club had membership subscriptions in arrears amounting to Rs.
48,000 and had received Rs. 12,000 subscriptions in advance.
During the year to 31 March 2013 the club received Rs. 624,000 including 26 memberships for
the year to 31 March 2014 at Rs. 1,200 per annum.
At 31 March 2013 16 members owed subscriptions of Rs. 1,200 each.
Half of the members who were in arrears at the end of the previous period still had not paid by
31 March 2013. It was decided to write these amounts off.
Required:
Calculate the amount of subscription income during the year.
Rs. ___________

19. Seaview Club started its operations on 1 February 2015. Total subscription received for the
period ended 31 December 2015 was Rs. 29,952,000
Annual subscription is Rs. 24,000. All new members pay three years’ subscription in advance.
The memberships were awarded as follows:
Month March June September December
No. of member 112 98 101 105
What amount of subscription income should be included in income and expenditure account for
the period ended 31 December 2015?
Rs. ___________

20. Seaview Club started its operations on 1 February 2015. Total subscription received for the
period ended 31 December 2015 was Rs. 29,952,000
Annual subscription is Rs. 24,000. All new members pay three years’ subscription in advance.
The memberships were awarded as follows:
Month March June September December
No. of member 112 98 101 105
What amount of advance subscription should be included in non-current liabilities as at 31
December 2015?
Rs. ___________

21. The main objective of a non-profit organization is;


(a) To earn profits
(b) To create monopoly
(c) Welfare of the society
(d) To provide for owner’s dividends

22. Non-profit organizations prepare all of the following accounts except the;
(a) Receipt and payment account
(b) Income and expenditure account
(c) Statement of financial position
(d) Profit or loss account

© Emile Woolf International 522 The Institute of Chartered Accountants of Pakistan


Chapter 11: Accounting for Not-for-Profit Organisations

23. Examples of non-profit organization is:


(a) Commercial banks
(b) Civil hospital
(c) Private educational institutions
(d) Association of person

24. The main source of income for non-profit organization is:


(a) Subscription
(b) Sales
(c) Dividends
(d) Other income

25. Income and expenditure accounts show;


(a) Cash available to an organization
(b) Closing capital of an organization
(c) Cash available to the bank account
(d) Surplus or deficit for an accounting period

26. The statement of financial position of a non-profit organization does not contain the;
(a) Owner’s equity
(b) Liability
(c) Asset
(d) Income

27. Rent expense of a non-profit organization paid in advance. Which of the following is the
correct classification of rent?
(a) Expense
(b) Liability
(c) Asset
(d) Equity

28. An advance receipt of subscription from a member of the non-profit organization is considered
as a/an?
(a) Expense
(b) Liability
(c) Asset
(d) Income

© Emile Woolf International 523 The Institute of Chartered Accountants of Pakistan


Financial accounting and reporting I

29. The capital of a non-profit organization is generally known as


(a) Equity
(b) Accumulated funds
(c) Retained earning
(d) Cash fund

30. When cash is received for life membership, which one of the following double entries is passed?
(a) Cash (debit) and capital (credit)
(b) Life membership (debit) and cash (credit)
(c) Cash (debit) and investment (credit)
(d) Cash (debit) and Life membership (credit)

31. If debit side of receipt and payment account exceeds credit, it represents:
(a) Cash at bank
(b) Bank overdraft
(c) Surplus
(d) Deficit

32. Receipt and payment account include:


(a) Revenue items
(b) Capital items
(c) Both capital and revenue items
(d) None of above

33. Sale of an old newspaper is classified as:


(a) Expense
(b) Liability
(c) Asset
(d) Income

34. Gift presented to Chief Guest at annual function by a non-profit organization is:
(a) Gift
(b) Reward
(c) Honorarium
(d) Grant

© Emile Woolf International 524 The Institute of Chartered Accountants of Pakistan


Chapter 11: Accounting for Not-for-Profit Organisations

35. Morning Football Club has a monthly subscription fee of Rs. 800 per member. The club has 240
members on 31 December 2018. No fresh members were admitted during 2018 but 30 members
left the club on 1 July 2018. As at 31 December 2018, the club has received subscription in
advance amounting to Rs. 60,000. The club’s subscription income for 2018 would be:
(a) Rs. 2,448,000
(b) Rs. 2,388,000
(c) Rs. 2,160,000
(d) Rs. 2,100,000

36. Alpha Club’s financial year ends on 31 December. Following information pertain to its members’
subscription:
Rupees
Subscription received in 2018 for 2019 180,000
Subscription received in 2019 for 2018 90,000
Subscription received in 2019 for 2019 1,400,000
Subscription received in 2019 for 2020 200,000
Subscription for 2018 outstanding as on 31 December 2018 150,000
Subscription for 2019 outstanding as on 31 December 2019 325,000

Subscription income for the year ended 31 December 2019 is:


(a) Rs. 1,845,000
(b) Rs. 1,705,000
(c) Rs. 1,905,000
(d) Rs. 1,665,000

37. The ‘Accounting Standard for NPOs’ has been issued by:
(a) The Institute of Chartered Accountants of Pakistan
(b) The Securities and Exchange Commission of Pakistan
(c) International Accounting Standards Board
(d) All Pakistan Association of NPOs

38. The primary source as basis of accounting for a large sized NPO is:
(a) Accounting Standard for NPOs
(b) Accounting and Financial Reporting Standards (AFRS)
(c) IFRSs for SMEs issued by IASB
(d) IFRSs issued by IASB

© Emile Woolf International 525 The Institute of Chartered Accountants of Pakistan


Financial accounting and reporting I

39. According to ASNPO, the financial statements of an NPO use the following concept of capital
maintenance:
(a) Physical capital maintenance
(b) Welfare capital maintenance
(c) Financial capital maintenance (real terms)
(d) Financial capital maintenance (money terms)

40. Statement of income and expenditures is based on:


(a) Cash accounting
(b) Accrual accounting
(c) Government accounting
(d) Management accounting

41. An NPO has a fund that is subject to externally imposed stipulations specifying the resources
contributed be maintained permanently, although the constituent assets may change from time
to time. Which type of fund it is?
(a) Unrestricted fund
(b) Capital assets fund
(c) Restricted fund
(d) Endowment fund

42. Which of the following statement is incorrect with respect to restrictions on contribution revenue
of an NPO?
(a) Restrictions may be externally imposed.
(b) Restrictions may be internally imposed.
(c) Restrictions may be explicit.
(d) Restrictions may be implicit.
43. An NPO received endowment contributions of Rs. 2 million. How should the receipt be
recognised under restricted fund method where all funds are separately reported?
(a) Recognise as revenue in statement of income and expenditure (in endowment fund
column) of the current period.
(b) Recognise as revenue in statement of income and expenditure (in general fund column)
of the current period.
(c) Recognise as direct increase in statement of changes in net assets (in endowment fund
column) of the current period.
(d) Recognise as direct increase in statement of changes in net assets (in general fund
column) of the current period.

© Emile Woolf International 526 The Institute of Chartered Accountants of Pakistan


Chapter 11: Accounting for Not-for-Profit Organisations

44. An NPO received endowment contributions of Rs. 2 million. How should the receipt be
recognised under deferral method?
(a) Recognise as revenue in statement of income and expenditure (in endowment fund
column) of the current period.
(b) Recognise as revenue in statement of income and expenditure (in general fund column)
of the current period.
(c) Recognise as direct increase in statement of changes in net assets (in endowment fund
column) of the current period.
(d) Recognise as direct increase in statement of changes in net assets (in general fund
column) of the current period.

45. An NPO received restricted contributions for expenses of current period (the corresponding
restricted fund is presented separately) of Rs. 2 million. How should the receipt be recognised
under restricted fund method?
(a) Recognise as revenue in statement of income and expenditure (in restricted fund
column) of the current period.
(b) Recognise as revenue in statement of income and expenditure (in general fund column)
of the current period.
(c) Recognise as direct increase in statement of changes in net assets (in restricted fund
column) of the current period.
(d) Recognise as direct increase in statement of changes in net assets (in general fund
column) of the current period.

46. An NPO received restricted contributions for expenses of current period (the corresponding
restricted fund is not presented separately) of Rs. 2 million. How should the receipt be recognised
under restricted fund method?
(a) Recognise as revenue in statement of income and expenditure (in restricted fund
column) of the current period.
(b) Recognise as revenue in statement of income and expenditure (in general fund column)
of the current period.
(c) Recognise as direct increase in statement of changes in net assets (in restricted fund
column) of the current period.
(d) Recognise as direct increase in statement of changes in net assets (in general fund
column) of the current period.

47. An NPO received restricted contributions for expenses of current period (corresponding
restricted fund is separately reported in financial statements) of Rs. 2 million. How should the
receipt be recognised under deferral method?
(a) Recognise as revenue in statement of income and expenditure (in restricted fund
column) of the current period.
(b) Recognise as revenue in statement of income and expenditure (in general fund column)
of the current period.
(c) Recognise as direct increase in statement of changes in net assets (in restricted fund
column) of the current period.
(d) Recognise as direct increase in statement of changes in net assets (in general fund
column) of the current period.

© Emile Woolf International 527 The Institute of Chartered Accountants of Pakistan


Financial accounting and reporting I

48. An NPO uses restrictive fund method to report its contributions revenue. The fund balance (or
deferred balance) related to _____________ shall not be presented in net assets in the
statement of financial position.
(a) Endowment contributions
(b) Restricted contributions reported in restricted fund
(c) Restricted contributions reported in general fund
(d) Unrestricted contributions reported in general fund

49. An NPO received restricted contribution for the repayment of debt. The related debt was incurred
for the payment of expenses expected to be incurred in next three years. How should the receipt
be recognised under deferral method?
(a) Recognise as revenue in statement of income and expenditures of the current period
(b) Defer and recognise as revenue in relevant period applying the matching concept
(c) Recognise as direct increase in net assets in the statement of changes in net assets
(d) Recognise as deduction from related capital asset or related expenses

50. An NPO received restricted contribution for the repayment of debt. The related debt was taken
to fund purchase of school’s furniture. How should the receipt be recognised under deferral
method?
(a) Recognise as revenue in statement of income and expenditures of the current period

(b) Defer and recognise as revenue in relevant period applying the matching concept

(c) Recognise as direct increase in net assets in the statement of changes in net assets

(d) Recognise as deduction from related capital asset or related expenses

51. An NPO received restricted contribution for the repayment of debt. The related debt was taken
to fund purchase of freehold land for construction of a school. How should the receipt be
recognised under deferral method?
(a) Recognise as revenue in statement of income and expenditures of the current period
(b) Defer and recognise as revenue in relevant period applying the matching concept
(c) Recognise as direct increase in net assets in the statement of changes in net assets
(d) Recognise as deduction from related capital asset or related expenses
52. An NPO received restricted contribution for the repayment of debt. The related debt was taken
neither to fund expenses of future periods nor to fund purchase of any capital assets. How should
the receipt be recognised under deferral method?
(a) Recognise as revenue in statement of income and expenditures of the current period
(b) Defer and recognise as revenue in relevant period applying the matching concept
(c) Recognise as direct increase in net assets in the statement of changes in net assets
(d) Recognise as deduction from related capital asset or related expenses

© Emile Woolf International 528 The Institute of Chartered Accountants of Pakistan


Chapter 11: Accounting for Not-for-Profit Organisations

53. An NPO earned investment income that is externally restricted to be held for endowment. How
should it be recognised under deferral method?
(a) Recognise as revenue in statement of income and expenditures of the current period
(b) Defer and recognise as revenue in relevant period applying the matching concept
(c) Recognise as direct increase in net assets in the statement of changes in net assets
(d) Recognise as deduction from related capital asset or related expenses

54. An NPO earned investment income that is externally restricted to be accumulated for purchase
of freehold land. How should it be recognised under deferral method?
(a) Recognise as revenue in statement of income and expenditures of the current period
(b) Defer and recognise as revenue in relevant period applying the matching concept
(c) Recognise as direct increase in net assets in the statement of changes in net assets
(d) Recognise as deduction from related capital asset or related expenses

55. An NPO has of 16,000 bags of rice to be sold only to members of an underprivileged community.
The cost of rice to the NPO is Rs. 600 per bag. However, the NPO sells one bag to the
underprivileged persons for Rs. 50 only. How should these 16,000 bags be measured?
(a) At cost
(b) At lower of cost and NRV
(c) At lower of current replacement cost and NRV
(d) At lower of cost and current replacement cost

56. An NPO had capital asset of furniture at carrying value of Rs. 800,000 and related unamortised
deferred contribution balance of Rs. 480,000 in statement of financial position. At that date, the
furniture was destroyed by fire completely and is now worth nothing. Which of the following is
correct treatment?
(a) Write down the furniture by Rs. 480,000 against deferred contribution.
(b) Write down the furniture by Rs. 800,000 and recognise revenue of Rs. 480,000 related
to deferred contribution provided that all restrictions have been complied with.
(c) Write down the furniture by Rs. 800,000 and transfer Rs. 480,000 related to deferred
contribution, directly in net assets provided that all restrictions have been complied with.
(d) Both (b) and (c) are acceptable accounting treatments.

© Emile Woolf International 529 The Institute of Chartered Accountants of Pakistan


Financial accounting and reporting I

8 OBJECTIVE BASED ANSWERS


01. (a)
02. (d)
03. (b)
04. (b)
05. (a)
06. (d)
07. (b)
08. (c)
09. (a)
10. (b)
11. (b)
Subscription a/c
Particulars Rs. Particulars Rs.
b/d 200,000 Cash received 1,000,000
I&E 900,000 c/d 100,000
1,100,000 1,100,000

12. (d)
13. (a)
14. (b)
15. (b)
16. Rs. 492,000
Subscriptions
Rs. Rs.
Balance b/d 18,000 Balance b/d 10,000
I&E 500,000 Cash 492,000
Balance c/d 6,000 Balance c/d 22,000
524,000 524,000

17. Rs. 576,000


Subscriptions
Rs. Rs.
Balance b/d 48,000 Balance b/d 12,000
I&E 576,000 Cash 624,000
Balance c/d Balance c/d
[26 x Rs. 1,200] 31,200 [16 x Rs. 1,200] 19,200
655,200 655,200

© Emile Woolf International 530 The Institute of Chartered Accountants of Pakistan


Chapter 11: Accounting for Not-for-Profit Organisations

18. Rs. 600,000


Subscriptions
Rs. Rs.
Balance b/d 48,000 Balance b/d 12,000
I&E 600,000 Cash 624,000
Bad debts
[48,000 x ½] 24,000
Balance c/d Balance c/d
[26 x Rs. 1,200] 31,200 [16 x Rs. 1,200] 19,200
679,200 679,200

19. Rs. 4,630,000 Subscription for 3 years is Rs. 72,000 so subscription for 1 year is Rs.
24,000 or Rs. 2,000 per month
Rs.000

Receipt / I&E Current Non-current


Members Months Rs. Months Rs. Months Rs.
Mar / 112 10 2,240 12 2,688 14 3,136
Jun / 98 7 1,372 12 2,352 17 3,332
Sep / 101 4 808 12 2,424 20 4,040
Dec / 105 1 210 12 2,520 23 4,830
4,630 9,984 15,338

20. Rs. 15,338,000 Subscription for 3 years is Rs. 72,000 so subscription for 1 year is Rs.
24,000 or Rs. 2,000 per month
Rs.000

Month / I&E Current Non-current


Members Months Rs. Months Rs. Months Rs.
Mar / 112 10 2,240 12 2,688 14 3,136
Jun / 98 7 1,372 12 2,352 17 3,332
Sep / 101 4 808 12 2,424 20 4,040
Dec / 105 1 210 12 2,520 23 4,830
4,630 9,984 15,338

21. (c)
22. (d)
23. (b)
24. (a)
25. (d)
26. (a)
27. (c)

© Emile Woolf International 531 The Institute of Chartered Accountants of Pakistan


Financial accounting and reporting I

28. (b)
29. (b)
30. (d)
31. (a)
32. (c)
33. (d)
34. (c)
35. (a) Rs. 2,448,000
36. (c) Rs. 1,905,000
37. (a) ASNPO has been issued by ICAP.
38. (d) A large sized NPO is required to apply IFRSs issued by IASB as applicable
in Pakistan. [ASNPO 1.9]
39. (d) Financial statements of an NPO are prepared with capital maintenance
measured in financial terms and with no adjustment being made for the
effect on capital of a change in the general purchasing power of the
currency during the period. [ASNPO Para 2.48]
40. (b) The statement of income and expenditures is based on accrual accounting.
41. (d) Such stipulations relate to endowment contribution and related fund would
be endowment fund. [ASNPO definitions]
42. (b) Restrictions (explicit or implicit) on contributions may only be externally
imposed. [ASNPO Para 6.5 and definitions]
43. (a) ASNPO Para 6.57
44. (c) ASNPO Para 6.28
45. (a) ASNPO Para 6.59
46. (b) ASNPO Para 6.62
47. (c) ASNPO Para 6.43 and 6.44
48. (c) ASNPO Para 6.64
49. (b) ASNPO Para 6.36
50. (b) ASNPO Para 6.34 and 6.39
51. (c) ASNPO Para 6.37
52. (a) ASNPO Para 6.38
53. (c) ASNPO Para 6.47 (b)
54. (c) ASNPO Para 6.47 (c)
55. (d) ASNPO Para 5.4
56. (b) ASNPO Para 8.12

© Emile Woolf International 532 The Institute of Chartered Accountants of Pakistan

You might also like